Sie sind auf Seite 1von 312

P

ICAP
Practice Kit

Advanced accounting
and financial reporting

© Emile Woolf International i The Institute of Chartered Accountants of Pakistan


Third edition published by
Emile Woolf Limited
Bracknell Enterprise & Innovation Hub
Ocean House, 12th Floor, The Ring
Bracknell, Berkshire, RG12 1AX United Kingdom
Email: info@ewiglobal.com
www.emilewoolf.com

© Emile Woolf International, November 2018

All rights reserved. No part of this publication may be reproduced, stored in a retrieval system, or transmitted,
in any form or by any means, electronic, mechanical, photocopying, recording, scanning or otherwise, without
the prior permission in writing of Emile Woolf Publishing Limited, or as expressly permitted by law, or under
the terms agreed with the appropriate reprographics rights organisation.

You must not circulate this book in any other binding or cover and you must impose the same condition on
any acquirer.

Notice
Emile Woolf International has made every effort to ensure that at the time of writing the contents of this study
text are accurate, but neither Emile Woolf International nor its directors or employees shall be under any
liability whatsoever for any inaccurate or misleading information this work could contain.

© Emile Woolf International ii The Institute of Chartered Accountants of Pakistan


Certified Finance and Accounting Professional

C
Advanced accounting and financial reporting

Contents
Page

Question and Answers Index v


Section A Questions 1
Section B Answers 109

© Emile Woolf International iii The Institute of Chartered Accountants of Pakistan


Advanced accounting and financial reporting

© Emile Woolf International iv The Institute of Chartered Accountants of Pakistan


Certified Finance and Accounting Professional

I
Advanced accounting and financial reporting

Index to questions and answers

CHAPTER 1 – BUSINESS COMBINATIONS AND CONSOLIDATION


1.1 HELLO 1 109
1.2 HASAN LIMITED 2 110
1.3 FLAMSTEED LTD AND HALLEY LTD 3 113
1.4 BRADLEY LTD 4 114
1.5 X LTD 5 116
1.6 KHAN LIMITED 6 118
1.7 WHITE LIMITED 7 122
CHAPTER 2 – BUSINESS COMBINATIONS ACHIEVED IN STAGES
2.1 STEP ACQUISITION 9 125
2.2 A LTD 9 125
2.3 X LTD GROUP 10 128
2.4 PLAIN LTD 12 130
2.5 MANGO LTD 13 133
CHAPTER 3 – CONSOLIDATED STATEMENTS OF PROFIT OR LOSS AND OTHER
COMPREHENSIVE INCOME
3.1 MILLARD LTD 15 136
3.2 SHERLOCK LIMITED 16 138
3.3 FAISAL LIMITED 17 141
3.4 GOLDEN LIMITED 19 145
CHAPTER 4 – COMPLEX GROUPS
4.1 PARVEZ LTD 21 147
4.2 HASAN, RIAZ AND SIDDIQ 22 150
4.3 LALIWALA GROUP 23 153
4.4 SHAKIR LIMITED 24 154
4.5 ANT, BEE AND FLY 25 156

© Emile Woolf International v The Institute of Chartered Accountants of Pakistan


Advanced accounting and financial reporting

CHAPTER 5 – ASSOCIATES AND JOINT VENTURES


5.1 JOINT ARRANGEMENTS 27 159
5.2 HELIUM 27 159
5.3 HAMACHI LTD 27 161
5.4 HIDE 28 163
5.5 HARK, SPARK AND ARK 29 164
5.6 P, S AND A 30 167
5.7 H LTD GROUP 31 169
5.8 ALPHA AND BETA 32 169
5.9 SNAKE LIMITED 33 171
CHAPTER 6 – OTHER GROUP STANDARDS
There are no specific questions in this area.
CHAPTER 7 – IAS 21: FOREIGN CURRENCY
7.1 DND LIMITED 34 172
7.2 STARLIGHT LIMITED 34 172
7.3 PERCEPT LTD 35 173
7.4 ORLANDO 36 174
7.5 MANCASTER AND STOCKPOT 36 174
7.6 A, B AND C 38 178
7.7 OMEGA LIMITED 39 181
7.8 PARENT COMPANY LIMITED 40 181
7.9 KANGAROO LIMITED 41 184
CHAPTER 8 – IAS 7: STATEMENTS OF CASH FLOWS
8.1 EVERNEW LTD 43 185
8.2 BISHOP GROUP 44 187
8.3 THE GRAPE GROUP 46 189
CHAPTER 9 – IFRS 9: FINANCIAL INSTRUMENTS: RECOGNITION AND
MEASUREMENT
9.1 AJI PANCA LTD 49 191
9.2 PASSILA LTD 49 192
9.3 FINANCIAL INSTRUMENTS 50 192
9.4 CASCABEL LTD 50 194
9.5 FAIR VALUE HEDGE ACCOUNTING 50 195
9.6 CASH FLOW HEDGE ACCOUNTING 51 196
9.7 WATERS LTD 51 197
9.8 ARIF INDUSTRIES LIMITED 52 199
9.9 QASMI INVESTMENT LIMITED 52 200
9.10 RASHID INDUSTRY LIMITED 53 201
9.11 LAHORE STEEL LIMITED 53 202

© Emile Woolf International vi The Institute of Chartered Accountants of Pakistan


Index to questions and answers

CHAPTER 10 – FINANCIAL INSTRUMENTS: PRESENTATION AND DISCLOSURE


10.1 SERRANO LIMITED 54 203
10.2 POBLANO LIMITED 54 203
10.3 PIQUIN LTD 54 204
10.4 AJI LTD 55 205
10.5 CHILTEPIN LTD 55 206
10.6 HABENERO LTD 55 206
10.7 PASHAM TELECOM LIMITED 56 207
CHAPTER 11 – IAS 19: EMPLOYEE BENEFITS
11.1 LABURNUM LIMITED 57 208
11.2 JABEL LIMITED 57 208
11.3 KAGHZI LIMITED 57 209
11.4 LASURA LTD 58 209
11.5 UNIVERSAL SOLUTIONS 58 210
11.6 DHA INTERIORS LTD 59 211
CHAPTER 12 – IFRS 2: SHARE BASED PAYMENTS
12.1 TOSHACK LTD 60 214
12.2 IFRS 2 60 214
12.3 SAVAGE LTD 60 216
12.4 YORATH LTD 61 216
12.5 QUALTECH LTD 61 217
12.6 BRIDGE LTD 61 217
12.7 CAPSTAN LTD 62 218
12.8 NEWTOWN LTD 62 218
12.9 SINDH TRANSIT LTD 62 219
12.10 XYZ LIMITED 63 220
12.11 RAVI LIMITED 64 221
CHAPTER 13 – DISPOSAL OF SUBSIDIARIES
13.1 PATCHE LTD 65 223
13.2 DISPOSAL 66 225
13.3 PART DISPOSAL 66 226
13.4 THE A GROUP 67 226
13.5 BARTLETT LTD 68 228
CHAPTER 14 – IFRS-5 : NON-CURRENT ASSETS HELD FOR SALE AND
DISCONTINUED OPERATIONS
14.1 SAUL 69 230
14.2 SHAHID HOLDINGS 70 231
14.3 PRIMA 71 233

© Emile Woolf International vii The Institute of Chartered Accountants of Pakistan


Advanced accounting and financial reporting

CHAPTER 15 – IFRS 13: FAIR VALUE MEASUREMENT


15.1 MONIBA LIMITED 72 235
CHAPTER 16 – IFRS 15: REVENUE FROM CONTRACTS WITH CUSTOMERS
16.1 PARVEZ LIMITED 73 236
16.2 SACHAL LIMITED 73 236
16.3 BRILLIANT LIMITED 73 238
16.4 WAQAS LIMITED 74 239
16.5 ZEBRA LIMITED 74 241
16.6 HAWKS LIMITED 75 241
CHAPTER 17 – IFRS 16: LEASES
17.1 X LTD 77 243
17.2 PROGRESS LIMITED 77 243
17.3 MIRACLE TEXTILE LIMITED 77 245
17.4 ACACIA LTD 78 246
17.5 SHOAIB LEASING LIMITED 78 247
17.6 AKBAR LIMITED 78 249
17.7 ALI LIMITED 79 250
17.8 MOAZZAM TEXTILE MILLS LIMITED 79 250
17.9 MODIFICATION THAT DECREASES THE SCOPE
OF THE LEASE (IFRS 16, ILLUSTRATIVE 79 252
EXAMPLE 17)
17.10 MODIFICATION THAT BOTH INCREASES AND
DECREASES THE SCOPE OF THE LEASE (IFRS 80 253
16, ILLUSTRATIVE EXAMPLE 18)
17.11 SUBLEASE CLASSIFIED AS A FINANCE LEASE
80 256
(IFRS 16, ILLUSTRATIVE EXAMPLE 20)
17.12 SUBLEASE CLASSIFIED AS AN OPERATING
80 256
LEASE (IFRS 16, ILLUSTRATIVE EXAMPLE 21)
17.13 TRACK LIMITED 80 256
17.14 PATEL LIMITED 81 257
CHAPTER 18 – IAS 12: INCOME TAXES
18.1 SHAKIR INDUSTRIES 82 260
18.2 DWAYNE LTD (PART 1) 83 261
18.3 DWAYNE LTD (PART 2) 83 261
18.4 COHORT 84 262
18.5 MODEL TOWN GROUP 85 263
18.6 ELEPHANT LIMITED 86 264
CHAPTER 19 – PRESENTATION OF FINANCIAL STATEMENTS (IAS 34, IAS 24)
19.1 FAZAL LIMITED 87 266
19.2 BABER LIMITED 87 266
19.3 GOLDEN LIMITED 87 267

© Emile Woolf International viii The Institute of Chartered Accountants of Pakistan


Index to questions and answers

19.4 METAL LIMITED 88 268


19.5 ENGINA 88 269
CHAPTER 20 – IAS 33: EARNINGS PER SHARE
20.1 AIRCON LTD 90 271
20.2 CACHET LTD 91 273
20.3 MARY 91 273
20.4 MANDY 92 274
20.5 AAZ LIMITED 92 275
20.6 ABC LIMITED 93 277
20.7 ALPHA LIMITED 93 279
20.8 SAJJAD LIMITED 94 280
20.9 TIGER LIMITED 95 281
CHAPTER 21 – IAS 36: IMPAIRMENT OF ASSETS
21.1 CHARLOTTE 96 283
21.2 ABA LIMITED 96 286
21.3 HUSSAIN ASSOCIATES LTD 97 287
21.4 IMPS 98 288
21.5 GYO MOVERS LIMITED 99 289
21.6 KHYBER LIMITED 100 291
CHAPTER 22 – IAS 40: INVESTMENT PROPERTY
22.1 VICTORIA 101 292
CHAPTER 23 – SUNDRY STANDARDS AND INTERPRETATIONS (IFRS 6, IFRS 14)
There are no specific questions in this area.
CHAPTER 24 – FIRST TIME ADOPTION OF IFRS
24.1 IFRS 1 102 294
CHAPTER 25 – SPECIALISED FINANCIAL STATEMENTS
25.1 LATEEF BANK LIMITED 103 296
25.2 AL-AMIN BANK LIMITED 103 297
25.3 IAS 26 104 297
25.4 SOGO LIMITED 104 298
25.5 JABBAR PVT LIMITED 105 299
25.6 KARACHI BANK LIMITED 106 300
25.7 LEOPARD INCOME FUND 106 300
CHAPTER 26 – INTERNATIONAL PUBLIC SECTOR ACCOUNTING STANDARDS
(IPSAS)
There are no specific questions in this area.
CHAPTER 27 – ACCOUNTING FOR HYPERINFLATION
There are no specific questions in this area
CHAPTER 28 – ISLAMIC ACCOUNTING STANDARDS
28.1 SALE AND LEASE BACK TRANSACTIONS 108 302

© Emile Woolf International ix The Institute of Chartered Accountants of Pakistan


Advanced accounting and financial reporting

© Emile Woolf International x The Institute of Chartered Accountants of Pakistan


Certified Finance and Accounting Professional

Q
Advanced accounting and financial reporting

SECTION
Questions
CHAPTER 1 - BUSINESS COMBINATIONS AND CONSOLIDATION
1.1. HELLO
On 1 January 2015, Hello acquired 60% of the ordinary share capital of Solong for Rs. 110,000.
At that date Solong had a retained earnings balance of Rs. 60,000.
The following statements of financial position have been prepared as at 31 December 2016.
Hello Solong
Rs. Rs.
Assets
Non-current assets
Property, plant and equipment 225,000 175,000
Investments in Solong 110,000

Current assets 271,000 157,000


———– ———–
606,000 332,000
———– ———–
Equity and liabilities
Capital and reserves
Share capital 100,000 100,000
Retained earnings 275,000 90,000
———– ———–
375,000 190,000
Current liabilities 231,000 142,000
———– ———–
606,000 332,000
———– ———–
The fair value of Solong’s net assets at the date of acquisition was determined to be Rs. 170,000.
The difference between the book value and the fair value of the new assets at the date of
acquisition was due to an item of plant which had a useful life of 10 years from the date of
acquisition.
Required
Prepare the consolidated statement of financial position of Hello and its subsidiary as at 31
December 2016.

© Emile Woolf International 1 The Institute of Chartered Accountants of Pakistan


Advanced accounting and financial reporting

1.2. HASAN LIMITED


On 1 April 2015, Hasan Limited acquired 90% of the equity shares in Shakeel Limited. On the
same day Hasan Limited accepted a 10% loan note from Shakeel Limited for Rs. 200,000 which
was repayable at Rs. 40,000 per annum (on 31 March each year) over the next five years.
Shakeel Limited’s retained earnings at the date of acquisition were Rs. 2,200,000.
Statements of financial position as at 31 March 2016
Hasan Limited Shakeel Limited
Rs.000 Rs.000 Rs.000 Rs.000
Non-current assets
Property, plant and equipment 2,120 1,990
Intangible – software – 1,800
Investments – equity in Shakeel Limited 4,110 –
Investments – 10% loan note Shakeel 200 –
Limited
Investments – others 65 210
6,495 4,000
Current assets
Inventories 719 560
Trade receivables 524 328
Shakeel Limited current account 75 –
Cash 20
1,338 888
Total assets 7,833 4,888

Equity and liabilities:


Capital and reserves
Equity shares of Rs. 1 each 2,000 1,500
Share premium 2,000 500
Retained earnings 2,900 1,955
6,900 3,955
Non-current liabilities
10% Loan note from Hasan Limited – 160
Government grant 230 40
230 200
Current liabilities
Trade payables 475 472
Hasan Limited current account – 60
Income taxes payable 228 174
Operating overdraft – 27
703 733
Total equity and liabilities 7,833 4,888

The following information is relevant:


(i) Included in Shakeel Limited’s property at the date of acquisition was a leasehold property
recorded at its depreciated historical cost of Rs. 400,000. The leasehold had been sub-let
for its remaining life of only four years at an annual rental of Rs. 80,000 payable in advance
on 1 April each year. The directors of Hasan Limited are of the opinion that the fair value of
this leasehold is best reflected by the present value of its future cash flows. An appropriate
cost of capital for the group is 10% per annum.

© Emile Woolf International 2 The Institute of Chartered Accountants of Pakistan


Questions

The present value of a Rs. 1 annuity received at the end of each year where interest rates
are 10% can be taken as:
3 year annuity Rs. 2.50
4 year annuity Rs. 3.20
(ii) The software of Shakeel Limited represents the depreciated cost of the development of an
integrated business accounting package. It was completed at a capitalised cost of Rs.
2,400,000 and went on sale on 1 April 2014. Shakeel Limited’s directors are depreciating
the software on a straight-line basis over an eight-year life (i.e. Rs. 300,000 per annum).
However, the directors of Hasan Limited are of the opinion that a five-year life would be more
appropriate as sales of business software rarely exceed this period.
(iii) The inventory of Hasan Limited on 31 March 2016 contains goods at a transfer price of Rs.
25,000 that were supplied by Shakeel Limited who had marked them up with a profit of 25%
on cost. Unrealised profits are adjusted for against the profit of the company that made them.
(iv) On 31 March 2016 Shakeel Limited remitted to Hasan Limited a cash payment of Rs. 55,000.
This was not received by Hasan Limited until early April. It was made up of an annual
repayment of the 10% loan note of Rs. 40,000 (the interest had already been paid) and Rs.
15,000 of the current account balance.
(v) The accounting policy of Hasan Limited for non-controlling interests (NCI) in a subsidiary is
to value NCI at a proportionate share of the net assets.
(v) An impairment test at 31 March 2016 on the consolidated goodwill concluded that it should
be written down by Rs. 120,000. No other assets were impaired.
Required
Prepare the consolidated statement of financial position of Hasan Limited as at
31 March 2016.
1.3. FLAMSTEED LTD AND HALLEY LTD
The draft Statement of Financial Position of Flamsteed Ltd and Halley Ltd on 30 June 2016 were
as follows:
Statement of financial position as at 30 June 2016
Flamsteed Ltd Halley Ltd
Rs.’000 Rs.’000
Assets:
Non-current Assets:
Property, plant and equipment 100,000 80,000
40,000 ordinary shares in Halley at cost 60,000 -
160,000 80,000
Current assets:
Inventory 6,000 16,000
Owed by Flamsteed Ltd - 20,000
Receivables 32,000 14,000
Cash 4,000 -
42,000 50,000
Total assets 202,000 130,000
Equity and liabilities:
Equity (ordinary shares @ Rs. 1) 90,000 50,000
Revaluation surplus 24,000 10,000
Retained earnings 52,000 56,000
166,000 116,000

© Emile Woolf International 3 The Institute of Chartered Accountants of Pakistan


Advanced accounting and financial reporting

Current Liabilities:
Owed to Halley Ltd 16,000 -
Trade payables 20,000 14,000
36,000 14,000
Total equity and liabilities 202,000 130,000
Additional information:
(i) Flamsteed Ltd acquired its investment in Halley Ltd on 1 July 2015, when the retained
earnings of Halley Ltd stood at Rs. 12,000,000.
(ii) The agreed consideration was Rs. 60,000,000 at the date of acquisition and a further Rs.
20,000,000 on 1 July 2017, Flamsteed Ltd’s cost of capital is 7%.
(iii) Halley Ltd has an internally developed brand name – TOLX – which was valued at Rs.
10,000,000 at the date of acquisition.
(iv) There have been no changes in the capital or revaluation surplus of Halley Ltd since the
date its shares were purchased.
(v) At 30 June 2016, Halley had invoiced Flamsteed Ltd for goods to the value of Rs.
4,000,000 and Flamsteed Ltd had sent payment in full but this had not been received by
Halley Ltd.
(vi) There is no impairment of goodwill.
(vii) It is the group’s policy to value non-controlling interest at full fair value.
(viii) At the acquisition date, the non-controlling interest was valued at Rs. 18,000,000.
Required
(a) Define Impairment loss in accordance with IAS 36 on Impairment of Assets.
(b) Explain any THREE sources of external information which an entity may consider in
assessing whether there is any indication that an asset may be impaired.
(c) Prepare an extract of consolidated Statement of Financial position of Flamsteed Ltd for the
year ended 30 June 2016, showing the assets side only.
1.4. BRADLEY LTD
Bradley Ltd’s purchased 960 million shares in Bliss Ltd a year ago when Bliss had a credit
balance of Rs. 190million in retained earnings. The fair value of the non-controlling interest at
the date of acquisition was Rs. 330million. At the date of acquisition, the freehold land of Bliss
Ltd was valued at Rs. 140million in excess of its carrying value. The revaluation has not been
recorded in the accounts of Bliss.
The statements of financial position of Bradley Ltd and Bliss Ltd as at 31 December 2016 are as
follows:
Bradley Ltd Bliss Ltd
Rs. Rs. Rs. Rs.
million million million million
Non Current Assets
Land and building 630 556
Machinery and equipment 570 440
Investment in Bliss Ltd. 1,320 -
2,520 996
Current Assets
Inventories 714 504
Trade receivables 1,050 252
Cash/bank 316 2,080 60 816
4,600 1,812
Ordinary Shares at Rs. 1 each 3,000 1,200

© Emile Woolf International 4 The Institute of Chartered Accountants of Pakistan


Questions

Bradley Ltd Bliss Ltd


Rs. Rs. Rs. Rs.
million million million million
Retained Earnings 1,160 424
Shareholders fund 4,160 1,624

Current Liabilities
Trade payables 440 188
4,600 1,812
Bliss Ltd owes Bradley Ltd Rs. 50million for goods purchased during the year. Inventory of Bliss
Ltd includes goods bought from Bradley Ltd at the price that includes a profit to Bradley Ltd of
Rs. 24million.
The management of Bradley Ltd wants the financial statements to be consolidated using the
acquisition method and wishes to know whether there is goodwill on acquisition of Bliss Ltd and
the amount involved.
Required
Prepare the consolidated statement of financial position as at 31 December 2016.
1.5. X LTD
The statements of financial position for X Ltd and Y Ltd as at 31 December 2016 are provided
below:
X Ltd Y Ltd
ASSETS Rs.000 Rs.000
Non-current assets
Property, plant and equipment 12,000 4,000
Available for sale investment (note 1) 4,000 -
Current assets 16,000 4,000
Inventories 2,200 800
Receivables 3,400 900
Cash and cash equivalents 800 300
6,400 2,000
Total assets 22,400 6,000
EQUITY AND LIABILITIES Equity
Share capital (Rs. 1 equity shares) 10,000 1,000
Retained earnings 7,500 4,000
Other reserves 200 -
Total equity 17,700 5,000
Non-current liabilities
Long term borrowings 2,700 -
Current liabilities 2,000 1,000
Total liabilities 4,700 1,000
Total equity and liabilities 22,400 6,000
Additional information:
1. X Ltd acquired a 75% investment in Y Ltd on 1 May 2016 for Rs. 3,800,000. The
investment has been classified as available-for-sale in the books of X Ltd. The gain on its
subsequent measurement as at 31 December 2016 has been recorded within other
reserves in X Ltd’s individual financial statements. At the date of acquisition Y Ltd had
retained earnings of Rs. 3,200,000.

© Emile Woolf International 5 The Institute of Chartered Accountants of Pakistan


Advanced accounting and financial reporting

2. It is the group policy to value non-controlling interest at fair value at the date of acquisition.
The fair value of the non-controlling interest at 1 May 2016 was Rs. 1,600,000.
3. As at 1 May 2016 the fair value of the net assets acquired was the same as the book value
with the following exceptions:
The fair value of property, plant and equipment was Rs. 800,000 higher than the book
value. These assets were assessed to have an estimated useful life of 16 years from the
date of acquisition. A full year’s depreciation is charged in the year of acquisition and none
in the year of sale.
The fair value of inventories was estimated to be Rs. 200,000 higher than the book value.
All of these inventories were sold by 31 December 2016.
On acquisition X Ltd identified an intangible asset that Y Ltd developed internally but which
met the recognition criteria of IAS 38 Intangible Assets. This intangible asset is expected
to generate economic benefit from the date of acquisition until 31 December 2017 and was
valued at Rs. 150,000 at the date of acquisition.
A contingent liability, which had a fair value of Rs. 210,000 at the date of acquisition, had a
fair value of Rs. 84,000 at 31 December 2016.
4. An impairment review was conducted at 31 December 2016 and it was decided that the
goodwill on the acquisition of Y Ltd was impaired by 20%.
5. X Ltd sold goods to Y Ltd for Rs. 300,000. Half of these goods remained in inventories at
31 December 2016. X Ltd makes 20% margin on all sales.
6. No dividends were paid by either entity in the year ended 31 December 2016.
Required
Prepare the consolidated statement of financial position as at 31 December 2016 for the X Ltd
Group.
1.6. KHAN LIMITED
On January 1, 2010, Khan Limited (KL) acquired 375 million ordinary shares and 40 million
preference shares in Gul Limited (GL) whose general reserve and retained earnings on
the date of acquisition, stood at Rs. 200 million and Rs. 1,000 million respectively.
The following balances were extracted from the records of KL and its subsidiary on
December 31, 2016:
KL GL
Dr Cr Dr Cr
Rs. m Rs. m Rs. m Rs. m
Ordinary share capital (Rs. 10 each) - 6,800 5,000
12% Preference share capital (Rs. 10 each) - - - 1,000
General reserve - 1,750 - 500
Retained earnings - 2,000 - 1,200
Loan from KL at 15% rate of interest - - - 2,000
14% Term Finance Certificates (TFCs) (Rs. 100 - 2,250 - -
each)
Accounts payable - 445 - 190
Dividend payable – preference shares - - - 60
Dividend payable – ordinary shares - 750 - 300
Property, plant and equipment - at cost 16,250 - 25,000 -
Property, plant and equipment - acc. - 9,750 - 17,000
depreciation
Investment in ordinary shares of GL 5,500 - - -

© Emile Woolf International 6 The Institute of Chartered Accountants of Pakistan


Questions

KL GL
Dr Cr Dr Cr
Rs. m Rs. m Rs. m Rs. m
Investment in preference shares of GL 400 - - -
Loan to GL at 15% rate of interest 2,000 - - -
Investment in KL's TFCs - - 1,500 -
(purchased at par value)
Profit before tax, interest and dividend - 2,865 - 1,550
Dividend income - 273 - -
Interest income - 300 - 210
Dividend receivable 249 - - -
Current assets 1,069 - 1,316 -
Interest on TFCs 315 - - -
Interest on loan from KL - - 300 -
Taxation 650 - 474 -
Preference dividend - - 120 -
Ordinary dividend – interim 750 - 300 -
27,183 27,183 29,010 29,010

Following relevant information is available:


(i) At the date of acquisition, the fair value of buildings, included in property, plant and
equipment of GL was assessed at Rs. 1,000 million above its carrying value. All other
identifiable assets and liabilities were considered to be fairly valued. GL provides for
depreciation on buildings at 10% per annum on the straight line basis.
(ii) GL purchased the TFCs in KL on January 1, 2016.
(iii) The non-controlling interests are measured at their proportionate share of the GL’s
identifiable net assets.
(iv) There is no impairment in the value of goodwill since its acquisition.
(v) There are no components of other comprehensive income.
Required
Prepare the following in accordance with the requirements of International Financial Reporting
Standards:
(a) Consolidated statement of financial position as at December 31, 2016.
(b) Consolidated statement of comprehensive income for the year ended December 31,
2016.
(c) Consolidated statement of retained earnings for the year ended December 31, 2016.
Note:
Ignore deferred tax and corresponding figures.
Notes to the above statements are not required. However, show workings wherever it is
necessary.
1.7. WHITE LIMITED
White Limited (WL) has investments in Green Limited (GL) and Yellow Limited (YL). YL is
registered and operates in a foreign country and its functional currency is T$. Following
information has been extracted from financial statements of the three companies for the year
ended 31 December 2016:

© Emile Woolf International 7 The Institute of Chartered Accountants of Pakistan


Advanced accounting and financial reporting

WL GL YL
Assets ------ Rs. in million ------ T$ in million
Property, plant and equipment 14,900 3,000 325
Investment property - 800 -
Investment in GL – at cost 4,200 - -
Investment in YL – at cost 1,500 5,400 -
Current assets 6,660 2,500 305
27,260 11,700 630
Equity & liabilities
Share capital (Rs./T$ 10 each) 11,400 1,500 225
Retained earnings 9,500 7,900 210
Current liabilities 6,360 2,300 195
27,260 11,700 630
Interim dividend paid on 30 June 2016 - - 10%

Other information:
(i) Details of investments made by WL and GL are as follows:
No. of Retained
Cost of
shares earnings at the
Investment date Investor Investee investment
acquired acquisition date
--------------- in million-------------------
1 Jan 2015 WL GL Rs. 4,200 135 Rs. 3,500
1 Jan 2016 WL YL T$ 75 4.5 T$ 50
1 Apr 2016 GL YL T$ 270 18 T$ 90
Fair values of each share of YL as on 1 January 2016 and 1 April 2016 were T$ 18 and T$ 23
respectively.
(ii) In the books of WL and GL, there is no movement in investment in YL since the date of
acquisition except the difference arising due to foreign currency translation at year end.
(iii) Investment property in GL was purchased on 1 January 2016 at a cost of Rs. 650 million
and rented to WL at an annual rent of Rs. 60 million on the same date. The property has a
useful life of 20 years. Both companies follow a policy of measuring their investment property
at fair value and property, plant and equipment at revalued amounts. Both companies also
charge depreciation on straight line method.
(iv) The relevant exchange rates per T$ are as follows:

Average rate
1-Jan-16 1-Apr-16 30-Jun-16 31-Dec-16
(1 Apr to 31 Dec)
Rs. 16 Rs. 17 Rs. 18.5 Rs. 20 Rs. 18

(v) WL values the non-controlling interest at its proportionate share of the subsidiaries’ net
identifiable assets.
Required:
Prepare WL’s consolidated statement of financial position as on 31 December 2016 in accordance
with the requirements of International Financial Reporting Standards. (Ignore taxation)

© Emile Woolf International 8 The Institute of Chartered Accountants of Pakistan


Questions

CHAPTER 2 - BUSINESS COMBINATIONS ACHIEVED IN STAGES


2.1. STEP ACQUISITION
On 1 January Year 1, H purchased 25% of the equity of AS for Rs. 80 million. H then acquired an
additional 40% of the equity of AS for Rs. 160 million on 30 June Year 1. At this date it was
estimated that the fair value of the original 25% shareholding in S was Rs. 95 million.
During the year S did not issue any new shares or make any distribution to its shareholders.
The carrying value of the net assets of AS were as follows:
Rs. million
At 1 January Year 1 260
At 30 June Year 1 300
H measures non-controlling interest at acquisition at fair value. This was estimated to be Rs.
120m.
The financial year of H ends on 30 June.
Required
For the consolidated financial statements of H for the year to 30 June Year 1, state:
(i) the total gain or profit attributable to the investment in AS for the year
(ii) total amount of goodwill arising with the acquisition
(iii) the amount of goodwill attributable to the NCI.
2.2. A LTD
The statements of financial position for A Ltd and B Ltd as at 30 September 2016 are provided
below:
A Ltd B Ltd
Rs.000 Rs.000
ASSETS
Non-current assets
Property, plant and equipment 22,000 5,000
Available for sale investment (note 1) 4,000 -
Current assets 26,000 5,000
Inventories 6,200 800
Receivables 6,600 1,900
Cash and cash equivalents 1,200 300
14,000 3,000
Total assets 40,000 8,000
EQUITY AND LIABILITIES Equity
Share capital (Rs. 1 equity shares) 20,000 1,000
Retained earnings 7,500 5,000
Other components of equity 500
Total equity 28,000 6,000
Non-current liabilities
5% Bonds 2019 (note 2) 3,900 -
Current liabilities 8,100 2,000
Total liabilities 12,000 2,000
Total equity and liabilities 40,000 8,000

© Emile Woolf International 9 The Institute of Chartered Accountants of Pakistan


Advanced accounting and financial reporting

Additional information:
1. A Ltd acquired a 15% investment in B Ltd on 1 May 2010 for Rs. 600,000. The investment
was classified as available for sale and the gains earned on it have been recorded within
other reserves in A Ltd’s individual financial statements. The fair value of the 15%
investment at 1 April 2016 was Rs. 800,000.
On 1 April 2016, A Ltd acquired an additional 60% of the equity share capital of B Ltd at a
cost of Rs. 2,900,000. In its own financial statements, A Ltd has kept its investment in B
Ltd as an available for sale asset recorded at its fair value of Rs. 4,000,000 as at 30
September 2016.
2. A Ltd issued 4 million Rs. 1 5% redeemable bonds on 1 October 2011 at par. The
associated costs of issue were Rs. 100,000 and the net proceeds of Rs. 3.9 million have
been recorded within non-current liabilities. The bonds are redeemable at Rs. 4.5 million
on 30 September 2019 and the effective interest rate associated with them is
approximately 8.5%. The interest on the bonds is payable annually in arrears and the
amount due has been paid in the year to 30 September 2016 and charged to the
statement of profit or loss.
3. An impairment review was conducted at the year end and it was decided that the goodwill
on the acquisition of B Ltd was impaired by 10%.
4. It is the group policy to value non-controlling interest at fair value at the date of acquisition.
The fair value of the non-controlling interest at 1 April 2016 was Rs. 1.25 million.
5. The profit for the year of B Ltd was Rs. 3 million, and profits are assumed to accrue evenly
throughout the year.
6. B Ltd sold goods to A Ltd for Rs. 400,000. Half of these goods remained in inventories at
30 September 2016. B Ltd makes 20% margin on all sales.
7. No dividends were paid by either entity in the year to 30 September 2016.
Required
(a) Explain how the investment in B Ltd should be accounted for in the consolidated financial
statements of A Ltd, following the acquisition of the additional 60% shareholding.
(b) Prepare the consolidated statement of financial position as at 30 September 2016 for the A
Ltd Group.
2.3. X LTD GROUP
Extracts from the financial statements of X Ltd, Y Ltd and Z Ltd are presented below.
Statements of comprehensive income for the year X Ltd Y Ltd Z Ltd
ended 31 December 2016
Rs.000 Rs.000 Rs.000

Revenue 1,200 290 150


Cost of sales (810) (110) (80)
Gross profit 390 180 70
Operating expenses (100) (40) (20)
290 140 50
Investment income 50 -
Finance costs (45) (10) (5)
Profit before tax 295 130 45
Income tax expense (80) (30) (15)
Profit for the year 215 100 30

© Emile Woolf International 10 The Institute of Chartered Accountants of Pakistan


Questions

Statements of comprehensive income for the year X Ltd Y Ltd Z Ltd


ended 31 December 2016
Rs.000 Rs.000 Rs.000
Other comprehensive income:
Revaluation of property, net of tax 60 20 10
Other comprehensive income for the year, net of tax 60 20 10
Total comprehensive income 275 120 40

Statements of changes in equity for the year X Ltd Y Ltd Z Ltd


ended 31 December 2016
Rs.000 Rs.000 Rs.000

Equity at 1 January 2016 1,700 840 500


Total comprehensive income for the year 275 120 40
Dividends (100) (50) -
Equity at 31 December 2016 1,875 910 540
Additional information
1 X Ltd acquired 80% of the ordinary share capital of Y Ltd for Rs. 620,000 on 1 January
2010 when the retained reserves of Y Ltd were Rs. 420,000. Y Ltd has 200,000 Rs. 1
ordinary shares in issue and there have been no share issues since the acquisition date.
The group policy is to measure the non- controlling interest at fair value at the date of
acquisition. The fair value of the non-controlling interest at 1 January 2010 was Rs.
180,000.
2 On 1 January 2010, the fair value of Y Ltd’s net assets was the same as their book value
with the exception of depreciable property, the fair value of which was Rs. 60,000 higher
than its book value. The property had a remaining useful life of 15 years at the date of
acquisition. Depreciation on property is charged to cost of sales.
3 Goodwill on the acquisition of Y Ltd was impaired for the first time by 25% in the year to 31
December 2015. An impairment review conducted at 31 December 2016 indicated a
further impairment of 10% of the remaining carrying value of goodwill. Impairment losses
on goodwill are charged to group operating expenses.
4 X Ltd acquired 40% of the ordinary share capital of Z Ltd on 1 July 2011, when the equity
was Rs. 435,000.
Required
(a) Prepare for the X Ltd Group for the year ended 31 December 2016:
(i) a consolidated statement of profit or loss and other comprehensive income; and
(ii) a consolidated statement of changes in equity.
X Ltd purchased a further 10% of the ordinary share capital of Y Ltd on 1 January 2017 for
Rs. 120,000.
(b) (i) Explain how the acquisition of this additional investment will be accounted for in the
consolidated financial statements of the X Ltd group for the year to 31 December
2017.
(ii) Calculate the debit or credit that will be made to the consolidated retained reserves
of the X Ltd group for the year to 31 December 2017 in respect of this additional
10% share purchase.
X Ltd purchased a further 20% of the ordinary share capital of Z Ltd on 1 January 2017.
(c) Explain how the acquisition of the additional investment in Z Ltd will be accounted for in
the consolidated financial statements of the X Ltd group for the year to 31 December 2017.

© Emile Woolf International 11 The Institute of Chartered Accountants of Pakistan


Advanced accounting and financial reporting

2.4. PLAIN LTD


The following statements of financial position are as at 31 March 2016:
Plain Stripes Spots
Rs. m Rs. m Rs. m
Assets
Tangible non-current assets 1,280 440 280
Investment in Stripes 413
Investment in Spots 60
Held to maturity investment 54
Current assets 477 190 130
Total assets 2,284 630 410
Equity and liabilities
Share capital of Rs. 1 800 240 200
Share premium 150 20 30
Revaluation reserve 90
Retained earnings 390 210 94
Total equity 1,430 470 324
Non-current liabilities 640 30 16
Current liabilities 214 130 70
Total equity and liabilities 2,284 630 410

Plain acquired the following shareholdings in Stripes and Spots.


Holding Fair value of Purchase
Date of acquisition
acquired net assets consideration
Rs. m Rs. m

Stripes 1 April 2013 30% 325 120


1 April 2015 50% 460 260

Spots 1 April 2015 25% 200 60


You are also provided with the following information which will be relevant to the consolidated
financial statements of Plain.
(1) None of the companies have issued any additional share capital since 1 April 2013.
(2) The financial statements of Plain have not yet been adjusted for the gain or loss arising on
gaining control of Stripes.
(3) At 1 April 2013, the carrying value of the net assets of Stripes was the same as their fair
value, Rs. 325 million.
(4) Plain wishes to use the full fair value method of accounting for the acquisition of Stripes, and
at 1 April 2015 the estimated value of goodwill attributable to non-controlling interests was
Rs. 3 million. The estimated fair value of the initial investment in 30% of the shares of Stripes
was Rs. 150 million at 31 March 2015.
(5) Included in the tangible non-current assets of Stripes is land valued at cost which on 1 April
2015 had a fair value of Rs. 25 million in excess of its carrying value. There has been no
subsequent significant change in that value.
(6) At 1 April 2014 the fair value of Spots’s land was Rs. 16 million in excess of its carrying
value. There has been no subsequent significant change in that value.

© Emile Woolf International 12 The Institute of Chartered Accountants of Pakistan


Questions

(7) Goodwill arising on acquisition is tested for impairment at each year end. At 31 March 2016
an impairment loss of Rs. 15 million was recognised for Stripes .
(8) There has been no impairment of the investment in Spots.
(9) During the year the directors of Plain decided to form a defined benefit pension scheme for
its employees. The company contributed cash to it of Rs. 250 million but the only accounting
entry for this has been to include it in receivables at 31 March 2016.
At 31 March 2016 the following details relate to the pension scheme:
Rs. m
Present value of obligation 317
Fair value of plan assets 302
Current service cost 276
Interest cost on pension scheme liabilities 41
Expected return on pension scheme assets 26
In the consolidated financial statements the directors wish to recognise any actuarial gain or
loss immediately.
(10) The ‘held to maturity’ investment in Plain’s financial statements is a zero coupon loan to an
unrelated third party. No interest has yet been recognised on this amount. The debt is
repayable in five years at Rs. 74 million. (Recognise interest on a straight line basis).
Required
Prepare the consolidated statement of financial position of the Plain group as at 31 March 2016.
2.5. MANGO LTD
The draft statements of financial position of Mango Ltd and its subsidiary at 30 November 2016
are as follows:

Mango Ltd Plum Ltd


Rs. m Rs. m
Assets:
Non-current assets
Property, plant and equipment 3,295 2,000
Investments in subsidiary 1,675
4,970 2,000
Current assets 1,685 861
Total assets 6,655 2,861
Equity and liabilities:
Share capital 850 1,020
Retained earnings 3,340 980
Other components of equity 250 80
Total equity 4,440 2,080
Non-current liabilities 1,895 675
Current liabilities 320 106
Total liabilities 2,215 781
Total equity and liabilities 6,655 2,861

© Emile Woolf International 13 The Institute of Chartered Accountants of Pakistan


Advanced accounting and financial reporting

The following information is relevant to the preparation of the group financial statements:
1. On 1 December 2013, Mango Ltd acquired 30% of the ordinary shares of Plum Ltd for a
cash consideration of Rs. 600 million. The fair value of Plum Ltd’s identifiable net assets
was Rs. 1,840 million at this date. The 30% holding gave Mango Ltd significant influence
over Plum Ltd and Mango Ltd accounted for the investment as an associate up to 1
December 2015. Mango Ltd’s share of Plum Ltd’s undistributed profit amounted to Rs. 90
million and its share of a revaluation gain amounted to Rs. 10 million.
On 1 December 2015, Mango Ltd acquired a further 40% of the ordinary shares of Plum
Ltd for a cash consideration of Rs. 975 million and gained control of the company. The
cash consideration has been added to the equity accounted balance for Plum Ltd at 1
December 2015 to give the carrying amount at 30 November 2016.
At 1 December 2015, the fair value of the equity interest in Plum Ltd held by Mango Ltd
before the business combination was Rs. 705 million.
At 1 December 2015, the fair value of Plum Ltd’s identifiable net assets was Rs. 2,250
million.
The retained earnings and other components of equity of Plum Ltd at 1 December 2015
were Rs. 900 million and Rs. 70 million respectively. It is group policy to measure the non-
controlling interest at fair value. The fair value of the non-controlling interest of 30% was
assessed as Rs. 620 million
2. At the time of the business combination with Plum Ltd, Mango Ltd has included in the fair
value of Plum Ltd’s identifiable net assets, an unrecognised contingent liability of Rs. 6
million in respect of a warranty claim in progress against Plum Ltd. In March 2016, there
was a revision of the estimate of the liability to Rs. 5 million. The amount has met the
criteria to be recognised as a provision in current liabilities in the financial statements of
Plum Ltd and the revision of the estimate is deemed to be a measurement period
adjustment.
3. The fair value of Plum Ltd’s identifiable net assets (Rs. 2,250 million) included an amount
of Rs. 200 million being the estimate of the fair value of buildings with the remainder
relating to non-depreciable land.
Mango Ltd had commissioned an independent valuation of the buildings of Plum Ltd which
was not complete at 1 December 2015 and therefore not considered in the fair value of the
identifiable net assets at the acquisition date. The valuations were received on 1 April 2016
and resulted in a decrease of Rs. 40 million in the fair value of property, plant and
equipment at the date of acquisition. The buildings have a remaining useful life of 20 years
at 1 December 2015. Buildings are depreciated on the straight-line basis and it is group
policy to leave revaluation gains on disposal in equity.
The decrease in the fair value of the buildings does not affect the fair value of the non-
controlling interest at acquisition and has not been entered into the financial statements of
Plum Ltd.
All goodwill arising on acquisitions has been impairment tested with no impairment being
required.
Required
Prepare the group consolidated statement of financial position of Mango Ltd as at 30
November 2016.

© Emile Woolf International 14 The Institute of Chartered Accountants of Pakistan


Questions

CHAPTER 3 - CONSOLIDATED STATEMENTS OF PROFIT OR LOSS AND OTHER


COMPREHENSIVE INCOME
3.1. MILLARD LTD
The profit and loss account of Millard Ltd and its subsidiary Fillmore Limited for the year ended
31 December 2016 are as follows:
Millard Fillmore
Ltd Ltd
Rs.’000 Rs.’000
Revenue 312,500 125.000
Cost of Sales (125,000) (50,000)
Gross Profit 187,500 75,000
Distribution Cost (25,000) (10,000)
Administrative Expenses (20,000) (8,000)
Operating Profit 142,500 57,000
Investment Income 7,950 -
Debenture Interest (47,500) (15,000)
Profit on ordinary activities before taxation 102,950 42,000
Taxation on ordinary activities (35,000) (17,500)
Profit on ordinary activities after taxation 67,950 24,500
Dividends:
Preference (13,750) (4,375)
Ordinary (20,000) (5,250)
Retained Profits 34,200 14,875
Retained Profits: 1/1/2016 66,750 19,500
Retained Profits: 31/12/2016 100,950 34,375
Additional information:
(1) Included in the revenue of Fillmore Limited is Rs. 12.5million in respect of sales to Millard
Ltd, giving Fillmore Limited a profit of 25% on cost. These are sales of components that
Fillmore Limited has been supplying to Millard Ltd on a regular basis for a number of
years. The amounts included in the inventories of Millard Ltd in respect of goods
purchased from Fillmore Limited at the beginning and end of the year were as follows:
Inventories of components in Millard Ltd’s books
Date Rs.’000
31/12/2016 2,000
31/12/2015 1,500
(2) Some years ago, Millard Ltd bought 50 million ordinary shares in Fillmore Limited at a cost
of Rs. 67million. On the same date, Millard Ltd bought 25% of the debentures of Fillmore
Limited at par.
At the date of Millard Ltd’s investment in Fillmore Limited, the statement of financial
position of Fillmore limited showed:
Rs.’000
Ordinary share capital 62,500
Preference share capital 43,750
Profit and loss account 12,500
118,750

© Emile Woolf International 15 The Institute of Chartered Accountants of Pakistan


Advanced accounting and financial reporting

The goodwill acquired by Millard Ltd in Fillmore Limited had been written off fully in December
2016 as a result of impairment losses.
Required
Prepare the consolidated profit and loss account of Millard Ltd for the year. Assume that
investment income is dealt with by Millard Ltd on an accrual basis.
3.2. SHERLOCK LTD
The following draft financial statements relate to Sherlock Ltd and its subsidiaries.
Draft statements of profit or loss and other comprehensive income for the year ended 31 December
2016.

Sherlock Mycroft Katie


Ltd Ltd Ltd
Rs. m Rs. m Rs. m
Revenue 400 115 70
Cost of sales (312) (65) (36)
Gross profit 88 50 34
Other income 21 7 2
Administrative costs (15) (9) (12)
Other expenses (35) (19) (8)
Operating profit 59 29 16
Finance costs (5) (6) (4)
Finance income 6 5 8
Profit before tax 60 28 20
Income tax expense (19) (9) (5)
Profit for the year 41 19 15
Other comprehensive income – revaluation
surplus 10
Total comprehensive income for year 51 19 15

The following information is relevant to the preparation of the group statement of profit or loss and other
comprehensive income:
1. On 1 January 2015, Sherlock Ltd acquired 60% of the equity interests of Mycroft Ltd. The
purchase consideration comprised cash of Rs. 80 million.
The fair value of the identifiable net assets acquired was Rs. 110 million at that date. The
excess of the fair value of the identifiable net assets at acquisition is due to non-
depreciable land.
Sherlock Ltd measures the non-controlling interest at acquisition at its fair value. The fair
value of the non-controlling interest (NCI) in Mycroft Ltd was Rs. 45 million on 1 January
2015.
Goodwill has been impairment tested annually and as at 31 December 2016 had reduced
in value by 20%. At 31 December 2016, goodwill was estimated to have a value of Rs. 2
million above its original value.
2. Sherlock Ltd acquired 60% of Katie Ltd on 30 June 2016. There has been no impairment
of goodwill since the date of acquisition.
3. Sherlock Ltd sold inventory to Mycroft Ltd for Rs. 12 million making a loss of Rs. 2 million
on the transaction. The sale was at fair value and Mycroft Ltd still holds half of the
inventory at the year end.

© Emile Woolf International 16 The Institute of Chartered Accountants of Pakistan


Questions

4. The following information relates to Sherlock Ltd’s pension scheme:


Rs. m
Plan assets at 1 January 2016 48
Defined benefit obligation at 1 January 2016 50
Service cost for year ended 31 December 2016 4
Discount rate at 1 January 2016 10%
Re-measurement loss in year ended 31 December 2016 2
Past service cost 1 January 2016 3
The pension costs have not yet been accounted for.
5. On 1 January 2015, Sherlock Ltd purchased plant for Rs. 12 million and this is being
depreciated using the straight line basis over 10 years with a zero residual value.
Sherlock Ltd measures plant of this type using the revaluation model. At 31 December
2015, the asset was revalued to Rs. 13 million but at 31 December 2016, the value of the
asset had fallen to Rs. 7 million. The effect of the revaluation at 31 December 2016 had
not yet been accounted for but depreciation for the year has been charged.
6. On 1 January 2015, Sherlock Ltd made an award of 8,000 share Katie Ltds to each of its
seven directors. The condition attached to the award is that the directors must remain
employed by Sherlock Ltd for three years.
The fair value of each Katie Ltd at the grant date was Rs. 100 and the fair value of each
Katie Ltd at 31 December 2016 was Rs. 110. At 31 December 2015, it was estimated that
three directors would leave before the end of four years.
The estimate of directors who were going to leave was revised to one director at 31
December 2016.
The share Katie Ltd expense for the year has not been included in profit or loss for the
current year and no directors had left by 31 December 2016.
7. A loss on an effective cash flow hedge of Mycroft Ltd of Rs. 3 million has been included in
the subsidiary’s finance costs.
8. Any expense adjustments should be made in other expenses.
Required
Prepare a consolidated statement of profit or loss and other comprehensive income for the year
ended 31 December 2016 for the Sherlock Ltd Group. (Ignore the deferred tax consequences of
the above events)
3.3. FAISAL LIMITED
Following is the summarised trial balance of Faisal Limited (FL) and its subsidiaries, Saqib Limited
(SL) and Ayaz Industries Limited (AIL) for the year ended December 31, 2016:
FL SL AIL
Rs. in million

Cash and bank balances 4,920 660 2,700


Accounts receivable 6,240 2,460 6,580
Closing inventory 14,460 4,200 5,680
Investment in SL (at cost) 9,000 - -
Investment in AIL (at cost) 10,500 - -
Other investments 11,100 - -
Property, plant and equipment 22,500 3,480 5,940
Dividend paid 600
Cost of sales 49,200 18,000 21,000
Operating expenses 3,600 2,100 5,400
131,520 30,900 47,900

© Emile Woolf International 17 The Institute of Chartered Accountants of Pakistan


Advanced accounting and financial reporting

FL SL AIL
Rs. in million
Accumulated depreciation 5,760 420 1,260
Ordinary share capital (Rs. 10 each) 30,000 12,000 6,000
Retained earnings – opening 33,780 - 5,400
Sales 57,600 16,500 33,800
Accounts payable 2,760 1,980 1,440
Gain on sale of non-current assets 540 - -
Dividend income 1,080 - -
131,520 30,900 47,900
Following additional information is available:
(i) SL was incorporated on February 1, 2016. 75% of the shares were acquired by FL at par
value on the same date.
(ii) FL acquired 80% of AIL on January 1, 2016
(iii) The following inter-company sales were made during the year 2016:
Included in Amount
Sales buyer’s closing receivable/payable Gross profit %
inventories at year enzd on sales
Rs. in million
FL to AIL 2,400 900 - 20
SL to AIL 1,800 600 800 10
AIL to FL 3,600 1,200 - 30
(iv) The gain on sale of non-current assets includes a sale of an item of property, plant and
machinery by FL to SL. This transaction occurred on July 1, 2016. SL. Details of the
transaction are as follows:
Rs. in million
Sales value 144
Less: Cost of plant and machineries 150
Accumulated depreciation (60)
Carrying amount at date of sale 90
Gain on sale of plant 54
The plant and machinery was purchased originally by FL on July 1, 2014, and was being
depreciated on the straight line method over a period of five years. SL computed
depreciation thereon using the same method based on the remaining useful life as at the
date of the transfer.
(v) FL billed Rs. 100 million to each subsidiary for management services provided during the
year 2016 and credited it to operating expenses. The invoices were paid on December 15,
2016.
(iv) Details of cash dividend are as follows:
Dividend
Date of declaration Date of payment %
FL Nov 25, 2016 Jan 5, 2017 20
AIL Oct 15, 2016 Nov 20, 2016 10
Required
Prepare the consolidated statement of financial position and the consolidated statement
of profit and loss of FL and its subsidiaries for the year ended December 31, 2016. Ignore
tax and corresponding figures.

© Emile Woolf International 18 The Institute of Chartered Accountants of Pakistan


Questions

3.4. GOLDEN LIMITED


Golden Limited (GL) is a listed company and has held shares in two companies, Yellow
Limited (YL) and Black Limited (BL), since July 1, 2014. The details of acquisition of shares in
these companies are as follows:
(a) GL acquired 18 million shares in YL at par, when YL’s reserves were Rs. 24 million.
The acquisition was made by issuing four shares in GL for every five shares in YL. The
market price of GL’s shares at July 1, 2014 was Rs. 20 per share. A fair value exercise
was carried out for YL’s assets and liabilities at the time of its acquisition with the following
results:

Book Value Fair Value


Rupees in million
Land 170 192
Machines 25 45
Investments 3 6

The remaining life of machine on acquisition was 5 years. The fair values of the assets
have not been accounted for in YL’s financial statements.
(b) 6 million shares in BL were acquired for Rs. 12 per share in cash. At the date of
acquisition, the reserves of BL stood at Rs. 40 million.
The summarized statements of profit or loss of the three companies for the year ended
June 30, 2016 are as follows:

GL YL BL
Rupees in million
Sales 875 350 200
Cost of sales (567) (206) (244)
Gross profit / (loss) 308 144 (44)
Selling expenses (33) (11) (15)
Administrative expenses (63) (40) (16)
Interest expenses (30) (22) (15)
Other income 65 - -
Profit/(loss) before tax 247 71 (90)
Income tax (73) (15) 8
Profit/(loss) for the period 174 56 (82)

The following relevant information is available:


(i) The share capital and reserves as at July 1, 2015 were as follows:

GL YL BL
Rupees in million
Ordinary share capital of Rs. 10 each 600 200 150
Reserves 652 213 108

The share capitals of all companies have remained unchanged since their incorporation.
(ii) During the year, GL sold goods amounting to Rs. 40 million to YL. The sales were made
at a mark-up of 25% on cost. 30% of these goods were still in the inventories of YL at
June 30, 2016.

© Emile Woolf International 19 The Institute of Chartered Accountants of Pakistan


Advanced accounting and financial reporting

(iii) GL manufactures a component used by BL. During the year, GL sold these components
amounting to Rs. 20 million to BL. Transfers are made at cost plus 15%. BL held Rs.
11.5 million of these components in inventories at June 30, 2016.
(iv) All assets are depreciated on straight line method.
(v) Other income includes dividend received from YL on April 15, 2016.
(vi) During the year, YL paid 20% cash dividend to its ordinary shareholders.
(vii) An impairment test was carried out on June 30, 2016 for the goodwill of YL and
investments in BL, appearing in the consolidated financial statements. The test indicated
that:
- goodwill of YL was impaired by 20%;
- due to recent losses, the fair value of investment in BL has been reduced to
Rs. 40 million.
No such impairment was required in previous years.
Required
Prepare, in a format suitable for inclusion in the annual report, a consolidated statement of
profit or loss for the year ended June 30, 2016.

© Emile Woolf International 20 The Institute of Chartered Accountants of Pakistan


Questions

CHAPTER 4 - COMPLEX GROUPS


4.1. PARVEZ LTD
Statements of profit or loss for Parvez Ltd, Saad Ltd and Vazir Ltd for the year ended 31
December 2016 are as follows:

Parvez Ltd Saad Ltd Vazir Ltd


Rs. 000 Rs. 000 Rs. 000
Revenues 45,600 24,700 22,800
Cost of sales (18,050) (5,463) (5,320)
Gross profit 27,550 19,237 17,480
Distribution costs (3,325) (2,137) (1,900)
Administrative expenses (3,475) (950) (1,900)
Operating profit 20,750 16,150 13,680
Interest paid (325)
Profit before tax 20,425 16,150 13,680
Tax (8,300) (5,390) (4,241)
Profit after tax 12,125 10,760 9,439

Statements of financial position as at 31 December 2016 are as follows:

Parvez Ltd Saad Ltd Vazir Ltd


Rs. 000 Rs. 000 Rs. 000
Property, plant and equipment 35,483 24,273 13,063
Investments
Shares in Saad Ltd 6,650
Shares in Vazir Ltd 3,800
Current assets 1,568 9,025 8,883
43,701 37,098 21,946
Equity and liabilities
Ordinary Rs. 1 shares 8,000 3,000 2,000
Retained earnings 22,638 24,075 19,898
30,638 27,075 21,898
Current liabilities 13,063 10,023 48
43,701 37,098 21,946

The following information is available relating to Parvez Ltd, Saad Ltd and Vazir Ltd:
(1) On 1 January 2010 Parvez Ltd acquired 2,700,000 Rs. 1 ordinary shares in Saad Ltd for
Rs. 6,650,000 at which date there was a credit balance of retained earnings of Saad Ltd of
Rs. 1,425,000. No shares have been issued by Saad Ltd since Parvez Ltd acquired its
interest.
(2) On 1 January 2010 Saad Ltd acquired 1,600,000 Rs. 1 ordinary shares in Vazir Ltd for Rs.
3,800,000 at which date there was a credit balance of retained earnings of Vazir Ltd of Rs.
950,000. No shares have been issued by Vazir Ltd since Saad Ltd acquired its interest.

© Emile Woolf International 21 The Institute of Chartered Accountants of Pakistan


Advanced accounting and financial reporting

(3) During 2016, Vazir Ltd had made inter-company sales to Saad Ltd of Rs. 480,000 making
a profit of 25% on cost and Rs. 75,000 of these goods were in inventory at 31 December
2016.
(4) During 2016, Saad Ltd had made inter-company sales to Parvez Ltd of Rs. 260,000
1
making a profit of 33 3 % on cost and Rs. 60,000 of these goods were in inventory at 31
December 2016.
(5) On 1 November 2016 Parvez Ltd sold warehouse equipment to Saad Ltd for Rs. 240,000
from inventory. Saad Ltd has included this equipment in its non-current assets. The
equipment had been purchased on credit by Parvez Ltd for Rs. 200,000 in October 2016
and this amount is included in its current liabilities as at 31 December 2016.
(6) Saad Ltd charges depreciation on its warehouse equipment at 20% on cost. It is company
policy to charge a full year’s depreciation in the year of acquisition to be included in the
cost of sales.
Required
(a) Prepare a consolidated statement of profit or loss for the Parvez Ltd Group for the year
ended 31 December 2016.
(b) Prepare statement of financial position as at that date.
4.2. HASAN, RIAZ AND SIDDIQ
The summarised balances extracted from the accounting records of Hasan (H) Ltd, Riaz (R) Ltd
and Siddiq (S) Ltd at 31 December 2016 are given below:

H Ltd R Ltd S Ltd

Rs. Rs. Rs.

Property, plant and equipment 1,102,500 271,950 122,550

Investments at cost

75% holding in shares of Riaz Ltd 367,500

40% holding in shares of Siddiq Ltd 49,000

20% holding in shares of Siddiq Ltd 24,500

Inventories 526,610 163,290 85,700

Receivables 241,920 129,680 29,750

Cash and bank balances 88,200 4,725 8,105

2,375,730 594,145 246,105

Share capital 1,750,000 420,000 175,000

Other reserves 350,000 70,000

Accumulated profits/(losses) 180,250 17,500 (17,500)

Payables 95,480 86,645 88,605

2,375,730 594,145 246,105

© Emile Woolf International 22 The Institute of Chartered Accountants of Pakistan


Questions

Further information:
(1) Hasan Ltd purchased its interest in Riaz Ltd and Siddiq Ltd in December 2013 at which
date Siddiq Ltd had accumulated losses of Rs. 35,000, and Riaz Ltd had accumulated
profits of Rs. 35,000.
(2) On 30 December 2016 Hasan Ltd despatched and invoiced goods for Rs. 12,500 to Riaz
Ltd which were not recorded by the latter until 3 January 2017. A mark-up of 25% is
added by Hasan Ltd to arrive at selling price. Riaz Ltd already had goods in inventories
which had been invoiced to them by Hasan Ltd at Rs. 10,400.
(3) Siddiq Ltd had accumulated losses of Rs. 52,500 when Riaz Ltd purchased 35,000 shares
in 2012.
(4) Hasan Ltd received a remittance of Rs. 8,000 on 2 January 2017 which had been sent by
Riaz Ltd on 29 December 2016.
(5) Included in Hasan’s receivables was a balance of Rs. 25,500 owed by Riaz Ltd.
(6) Neither Riaz Ltd nor Siddiq Ltd had any other reserves when their shares were purchased
by Hasan Ltd and Riaz Ltd.
(7) Payables of Riaz Ltd included an amount of Rs. 5,000 due to Hasan Ltd.
Required
Prepare the consolidated statement of financial position of Hasan Ltd and its subsidiaries at 31
December 2016.

4.3. LALIWALA GROUP


On 1 January 2016, Laliwala Group (LG) acquired 100% holding in PA Limited (PAL) at a
consideration of Rs. 2,000 million in cash plus replacement awards as discussed in (iv) below.
LG also paid Rs. 20 million to its bankers and lawyers in connection with the deal.
The fair value of the assets and liabilities of PAL together with their carrying values and tax base
on acquisition date are given below.
Fair value Carrying value Tax base
-------- Rs. in million --------
Property, plant and equipment 1,532 1,259 887
Investments 490 367 290
Deferred tax asset - net 24 24 N/A
Current assets 1,572 1,572 1,572
Total assets 3,618 3,222 2,749

Long term debt 634 634 634


Retirement benefit obligations 60 17 -
Current liabilities 1,194 1,194 1,194
Total liabilities 1,888 1,845 1,828

Other information relating to acquisition of PAL:


(i) LG recognized development cost previously incurred by PAL as an intangible asset at its fair
value of Rs. 153 million. PAL had charged off these costs in 2014.
(ii) A contingent liability of Rs. 39 million is appearing in the financial statements of PAL. LG’s
legal adviser had estimated that PAL is likely to pay Rs. 25 million to settle the claim.
(iii) PAL had assessed brought forward losses of Rs. 940 million. It is estimated that PAL would
be able to utilise losses of Rs. 500 million only. LG expects that losses of Rs. 300 million can
be used against future taxable profits of the group.

© Emile Woolf International 23 The Institute of Chartered Accountants of Pakistan


Advanced accounting and financial reporting

(iv) PAL had outstanding employee share options with a market based measure of Rs. 140
million. The share options were fully vested. As part of the business combination, PAL’s
outstanding share options were replaced by share options of LG with a market based
measure of Rs. 140 million and an intrinsic value of Rs. 90 million. The replacement awards
are fully vested.
On 31 December 2016 intrinsic value of replacement awards has increased to Rs. 150
million. According to the tax law, intrinsic value of the option on the exercise date is an
admissible expense.
Additional information:
(i) During the year, the following inter-company transactions took place between LG and PAL:

Included in buyer’s
Sales Profit % on
closing inventories
sales
-------- Rs. in million --------
LG to PAL 520 80 20%
PAL to LG 790 140 15%

(ii) On 1 January 2016, LG acquired 30% shareholdings in NA Limited (NAL) at a cash


consideration of Rs. 165 million. During 2016, NAL reported net income of Rs. 50 million out
of which it distributed dividend of Rs. 20 million.
LG plans to dispose of 40% of its total shareholdings in NAL by 30 June 2017.
(iii) Applicable tax rates for LG and PAL are 25% and 35% respectively. Both companies are
subject to tax on dividend income and capital gain at 12.5% and 17.5% respectively.
Required:
Determine the amounts of goodwill and deferred tax to be recognised in the consolidated
financial statements of LG for the year ended 31 December 2016, as a result of the above
transactions.
4.4. SHAKIR LIMITED
The draft statements of financial position of Shakir Limited (SL), Mashkoor Limited (ML) and
Baqir Limited (BL) as at 30 June 2017 are as follows:
Particulars SL ML BL
Assets: ----------- Rs. in million -----------
Property, plant & equipment 16,500 5,600 11,000
Investment in ML – at cost 1,375 - -
Investment in BL – at cost 7,500 - -
Investment in joint operation – at cost 620
Stock-in-trade 2,414 1,460 1,750
Trade and other receivables 2,200 2,060 1,800
Cash and bank 1,600 800 1,900
31,589 9,920 17,070
Equity and liabilities:
Share capital (Rs. 10 per share) 20,000 2,200 10,000
Share premium 1,000 900 -
Retained earnings 6,189 3,200 6,000
Trade and other payables 4,400 3,620 1,070
31,589 9,920 17,070

© Emile Woolf International 24 The Institute of Chartered Accountants of Pakistan


Questions

(i) On 1 July 2014 SL acquired 80% shares of ML when ML’s retained earnings were Rs. 1,400
million, at a cash consideration of Rs. 4,400 million. On acquisition date, fair value of net
assets was equal to their carrying value. 20% of the goodwill has been impaired till 30 June
2016.
(ii) Following information in respect of ML is available for the year ended 30 June 2017:
On 1 July 2016 SL disposed of 20% holding in ML (leaving 60% with SL) for Rs. 1,188
million when ML’s share price was Rs. 26 per share.
On 30 June 2017 SL further disposed of 35% holding in ML (leaving 25% with SL) for Rs.
2,926 million when ML’s share price was Rs. 36 per share.
On both disposals, SL credited investment in ML with related cost and took the difference to
profit or loss account.
ML made a net profit of Rs. 700 million during the year. No dividend was declared during the
year.
SL’s receivables include Rs. 200 million due from ML.
(iii) On 1 July 2015 SL acquired 60% holding in BL which resulted in bargain purchase of Rs. 180
million. On acquisition date, fair value of BL’s net assets was equal to their carrying value
except a building whose fair value was Rs. 200 million higher than its carrying value. Its
remaining life at the date of acquisition was 16 years.
(iv) SL’s closing stock includes goods sold by BL at 20% margin. These were invoiced at Rs. 50
million but are included in SL’s stock at NRV of Rs. 44 million.
(v) BL has 40% share in a joint operation, a power generation unit. The following information
relates to activities of the joint operation for the year ended 30 June 2017:
The unit was constructed at a cost of Rs. 1,550 million and commenced its operation from 1
July 2016. It has a useful life of 10 years.
Revenue from generation of electricity was Rs. 1,100 million. Power generation cost and
operating expenses paid amounted to Rs. 670 million and Rs. 130 million respectively.
All revenues and expenses of the operation have been settled during the year. However,
entries in respect of revenues/costs have not been made in the books of BL because they
have been received/paid by the other joint operator. SL and the other joint operator have
agreed to settle the outstanding balance after year end.
(vi) SL follows a policy of valuing the non-controlling interest at its proportionate share of the fair
value of the subsidiary’s identifiable net assets.
(vii) No further shares have been issued by ML and BL since their acquisition by SL.
Required:
Prepare sl’s consolidated statement of financial position as on 30 june 2017 in accordance With
the international financial reporting standards
4.5. ANT, BEE AND FLY
The draft statements of financial position of Ant Limited (AL), Bee Limited (BL) and Fly Limited
(FL) as at 31 December 2017 are as follows:
AL BL FL
------------ Rs. in million ------------
Assets
Property, plant and equipment 3,510 2,835 2,200
Investment property 130 45 -
Investment in BL at cost 3,540 - -
Investment in FL at cost - 2,400 -
Current assets 2,120 1,420 2,800
Total assets 9,300 6,700 5,000

© Emile Woolf International 25 The Institute of Chartered Accountants of Pakistan


Advanced accounting and financial reporting

Equity and liabilities


Share capital (Rs. 10 each) 5,500 4,000 2,500
Retained earnings 2,000 1,314 1,000
Gratuity 25 - -
Current liabilities 1,775 1,386 1,500
Total equity and liabilities 9,300 6,700 5,000

Other information:
(i) Details of investments are as follows:
Cost of Retained earnings
Date of investment of investee
Investor % holding Investee
investment
------ Rs. in million ------
1-Jan-2015 AL 65% BL 3,100 520
1-Apr-2017 AL 10% BL 440 815
30-Jun-2017 BL 60% FL 2,400 1,150
(ii) On acquisition date of BL, fair value of its net assets was equal to their carrying value
except a plant whose fair value was Rs. 120 million whereas its carrying amount was Rs.
140 million. Value in use and remaining useful life of the plant were Rs. 150 million and 10
years respectively at that date.
(iii) At the date of acquisition of FL, fair value of its net assets recorded in the books was equal
to their carrying value. Further, a contingent liability of Rs. 70 million was disclosed in the
financial statements of FL. AL's legal adviser had at that time estimated that this claim
would be settled at Rs. 50 million. However, it was actually settled on 15 February 2018 at
Rs. 40 million. Date of authorisation of FL's financial statements was 10 February 2018
and the claim was disclosed as contingent liability in FL's financial statements.
(iv) On 1 July 2017 AL sold its office building having carrying value of Rs. 43 million to BL at its
fair value of Rs. 50 million. The building had a remaining useful life of 5 years on the date
of disposal. On the same date, BL rented out the building to Monkey Limited for one year.
AL group follows fair value model for investment property whereas BL uses cost model for
investment property. Fair value of the building on 31 December 2017 was Rs. 58 million.
(v) On 31 December 2017 FL’s recoverable amount was estimated at Rs. 3,700 million.
(vi) AL group follows a policy of valuing the non-controlling interest at its proportionate share of
the fair value of the subsidiary's identifiable net assets.
(vii) The following information relates to AL's gratuity scheme for the year ended 31
December 2017:
Rs. in million
Contribution paid 70
Benefits paid 55
Current service cost 85
Re-measurement gain 10
During the year, payments made by AL were charged to profit or loss account. No further
adjustments have been made.
Discount rate and fair value of plan assets at 1 January 2017 were 12% per annum and Rs.
320 million respectively.
Required:
Prepare AL's consolidated statement of financial position as on 31 December 2017 in
accordance with the requirements of IFRSs.

© Emile Woolf International 26 The Institute of Chartered Accountants of Pakistan


Questions

CHAPTER 5 - ASSOCIATES AND JOINT VENTURES


5.1. JOINT ARRANGEMENTS
(a) State and explain the TWO types of joint arrangement identified in IFRS 11
(b) A joint operator is expected to recognise and account for certain elements in relation to the
joint operations. State FIVE elements to be recognised.
(c) State TWO characteristics of a joint arrangement.
5.2. HELIUM
The draft statements of financial position as at 31 December 2016 of three companies are set out
below.
Helium Sulphur Arsenic
Rs.000 Rs.000 Rs.000
Assets
Non-current assets
Property, plant and equipment 400 100 160
Investments:
- shares in Sulphur (60%) 75 – –
- shares in Arsenic (30%) 30 – –

Current assets 445 160 80


—— —— ——
950 260 240
—— —— ——
Equity and liabilities
Share capital 100 30 60
Retained earnings 650 180 100
Non-current loans 200 50 80
—— —— ——
950 260 240
—— —— ——
The reserves of Sulphur and Arsenic when the investments were acquired were Rs. 70,000 and
Rs. 30,000 respectively
Required
Prepare the consolidated statement of financial position as at 31 December 2016.
5.3. HAMACHI LTD
Hamachi Ltd acquired 90% of Saba Ltd’s Rs. 1 ordinary shares on 1 April 2014 paying Rs. 3.00
per share. The balance on Saba Ltd’s retained earnings at this date was Rs. 800,000. On 1
October 2015, Hamachi Ltd acquired 30% of Anogo Ltd’s Rs. 1 ordinary shares for Rs. 3.50 per
share. The statements of financial position of the three companies at 31 March 2016 are shown
below:
Hamachi Ltd Saba Ltd Anogo Ltd
Rs.000 Rs.000 Rs.000 Rs.000 Rs.000 Rs.000
Non-current assets
Property, plant and 8,050 3,600 1,650
equipment
Investments 4,000 910 nil
12,050 4,510 1,650
Current assets
Inventory 830 340 250
Accounts receivable 520 290 350
Bank 240 nil 100
1,590 630 700
Total assets 13,640 5,140 2,350

© Emile Woolf International 27 The Institute of Chartered Accountants of Pakistan


Advanced accounting and financial reporting

Hamachi Ltd Saba Ltd Anogo Ltd


Rs.000 Rs.000 Rs.000 Rs.000 Rs.000 Rs.000
Equity and liabilities
Equity:
Ordinary shares of Rs. 1 5,000 1,200 600
each
Reserves:
Retained earnings b/f 6,000 1,400 800
Profit year to 31 March 2016 1,500 900 600
7,500 2,300 1,400
12,500 3,500 2,000
Non-current liabilities
10% Loan notes 500 240 nil
Current liabilities
Accounts payable 420 960 200
Taxation 220 250 150
Overdraft nil 190 nil
640 1,400 350
Total equity and liabilities 13,640 5,140 2,350
The following information is relevant
(i) Fair value adjustments
On 1 April 2014 Saba Ltd owned an investment property that had a fair value of Rs.
120,000 in excess of its carrying value (book value). The value of this property has not
changed since acquisition. This property is included within investments in the balance
sheet.
Just prior to its acquisition, Saba Ltd was successful in applying for a six-year licence to
dispose of hazardous waste. The licence was granted by the government at no cost,
however Hamachi Ltd estimated that the licence was worth Rs. 180,000 at the date of
acquisition.
(ii) In January 2016 Hamachi Ltd sold goods to Anogo Ltd for Rs. 65,000. These were
transferred at a mark-up of 30% on cost. Two thirds of these goods were still in the
inventory of Anogo Ltd at 31 March 2016.
(iii) To facilitate the consolidation procedures the group insists that all inter company current
account balances are settled prior to the year-end. However a cheque for Rs. 40,000 from
Saba Ltd to Hamachi Ltd was not received until early April 2016. Inter company balances
are included in accounts receivable and payable as appropriate.
(iv) Anogo Ltd is to be treated as an associated company of Hamachi Ltd.
(v) An impairment test at 31 March 2016 on the consolidated goodwill of Saba Ltd concluded
that it should be written down by Rs. 468,000. No other assets were impaired.
Required
(a) Prepare the consolidated statement of financial position of Hamachi Ltd as at 31 March
2016.
(b) Discuss the matters to consider in determining whether an investment in another company
constitutes associated company status.
5.4. HIDE
Hide holds 80% of the ordinary share capital of Seek (acquired on 1 February 2016) and 30% of
the ordinary share capital of Arrive (acquired on 1 July 2015).
Hide had no other investments.

© Emile Woolf International 28 The Institute of Chartered Accountants of Pakistan


Questions

The draft statements of profit or loss for the year ended 30 June 2016, are set out below.
Hide Seek Arrive
Rs.000 Rs.000 Rs.000
Revenue 12,614 6,160 8,640
Operating expenses (11,318) (5,524) (7,614)
Dividends receivable 150 – –
——— ——– ——–
1,446 636 1,026
Income tax (621) (275) (432)
——— ——– ——–
Profit after taxation 825 361 594
——— ——– ——–
Included in the inventory of Seek at 30 June 2016 was Rs. 50,000 for goods purchased from
Hide in May 2016 which the latter company had invoiced at cost plus 25%. These were the only
goods sold by Hide to Seek but it did make sales of Rs. 180,000 to Arrive during the year. None
of these goods remained in Arrive’s inventory at the year end.
Required
Prepare a consolidated statement of profit or loss for Hide for the year ended 30 June 2016.
5.5. HARK, SPARK AND ARK
Hark acquired the following non-current investments on 1 April 2015:
(1) 4 million equity shares in Spark, by means of an exchange of one share in Handel for
every one share in Spark, plus Rs. 6.05 million in cash. The professional fees associated
with the acquisition amounted to Rs. 1 million. The market price of shares in Hark at the
date of the acquisition was Rs. 9 per share. The market price of Spark shares just before
the acquisition was Rs. 7. The cash part of the consideration is deferred and will not be
paid until two years after the acquisition.
(2) 25% of the equity shares in Ark, at a cost of Rs. 6 per share. The money to make this
payment was obtained by issuing one million new shares in Hark at Rs. 9 per share.
None of these transactions has yet been recorded in the summary statements of financial
position that are shown below.
The summarised draft statements of financial position of the three companies at 31 March 2016
are as follows.
Statement of financial position Hark Spark Ark
Rs. million Rs. million Rs. million
Assets
Non-current assets
Property, plant and equipment 60.0 31.0 16.0
Other equity investments 0.8 nil nil
60.8 31.0 16.0
Current assets 18.2 8.0 9.0
Total assets 79.0 39.0 25.0
Equity and liabilities
Equity shares of Rs. 1 each 16.0 5.0 6.0
Share premium 2.0 4.0 4.0
Retained earnings: at 1 April 2015 36.0 16.0 8.0
- for year ended 31 March 2016 8.0 3.0 2.0
62.0 28.0 20.0
Non-current liabilities
6% loan notes 10.0 - -
7% loan notes - 6.0 3.0
Current liabilities 7.0 5.0 2.0
Total equity and liabilities 79.0 39.0 25.0

© Emile Woolf International 29 The Institute of Chartered Accountants of Pakistan


Advanced accounting and financial reporting

The following information is relevant:


(1) Hark has chosen to value the non-controlling interest in Spark using the fair value method
permitted by IFRS 3 (revised). The fair value of the non-controlling interests at the
acquisition date is estimated to be the market value of the shares before the acquisition.
(2) At the date of acquisition of Spark, the fair values of its assets were equal to their carrying
amounts.
(3) The cost of capital of Hark is 10% per year.
(4) During the year ended 31 March 2016, Spark sold goods to Hark for Rs. 3.6 million, at a
mark-up of 50% on cost. Hark had 75% of these goods in its inventory at 31 March 2016.
(5) There were no intra-group receivables and payables at 31 March 2016.
(6) On 1 April 2015, Hark sold a group of machines to Spark at their agreed fair value of Rs. 3
million. At the time of the sale, the carrying amount of the machines was Rs. 2 million. The
estimated remaining useful life of the plant at the date of the sale was four years. Plant and
machinery is depreciated to a residual value of nil using straight-line depreciation and at 1
April 2015 the machines had an estimated remaining life of five years.
(7) “Other equity investments” are included in the summary statement of financial position of
Hark at their fair value on 1April 2015. Their fair value at 31 March 2016 is Rs.0.65 million.
(8) Impairment tests were carried out on 31 March 2016. These show that there is no
impairment of the value of the investment in Ark or in the consolidated goodwill.
(9) No dividends were paid during the year by any of the three companies.
Required
Prepare the consolidated statement of financial position for Hark as at 31 March 2016.
5.6. P, S AND A
The statements of financial position of three entities P, S and A are shown below, as at 31
December Year 5. However, the statement of financial position of P records its investment in
Entity A incorrectly.
P S A
Rs. Rs. Rs.
Non-current assets
Property, plant and equipment 450,000 240,000 460,000
Investment in S at cost 320,000 - -
Investment in A at cost 140,000 - -
––––––––– ––––––––– –––––––––
910,000 240,000 460,000
Current assets
Inventory 70,000 90,000 70,000
Current account with P - 60,000 -
Current account with A 20,000 - -
Other current assets 110,000 130,000 40,000
––––––––– ––––––––– –––––––––
Total assets 1,110,000 520,000 570,000
––––––––– ––––––––– –––––––––
Equity and reserves
Equity shares of Rs. 1 100,000 200,000 100,000
Share premium 160,000 80,000 120,000
Accumulated profits 650,000 140,000 250,000
––––––––– ––––––––– –––––––––
910,000 420,000 470,000
Long-term liabilities 40,000 20,000 30,000

© Emile Woolf International 30 The Institute of Chartered Accountants of Pakistan


Questions

P S A
Rs. Rs. Rs.
Current liabilities
Current account with P - - 20,000
Current account with S 60,000 - -
Other current liabilities 100,000 80,000 50,000
––––––––– ––––––––– –––––––––
1,110,000 520,000 570,000
––––––––– ––––––––– –––––––––
Additional information
P bought 150,000 shares in S several years ago when the fair value of the net assets of S was
Rs. 340,000.
P bought 30,000 shares in A several years ago when A’s accumulated profits were Rs. 150,000.
There has been no change in the issued share capital or share premium of either S or A since P
acquired its shares in them.
There has been impairment of Rs. 20,000 in the goodwill relating to the investment in S, but no
impairment in the value of the investment in A.
At 31 December Year 5, A holds inventory purchased during the year from P which is valued at
Rs. 16,000 and P holds inventory purchased from S which is valued at Rs. 40,000. Sales from P
to A and from S to P are priced at a mark-up of one-third on cost.
None of the entities has paid a dividend during the year.
P uses the partial goodwill method to account for goodwill and no goodwill is attributed to the
non-controlling interests in S.
Required
Prepare the consolidated statement of financial position of the P group as at 31 December Year
5.
5.7. H LTD GROUP
The statements of comprehensive income for H Ltd, S Ltd and A Ltd for the year ended 31 May
2016 are shown below:

H Ltd S Ltd A Ltd


Rs.000 Rs.000 Rs.000
Revenue 6,000 3,000 1,000
Cost of sales (4,800) (2,400) (800)
Gross profit 1,200 600 200
Distribution costs (64) (32) (10)
Administrative expenses (336) (168) (52)
Finance costs (30) (15) (5)
Profit before tax 770 385 133
Income tax expense (204) (102) (33)
PROFIT FOR THE YEAR 566 283 100

© Emile Woolf International 31 The Institute of Chartered Accountants of Pakistan


Advanced accounting and financial reporting

H Ltd S Ltd A Ltd


Rs.000 Rs.000 Rs.000
Other comprehensive income:
Revaluation of property 200 100 30
Tax effect of revaluation (42) (21) (6)
Other comprehensive income for the year, net of tax 158 79 24
TOTAL COMPREHENSIVE INCOME FOR THE YEAR 724 362 124
Additional information:
1. H Ltd operates a defined benefit pension plan for its employees. At the year end, there is
an actuarial loss of Rs. 52,000 on the pension plan liabilities and an actuarial gain of Rs.
40,000 on pension plan assets. These amounts are not reflected in the above statements.
In accordance with the amendment to IAS 19 Employee Benefits, H Ltd recognises
actuarial gains and losses from the defined benefit plan in other comprehensive income in
the period that they occur.
2. H Ltd holds a 15% investment in XY which is designated as available for sale. The fair
value of this investment at 31 May 2016 was Rs. 106,000. The investment is currently
recorded in the financial statements at Rs. 92,000.
3. H Ltd owns 80% of the ordinary share capital of S Ltd and exercises control over its
operating and financial policies. H Ltd owns 30% of the ordinary share capital of A Ltd and
exerts significant influence over its operating and financial policies.
Required
Prepare the consolidated statement of profit or loss and other comprehensive income for the H
Ltd Group, taking account of the information provided in the notes above. Ignore any further
taxation effects of notes 1 and 2.
5.8. ALPHA AND BETA
On 1 July 2012 Alpha Limited (AL) and Beta Limited (BL) entered into an agreement to set up
two Separate Vehicles (SVs) to manufacture and distribute their products. Each company has
50% share in both SVs. The following are the extracts from draft statements of financial position
and comprehensive income of AL and the SVs for the year ended 30 June 2016.
Statements of financial position
AL SV-1 SV-2 AL SV-1 SV-2
Rs. in millon Rs. in millon
Property, plant and 2,650 750 365 Capital 2,000 400 200
equipment
Investment in SVs - at 443 - - Accumulated profit 1,193 55 305
cost
Stock in hand 695 250 140 10% bank loan 500 320 -
Other assets 570 180 80 Current liabilities 665 405 80
4,358 1,180 585 4,358 1,180 585
Statement of comprehensive income
AL SV-1 SV-2
Rs. in millon
Sales 4,250 650 1,000
Less: Cost of sales (2,993) (480) (750)
Gross profit 1,257 170 250
Less: Expenses (657) (145) (200)
Net profit 600 25 50

© Emile Woolf International 32 The Institute of Chartered Accountants of Pakistan


Questions

Additional information:
(i) SV-1 is classified as joint operation whereas SV-2 is classified as joint venture.
(ii) On 1 July 2015, AL acquired 60% of BL’s ownership in SV-1 at Rs. 140 million. AL also
incurred acquisition related costs amounting to Rs. 3 million which were capitalized.
(iii) The details of transactions made during the year 2016 between AL and the SVs and their
subsequent status are given below:

Included in Amount
Sales buyer’s closing receivable/(payable) Profit % on
inventories in the books of AL sales
-------------- Rs. in million --------------
AL to SV-1 350 220 320 10
AL to SV-2 250 110 70 20
SV-1 to AL 190 150 (150) 30
SV-2 to AL 60 38 (20) 15

(iv) AL follows the equity method for recording its investment in joint venture whereas investment
in joint operations is recorded in accordance with IFRS-11.
Required:
In accordance with the requirements of International Financial Reporting Standards, prepare AL’s
separate statements of financial position and comprehensive income for the year ended 30 June
2016.
5.9. SNAKE LIMITED
Draft consolidated financial statements of H Limited (HL) for the year ended 31 December 2017
show the following amounts:
Rs. in million
Total assets 2,500
Total liabilities 1,610
Total comprehensive income 659

During the process of finalisation, following matters have been noted:


On 31 December 2017 HL disposed of 2.2 million shares of Snake Limited (SL) for Rs. 290 million.
HL had acquired 3 million shares of SL at fair value on 1 January 2015. SL’s paid-up capital
consists of 10 million shares. Due to recurring losses made by SL, HL had booked impairment of
Rs. 90 million against this investment on 31 December 2016.
Fair value per share and retained earnings of SL at respective dates were as follows:
Fair value Retained earnings
Date
Rs. per share Rs. in million
1 January 2015 200 1,700
31 December 2016 118 1,200
31 December 2017 128 1,350

Disposal proceeds have been credited to profit or loss account. No other adjustment has been
made during the year ended 31 December 2017.
Required:
Determine the revised amounts of total assets, total liabilities and total comprehensive income after
incorporating impact of the above adjustments, if any.

© Emile Woolf International 33 The Institute of Chartered Accountants of Pakistan


Advanced accounting and financial reporting

CHAPTER 7 - IAS 21: FOREIGN CURRENCY


7.1. DND LIMITED
DND Limited is a listed company, having its operations within Pakistan. During the year ended
December 31, 2016, the company contracted to purchase plants and machineries from a US
Company. The terms and conditions thereof , are given below:
Total cost of contract = US$ 100,000.
Payment to be made in accordance with the following schedule:
Payment Dates Amount Payable
On signing the
1-Jul-16 US$ 20,000
contract
On shipment* 30-Sep-16 US$ 50,000
After installation
31-Jan-17 US$ 30,000
and test run
*(risk and rewards of ownership are transferred on
shipment)
The contract went through in accordance with the schedule and the company made all the
payments on time. The following exchange rates are available:
Dates Exchange Rates
1-Jul-16 US$ 1 = Rs. 103
30-Sep-16 US$ 1 = Rs. 103.5
31-Dec-16 US$ 1 = Rs. 104
31-Jan-17 US$ 1 = Rs. 104.5
Required
Prepare journals to show how the above contract should be accounted for under IAS 21.

7.2. STARLIGHT LIMITED


In December 2014, Lahore Holdings Ltd acquired an 80% interest in a Qatari investment
company, Starlight Limited. Starlight Limited has the Qatari Rial (QR) as its functional currency.
The acquisition cost was Rs. 2,500,000 and the revenue reserves balance of Starlight Limited
stood at QR 49,300 at the acquisition date.
The following financial information was extracted from the books for the year ended 31
December 2016.
Starlight Limited
Qatari Riyal
Turnover 344,880
Cost of sales (249,710)
Gross profit 95,170
Expenses (29,490)
Profit before tax 65,680
Taxation (17,325)
Profit after tax 48,355
Interim dividend (16,300)
Retained profit for the year 32,055

© Emile Woolf International 34 The Institute of Chartered Accountants of Pakistan


Questions

Extract of statement of financial position at 31 December, 2016

Qatari Riyal

Share capital 20,250

Revenue reserve 103,200

Liabilities 34,480

Note

Exchange rate (PKR to Qatari Riyal)

31-Dec-14 30

31-Dec-15 31

31-Dec-16 33

Average rate for 2016 32

Required
(a) Prepare the translated profit and loss account of Starlight Limited.
(b) Calculate the goodwill on consolidation and the non-controlling interest that would appear
in the consolidated statement of profit or loss.

7.3. PERCEPT LTD


Percept Ltd acquired 70% of the share capital of Trint on 1 January 2016, a company based in
Japan for 6,900,000 Yen. The retained earnings on this date was 4,500,000 Yen. The fair value
of the identifiable net assets of the company was 12,375,000 Yen and the net asset relates to
items of property plant and equipment. Similarly, the fair value of the non-controlling Interest
(NCI) in Trint Ltd as at 1 January, 2016 was 6,250,000 Yen. It is the policy of Percept Ltd to use
the “full goodwill method” in the preparation of the group’s financial statements. Trint Ltd’s profit
for the year ended 31 December 2016 was 2,000,000 Yen. The acquisition on 1 January 2016
was done in Japan when the following exchange rates were in force:

Yen to rupees

1/1/2016 0.9

31/12/2016 0.8

The average rate for the year ended 31 December 2016 was 0.85 Yen to Rs. 1.
Trint Ltd: Statement of Financial Position as at 31 December 2016.

Yen (000)
Assets:
Non-current assets 9,500
Financial assets 1,250

10,750
Current assets 8,250

Total assets 19,000

© Emile Woolf International 35 The Institute of Chartered Accountants of Pakistan


Advanced accounting and financial reporting

Equity and liabilities


Share capital 5,000
Retained earnings 7,500
12,500
Non-current liabilities 4,000
Current liabilities 2,500
6,500
Total equity and liabilities 19,000
Required
(a) Translate the statement of financial position of Trint Ltd. as at 31 December 2016
(b) Calculate the goodwill arising on acquisition of Trint and any gain/loss arising on
retranslation of the goodwill as at 31 December 2016
(c) Calculate the exchange difference arising from the translation of Trint Ltd’s net assets.

7.4. ORLANDO
Orlando is an entity whose functional currency is the US dollar. It prepares its financial
statements to 30 June each year. The following transactions take place on 21 May Year 4 when
the spot exchange rate was $1 = €0.8.
Goods were sold to Koln, a customer in Germany, for €96,000.
A specialised piece of machinery was bought from Frankfurt, a German supplier. The invoice for
the machinery is for €1,000,000.
The company receives €96,000 from Koln on 12 June Year 4.
At 31 June Year 4 it still owns the machinery purchased from Frankfurt. No depreciation has
been charged on the asset for the current period to 30 June Year 4.
The liability for the machine is settled on 31 July Year 4.
Relevant $/€ exchange rates are:
12 June Year 4 $1 = €0.9
30 June Year 4 $1 = €0.7
31 July Year 4 $1 = €0.8
Required
Show the effect on profit or loss of these transactions for:
(a) the year to 30 June Year 4
(b) the year to 30 June Year 5

7.5. MANCASTER AND STOCKPOT


Part A
Required
(1) Define and explain the following terms as used in IAS 21.
The effects of changes in foreign exchange rates.
(a) Functional currency
(b) Presentation currency
(2) Outline the factors to be considered when determining the functional currency of an
overseas subsidiary.

© Emile Woolf International 36 The Institute of Chartered Accountants of Pakistan


Questions

Part B
The statements of financial position of Manchester and its subsidiary Stockpot at 31 March Year
4 and their statement of profit or loss for year ended on that date are set out below:

Statements of financial position at 31 March Year 4


Mancaster Stockpot
$000 $000 €000 €000
Non-current assets:
Property, plant and equipment 20,000 30,000
Investments (notes 1 and 2) 5,500 -
25,500 30,000
Current assets:
Inventories 10,000 18,000
Trade receivables 10,000 15,000
20,000 33,000
Total assets 45,500 63,000
Capital and reserves:
Share capital ($1/€1 Shares) 9,000 15,000
Accumulated profits 12,500 10,000
21,500 25,000
Non-current liabilities (note 4) 10,000 20,000
Current liabilities
Trade payables 7,900 10,400
Bank overdraft 6,100 7,600
14,000 18,000
Total equity and liabilities 45,500 63,000

Statement of profit or loss – year ended 31 March Year 4


Mancaster Stockpot
$000 €000
Revenue 50,000 60,000
Cost of sales (notes 2 and 5) (25,000) (30,000)
Gross profit 25,000 30,000
Other operating expenses (15,000) (16,000)
Operating profit 10,000 14,000
Investment income (note 3) 1,500 -
Interest payable (1,000) (2,000)
Profit before tax 10,500 12,000
Tax (3,600) (4,200)
Profit after tax 6,900 7,800

© Emile Woolf International 37 The Institute of Chartered Accountants of Pakistan


Advanced accounting and financial reporting

Statement of changes in equity – year ended 31 March Year 4


Mancaster Stockpot
Share Reserves Share Reserves
capital capital
$000 $000 €000 €000
Balance at 1 April Year 3 9,000 9,500 15,000 6,600
Profit for the period 6,900 7,800
Dividends paid (3,900) (4,400)
Balance at 31 March Year 4 9,000 12,500 15,000 10,000

You are provided with the following additional information:


(1) Investments represent the acquisition of 11.25 million shares in Stockpot on 31 March Year
0. The retained profits of Stockpot on this date stood at €5 million. Any goodwill arising on
the acquisition is to be treated as a foreign currency asset. Stockpot operates as a
reasonably autonomous entity on a day-to-day basis although Mancaster does control the
long-term strategy of Stockpot.
(2) Exchange rates have been as follows:

Date € to $1
31 March Year 0 3.0
31 March Year 3 2.4
31 March Year 4 2.2
Average for Year 4 2.3

(3) Investment income represents dividends received from Stockpot.


(4) The non-current liabilities represent long-term borrowings.
Required
(a) Translate the statement of financial position of Stockpot into the presentation currency of
dollars and prepare the consolidated statement of financial position of the group at 31
March Year 4.
(b) Translate the statement of profit or loss of Stockpot into dollars and prepare the
consolidated statement of profit or loss of the group for the year ended 31 March Year 4.

7.6. A, B AND C
Extracts from the financial statements of A, its subsidiary, B and its associate, C for the year to 30
September 2016 are presented below:
Summarised statement of profit or loss and other A B C
comprehensive income
Rs.000 A$000 Rs.000
Revenue 4,600 2,200 1,600
Cost of sales and operating expenses (3,700) (1,600) (1,100)
Profit before tax 900 600 500
Income tax (200) (150) (100)
Profit for the year 700 450 400
Other comprehensive income:
Revaluation of property, plant and equipment 200 120 70
Total other comprehensive income 200 120 70
Total comprehensive income 900 570 470

© Emile Woolf International 38 The Institute of Chartered Accountants of Pakistan


Questions

Statement of financial position A B C


Rs.000 A$000 C$000
Assets
Non-current assets
Property, plant and equipment 7,000 4,000 2,000
Investment in B 5,200
Investment in C 900 _
13,100 4,000 2,000
Current assets 3,000 2,000 1,000
Total assets 16,100 6,000 3,000
Equity and liabilities
Share capital 2,000 1,000 1,000
Reserves 12,100 3,500 1,500
14,100 4,500 2,500
Current liabilities 2,000 1,500 500
Total equity and liabilities 16,100 6,000 3,000

Additional information:
1. The functional currency of both A and C is the Rs. and the functional currency of B is the
A$.
2. A acquired 80% of B on 1 October 2013 for Rs. 5,200,000 when the reserves of B were
A$1,800,000. The investment is held at cost in the individual financial statements of A.
3. A acquired 40% of C on 1 October 2011 for Rs. 900,000 when the reserves of C were Rs.
700,000. The investment is held at cost in the individual financial statements of A.
4. No impairment to either investment has occurred to date.
5. The group policy is to value the non-controlling interest at fair value at the date of
acquisition. The fair value of the non-controlling interest of B at 1 October 2013 was
A$600,000.
6. Relevant exchange rates are as follows:

1 October 2013 Rs./A$0.5000

30 September 2015 Rs./A$0.7100

30 September 2016 Rs./A$0.6300

Average rate for year ended 30 September 2016 Rs./A$0.6500

Required
Prepare the consolidated statement of profit or loss and other comprehensive income for the A
Group for the year ended 30 September 2016 and the consolidated statement of financial
position as at that date.

7.7. OMEGA LIMITED


Omega Limited (OL) is incorporated and listed in Pakistan. On 1 May 2012, it acquired 20,000
ordinary shares (2% shareholding) in Al-Wadi Limited (AWL), a Dubai based company at a cost
of AED 240,000 which was equivalent to Rs. 6,000,000. The face value of the shares is AED 10
each. OL intends to hold the shares to avail benefits of regular dividends and capital gains.

© Emile Woolf International 39 The Institute of Chartered Accountants of Pakistan


Advanced accounting and financial reporting

On 1 June 2013, AWL was acquired by Hilal Limited (HL), which issued three shares in HL in
exchange for every four shares held in AWL.
Other relevant information is as under:
AWL HL
Final dividend received on 31 March 2013:
Cash 15% -
Bonus shares 10% -
Final cash dividend received on 10 April 2014 - 20%
Fair value per share as at: 31 December 2012 AED 13.00 -
1 June 2013 AED 14.00 AED 18.00
31 December 2013 - AED 19.50
Exchange rates on various dates were as follows:
31-Dec-2012 31-Mar-2013 1-Jun-2013 31-Dec-2013 10-Apr-2014
1 AED Rs. 25.00 Rs. 26.50 Rs. 28.00 Rs. 28.70 Rs. 28.20
Required
Determine the amounts (duly classified under appropriate heads) that would be included in
OL’s statement of comprehensive income for the year ended 31 December 2013 in respect of the
above investment.

7.8. PARENT COMPANY LIMITED


Parent Company Limited (PCL) is a listed company and owns 80% and 75% equity in LS Limited
and FS Limited respectively. FS is registered and operates in a foreign country and its functional
currency is CU. Summarised statements of financial position as at 30 June 2014 and other
information relating to the group companies are as under:

PCL LS FS
Rs. in million CU in million
Assets
Property, plant and equipment 4,200 3,500 250
Investments in LS and FS 6,500 - -
Current assets 3,500 4,000 450
14,200 7,500 700
Equity and liabilities
Share capital (Rs. 10/CU 10
each 6,000 1,800 120
Retained earnings 3,500 900 280
Current liabilities 4,700 4,800
14,200 7,500 700
Profit after tax for the year ended
30 June 2014 700 400 30
Final dividend for the year ended
30 June 2013: 12% - 15%
Cash (paid on 1 January 2014) 10% 20% -
700 400 30

© Emile Woolf International 40 The Institute of Chartered Accountants of Pakistan


Questions

The following information is also available:

i.
At the acquisition date
No. of
Investment Fair value
Company shares Cost Retained
date of
acquired earnings *
NCI
---- in million ----
LS 1-Jan-2012 120 Rs. 2,000 Rs. 250 Rs. 540
FS 1-Jul-2012 9 CU 300 CU 160 CU 90

*NCI stands for Non-controlling interest


ii. On the date of acquisition, fair value of the net assets of LS and FS were equal to their
book value. However, a contingent liability of Rs. 25 million was disclosed in the financial
statements of LS. PCL's legal adviser had at that time estimated that LS would be liable
to pay Rs. 6 million to settle the claim and it was finally settled at the same amount in
May 2014.
iii. No further shares have been issued by LS and FS since their acquisitions, except for the
bonus issue as mentioned above.
iv. An impairment test carried out on 30 June 2014 revealed that goodwill of FS is impaired
by CU 10 million.
v. PCL values non-controlling interest on the date of acquisition at fair value.
vi. The exchange rates in terms of Rs. per CU, were as follows:

1-Jul-2012 30-Jun-2013 1-Jan-2014 30-Jun-2014 Average for 2013-14


Rs. 15.00 Rs. 16.80 Rs. 16.90 Rs. 17.30 Rs. 17.00

vii. The break-up of exchange reserve in the consolidated financial statements for the year
ended 30 June 2013 is as follows:

Relating to goodwill Rs. 148.50 million


Relating to translation of foreign operations Rs. 463.05 million

Required:
In accordance with the requirements of the International Financial Reporting Standards, prepare:
(a) Consolidated statement of financial position as at 30 June 2014; and
(b) Consolidated statement of other comprehensive income for the year ended 30 June
2014. (Ignore taxation)

7.9. KANGAROO LIMITED


Kangaroo Limited (KL), a Pakistan based company, is preparing its financial statements for the
year ended 31 December 2017. Following transactions were carried out during the year.
(i) Foreign currency transactions:
KL purchased an investment property in United States for USD 2.6 million. 10% advance
payment was made on 1 May 2017 and 70% payment was made on 1 July 2017 on
transfer of title and possession of the property. The remaining amount was paid on 1
August 2017.
On 1 September 2017, KL rented out this property at annual rent of USD 0.24 million for
one year and received full amount in advance on the same date.

© Emile Woolf International 41 The Institute of Chartered Accountants of Pakistan


Advanced accounting and financial reporting

KL uses fair value model for its investment property. On 31 December 2017, an
independent valuer determined that fair value of the property was USD 2.5 million.
Following spot exchange rates are available:
Date 1-May-2017 1-Jul-2017 1-Aug-2017 1-Sept-2017 31-Dec-2017
USD 1 Rs. 100 Rs. 105 Rs. 108 Rs. 110 Rs. 116

Following average exchange rates are also available:


Period 2017 Jul to Dec 2017 Sep to Dec 2017
USD 1 Rs. 105 Rs. 111 Rs. 113

(ii) Equity investments:


On 1 May 2017 KL acquired following equity investments:

Purchase price Transaction cost Total


-------------------- Rs. in million -- ------------------
Investment A 100 2 102
Investment B 150 3 153

Investment A was designated as measured at fair value through profit or loss whereas
investment B was irrevocably elected at initial recognition as measured at fair value through other
comprehensive income.
In October 2017, KL earned dividend of Rs. 12 million and Rs. 9 million on investment A and B
respectively.
20% of investment A and 30% of investment B were sold for Rs. 23 million and Rs. 50 million
respectively in November 2017. Transaction cost was paid at 2%.
As on 31 December 2017, fair values of the remaining investments are given below:

Fair value Transaction cost on disposal Net amount


------------- Rs. in million -------------
Investment A 105 2.1 102.9
Investment B 130 2.6 127.4

Required:
Prepare the extracts relevant to the above transactions from KL’s statements of financial position
and comprehensive income for the year ended 31 December 2017, in accordance with the
IFRSs. (Comparative figures and notes to the financial statements are not required)

© Emile Woolf International 42 The Institute of Chartered Accountants of Pakistan


Questions

CHAPTER 8 - IAS 7: STATEMENTS OF CASH FLOWS


8.1. EVERNEW LTD
The following relates to the financial statements of Evernew Ltd.
(a) Consolidated statement of profit or loss for the year ended 31 December 2016
Rs.000
Operating profit 144,000
Interest expenses (10,080)
Profit after interest 133,920
Profit from disposal of subsidiary 5,040
Profit before taxation 138,960
Taxation (46,800)
Profit after taxation 92,160

Profit for the year attributable to:


Owners of the parent 84,960
Non-controlling interest 7,200
Retained profit c/f 92,160

(b) Consolidated statement of financial position as at 31 December, 2016


2016 2015
Rs.’000 Rs.’000 Rs.’000 Rs.’000
Assets
Non-current assets 369,720 360,000
Current assets:
Inventory 180,000 165,600
Trade receivables 151,200 136,800
Cash in hand 63,360 14,400
394,560 316,800
Total assets 764,280 676,800
Equity and liabilities
Ordinary share capital 144,000 144,000
Accumulated profits 317,520 232,560
461,520 376,560
Non-controlling interest 36,360 41,400
10% debenture 68,400 90,000

Current liabilities
Trade creditors 108,000 93,600
Taxation 46,800 39,240
Bank overdraft 43,200 36,000
Net current assets 198,000 168,840
Total equity and liabilities 764,280 676,800
(c) The following additional information is relevant:
(i) Depreciation for the year in the consolidated profit and loss account was Rs.
72,720,000. Non-current assets were not disposed by the group except those made
during disposal of the investment in the shares of Pastit Limited.

© Emile Woolf International 43 The Institute of Chartered Accountants of Pakistan


Advanced accounting and financial reporting

(ii) Evernew Ltd sold its investment in Pastit Limited in July 2016. The entire 80%
shareholding in the subsidiary was sold for Rs. 39.6million. Information about the
disposal is as follows:
Rs.’000 Rs.’000
Inventories 14,400
Receivables 18,000
Non-current assets 28,800
Trade creditors (10,800)
Taxation (2,160)
Bank overdraft (1,440)
Debenture stock (3,600) (18,000)
43,200
Non-controlling interest (8,640)
34,560

The investment was acquired many years ago for Rs. 13.68million when the net assets of
Pastit Limited were Rs. 14.4million. Goodwill had been fully written off before due to
impairment.
Required
Prepare the Evernew Ltd group consolidated cash flow statement for the year ended 31
December, 2016.
8.2. BISHOP GROUP
You are provided with the information set out below relating to a group of companies.
Consolidated statement of profit or loss for Bishop Group for the year ended 31
December 20X2
20X2 20X1
Rs.000 Rs.000
Revenue 19,308 18,173
Cost of sales (4,315) (4,620)
––––––––– –––––––––
Gross profit 14,993 13,553
Distribution costs (6,439) (6,126)
Administrative expenses (5,705) (6,719)
––––––––– –––––––––
Profit before tax and finance costs (note 1) 2,849 708
Finance income 90 75
Finance costs (note 2) (350) (230)
––––––––– –––––––––
Profit before taxation 2,589 553
Income tax expense (note 3) (800) (125)
Profit for the year 1,789 428
––––––––– –––––––––
Attributable to:
Equity holders of the parent 1,369 318
Non-controlling interest 420 110
––––––––– –––––––––
1,789 428
––––––––– –––––––––

© Emile Woolf International 44 The Institute of Chartered Accountants of Pakistan


Questions

Summary statement of changes in equity for the year to 31 December 20X2


Foreign
Issued Share currency Retained
capital premium translation earnings Total NCI Equity
B/fwd 1 Jan 7,500 77 2,100 4,905 14,582 2,500 17,082
Issue of shares 3,500 324 3,824 3,824
Profit for year 1,369 1,369 420 1,789
Dividends (600) (600) (295) (895)
Exchange gain 700 700 175 875
–––––– –––––– –––––– –––––– –––––– –––––– ––––––
C/fwd 31 Dec 11,000 401 2,800 5,674 19,875 2,800 22,675
–––––– –––––– –––––– –––––– –––––– –––––– ––––––
Notes: NCI = non-controlling interest
Exchange gain = exchange gain on translation of subsidiary
Consolidated statement of financial position as at 31 December 20X2
20X2 20X1
Rs.000 Rs.000
Non-current assets
Tangible assets 11,720 7,520
Investments 3,000 2,700
Current assets
Inventories 6,135 5,740
Receivables 5,720 4,380
Cash at bank and in hand 820 169
27,395 20,509
Equity and liabilities
Ordinary share capital 11,000 7,500
Share premium 401 77
Foreign currency translation 2,800 2,100
Retained earnings 5,674 4,905
Equity attributable to owners of parent 19,875 14,582
Non-controlling interest 2,800 2,500
Total equity 22,675 17,082
Current liabilities
Payables 1,420 1,760
Tax 700 167
Obligations under finance leases 110 50
Non- current liabilities
Loans 1,200 800
Obligations under finance leases 740 250
Provisions for liabilities and charges
Deferred tax 550 400
27,395 20,509
Notes to the accounts
(1) Operating profit is stated after charging Rs.000 Rs.000
Depreciation: Owned assets 960 840
Assets held under finance leases 240 120
1,200 960

© Emile Woolf International 45 The Institute of Chartered Accountants of Pakistan


Advanced accounting and financial reporting

(2) Finance costs Rs.000 Rs.000


Loan interest 120 80
Finance charge on finance leases 205 132
Exchange rate losses on long-term loans 25 18
350 230

(3) Taxation on profits on ordinary activities Rs.000 Rs.000


Tax on income at 30% 600 90
Deferred tax 150 35
Under provision in respect of previous years 50 -
800 125

(4) Foreign exchange differences Rs.000 Rs.000


Gains arising on re-translation 700 400
The exchange rate gain relates to the translation of an 80% owned overseas
subsidiary, Louise, under the closing rate method. The gain comprises:
Non-current assets 424
Inventories 117
Receivables 339
Cash 53
Trade payables (58)
875
Attributable to NCI (175)
Attributable to owners of parent company 700
(5) During the year non-current assets additions of Rs. 700,000 were acquired under finance
leases. Payments on finance leases are made in arrears. The net book value of non-
current assets disposed of was Rs. 720,000, with sale proceeds of Rs. 810,000.

Required
(a) Prepare the group statement of cash flows of Bishop in accordance with IAS 7 together with
any required notes for the year ended 31 December 20X2.
(b) Explain why external users of financial statements benefit from receiving a statement of cash
flows.
8.3. THE GRAPE GROUP
The draft statements of financial position and statement of profit or loss of the Grape Group at 31
March Year 4 and 31 March Year 3 are as follows:
Notes Year 4 Year 3
Rs.000 Rs.000

Non-current assets
Intangible assets 24 -
Property, plant and equipment (1) 13,515 12,990
Investments – associated undertakings 1,966 1,920
15,505 14,910

© Emile Woolf International 46 The Institute of Chartered Accountants of Pakistan


Questions

Notes Year 4 Year 3


Rs.000 Rs.000
Current assets
Inventory 11,657 10,530
Receivables 7,209 6,936
Cash at bank and in hand 5,190 1,728
24,056 19,194
39,561 34,104
Capital and reserves
Share capital 8,100 7,425
Share premium 1,989 1,470
Accumulated profits 13,200 8,700
23,289 17,595
Non-current liabilities (2) 6,900 7,890
Current liabilities (3) 9,372 8,619
39,561 34,104

Statement of profit or loss: Year to 31 March Year 4 Year 3


Rs.000 Rs.000
Sales revenue 74,100 59,400
Cost of sales (54,138) (42,240)
Gross profit 19,962 17,160
Distribution costs (5,889) (4,869)
Administrative expenses (4,092) (3,384)
Operating profit 9,981 8,907
Income from interests in associates 139 144
Loss on sale of tangible non-current assets (18) -
Interest expense (552) (651)
Profit before tax 9,550 8,400
Income tax expense (2,950) (2,400)
Profit after tax 6,600 6,000

Notes
(1) Property, plant and equipment
Year 4 Year 3
Rs.000 Rs.000
Cost
At 1 April 20,598 19,416
Additions 1,875 2,022
Disposals (429) (840)
At 31 March 22,044 20,598
Depreciation
At 1 April 7,608 6,984
Charge for year 1,176 936
Disposals (255) (312)
At 31 March 8,529 7,608
Net book value 13,515 12,990

© Emile Woolf International 47 The Institute of Chartered Accountants of Pakistan


Advanced accounting and financial reporting

(2) Non-current liabilities


Year 4 Year 3
Rs.000 Rs.000
8% Loan notes 6,900 6,900
10% Unsecured loan notes - 990
6,900 7,890
(3) Current liabilities
Year 4 Year 3
Rs.000 Rs.000
Accounts payable 6,422 6,219
Income tax 2,950 2,400
9,372 8,619
You are provided with the following additional information:
Interest on the 8% loan note is payable half-yearly on 30 September and 31 March.
Income tax is payable nine months after the year-end.
Dividends of Rs. 2.1m and Rs. 2.4m were proposed for Year 3 and Year 4 respectively.
During the year, the Grape Group acquired 100% of the ordinary share capital of Pip. The
purchase was financed by Rs. 346,000 in cash and the issue of 54,000 ordinary shares of
Rs. 1 each. The ordinary shares had a market value of Rs. 152,000. The following figures
related to Pip at the date of acquisition:
Rs.000
Property, plant and equipment 315
Inventory 139
Receivables 85
Cash at bank and in hand 3
Payables (68)
474
Share capital 180
Reserves 294
474

(4) The share capital consists of Rs. 1 ordinary shares.


Required
Prepare a statement of cash flows and related notes for the Grape Group for the
year ended 31 March Year 4.

© Emile Woolf International 48 The Institute of Chartered Accountants of Pakistan


Questions

CHAPTER 9 - IFRS 9: FINANCIAL INSTRUMENTS: RECOGNITION AND


MEASUREMENT
9.1. AJI PANCA LTD
On 1 January Year 1 Aji Panca Ltd has the following capital and reserves.
Equity Rs.
Share capital (Rs. 1 ordinary shares) 1,000,000
Share premium 200,000
Retained earnings 5,670,300
––––––––––
6,870,300
––––––––––
During Year 1 the following transactions took place.
1 January An issue of Rs. 100,000 8% Rs. 1 redeemable preference shares at a
premium of 60%. Issue costs are Rs. 2,237. Redemption is at 100% premium
on 31 December Year 5. The effective rate of interest is 9.5%.
31 March An issue of 300,000 ordinary shares at a price of Rs. 1.30 per share. Issue
costs, net of tax benefit, were Rs. 20,000
30 June A 1 for 4 bonus issue of ordinary shares.
Profit for the year, before accounting for the above, was Rs. 508,500. The dividends on the
redeemable preference shares have been charged to retained earnings.
Required
Set out capital and reserves and liabilities resulting from the above on 31 December Year 1.
9.2. PASSILA LTD
On 1 July 2016, Passila Ltd, issued 20,000 8% debentures at a discount of 2.5% to
Rs. 97.50 each. The security is redeemable in five years’ time. The interest on the debentures is
payable bi-annually on 30 June and 31 December.
On 31 December 2016, the Company’s year-end date, the debentures were quoted on the
Karachi Stock Exchange for Rs. 96. The company accountant has suggested each of the
following as possible valuation basis for reporting the debentures liability on the statement of
financial position as at 31 December 2016:
(i) Face value of the debentures.
(ii) Face value of the debenture plus interest payment for five years.
(III) Market value on the statement of financial position as at the year end.
Required
(a) Determine the face value of the debentures and the proceeds accruing to the company.

(b) Determine the amount and explain the nature of the differences between the face value
and the market value of the debentures on 1 July, 2016.
(c) Distinguish between nominal and effective rate of interest.
(d) Determine the nominal interest payable on the debentures for the year ended 31
December 2016.
(e) State arguments for or against each of the suggested alternatives for reporting the
debentures liability on the statement of financial position as at 31 December 2016.

© Emile Woolf International 49 The Institute of Chartered Accountants of Pakistan


Advanced accounting and financial reporting

9.3. FINANCIAL INSTRUMENTS


(a) Explain the IFRS 9 rules on the recognition and measurement of financial assets and
financial liabilities.
(b) Explain how the following should be accounted for in accordance with IAS 32 and IFRS 9
in the financial statements to 31 December Year 3.
(i) A 3% bond was purchased on 1 January Year 3 for Rs. 250,000. The nominal value
is Rs. 300,000 and redemption will be at par on 31 December Year 6. The coupon is
received annually in arrears. The effective interest rate on the bond is 9.7%. The
company intends to hold the bond until its maturity. The market value of the bond at
31 December Year 3 is Rs. 275,000.
(ii) An investment was made in the equity shares of XYZ. 3,000 shares were purchased
(a 1% stake) at a cost of Rs. 10 per share on 1 April Year 1. A transaction fee of Rs.
300 was charged on the purchase. The entity intended to sell the shares within three
months but the market value of the investment continued to rise and so the
company decided not to sell in the near term. The market value of the shares over
the three years has been as follows:
Rs.000
31 December Year 1 32
31 December Year 2 34
31 December Year 3 35
(iii) The company issued a convertible bond at par on 31 December Year 3, raising Rs.
500,000. The coupon on the bond is 4%. The rate on an equivalent redeemable
bond is 7%. The bond can be redeemed at par on 31 December Year 6 or converted
into equity shares at a rate of five shares per Rs. 100. The bond has not been
classed as fair value through profit and loss.
9.4. CASCABEL LTD
Cascabel Ltd entered into a forward contract on 31 July 2016 to purchase A$.2 million at a
contracted rate of Rs. 1: A$0.64 on 31 October 2016. The contract cost was nil. Cascabel Ltd
prepares its financial statements to 31 August 2016. At 31 August 2016 an equivalent contract
for the purchase of A$2 million could be acquired at a rate of Rs. 1: A$0.70.
Required
(a) Explain how this financial instrument should be classified and prepare the journal entry
required for its measurement as at 31 August 2016.
(b) Assume now that the instrument described above was designated as a hedging instrument
in a cash flow hedge, and that the hedge was 100% effective.
Explain how the gain or loss on the instrument for the year ended 31 August 2016 should
now be recorded and why different treatment is necessary.
9.5. FAIR VALUE HEDGE ACCOUNTING
X Ltd holds an inventory of tin. At 30th September 2015 X Ltd decided to hedge this position with
a 6 month forward sale of 30 tonnes of the same grade of tin.
Relevant information
Tin inventory Forward
(Rs.) contract (Rs.)
Carrying amount of inventory at 30 Sept 2015, at the
1,000,000 N/A
lower of cost and NRV
Fair values at 30 Sept 2015 1,300,000 NIL
Fair values at 31 Dec 2015 1,200,000 95,000
Fair values at 31 March 2016, when the tin is sold and
1,150,000 142,000
the contract is closed

© Emile Woolf International 50 The Institute of Chartered Accountants of Pakistan


Questions

Required
(a) Prepare journals for the year ended 31 December 2015
(b) Prepare the journals that are necessary at 31 March 2016
9.6. CASH FLOW HEDGE ACCOUNTING
At 30th November 2015, the company decided to acquire a new drilling machine from a foreign
supplier. The machine is essential, and there is absolutely no likelihood that the purchase will be
delayed.
The price of the machine is A$400,000, payable upon delivery which is anticipated to be 28
February 2016.
Spot rate a 30th November 2015 is Rs.0.70 = A$1.
In order to hedge the exchange rate risk, the company enters into a forward foreign exchange
contract to buy A$400,000 forward 3 months, at a rate of 0.70 (Rs. = A$1).
At 31 December 2015, the forward rate in the market for 2 month delivery is 0.75.
The machine was duly delivered on 28 February, and the exchange rate ruling on that day of
payment was 0.80. The forward contract was closed out. It is the company’s accounting policy to
take any deferred gains/losses on a cash flow hedge of the acquisition of a non-financial asset,
against the cost of that asset (a basis adjustment).
For both situations, ignore the effect of time value of money and transaction costs.
Required
(a) Prepare journals for the year ended 31 December 2015
(b) Prepare the journals that are necessary at 28 February 2016)
(Note: A$ is a fictional currency used for the purposes of this example)
9.7. WATERS LTD
Waters Ltd acquired the following financial assets and liabilities in 2016.
1 On 1 September, Waters acquired 2,000 Rs. 100 nominal units of 7% treasury stock 2022
for Rs. 104.10 per unit. The gross redemption yield at the date of purchase was 6.30%.
Waters does not intend to hold the treasury stock until maturity, as the cash may be
required in the meantime. Interest is paid annually in arrears.
2 Waters buys and sells goods in Constantia, a country whose currency is the Prif (PR). On
3 December Waters enters into a futures contract to sell PR500,000 on 30 April 2017 at an
agreed price of PR1.98/Rs. 1. This contract is not part of a designated hedge. The cost of
entering into the contract was Rs. 750.
3 On 5 February Waters acquired 250,000 ordinary shares in Gilmour Ltd at Rs. 4.85 per
share incurring Rs. 35,000 attributable transaction costs.
4 On 1 July Waters sells goods to Mason for Rs. 500,000 on interest free credit payable 30
June 2017. The imputed rate of interest is 11%.
5 On 30 April Waters acquired 1,000 Rs. 100 nominal units of 8.5% treasury stock 2018 at
Rs. 107.10 per unit. The gross redemption yield is 5.9%. Waters intends to hold the
investment to maturity. Interest is paid annually in arrears.
6 On 26 December Waters purchased Rs. 25,000 of quoted company loan notes. This asset
has been designated as being held for short-term trading purposes.
7 On 24 December Waters sold 10,000 shares 'short' in Wright Ltd for Rs. 3.60 each, hoping
that the share price would fall so that it could clear its position by buying the shares in
January 2017 at a lower price.
On 31 December 2016, the values are as follows:
1 Rs. 100 nominal units of 7% treasury stock 2022 are trading at Rs. 98.07 per unit at 31
December 2016. The gross redemption yield at that date is 7.3%.

© Emile Woolf International 51 The Institute of Chartered Accountants of Pakistan


Advanced accounting and financial reporting

2 The futures rate for a Prif contract with a delivery date of 30 April 2017 is PR1.99/Rs. 1.
3 The shares in Gilmour are now trading at Rs. 5.20 – Rs. 5.25 per share, having an
average of Rs. 5.05 during the year. Disposal costs would be 2% of the sale proceeds.
4 Amounts receivable from Mason remain outstanding at the reporting date. The imputed
interest rate for current sales is 12%.
5 The 8.5% treasury stock 2018 is now trading at Rs. 101.50 per unit and the gross
redemption yield is currently quoted at 7.48%.
6 The loan notes are now worth Rs. 25,500 due to the market being more confident that the
interest will be paid in full and on time.
7 Shares in Wright Ltd are now trading at Rs. 3.30 each.
Required
Explain and calculate the impact of the above transactions on the financial statements of Waters
Ltd for the year ended 31 December 2016.
9.8. ARIF INDUSTRIES LIMITED
Arif Industries Limited (AIL) owns and operates a textile mill with spinning and weaving units.
Due to recurring losses, AIL disposed of the weaving unit for an amount of Rs. 100 million on
July 1, 2015 and invested the proceeds in Pakistan Investment Bonds (PIBs).
Details of investment in PIBs are as follows:
(i) The PIBs were purchased through a commercial bank at face value. The bank initially
charged premium and investment handling charges of Rs. 4,641,483. At the time of
purchase, AIL’s intention was to liquidate the investment after four years and utilize the
realized amount for expansion of its spinning business. The bank has agreed to
repurchase the PIBs on June 30, 2019, at their face value.
(ii) The markup on PIBs is 15% for the initial two years and 20% for the remaining two years.
The effective yield on investment at the time of purchase was 15.50%.
Required
(a) Prepare an amortisation table showing the amortised cost and interest income over the life of
the loan asset.
Following on from the facts in part (a), suppose that on June 30, 2017 AIL decided to defer the
expansion plan by one year. The bank agreed to extend the holding period accordingly and pay
20% interest in year 5 but reduced the repurchase price by 2%.
(b) Prepare an amortisation table showing the amortised cost and interest income over the life of
the loan after taking account of any necessary adjustment to the carrying amount of the loan
asset.
9.9. QASMI INVESTMENT LIMITED
On 1 January 2009 Qasmi Investment Limited (QIL) purchased 1 million 12% Term Finance
Certificates (TFCs) issued by Taj Super Stores (TSS), which operates a chain of five Super
Stores. The terms of the issue are as under:
The TFCs have a face value of Rs. 100 each and were issued at a discount of 5%. These are
redeemable at a premium of 20% after five years.
Interest on the TFCs is payable annually in arrears on 31 December each year.
Effective interest rate calculated on the above basis is 16.426% per annum.
Due to a property dispute, TSS had to temporarily discontinue operations of two stores in 2010.
Consequently, TSS was unable to pay interest due on 31 December 2010 and 31 December
2011.
At the time of finalization of accounts for the year ended 31 December 2010, QIL was quite
hopeful of recovery of the interest and therefore, no impairment was recorded. However, in 2011,
after a thorough review of the whole situation, QIL’s management concluded that it would be able

© Emile Woolf International 52 The Institute of Chartered Accountants of Pakistan


Questions

to recover the face value of the TFCs along with the premium on the due date i.e. 31 December
2013, but the interest for the years 2010 to 2013 would not be received. Accordingly, QIL
recorded impairment in the value of the TFCs on 31 December 2011.
In 2012, TSS reached an out of court settlement of the property dispute and the stores became
operational. Subsequently, QIL and TSS agreed upon a revised payment schedule according to
which the present value of the agreed future cash flows on 31 December 2012 is estimated at
Rs. 115 million.
Required
Prepare journal entries in the books of QIL for the years ended 31 December 2011 and 2012.
Show all the relevant computations.
9.10. RASHID INDUSTRIES LIMITED
On 15 October 2016, Rashid Industries Limited (RIL) made the following investments:

Percentage of *Cost of investment


Name of Investees No. of shares
shareholding acquired (Rs. in million)
Karim Limited (KL) 155,000 4% 20
Bashir Limited (BL) 135,000 2% 65
* including transaction cost
Investment in KL was made with no intention to sell the shares while investment in BL was made
with the intention to sell the shares before 31 December 2016.
The board of directors in its meeting held on 30 November 2016 decided that since the future
prospects of BL are quite attractive, its shares should be held till 30 June 2018. The market rate
on 30 November 2016 was Rs. 621.
On 31 December 2016, RIL decided to record an impairment loss of Rs. 5 million against
investment in KL. The market price of shares of KL and BL as on 31 December 2016 was Rs. 80
and Rs. 600 respectively.
RIL’s broker normally charges transaction costs of 0.2%.
Required:
Explain the accounting treatment of above transactions in accordance with International Financial
Reporting Standards.
9.11. LAHORE STEEL LIMITED
Lahore Steel Limited (LSL) issued 1 million six-year debentures on 1 January 2015 at par value
of Rs. 100 each at a fixed rate of 6% per annum. Interest payable at the end of each year
whereas the principal is to be repaid in two equal instalments at the end of 2019 and 2020.
Debentures were issued with an option to convert 10 debentures into 4 ordinary shares of LSL till
the date of first principal redemption. The liability was not designated as measured at fair value
through profit or loss on initial recognition.
The market interest rate for non-convertible debentures issued by entities having similar credit
risk and loan tenor is 1-Year KIBOR + 2% per annum.
On 1 January 2016 LSL repurchased 100,000 debentures at a premium of Rs. 5 per debenture.
Transaction cost of Rs. 2 per debenture was incurred on this redemption.
The market interest rates and market values of LSL’s shares are given below:

Date 1-Year KIBOR Market value per share (Rs.)


1 January 2015 5% 200
1 January 2016 6% 250
Required:
Prepare journal entries in the books of LSL for the year ended 31 December 2016.

© Emile Woolf International 53 The Institute of Chartered Accountants of Pakistan


Advanced accounting and financial reporting

CHAPTER 10 - FINANCIAL INSTRUMENTS: PRESENTATION AND DISCLOSURE


10.1. SERRANO LTD
On 1 October 2015 Serrano Ltd issued Rs. 10 million 6% convertible loan stock on the following
terms:
The issue price was at par.
The loan stock is convertible into the company’s equity shares at the option of the stockholders
four years after the date of its issue (30 September 2019) on the basis of 20 shares for each Rs.
100 of loan stock. Alternatively it will be redeemed at par.
Ancho Services had advised that if Serrano Ltd had issued similar loan stock without the
conversion rights, then it would have had to pay an interest (coupon) rate of 10% on the loan
stock. This is because the terms of conversion to equity shares are favourable.
Ancho Services further advised that because it is almost certain that the loan stock holders will
exercise their right to convert to equity shares, the loan stock has the substance of equity and
can be included as such on the statement of financial position. This has the added advantage of
improving/reducing the company’s gearing (debt/equity) in comparison to what would be the case
with the issue of ‘straight’ loan stock.
The present value of Rs. 1 receivable at the end of each year, based on discount rates of 6% and
10% can be taken as:
Required
In relation to the 6% convertible loan stock, calculate the finance cost to be shown in the
statement of profit or loss and the extracts from the statement of financial position for the year to
30 September 2016; and comment on Ancho Services’ advice.
10.2. POBLANO LTD
Poblano Ltd issued Rs. 10 million of 4% convertible loan notes on 1 October 2015, on which
interest is paid annually in arrears on 30 September. The loan notes are convertible into equity
shares of Poblano Ltd on 30 September 2018 at the rate of 20 shares in Poblano Ltd for every
Rs. 100 of notes. Alternatively the notes can be redeemed on that date for cash at par, at the
option of the note holder.
If Poblano Ltd had issued straight loan notes, redeemable at par after 3 years, it would have had
to pay interest at the rate of 7% in order to persuade investors to subscribe for them.
Required
Show how the convertible loan notes would be accounted for in the financial statements of
Poblano Ltd for the year to 30 September 2016.
10.3. PIQUIN LTD
(a) Piquin Ltd issued 10 million 5% convertible Rs. 1 bonds 2021 on 1 January 2016. The
proceeds of Rs. 10 million were credited to non-current liabilities and debited to bank. The
5% interest paid has been charged to finance costs in the year to 31 December 2016.
The market rate of interest for a similar bond with a five year term but no conversion terms
is 7%.
Required
Explain AND demonstrate how this convertible instrument would be initially measured in
accordance with IAS 32 Financial Instruments: Presentation AND subsequently measured
in accordance with IAS 39 Financial Instruments: Recognition and Measurement in the
financial statements for the year ended 31 December 2016.
(b) The directors of Piquin Ltd want to avoid increasing the gearing of the entity. They plan to
issue 5 million 6% cumulative redeemable Rs. 1 preference shares in 2017.
Required
Explain how the preference shares would be classified in accordance with IAS 32 Financial
Instruments: Presentation, AND the impact that this issue will have on the gearing of
Piquin Ltd.

© Emile Woolf International 54 The Institute of Chartered Accountants of Pakistan


Questions

10.4. AJI LTD


Aji Ltd issued Rs. 6 million 7% convertible bonds on 1 January 2016 at par. The bonds are
redeemable at par on 31 December 2019 or convertible at that date on the basis of two Rs. 1
ordinary shares for every nominal Rs. 10 of bonds.
At the date of issue the prevailing market rate for similar debt without conversion rights was 9%.
The interest due was paid on 31 December 2016 and recorded within finance costs during the
year.
Required
(a) Explain how convertible instruments are initially recognised, in accordance with IAS 32
Financial Instruments: Presentation, and prepare the journal entry to record the issue of
the bonds by Aji Ltd.
(b) (i) Explain how the bonds will be subsequently measured, in accordance with IFRS 9
Financial Instruments: Recognition and Measurement, and prepare the journal entry
to record the subsequent measurement of the bonds in the financial statements of
Aji Ltd for the year to 31 December 2016.
(ii) Prepare extracts that illustrate how the bonds will be presented in the statement of
financial position of Aji Ltd as at 31 December 2016.
10.5. CHILTEPIN LTD
Chiltepin Ltd issued Rs. 4 million 5% convertible bonds on 1 October 2015 for Rs. 3.9 million.
The bonds have a four year term and are redeemable at par. At the time the bonds were issued
the prevailing market rate for similar debt without conversion rights was 7%. The effective interest
rate associated with the bonds is 7% and the liability is measured, in accordance with IAS 39
Financial Instruments: recognition and measurement, at amortised cost. The interest due was
paid and recorded within finance costs during the year.
Required
Prepare the accounting entries to record the issue of the convertible bonds and to record the
adjustment required in respect of the interest expense on the bonds for the year ended 30
September 2016.
10.6. HABENERO LTD
Habenero Ltd issued 6 million 5% cumulative Rs. 1 preference shares on 1 January 2016 and 2
million Rs. 1 ordinary shares on 1 May 2016.
Required
(a) Explain, with reference to the principles of IAS 32 Financial Instruments:
(b) Presentation how both of these instruments would be classified AND how their associated
dividends would be recorded in the financial statements of Habenero Ltd for the year
ended 31 December 2016.
Habenero Ltd acquired 500,000 shares in X on 1 November 2016 for Rs. 2.80 per share and
classified this investment as held for trading. Habenero Ltd paid 0.5% commission on the value
of the transaction to its broker. X’s shares were trading at Rs. 3.42 on 31 December 2016.
Required
Prepare the journal entries to record:
(i) the initial measurement of the investment at 1 November 2016; and
(ii) the subsequent measurement of this investment in the financial statements of
Habenero Ltd at 31 December 2016.

© Emile Woolf International 55 The Institute of Chartered Accountants of Pakistan


Advanced accounting and financial reporting

10.7. PASHAM TELECOM LIMITED


On 1 October 2016, Pasham Telecom Limited (PTL) raised Rs. 900 million by issuing 5-year
Term Finance Certificates (TFCs) at par value of Rs. 1,000 each carrying interest at a fixed rate
of 8% per annum. The interest is payable at the end of each quarter whereas principal will be
repaid in lump sum at the end of 5 years.
Considering the expected decline in interest rate, PTL entered into swap agreements (at market
rates) of an equal amount i.e. Rs. 900 million. The brokerage house which facilitated the
agreements was paid a brokerage of Rs. 1.0 million. The swap agreements would allow PTL to
receive a fixed rate of 6.5% per annum whereas PTL would pay a variable rate. Both payments
would be made at the beginning of each quarter. The swap agreements have the same maturity
dates as the TFCs.
All necessary documentation was completed on 1 October 2016 when the variable interest rate
was 6.27% per annum.
On 31 December 2016, as a result of a rise in market interest rates, the fair value of the TFCs
fell to Rs. 992 per TFC and the net fair value of the swap was Rs. 7.29 million (loss).
Required:
Explain how the above transactions should be accounted for in the books of PTL during the year
ended 31 December 2016 assuming that hedging criteria are met. Show all relevant
calculations.

© Emile Woolf International 56 The Institute of Chartered Accountants of Pakistan


Questions

CHAPTER 11 - IAS 19: EMPLOYEE BENEFITS

11.1. LABURNUM LIMITED


Laburnum Limited operates a defined benefit pension plan for its employees. At 1 January 2016
the fair value of the pension plan assets was Rs. 2,600,000 and the present value of the plan
liabilities was Rs. 2,900,000.
The actuary estimates that the current and past service costs for the year ended 31 December
2016 is Rs. 450,000 and Rs. 90,000 respectively. The past service cost is caused by an increase
in pension benefits. The plan liabilities at 1 January and 31 December 2016 correctly reflect the
impact of this increase.
The interest cost on the plan liabilities is estimated at 8% and the expected return on plan assets
at 5%.
The pension plan paid Rs. 240,000 to retired members in the year to 31 December 2016.
Laburnum Limited paid Rs. 730,000 in contributions to the pension plan and this included Rs.
90,000 in respect of past service costs.
At 31 December 2016 the fair value of the pension plan assets is Rs. 3,400,000 and the present
value of the plan liabilities is Rs. 3,500,000.
In accordance with the amendment to IAS 19 Employee Benefits, Laburnum Limited recognises
actuarial gains and losses in other comprehensive income in the period in which they occur.
Required
Calculate the actuarial gains or losses on pension plan assets and liabilities that will be included
in other comprehensive income for the year ended 31 December 2016. (Round all figures to the
nearest Rs.000).
11.2. JABEL LIMITED
Jabel Limited operates a defined benefit pension plan for its employees. At 1 July 2015 the fair
value of the pension plan assets was Rs. 1,200,000 and the present value of the plan liabilities
was Rs. 1,400,000. The interest cost on the plan liabilities was estimated at 7% and the expected
return on plan assets at 4%.
The actuary estimates that the current service cost for the year ended 30 June 2016 is Rs.
300,000. Jabel Limited made contributions into the pension plan of Rs. 400,000 in the year. The
pension plan paid Rs. 220,000 to retired members in the year to 30 June 2016.
At 30 June 2016 the fair value of the pension plan assets was Rs. 1,400,000 and the present
value of the plan liabilities was Rs. 1,600,000.
In accordance with the amendment to IAS 19 Employee Benefits, Jabel Limited recognises
actuarial gains and losses in other comprehensive income in the period in which they occur.
Required
Calculate the net expense that will be included in Jabel Limited’s profit or loss AND the amounts
that would be included in other comprehensive income in respect of actuarial gains or losses for
the year ended 30 June 2016. (Round all workings to the nearest Rs.000)
11.3. KAGHZI LIMITED
Kaghzi Limited operates a defined benefit pension plan for its employees. At 1 January 2016 the
fair value of the pension plan assets was Rs. 1,400,000 and the present value of the pension
plan liabilities was Rs. 1,700,000.
The actuary estimates that the service cost for the year to 31 December 2016 is Rs. 320,000.
The interest cost on the plan liabilities is estimated at 7% and the expected return on plan assets
at 4% for the year to 31 December 2016. The pension plan paid Rs. 170,000 to retired members
and Kaghzi Limited paid Rs. 580,000 in contributions to the pension plan for the year to 31
December 2016.

© Emile Woolf International 57 The Institute of Chartered Accountants of Pakistan


Advanced accounting and financial reporting

At 31 December 2016 the fair value of the pension plan assets is Rs. 2,100,000 and the present
value of the pension plan liabilities is Rs. 2,400,000. In accordance with the amendment to IAS
19 Employee Benefits, Kaghzi Limited recognises actuarial gains and losses within other
comprehensive income in the period in which they occur.
Required
Calculate the actuarial gains or losses on pension plan assets and liabilities that will be included
in Kaghzi Limited’s other comprehensive income for the year ended 31 December 2016. (Round
all figures to the nearest Rs.000).
11.4. LASURA LTD
Lasura Ltd operates a defined benefit pension plan for its employees. At 1 July 2015 the fair
value of the pension plan assets was Rs. 2,200,000 and the present value of the pension plan
liabilities was Rs. 2,400,000. The interest cost on the pension plan liabilities was estimated at 8%
and the expected return on pension plan assets at 5%.
The actuary estimates that the current service cost for the year ended 30 June 2016 is Rs.
500,000. Lasura Ltd made contributions into the pension plan of Rs. 300,000 and the pension
plan paid Rs. 450,000 to retired members in the year to 30 June 2016. At 30 June 2016 the fair
value of the pension plan assets was Rs. 2,300,000 and the present value of the pension plan
liabilities was Rs. 2,700,000.
Actuarial gains and losses are included within the other comprehensive income of Lasura Ltd as
incurred.
Required
(i) Calculate the net expense that will be included in Lasura Ltd’s profit or loss for the year
ended 30 June 2016, in accordance with IAS 19 Employee benefits.
(ii) Calculate the amount that will be included in Lasura Ltd’s other comprehensive income for
the year ended 30 June 2016, in accordance with IAS 19 Employee benefits.
11.5. UNIVERSAL SOLUTIONS
(a) Explain the following as used in IAS 19 Employee Benefits:
(i) The term ‘defined benefit pension plan’
(ii) The basis to be adopted in measuring scheme assets
(iii) The basis to be adopted in measuring scheme liabilities
(iv) Actuarial gains and losses.
(b) Universal Solutions operates a defined benefit pension scheme on behalf of its employees.
The company conducts an annual review of funding in conjunction with their actuaries who
have supplied the following information:
At 31 Dec Year 3 At 31 Dec Year 4
Rs. Rs.
Present value of pension fund obligations 1,200 1,300
Market value of pension fund assets 1,000 1,100
Information relevant to the actuarial valuation:
Discount rate used to determine pension fund liabilities 5%
Current service cost Rs. 100
Contributions to the pension fund Rs. 140
Benefits paid out amounted to Rs. 95
Required
(i) Show the figures that would appear on the face of the statement of financial performance
as at 31 December Year 3 and Year 4.
(ii) Construct a journal to explain the movement on the defined benefit net asset (or net
liability) during the year ended 31 December Year 4

© Emile Woolf International 58 The Institute of Chartered Accountants of Pakistan


Questions

11.6. DHA INTERIORS LTD


DHA Interiors Ltd operates two pension plans.
Pension Plan 1
The terms of the plan are as follows:
(i) employees contribute 6% of their salaries to the plan
(ii) DHA Interiors Ltd contributes, currently, the same amount to the plan for the benefit of the
employees
(iii) On retirement, employees are guaranteed a pension which is based upon the number of
years’ service with the company and their final salary.
(iv) This plan was closed to new entrants from 31 October 2015, but which was open to future
service accrual for the employees already in the scheme.
The following details relate to the plan in the year to 31 October 2016:
Rs. m
Present value of obligation at 1 November 2015 200
Present value of obligation at 31 October 2016 240
Fair value of plan assets at 1 November 2015 190
Fair value of plan assets at 31 October 2016 225
Current service cost 20
Pension benefits paid 19
Total contributions paid to the scheme for year to 31 October 2016 17
Actuarial gains and losses are recognised in the ‘statement of other comprehensive income
(OCI)’.
Pension Plan 2
Under the terms of the plan, DHA Interiors Ltd does not guarantee any return on the contributions
paid into the fund. The company’s legal and constructive obligation is limited to the amount that is
contributed to the fund. The following details relate to this scheme:
Rs. m
Fair value of plan assets at 31 October 2016 21
Contributions paid by company for year to 31 October 2016 10
Contributions paid by employees for year to 31 October 2016 10
The discount rates for the two plans are:
1 November 2015 31 October 2016
Discount rate 5% 6%
Required
(a) Explain the nature of and differences between a defined contribution plan and a defined
benefit plan with specific reference to the company’s two schemes.
(b) Show the accounting treatment for the two DHA Interiors Ltd pension plans for the year
ended 31 October 2016 under IAS 19 ‘Employee Benefits’.

© Emile Woolf International 59 The Institute of Chartered Accountants of Pakistan


Advanced accounting and financial reporting

CHAPTER 12 - IFRS 2: SHARE BASED PAYMENTS


12.1. TOSHACK LTD
Toshack Ltd has granted 50 share appreciation rights to each of its 1000 employees on 1
January 2013. The rights are due to vest on 31 December 2016, with payment being made on 31
December 2017. Assume that 75% of the awards vest. Shares prices were:

Rs.
01/01/2013 22
31/12/2013 27
31/12/2016 31
31/12/2017 28
Required
In accordance with IFRS 2, Share Based Payment;
(i) What liability would be recorded on 31 December 2016 for the share appreciation rights?
(ii) How would the settlement of the transaction be accounted for on 31 December 2017.
12.2. IFRS 2
(a) IFRS 2 requires an entity to recognise share-based payment transactions in its financial
statements. These include transactions with the employees or other parties where they are
to be settled in cash, other assets or equity instruments of the entity.
The IFRS identifies three types of share-based payment transaction and sets out the
measurement principles and specific requirements for each.
Required
(i) Suggest why there was a need for a standard in this area.
(ii) Identify and briefly explain the three types of share based payments recognised by
IFRS 2.
(b) A client of your firm, a listed company with a 31 December year end, contacts you for
advice on a proposed share option scheme for its employees.
On 1 January Year 5, the client granted 100 options to each of its 500 employees. The
grant is conditional upon the employee working for the client over the next three years. At
the grant date, it is estimated that the fair value of each option is Rs. 15.
Calculate the expense in profit or loss for each year of the vesting period:
(i) assuming that the client’s expectations throughout the vesting period are that all
options will vest; and alternatively
(ii) assuming that the client’s best estimates of the proportion of options that will vest
are as follows:
Estimate at 31 December Year 5 85%
Estimate at 31 December Year 6 88%
With 44,300 options actually vesting at 31 December Year 7.
12.3. SAVAGE LTD
Savage Ltd granted share options to its 300 employees on 1 October 2014. Each employee will
receive 1,000 share options provided they continue to work for Savage Ltd for the following three
years from the grant date. The fair value of the options at the grant date was Rs. 11 each. In the
year ended 30 September 2015, 10 employees left and another 30 were expected to leave over
the next two years. For the year ended 30 September 2016, 20 employees left and another 15
are expected to leave in the year to 30 September 2017.

© Emile Woolf International 60 The Institute of Chartered Accountants of Pakistan


Questions

Required
Discuss the accounting treatment to be adopted for the share options and calculate the amount
to be recognised in the statement of profit or loss in respect of these options for the year ended
30 September 2016. Prepare appropriate accounting entries.
12.4. YORATH LTD
Yorath Ltd granted share options to its 600 employees on 1 October 2013. Each employee will
receive 500 share options provided they continue to work for Yorath Ltd for four years from the
grant date. The fair value of each option at the grant date was Rs. 148.
The actual and expected staff movement over the 4 years to 30 September 2017 is given below:
2014 20 employees left and another 50 were expected to leave over the next three years.
2015 A further 25 employees left and another 40 were expected to leave over the next two years.
2016 A further 15 employees left and another 20 were expected to leave the following year.
2017 No actual figures are available to date.
The sales director of Yorath Ltd has stated in the board minutes that he disagrees with the
treatment of the share options. No cash has been paid out to employees, therefore he fails to
understand why an expense is being charged against profits.
Required
(a) Calculate the charge to the statement of profit or loss for the year ended 30 September
2016 for Yorath Ltd in respect of the share options and prepare the journal entry to record
this.
(b) Explain the principles of recognition and measurement for share-based payments as set
out in IFRS 2 Share-based Payments so as to address the concerns of the sales director.
12.5. QUALTECH LTD
(a) Qualtech Ltd granted share options to its 300 employees on 1 January 2015. Each
employee will receive 1,000 share options provided they continue to work for Qualtech Ltd
for 3 years from the grant date. The fair value of each option at the grant date was Rs.
122.
The actual and expected staff movement over the 3 years to 31 December 2017 is
provided below:
2015: 25 employees left and another 40 were expected to leave over the next two years.
2016: A further 15 employees left and another 20 were expected to leave the following
year.
Required
(i) Calculate the charge to Qualtech Ltd’s statement of profit or loss for the year ended
31 December 2016 in respect of the share options and prepare the journal entry to
record this.
(ii) Explain how the recognition and measurement of a share-based payment would
differ if it was to be settled in cash rather than in equity, in accordance with IFRS 2
Share-based Payments.
12.6. BRIDGE LTD
Bridge Ltd granted 1,000 share options to each of its 300 employees on 1 January 2015, with the
condition that they continue to work for Bridge Ltd for 4 years from the grant date. The fair value
of each option at the grant date was Rs. 50.
20 employees left in the year to 31 December 2015 and at that date another 65 were expected to
leave over the next three years. 23 employees left in the year to 31 December 2016 and at that
date another 44 were expected to leave over the next two years.

© Emile Woolf International 61 The Institute of Chartered Accountants of Pakistan


Advanced accounting and financial reporting

Required
(i) Calculate the charge to Bridge Ltd’s statement of profit or loss for the year ended 31
December 2016 in respect of the share options and prepare the journal entry to record
this.
(ii) Explain why, in accordance with IFRS 2 Share-based Payment, share options, such as
those granted by Bridge Ltd, generate a charge to the statement of profit or loss despite no
cash transaction having occurred.
12.7. CAPSTAN LTD
Capstan Ltd granted 1,000 share appreciation rights (SARs), to its 300 employees on 1 January
2015. To be eligible, employees must remain employed for 3 years from the date of issue and the
rights must be exercised in January 2014, with settlement due in cash.
In the year to 31 December 2015, 32 staff left and a further 35 were expected to leave over the
following two years.
In the year to 31 December 2016, 28 staff left and a further 10 were expected to leave in the
following year.
The fair value of each SAR was Rs. 80 at 31 December 2015 and Rs. 120 at 31 December 2016.
Required
Prepare the accounting entry to record the expense associated with the SARs, for the year to 31
December 2016, in accordance with IFRS 2 Share-based Payments.
12.8. NEWTOWN LTD
Newtown Ltd granted 1,000 share appreciation rights (SARs) to each of its 500 employees on 1
July 2014. To be eligible for the rights, employees must remain employed by Newtown Ltd for 3
years from the date of grant. The rights must be exercised in July 2017, with settlement due in
cash.
In the year to 30 June 2015, 42 employees left and a further 75 were expected to leave over the
following two years.
In the year to 30 June 2016, 28 employees left and a further 25 were expected to leave in the
following year.
The fair value of each SAR was Rs. 90 at 30 June 2015 and Rs. 110 at 30 June 2016.
Required
(i) Prepare the journal entry to record the expense associated with the SARs for the year
ended 30 June 2016, in accordance with IFRS 2 Share-based payment.
(ii) Explain, in accordance with IFRS 2 Share-based payment, how the recognition and
measurement of a share-based payment would differ, if it was to be settled in equity rather
than cash.
12.9. SINDH TRANSIT LTD
Sindh Transit Ltd granted share options to all of its 400 employees on 1 January 2015. Each
employee will receive 1,000 share options provided they continue to be employed by Sindh
Transit Ltd for four years from the grant date. The fair value of an option at the grant date was
Rs. 220.
On the same date Sindh Transit Ltd granted 500 share appreciation rights to each of its
employees. To be eligible, employees again have to be employed by Sindh Transit Ltd for four
years from the grant date.
The rights are exercisable in the two-month period from 1 January 2019 and will be settled in
cash. The fair value of each share appreciation right was Rs. 120 at 31 December 2015 and Rs.
140 at 31 December 2016.
The actual and expected future staff movements as at 31 December 2015 and 31 December
2016 are provided below.

© Emile Woolf International 62 The Institute of Chartered Accountants of Pakistan


Questions

2015: 15 left and another 55 were expected to leave over the next three years.
2016: a further 22 left and another 36 were expected to leave over the next two years.
Required
(a) Prepare, in accordance with IFRS 2 Share-based Payment, the accounting entries
required in the financial statements of Sindh Transit Ltd for the year to 31 December 2016
in respect of the two financial instruments identified above.
(b) Explain the main principle of recognition set out by IFRS 2 Share-based Payment for share
based payments AND why the treatment of the two financial instruments identified above
will differ in the statement of financial position.
12.10. XYX LIMITED
The financial statements of XYZ Limited for the year ended 30 June 2016 are in the final stage of
preparation and the following matters are under consideration:
(a) On 1 July 2013, XYZ offered 5000 share options each to its 10 marketing managers and
10 back office managers. The offer is conditional upon completion of three years’ service
from the date the offer was given. It was estimated at the time of offer that two managers
from each department would leave the company before the completion of 3 years. The fair
market value of the company’s shares on 1 July 2013 was Rs. 50 per share.
Other conditions and information are as follows:
(i) Conditions specific to marketing managers:
 Marketing manager can exercise the offer if the profit of the company increases by
10% per annum on average over the next three years.
 The offer can be exercised at Rs. 18 per share at the completion of vesting period.

Profit for the first two years increased by 12% and 10% respectively. However profit for the third
year has increased by 3% only.
(ii) Conditions specific to back office managers:
 Back office managers can exercise the offer if share price of the company increases
by 10% per annum on average over the next three years.
 The offer can be exercised at Rs. 23 per share at the completion of vesting period.
 On 1 July 2013, fair value of these share options was Rs. 30 per option taking into
account the estimated probability that the necessary share price growth would be
achieved.

On 1 January 2016, the share price declined. Considering the decline, XYZ modified the share
option scheme for back office managers by reducing the exercise price to Rs. 10 per share. The
fair value of the option immediately before and after the reduction in exercise price was Rs. 5 and
Rs. 14 respectively.
(iii) Upto 30 June 2015, there was no change in estimate regarding number of managers
leaving the company. However, during the year ended 30 June 2016, three managers left
the company i.e. two from marketing and one from back office.
(b) On 1 July 2013, XYZ purchased 1 million five year bonds issued by Ali Manufactures
Limited (AML) at a premium of Rs. 5 per bond with the intention to hold them till maturity
i.e. 30 June 2018. The bonds will be redeemed at their face value i.e. Rs. 100 per bond.
The transaction costs associated with the acquisition of the bonds were Rs. 1.5 million.
The coupon interest rate is 6% per annum while the effective interest yield at the time of
purchase was 4.5186%.
Due to certain financial and liquidity issues, AML restructured the payment plan with effect from
30 June 2016, after due consultation with bondholders. Under the revised plan the maturity date
was extended by one year. Further, the coupon rate was increased to 6.25% for 2017 and 2018
and 6.5% for 2019.

© Emile Woolf International 63 The Institute of Chartered Accountants of Pakistan


Advanced accounting and financial reporting

The management of XYZ is of the view that due to restructuring the credit risk on the loan has
increased significantly. As a result, it estimates lifetime expected credit losses of Rs. 5 million on
the investment.
Required:
In accordance with the requirement of International Financial Reporting Standards, describe the
accounting treatment in respect of the above transactions in the financial statements of XYZ
Limited for the year ended 30 June 2016.
12.11. RAVI LIMITED
On 1 July 2016 Ravi Limited (RL) offered 1000 share options to each of its 500 employees. The
offer is conditional upon completion of five years’ service from the date the offer was given. The
award of options would depend on attainment of the following additional conditions:
Condition 1: Average sales for the next five years is Rs. 300 million or more.
Condition 2 : At the end of the 5th year, share price of the company exceeds Rs. 200 per share.
Market values of the options at grant date were estimated as under:

Rupees
Without taking into account any of the two conditions 50
Taking into account only condition 1 44
Taking into account only condition 2 38
Taking into account both the conditions 36

Following information is available at year end:


(i) Sales for the year ended 30 June 2017 was Rs. 210 million however it was estimated that
sales would increase by 20% each year.
(ii) The share price was Rs. 160 per share.
(iii) It was estimated that 15% of the employees would leave the company before completion
of five years.
Required:
Discuss how this transaction should be recorded in RL’s books of accounts for the year ended 30
June 2017.

© Emile Woolf International 64 The Institute of Chartered Accountants of Pakistan


Questions

CHAPTER 13 - DISPOSAL OF SUBSIDIARIES


13.1. PATCHE LTD
Patche Ltd owns 85% of the ordinary share capital of Somers Ltd for many years. The shares
were bought for Rs. 765 million and Somers Ltd’s reserves at the time of purchase amounted Rs.
60million.
On 31 March 2016, Patche Ltd. sold 120 million of Somers Ltd shares for Rs. 480 million. The
only entry made in respect of this transaction was the receipt of the cash, which was credited to
the “investment in subsidiary” account. No dividend was paid by either entity during the period.
The summarised financial statements of the companies are as follows:
Statements of profit or loss and other comprehensive income for the year ended to 30 June
2016.

Patche Ltd Somers Ltd


Rs.’m Rs.‘m
Profit before tax 390 180
Income tax expenses (120) (60)

Profit for the year 270 120


Other comprehensive income that will not be
reclassified to profit or loss net of tax 60 30

330 150
STATEMENTS OF FINANCIAL POSITION AS AT 30 JUNE 2016
Non-current assets:
Property, plant & equipment 1,605 534
Investment in Somers Ltd 285 -
Current assets:
Inventories 960 570
Trade receivables 750 525
Cash and bank 240 267

3840 1896

Equity interest
Share capital: Rs. 1 ordinary shares 1,500 600
Reserves 930 510

2,430 1,110
Current liabilities:
Trade payables 885 513
Income tax 240 180
Provisions 285 93

3,840 1,896

© Emile Woolf International 65 The Institute of Chartered Accountants of Pakistan


Advanced accounting and financial reporting

There was no impairment loss in the group during the year.


Assume that the gain as calculated in the parent’s separate financial statement will be subject to
companies’ income tax at a rate of 30%, and that profit and other comprehensive income accrue
evenly throughout the year.
Patche Ltd. group policy is to measure non-controlling interest at fair value at the date of
acquisition.
The fair value of the non-controlling interest in Somers Ltd. was Rs. 135million at the date of
acquisition.
Required
Prepare the following
(a) Statement of profit or loss and comprehensive income and statement of changes in equity
of Patche Ltd for the year ended 30 June 2016.
(b) Consolidated statement of profit or loss and comprehensive income of Patche Ltd. for the
same period.
(c) Consolidated statement of financial position as at 30 June 2016.

13.2. DISPOSAL
At 31 December Year 1, Hoo owned 90% of the shares in Spool. At this date the carrying amount
of the net assets of Spool in the consolidated financial statements of the Hoo Group was Rs. 800
million. None of the assets of Spool are re-valued.
On 1 January Year 2, Hoo sold 80% of the equity of Spool for Rs. 960 million in cash.
The remaining shares in Spool held by Hoo are estimated to have a fair value of Rs. 100 million.
Required
Explain how the disposal of the shares in Spool should be accounted for in the consolidated
financial statements of the Hoo Group.

13.3. PART DISPOSAL


On 1 January Year 2, P acquired 80% of the equity of S for Rs. 620 million in cash. On 30 June
Year 2 it sold 10% of the equity in S for Rs. 94 million. S did not issue any shares or make any
distribution to its shareholders in the year to 31 December Year 2. P uses the partial goodwill
method to account for the acquisition of S and no goodwill is attributed to the non-controlling
interest.
The net assets of S were as follows, at carrying value:

Rs. million

At 1 January Year 2 700

At 31 December Year 2 900

At 31 December Year 2, P carries out an impairment review and decides that the goodwill in its
investment in S has been impaired by Rs. 8 million.
Required
Explain how the disposal of the shares in S should be accounted for.

© Emile Woolf International 66 The Institute of Chartered Accountants of Pakistan


Questions

13.4. THE A GROUP


The summarised statements of financial position of A and its two subsidiaries B and C at 31
December Year 3 are shown below:
Summarised statements of financial position at 31 December Year 3

A B C
Rs.000 Rs.000 Rs.000
Investment in subsidiaries:
B 1,164
C 1,120
Other net assets 2,516 1,260 1,400
4,800 1,260 1,400
Ordinary share capital
(Rs. 1 shares) 1,500 500 400
Accumulated profits 3,300 760 1,000
4,800 1,260 1,400
The summarised statement of profit or loss for A and B for the year ended 31 December Year 4
are as follows:
A B
Rs.000 Rs.000
Profit before tax 1,200 250
Taxation (360) (60)
–––––– ––––––
Profit after tax 840 190
Dividends paid (50) (20)
–––––– ––––––
Retained profit for year 790 170
Retained profit at start of year 3,300 760
–––––– ––––––
Retained profits at end of year 4,090 930
–––––– ––––––
Additional information:
(i) A acquired 80% of the ordinary share capital of B on 1 January Year 0 when the reserves
of B were Rs. 420,000.
(ii) A acquired 90% of the ordinary share capital of C on 1January Year 1 when the reserves
of C were Rs. 320,000.
(iii) On 1 January Year 4, A disposed of 350,000 shares in C for Rs. 1,925,000. This
transaction has not yet been accounted for by A. The remaining investment in shares of C
at this date had a fair value of Rs. 44,000.
(iv) There were no changes in the issued share capital of the subsidiaries since acquisition by
A.
(v) None of the companies re-value any of their non-current assets.
(vi) The A Group uses the partial goodwill method of accounting for acquisitions and no
goodwill is attributed to non-controlling interests. There has been no impairment of
goodwill.

© Emile Woolf International 67 The Institute of Chartered Accountants of Pakistan


Advanced accounting and financial reporting

Required
Prepare A’s consolidated statement of profit or loss and show the movement on consolidated
equity reserves for the year to 31 December Year 4 and a consolidated statement of financial
position as at that date.
13.5. BARTLETT LTD
Many years ago Bartlett Ltd bought 80% of the ordinary shares of Lymon Ltd for Rs. 175,000.
On 1 July 2016 Bartlett sold all of these shares and used the proceeds (Rs. 212,000) to purchase
65% of the ordinary shares of Zeigler Ltd on the same date.
Statements of profit or loss for all three companies for the year ended 31 December 2016 were
as follows.

Bartlett Ltd Lymon Ltd Zeigler Ltd


Rs. Rs. Rs.
Revenue 1,926,500 521,600 792,400
Cost of sales 1,207,200) (386,200) (405,900)
Gross profit 719,300 135,400 386,500
Distribution costs (207,500) (79,200) (198,200)
Administrative expenses (192,600) (26,100) (107,100)
Profit before tax 319,200 30,100 81,200
Taxation (110,000) (9,500) (27,500)
Profit after tax 209,200 20,600 53,700

No entries have been made in Bartlett Ltd’s statement of profit or loss relating to the sale of
Lymon Ltd. Lymon’s net assets were Rs. 140,000 at the 1st January 2016.
Goodwill arising on the acquisition of Lymon Ltd was Rs. 25,400.
Required
Prepare the consolidated statement of profit or loss for Bartlett Ltd for the year ended 31
December 2016.

© Emile Woolf International 68 The Institute of Chartered Accountants of Pakistan


Questions

CHAPTER 14 - IFRS-5: NON-CURRENT ASSETS HELD FOR SALE AND


DISCONTINUED OPERATIONS
14.1. SAUL
Saul operates its business through a number of divisions. It has a year end of 31 December. Set
out below are extracts from the draft financial statements of Saul for the year ended 31
December Year 1.
Statement of profit or loss for the year ended 31 December Year 1
Rs.000
Revenue 3,900
Cost of sales (2,500)
––––––
Gross profit 1,400
Distribution costs (300)
Administrative expenses (800)
––––––
Profit before tax 300
Income tax expense (90)
––––––
Profit for the period 210
––––––

Statement of financial position at 31 December Year 1


Assets Rs.000 Rs.000
Non-current assets
Property, plant and equipment 1,900
Intangible assets 40
––––––
1,940
Current assets
Inventories 350
Trade and other receivables 190
Cash 90
––––––
630
u ––––––
Total assets 2,570
––––––
Equity and liabilities
Equity
Share capital 600
Retained earnings 1,700
––––––
2,300
Current liabilities
Trade and other payables 195
Current tax payable 75
––––––
270
––––––
Total equity and liabilities 2,570
––––––

© Emile Woolf International 69 The Institute of Chartered Accountants of Pakistan


Advanced accounting and financial reporting

On 30 November Year 1 Saul made the decision to close Division A, which is located in a
different part of the country and covers a separate major line of business. This decision was
immediately announced to the press and to the workforce and, by the end of Year 1, a buyer had
been found.
The directors of Saul have calculated the following.
15% of the entity’s income and expenses for the year was attributable to Division A.
No tax is attributable to Division A.
Property, plant and equipment of Rs. 510,000 and payables of Rs. 10,000 in the above statement
of financial position relate to Division A. The fair value minus costs to sell of the property, plant
and equipment is Rs. 450,000.
Required
Redraft the above financial statements to meet the provisions of IFRS 5:Non-current assets held
for sale and discontinued operations. Work to the nearest Rs.000.
14.2. SHAHID HOLDINGS
(a) State the definition of both non-current assets held for sale and discontinued operations
and explain the usefulness of information for discontinued operations.
Shahid Holdings is in the process of preparing its financial statements for the year ended
31 October 2016. The company’s main activity is in the travel industry mainly selling
package holidays (flights and accommodation) to the general public through the Internet
and retail travel agencies. During the current year the number of holidays sold by travel
agencies declined dramatically and the directors decided at a board meeting on 15
October 2016 to cease marketing holidays through its chain of travel agents and sell off
the related high-street premises. Immediately after the meeting the travel agencies’ staff
and suppliers were notified of the situation and an announcement was made in the press.
The directors wish to show the travel agencies’ results as a discontinued operation in the
financial statements to 31 October 2016. Due to the declining business of the travel
agents, on 1 August 2016 (three months before the year end) Shahid Holdings expanded
its Internet operations to offer car hire facilities to purchasers of its Internet holidays.
The following are Shahid Holdings’s summarised statement of profit or loss results – years
ended:
31 October 2016 31 October 2015
Travel
Internet Car hire Total Total
agencies
Rs.’000 Rs.’000 Rs.’000 Rs.’000 Rs.’000
Revenue 23,000 14,000 2,000 39,000 40,000
Cost of sales (18,000) (16,500) (1,500) (36,000) (32,000)
    
Gross profit/(loss) 5,000 (2,500) 500 3,000 8,000
Operating expenses (1,000) (1,500) (100) (2,600) (2,000)
    
Profit/(loss) before tax 4,000 (4,000) 400 400 6,000
    
The results for the travel agencies for the year ended 31 October 2015 were: revenue Rs.
18 million, cost of sales Rs. 15 million and operating expenses of Rs. 1·5 million.
Required
(b) Discuss whether the directors’ wish to show the travel agencies’ results as a
discontinued operation is justifiable.
(c) Assuming the closure of the travel agencies is a discontinued operation, prepare the
(summarised) statement of profit or loss of Shahid Holdings for the year ended 31
October 2016 together with its comparatives.
Note: Shahid Holdings discloses the analysis of its discontinued operations on the
face of its statement of profit or loss.

© Emile Woolf International 70 The Institute of Chartered Accountants of Pakistan


Questions

14.3. PRIMA
Prima is a listed company with a year end of 31 December. It operates two businesses, the first
is the rental of luxury yachts and the second is a chain of luxury holiday villas in Europe. The
directors have requested your advice on the following matters.
Holiday villas
Prima’s policy is to carry the holiday villas at their re-valued amount, which, based on the most
recent valuation in 20X0, was Rs. 20m (historical cost was Rs. 10m). Prima is unsure how
frequently a revaluation of such properties is required and so has instructed a surveyor to provide
an up-to-date valuation as at 31 December Year 4. This valuation has provided the following
information:

Rs. million
Replacement cost 17
Value in use 28
Open market value 25

One of the villas has received very few bookings over the past two years and so a decision was
reached to exclude it from the Year 5 brochure. It is currently up for sale. The villa has a carrying
value of Rs. 1.25m. Its value in use is only Rs.0.85m and its expected market value is Rs. 1m,
before expected agents and solicitor’s fees of Rs. 50,000. The directors are unsure as to the
accounting treatment of this villa. A number of potential buyers have expressed an interest in the
property, and it is hoped that a deal will be negotiated in the first few months of Year 5.
Prima’s accounting policy is to not charge depreciation on the villas. Its justification is that the
villas are maintained to a high standard and have useful lives of at least 50 years.
Head Office
Over the past two years, Prima has built its own head office. Construction began on 1 October
Year 2 and finished on 1 June Year 4, although minor modifications meant that the company did
not relocate until 1 September Year 4.
The site cost Rs. 1m and the costs of construction were a further Rs. 8m. Prima took out a two
year loan of Rs. 5m on 1 October Year 2, at an interest rate of 9% per annum, to help fund the
work. In order to encourage businesses to operate in areas of high unemployment, the
government offered a Rs. 1.5m grant towards the cost of construction. The terms of settlement
were that payment would only be made upon completion of the building once a government
inspection had taken place. This inspection had not taken place by the year end, but Prima is
confident that the grant will be received shortly after the year end.
The company intends to use the head office for the next 50 years and, as for the villas, does not
intend to depreciate the land or buildings.
Yachts
Prima has spent the past year designing a new range of luxury yachts. Work was completed on 1
April Year 4 at a cost of Rs. 20m. During the construction, the economy took a downturn and the
company now believes that the market value of the yachts is only Rs. 17m, although the value in
use is estimated to be Rs. 18m. The engines of the yachts have a three year life, the interior has
a two year life, and the remainder should have a life of 15 years. The engine cost is believed to
represent 15% of the total cost of manufacture and the interior approximately 25%.
Required
Explain the accounting issues relating to the villas, head office and yachts, referring to relevant
IFRS guidance. Where possible, numerical information relating to the 31 December Year 4
financial statements should be provided.

© Emile Woolf International 71 The Institute of Chartered Accountants of Pakistan


Advanced accounting and financial reporting

CHAPTER 15 - IFRS 13: FAIR VALUE MEASUREMENT


15.1. MONIBA LIMITED
Moniba Limited holds an asset that is traded in three different active markets.
Relevant information about the asset in the three markets is as follows:

Market A Market B Market C

Market share in % 50% 30% 20%

--------------- Rs. per unit - --------------

Entry price 30,500 31,500 30,600

Exit price 29,500 30,500 29,600

Transaction cost 700 1,500 1,000

Transport cost 800 1000 400

Required:
Identify principal and most advantageous markets along with reasons thereof. Also calculate the
fair value of the asset in these markets

© Emile Woolf International 72 The Institute of Chartered Accountants of Pakistan


Questions

CHAPTER 16 - IFRS 15: REVENUE FROM CONTRACTS WITH CUSTOMERS


16.1. PARVEZ LIMITED
The following transactions took place at Parvez Limited (PL).
(1) On 5 March 2017 PL sold goods to a bank for Rs.18m cash and agreed to repurchase the
goods for Rs.19m cash on 5 July 2017. The goods will be shifted to a storage facility under
bank’s control and security.
(2) On 31 March PL’s car manufacturing division consigned several vehicles to independent
dealers for sale to third parties. The sales price to the dealer is PL’s list price at the date of
sale to third parties. If a vehicle is unsold after six months, the dealer has a right to return
the vehicle to PL within next fifteen days.
Required
Discuss how the above transactions should be accounted for in the books of accounts of Parvez
Limited.
16.2. SACHAL LIMITED
Sachal Limited (SL):
(a) Sells standard computer software package meant for small and medium sized restaurant
management. This software package is sold:
 at price of Rs. 1.5 million payable before delivery,
 with thirty days trial time, and
 without any maintenance support after trial time
As per practice, it takes around six months for the customers to use the package
independent of any support from SL. Practically, SL has to provide on-site support service
for at least six months to almost all customers free-of-cost. However, in case of customer’s
request for support beyond six months, SL provides services under a formal paid service
contract.
(b) Provides maintenance and support for the above standard software package at a price of
Rs. 0.3 million per annum.
(c) Provides designing and development of customized software to customers. Payment is
made monthly by customers on the basis of chargeable hours of developers of SL. First
year maintenance service is provided free-of-cost. Subsequent maintenance service is
provided at the rate of 10% of the total contract price. Thereafter, for next three years
maintenance service is provided at 5% of the contract price per annum.
Required
Explain the considerations to be taken into account in determining accounting for revenue by
Sachal Limited.
16.3. BRILLIANT LIMITED
Brilliant Limited (BL) manufactures and sells plastic card printing machines with laminators. A
machine-specific card printing software is provided as a must part of the printing machine. BL
also sells plastic cards imported from Thailand.
BL agreed to supply the following to, Proud Learners (PL), a country-wide school network:
 15 Card printing machines – Available in ready stock
 8 Laminators – Would require 30 days to deliver
 100,000 Plastic cards – Available in ready stock
A lump sum price of Rs.9.2 million for the total contract has been agreed between BL and school
network.

© Emile Woolf International 73 The Institute of Chartered Accountants of Pakistan


Advanced accounting and financial reporting

Cost and list prices of the goods are:

Item Price (Rs.) Cost (Rs.)

Card printing machines 800,000 400,000

Laminators 200,000

Plastic cards 12 5

BL does not sell printing machine without laminator. However, in order to get this order BL went
against its policy. There is another supplier of imported card printing machine of almost similar
specification. This supplier sells the machine at Rs.750,000.
In most recent customers’ surveys printing machine of BL has been given 7 out of 10 points as
against 9 out of 10 given to competitors’ imported machine. There is no supplier of laminator in
the market.
Required
Identify performance obligations and allocate the transaction price to the identified performance
obligations.
16.4. WAQAS LIMITED
Waqas Limited (WL) enters into a contract of construction of a reverse osmosis plant for the
manufacturing unit of Ali Chemical Limited (ACL) for Rs.20 million, for which WL estimated cost
is Rs.12 million. This included supply and installation of plant and related construction work. The
project is to be completed within 18 months. WL measures performance on the basis of cost
incurred.
At the end of seventh month ACL and WL agreed to modify the contract by adding construction
of an additional water reservoir at a price of Rs.2.5 million, which will supply drinking water to a
sister concern of ACL. The additional cost is estimated as Rs.1.8 million by WL. At the end of
seventh month WL incurred 4.2 million on the project.
At the end of tenth month ACL and WL agreed to modify the contract by increasing the size of
water reservoir that was included in the original design of the project. ACL and WL agreed to an
additional consideration of Rs.1 million, for which WL will incur an additional cost of Rs.1 million.
At the end of seventh month WL incurred Rs. 7.2 million on the plant project and Rs. 0.72 million
on additional reservoir.
At the end of sixteenth month ACL and WL agreed to modify the contract by adding pumping and
piping facility from plant to the manufacturing unit of ACL for a consideration of Rs.3 million. This
facility was part of the project, but at the inception this contract was awarded to another
contractor, which was terminated by ACL. The cost to be incurred by WL was estimated as
Rs.2.8 million. At the end of sixteenth month WL incurred Rs.11.7 million on the plant project and
Rs.1.35 million on additional reservoir.
Required
Advise how these transactions should be recognized in the books of Waqas Limited.
16.5. ZEBRA LIMITED
During the year ended 31 December 2017, following transactions were made by Zebra Limited
(ZL):
i. On 1 October 2017 ZL purchased a piece of land from Cow Limited (CL) having fair value of
Rs. 230 million. According to the agreement, CL has the option to receive:
 75,000 shares of ZL to be issued on 30 April 2018; or
 Cash equivalent to the value of 70,000 ZL’s shares to be paid on 28 February 2018.

© Emile Woolf International 74 The Institute of Chartered Accountants of Pakistan


Questions

The actual/estimated fair values of ZL’s share at various dates were as follows:

Date 1-Oct-2017 31-Dec-2017 28-Feb-2018 30-Apr-2018


Fair value per share Rs. 3,000 Rs. 2,900 Rs. 3,300 Rs. 3,400

ii. On 1 April 2017 ZL acquired a licence for operating a TV channel for Rs. 86.3 million out of
which Rs. 50 million was paid immediately. The balance amount is payable on 1 April 2019.
A mega social media and print media campaign was launched to promote the channel at a
cost of Rs. 10 million. The transmission of the channel started on 1 August 2017.
The license is valid for 5 years but is renewable every five years at a cost of Rs. 35 million.
Since the renewal cost is significant, the management intends to renew the license only once and
sell it at the end of 8 years.
In the absence of any active market, the management has estimated that residual value of the
license would be Rs. 15 million and Rs. 20 million at the end of 5 years and 8 years respectively.
Applicable discount rate is 10% p.a.
Required:
Discuss how these transactions should be recorded in ZL’s books of accounts for the year ended
31 December 2017.
16.6. HAWKS LIMITED
Draft consolidated financial statements of Hawks Limited (HL) for the year ended 31 December
2017 show the following amounts:

Rs. in million
Total assets 2,500
Total liabilities 1,610
Total comprehensive income 659

During the process of finalisation, following matters have been noted:


i. HL signed a contract with one of its customers, Rhino Limited (RL). Under the terms of the
contract, HL is required to:
 produce a series of 5 television advertisements. Each completed advertisement has to
be approved by an independent agency for a minimum 3-star rating. After approval, copy
of the advertisement would be provided to RL who can then use it for other campaigns.
HL has no enforceable right to payment against any under production advertisement.
 arrange airtime of 120 minutes for broadcasting of each advertisement. The primary
responsibility for broadcasting of these advertisements lies with HL.
HL is entitled to Rs. 80 million for the whole contract and bonus of Rs. 2 million for each
advertisement if a 5-star rating is attained.
HL considers all advertisements as equal units. The expected cost of producing each
advertisement and its broadcasting is Rs. 5 million and Rs. 9 million respectively. HL expects to
earn mark-up of 30% and 20% respectively on similar services to other clients. Historically,
advertisements produced by HL have received the minimum 3-star rating but 5-star rating is
received occasionally.
As at 31 December 2017:
 production of 3 advertisements has been completed. Two of them have received 5-star
rating whereas one has received 3-star rating. HL expects that at least one of the remaining
advertisements would get 5-star ratings.
 broadcasting of first two advertisements has been completed whereas 70% time of the third
advertisement has been broadcasted. Bookings have been made for the broadcasting of
remaining time of third advertisement and entire time of fourth advertisement.

© Emile Woolf International 75 The Institute of Chartered Accountants of Pakistan


Advanced accounting and financial reporting

 details of the actual cost incurred on this project are as follows:

Production cost Broadcasting cost


Advertisement
----------- Rs. in million -----------
1 4.7 8.5
2 5.6 9.2
3 4.8 8.9
4 3.1* 9.0

* in process

All the above costs have been paid and charged to profit or loss account. HL had received Rs. 40
million from RL by 31 December 2017 which has been credited to advance from customers
account.
Required:
Determine the revised amounts of total assets, total liabilities and total comprehensive income
after incorporating impact of the above adjustments, if any.

© Emile Woolf International 76 The Institute of Chartered Accountants of Pakistan


Questions

CHAPTER 17 - IFRS 16: LEASES


17.1. X LTD
X Ltd is considering acquiring a machine. It has two options; cash purchase at a cost of Rs.
11,420,000 or a lease.
The terms of the lease are as follows:
(i) The lease period is for four years from 1 January 2016 with an annual rental of Rs.
4,000,000 payable on 31 December each year.
(ii) The lessee is required to pay all repairs, maintenance and other incidental costs.
(iii) The interest rate implicit in the lease is 15% p.a.
Note:
Estimated useful economic life span of the machine is four years.
Required
(a) Prepare a schedule of the allocation of the finance charges in the books of X Limited for
the entire lease period.
(b) Prepare an extract of the Statement of Financial Position of X Limited for the year ended
31 December 2016.
17.2. PROGRESS LTD
Progress Ltd. acquired a machine from Fine Rentals Ltd. on January 3, 2016 under a lease
agreement extending over three years.
The agreement required them to make an initial deposit of Rs. 1,280,000 to be followed by three
annual payments of Rs. 800,000 on 31 December each year starting from 2016.
The cash price of the machinery was Rs. 3,200,000 and Fine Rentals Ltd. added 12% interest
which was duly communicated to Progress Ltd.
The annuity method is used to allocate interest.
Required
(a) Compute the interest element and the capital portion of the annual repayments; and
(b) Show the journal entries that will record the transaction resulting from the lease
agreement.
17.3. MIRACLE TEXTILE LIMITED
On 1 July 2014, Miracle Textile Limited (MTL) acquired a machine on lease, from a bank.
Details of the lease are as follows:
(i) Cost of machine is Rs. 20 million.
(ii) The lease term and useful life is 4 years and 10 years respectively.
(iii) Instalment of Rs. 5.80 million is to be paid annually in advance on 1 July.
(iv) The interest rate implicit in the lease is 15.725879%.
(v) At the end of lease term, MTL has an option to purchase the machine on payment of
Rs. 2 million. The fair value of the machine at the end of lease term is expected to be
Rs. 3 million.
MTL depreciates the machine on the straight line method to a nil residual value.
Required
Prepare relevant extracts of the statement of financial position and related notes to the financial
statements for the year ended 30 June 2016 along with comparative figures. Ignore taxation.

© Emile Woolf International 77 The Institute of Chartered Accountants of Pakistan


Advanced accounting and financial reporting

17.4. ACACIA LTD


On 1 April 2015 Acacia Ltd entered into the following lease agreements. The terms of each lease
are as follows:
(1) Plant with a fair value of Rs. 275,000 was leased under an agreement which requires
Acacia Ltd to make annual payments of Rs. 78,250 on 1 April each year, commencing on
1 April 2015, for four years. After the four years Acacia Ltd has the option to continue to
lease the plant at a nominal rent for a further three years and is likely to do so as the asset
has an estimated useful life of six years. The present value of the lease payments is Rs.
272,850. Acacia Ltd is responsible for insuring and maintaining the plant during the period
of the lease.
(2) Office equipment with a fair value of Rs. 24,000 was leased under a non-cancellable
agreement which requires Acacia Ltd to make annual payments of Rs. 6,000 on 1 April
each year, commencing on 1 April 2015, for three years. The lessor remains responsible
for insuring and maintaining the equipment during the period of the lease. The equipment
has an estimated useful life of ten years. The present value of the lease payments is Rs.
16,415.
Acacia Ltd allocates finance charges on an actuarial basis. The interest rate implicit in both of the
leases is 10%.
Required
Prepare all relevant extracts from Acacia Ltd's financial statements for the year ended 31 March
2016 in respect of the above leases. The only notes to the financial statements required are
those in respect of lease liabilities or commitments.
17.5. SHOAIB LEASING LIMITED
Shoaib Leasing Limited (the lessor) has entered into a three year agreement with Sarfaraz
Limited (the lessee) to lease a machine with an expected useful life of 4 years. The cost of
machine is Rs. 2,100,000.
The following information relating to lease transaction is available:
(i) Date of commencement of lease is July 1, 2016.
(ii) The lease contains a purchase bargain option at Rs. 100,000. At the end of the lease term,
the value of the machine will be Rs. 300,000.
(iii) Lease instalments of Rs. 860,000 are payable annually, in arrears, on June 30.
(iv) The implicit interest rate is 12.9972%.
Required
(a) Prepare the journal entries for the years ending June 30, 2017, 2018 and 2019 in the
books of lessor. Ignore tax.
(b) Produce extracts from the statement of financial position including relevant notes as at
June 30, 2017 to show how the transactions carried out in 2017 would be reflected in
the financial statements of the lessor.
(Disclosure of accounting policy is not required.)
17.6. AKBAR LTD
Akbar Ltd. (AL) prepares financial statements on 31 March each year. On
1 April Year 4, AL sold a machine to another company, Shahwez Ltd. (SL), for Rs.850,000 and
then leased it back under a ten year arrangement. AL had purchased the machine exactly ten
years previously for Rs.500,000 and had charged total depreciation of Rs.60,000 on the machine
up to the date of disposal. Assume that the transfer of machine by the seller-lessee satisfies the
requirements of IFRS 15.
Details of the sale and leaseback arrangement are as follows:
Consideration received from SL Rs.850,000
Fair value at date of disposal Rs.550,000

© Emile Woolf International 78 The Institute of Chartered Accountants of Pakistan


Questions

Lease rentals (payable at the end of each year) is Rs.100,000 and interest rate implicit in the
lease is 10% p.a
Required
How AL should reflect in its books of accounts:
a) Right-of-use retained by AL
b) Gain / loss on rights transferred
17.7. ALI LIMITED
Ali Limited entered into a sale and leaseback arrangement with a bank on
1 April 2015. The arrangement involved the sale at fair value of plant and machinery to the bank
for Rs.1,440,000.
This amount has been credited to Ali Limited’s operating income. The carrying amount of the
plant and machinery was Rs.840,000 and its remaining useful life was five years at 1 April 2015.
No depreciation has been charged in respect of this plant and machinery for the year ended 31
March 2016.
Under the terms of the lease, Ali Limited is to pay five annual payments at
31 March each year, of Rs.360,000 (in arrears). The first payment has been made and has been
debited to operating costs. The interest rate implicit in the lease is 8%. The transfer of asset does
not satisfy the requirements of IFRS 15.
Required
Explain how the above transaction should be accounted for, with all relevant calculations, in the
financial statements for the year ended 31 March 2016.
17.8. MOAZZAM TEXTILE MILLS LIMITED
Moazzam Textile Mills Limited (MTML) is facing severe financial difficulties. To improve the cash
flows, the management has decided to sell and lease back three power generators of the
company under three different sale and lease back arrangements which were signed on August
15, 2016. At the same time, MTML enters into a contract with the buyer-lessor for the right to use
the generators for 5 years, with annual payments of Rs.1,000,000 each for Generator A and
Generator B and Rs.1,500,000 for Generator C, payable at the end of each year. The interest
rate implicit in the lease is 4.5%, The related information as on
August 15, 2016 is given below:
Cost Carrying Fair Value Value in Amount
Value Use of Financing
Rs.000 Rs.000 Rs.000 Rs.000 Rs.000
Generator A 10,000 7,500 6,000 6,500 6,000
Generator B 12,000 6,000 5,000 5,000 6,000
Generator C 10,000 7,000 10,000 12,000 10,000
Required
Prepare the accounting entries that should be recorded by the company on August 15, 2016 in
respect of the above transactions.
Note: Cost of making sale is negligible. Ignore tax and deferred tax implications, if any.
17.9. MODIFICATION THAT DECREASES THE SCOPE OF THE LEASE (IFRS 16,
ILLUSTRATIVE EXAMPLE 17)
Lessee enters into a 10-year lease for 5,000 square metres of office space. The annual lease
payments are CU 50,000 payable at the end of each year. The interest rate implicit in the lease
cannot be readily determined. Lessee’s incremental borrowing rate at the commencement date is
6 per cent per annum. At the beginning of Year 6, Lessee and Lessor agree to amend the
original lease to reduce the space to only 2,500 square metres of the original space starting from
the end of the first quarter of Year 6. The annual fixed lease payments (from Year 6 to Year 10)
are CU30,000. Lessee’s incremental borrowing rate at the beginning of Year 6 is 5 per cent per
annum.

© Emile Woolf International 79 The Institute of Chartered Accountants of Pakistan


Advanced accounting and financial reporting

Required
How the lessee should reflect in its books of accounts:
a) Right-of-use retained
b) Lease liability
17.10. MODIFICATION THAT BOTH INCREASES AND DECREASES THE SCOPE OF THE
LEASE (IFRS 16, ILLUSTRATIVE EXAMPLE 18)
Lessee enters into a 10-year lease for 2,000 square metres of office space. The annual lease
payments are CU100,000 payable at the end of each year. The interest rate implicit in the lease
cannot be readily determined. Lessee’s incremental borrowing rate at the commencement date is
6 per cent per annum. At the beginning of Year 6, Lessee and Lessor agree to amend the
original lease to;
a) include an additional 1,500 square metres of space in the same building starting from the
beginning of Year 6 and
b) reduce the lease term from 10 years to eight years. The annual fixed payment for the 3,500
square metres is CU150,000 payable at the end of each year (from Year 6 to Year 8).
Lessee’s incremental borrowing rate at the beginning of Year 6 is 7 per cent per annum.
Required
How the lessee should account for;
a) Pre-modification right-of-use and lease liability
b) At the effective date of modification
c) Decrease in the lease term
d) Increase in the leased space
17.11. SUBLEASE CLASSIFIED AS A FINANCE LEASE (IFRS 16, ILLUSTRATIVE EXAMPLE
20)
Head lease — An intermediate lessor enters into a five-year lease for 5,000 square metres of
office space (the head lease) with Entity A (the head lessor).
Sublease — At the beginning of Year 3, the intermediate lessor subleases the 5,000 square
metres of office space for the remaining three years of the head lease to a sublessee.
Required
How this transaction is accounted for in the books of intermediate lessor.
17.12. SUBLEASE CLASSIFIED AS AN OPERATING LEASE (IFRS 16, ILLUSTRATIVE
EXAMPLE 21)
Head lease — An intermediate lessor enters into a five-year lease for 5,000 square metres of
office space (the head lease) with Entity A (the head lessor).
Sublease — At commencement of the head lease, the intermediate lessor subleases the 5,000
square metres of office space for two years to a sublessee.
Required
How this transaction is accounted for in the books of intermediate lessor
17.13. TRACK LIMITED
On 1 July 2014 Track Limited (TL) sold its property to Strong Bank Limited (SBL) for Rs. 600
million. The net carrying amount and market value of the property on 1 July 2014 were Rs. 240
million and Rs. 800 million respectively. The remaining useful economic life of the property was
15 years. Under the terms of agreement, TL continues to occupy the property and is also
responsible for its maintenance. As consideration of occupation rights,
TL pays rent of Rs. 90 million per annum, payable in arrears.
TL has the option to repurchase the property on 30 June 2016 at Rs. 550 million. TL charges
depreciation on straight-line basis.

© Emile Woolf International 80 The Institute of Chartered Accountants of Pakistan


Questions

TL’s cost of equity is 10% whereas incremental borrowing rate is 11.052% per annum.
Applicable income tax rate is 30%.
Required:
a) Prepare accounting entries to record the above transaction for the year ended 30 June 2015
and give brief explanation of the accounting treatment worked out by you with reference to
the relevant International Financial Reporting Standards.
b) Prepare accounting entries to record the transactions for the year ended 30 June 2016
if TL does not exercise the option to repurchase the property on 30 June 2016.
17.14. PATEL LIMITED
a) Following are the details of lease related transactions of Patel Limited (PL):
On 1 July 2015 PL acquired a plant for lease term of 5 years at Rs. 18 million per annum,
payable in arrears. Fair value and useful life of this plant as on 1 July 2015 were Rs. 60
million and 6 years respectively. Bargain purchase option at the end of lease term would be
exercisable at Rs. 1 million. On July 2015 PL’s incremental borrowing rate was 9% per
annum.
After one year, PL sub-let this plant for Rs. 21 million per annum, payable in arrears for lease
term of 5 years. Implicit rate of this transaction was 11% per annum.
b) On 1 July 2014, PL acquired a building for its head office for lease term of 8 years at Rs. 50
million per annum, payable in arrears.
However, after the board’s decision of constructing own head office building, PL negotiated
with the lessor and the lease contract was amended on July 2016 by reducing the original
lease term from 8 to 6 years with same annual payments.
Incremental borrowing rates on 1 July 2014 and 1 July 2016 were 12% and 10% per annum
respectively.
Required:
Prepare the extracts relevant to the above transactions from PL’s statements of financial
position and profit or loss for the year ended 30 June 2017, in accordance with the
International Financial Reporting Standards. (Comparatives figures and notes to the financial
statements are not required)

© Emile Woolf International 81 The Institute of Chartered Accountants of Pakistan


Advanced accounting and financial reporting

CHAPTER 18 - IAS 12: INCOME TAXES


18.1. SHAKIR INDUSTRIES
Given below is the statement of profit or loss and other comprehensive income of Shakir
Industries for the year ended December 31, 2016:
2016
Rs. m
Sales 143.00
Cost of goods sold (96.60)
Gross profit 46.40
Operating expenses (28.70)
Operating profit 17.70
Other income 3.40
Profit before interest and tax 21.10
Financial charges (5.30)
Profit before tax 15.80
Following information is available:
(i) Operating expenses include an amount of Rs.0.7 million paid as penalty to SECP on non-
compliance of certain requirements of the Companies Act, 2017.
(ii) During the year, the company made a provision of Rs. 2.4 million for gratuity. The
actual payment on account of gratuity to outgoing members was Rs. 1.6 million.
(iii) Lease payments made during the year amounted to Rs.0.65 million which include
financial charges of Rs.0.15 million. As at December 31, 2016, obligations against assets
subject to finance lease stood at Rs. 1.2 million. The movement in assets held under
finance lease is as follows:
Rs. m
Opening balance – 01/01/2016 2.50
Depreciation for the year (0.7)
Closing balance – 31/12/2016 1.80
(iv) The details of owned fixed assets are as follows:

Accounting Tax
Rs. m Rs. m
Opening balance – 01/01/2016 12.50 10.20
Purchased during the year 5.3 5.3
Depreciation for the year (1.10) (1.65)
Closing balance – 31/12/2016 16.70 13.85

(v) Capital work-in-progress as on December 31, 2016 include financial charges of Rs. 2.3
million which have been capitalised in accordance with IAS-23 “Borrowing Costs”.
However, the entire financial charges are admissible, under the Income Tax Ordinance,
2002.
(vi) Deferred tax liability and provision for gratuity as at January 1, 2016 was Rs.0.55
million and Rs.0.7 million respectively.
(vii) Applicable income tax rate is 35%.
Required
Based on the available information, compute the current and deferred tax expenses for the
year ended December 31, 2016.

© Emile Woolf International 82 The Institute of Chartered Accountants of Pakistan


Questions

18.2. DWAYNE LTD (PART 1)


The following information has been obtained in order to allow completion of Dwayne Ltd’s
deferred tax balances as at 31st December 2015.
Statement of financial position at 31st December 2015 - Extracts
Carrying amount Tax base
Rs. 000 Rs. 000
Assets
Land & buildings 45,500 17,500
Plant and equipment 68,000 26,000
Cost of investment in Larry 750 750
Investments 72,000 65,000
Dividend receivable 150 -

Liabilities
Long-term debt 20,500 21,000
Trade payables 9,500 9,500
Defined benefit liability 1,000
Deferred tax liability (31st December 2014) 13,500
(i) Dwayne revalues its land and buildings on an annual basis. It has no investment
properties. The fair value of land and buildings was Rs. 60 million at 31st December
2015.The 2015 revaluation has not yet been accounted for in Dwayne’s financial
statements. The pre-tax revaluation surplus as at 31st December 2014 stood at Rs. 24m.
(ii) The balance on the investments line relates to a portfolio of equity holdings. Some of these
are categorised as fair value through profit or loss and the balance as available-for-sale.
The fair value loss on AFS investments was Rs. 1m during 2015. This loss is considered to
be temporary in nature. The entire portfolio of equity holdings was acquired during 2015.
(iii) Tax relief on the defined benefit expense is given on a cash basis.
(iv) Dividend income is not taxed in the jurisdiction in which Dwayne operates.
(v) Dwayne borrowed Rs. 21m just before the year end and incurred transaction costs of
500k. Transaction costs are allowable in full in the year in which a loan is raised.
(vi) The tax rate changed from 30% to 28% in the current year.
Required
(a) Prepare a schedule of temporary differences and resultant deferred tax for Dwayne.
(b) Prepare a note showing the movement on the consolidated deferred tax balance for the
year ending 31st December 2015.
(c) Prepare a journal showing the movement on the deferred taxation account showing the
entries due to rate changes and temporary differences arising during the period.
18.3. DWAYNE LTD (PART 2)
The investment in Dwayne’s statement of financial position is the cost of 80% of Larry. The date
of this acquisition was 31st December 2015.
The following statement of net assets relates to Larry on 31st December 2015.
Fair value Carrying amount Tax base
Rs. 000 Rs. 000 Rs. 000
Buildings 600 400 300
Plant and equipment 56 46 25
Inventory 152 162 144
Trade receivables 120 120 120
Defined benefit liability (100) (150) –
Current liabilities (50) (50) (50)
778 528 539

© Emile Woolf International 83 The Institute of Chartered Accountants of Pakistan


Advanced accounting and financial reporting

Required
(a) Prepare a schedule of temporary differences and resultant deferred tax for Larry from the
point of view of the group.
(b) Combine the deferred tax figures to obtain the group deferred tax balance.
(c) Prepare a note showing the movement on the consolidated deferred tax balance for the
year ending 31st December 2015.
(d) Calculate the goodwill arising on acquisition of Larry.
18.4. COHORT
Cohort is a private limited company and has two 100% owned subsidiaries, Legion and Air, both
themselves private limited companies. Cohort acquired Air on 1 January 20X2 for Rs. 5 million
when the fair value of the net assets was Rs. 4 million, and the tax base of the net assets was
Rs. 3.5 million. The acquisition of Air and Legion was part of a business strategy whereby Cohort
would build up the value of the group over a three-year period and then list its share capital on
the Stock Exchange.
(a) The following details relate to the acquisition of Air, which manufactures electronic goods:
(i) Part of the purchase price has been allocated to intangible assets because it relates
to the acquisition of a database of key customers of Air. The recognition and
measurement criteria for an intangible asset under IFRS 3 Business Combinations
and IAS 38 Intangible Assets do not appear to have been met but the directors feel
that the intangible asset of Rs. 500,000 will be allowed for tax purposes and have
computed the tax provision accordingly. However, the tax authorities could possibly
challenge this opinion.
(ii) Air has sold goods worth Rs. 3 million to Cohort since acquisition and made a profit
of Rs. 1 million on the transaction. The inventory of these goods recorded in
Cohort’s statement of financial position at the year ending 31May 20X2 was Rs. 1.8
million.
(iii) The retained earnings of Air at acquisition were Rs. 2 million. The directors of
Cohort have decided that, during the three years leading up to the date that they
intend to list the shares of the company, they will realise earnings through future
dividend payments from the subsidiary amounting to Rs. 500,000 per year. Tax is
payable on any remittance of dividends and no dividends have been declared for the
current year.
(b) Legion was acquired on 1 June 20X1 and is a company which undertakes various projects
ranging from debt factoring to investing in property and commodities. The following details
relate to Legion for the year ending 31 May 20X2:
(i) Legion has a portfolio of readily marketable government securities which are held as
current assets. These investments are stated at market value in the statement of
financial position with any gain or loss taken to profit or loss. These gains and losses
are taxed when the investments are sold. Currently the accumulated unrealised
gains are Rs. 4 million.
(ii) Legion has calculated that it requires a general allowance of Rs. 2 million against its
total loan portfolio. Tax relief is available when the specific loan is written off.
Management feel that this part of the business will expand and thus the amount of
the general provision will increase.
(iii) When Cohort acquired Legion it had unused tax losses brought forward. At 1 June
20X1, it appeared that Legion would have sufficient taxable profit to realise the
deferred tax asset created by these losses but subsequent events have proven that
the future taxable profit will not be sufficient to realise all of the unused tax loss.
Impairment of goodwill is not allowed as a deduction in determining taxable profit.
Required
Write a note suitable for presentation to the partner of an accounting firm setting out the
deferred tax implications of the above information for the Cohort Group of companies.

© Emile Woolf International 84 The Institute of Chartered Accountants of Pakistan


Questions

18.5. MODEL TOWN GROUP


The following statement of financial position relates to Model Town Group, a public limited company
at 30 June 2016:
Rs.000
Assets:
Non-current assets:
Property, plant, and equipment 10,000
Goodwill 6,000
Other intangible assets 5,000
Financial assets (cost) 9,000
30,000
Trade receivables 7,000
Other receivables 4,600
Cash and cash equivalents 6,700
18,300
Total assets 48,300
Equity and liabilities
Share capital 9,000
Other reserves 4,500
Retained earnings 9,130
Total equity 22,630
Non-current liabilities
Long term borrowings 10,000
Deferred tax liability 3,600
Employee benefit liability 4,000
Total non-current liabilities 17,600
Current tax liability 3,070
Trade and other payables 5,000
Total current liabilities 8,070
Total liabilities 25,670
Total equity and liabilities 48,300

The following information is relevant to the above statement of financial position:


(i) The financial assets are investments in equity. Model Town has made an irrevocable
election to recognise gains and losses on these assets in other comprehensive income.
However, they are shown in the above statement of financial position at their cost on 1 July
2015. The market value of the assets is Rs. 10.5 million on 30 June 2016. Taxation is
payable on the sale of the assets.
(ii) The stated interest rate for the long term borrowing is 8 per cent. The loan of Rs. 10 million
represents a convertible bond which has a liability component of Rs. 9.6 million and an equity
component of Rs.0.4 million. The bond was issued on 30 June 2016.
(iii) The tax bases of the assets and liabilities are the same as their carrying amounts in the
statement of financial position at 30 June 2016 except for the following:
(a) Rs.000
Property, plant, and equipment 2,400
Trade receivables 7,500
Other receivables 5,000
Employee benefits 5,000
(b) Other intangible assets were development costs which were all allowed for tax
purposes when the cost was incurred in 2015.

© Emile Woolf International 85 The Institute of Chartered Accountants of Pakistan


Advanced accounting and financial reporting

(c) Trade and other payables include an accrual for compensation to be paid to
employees. This amounts to Rs. 1 million and is allowed for taxation when paid.
(iv) Goodwill is not allowable for tax purposes in this jurisdiction.
(v) Assume taxation is payable at 30%.
Required
Calculate the provision for deferred tax at 30 June 2016 after any necessary adjustments to the
financial statements showing how the provision for deferred taxation would be dealt with in the
financial statements.
(Assume that any adjustments do not affect current tax. You should briefly discuss the
adjustments required to calculate the provision for deferred tax).
18.6. ELEPHANT LIMITED
Elephant Limited (EL) is in process of finalizing its financial statements for the year ended 31
December 2017. The following information has been gathered for preparing the disclosures relating
to taxation:
(i) Profit before tax for the year after making all necessary adjustments was Rs. 103 million.
(ii) Expenses include:
 donations of Rs. 12 million not allowable for tax purposes.
 accruals of Rs. 30 million which will be allowed in tax on payment basis.
(iii) Other income includes government grant of Rs. 10 million and dividend of Rs. 4 million.
Government grant is not taxable while dividend income is subject to tax rate of 10%.
(iv) Accounting depreciation for the year exceeds tax depreciation by Rs. 20 million.
(v) On 31 December 2017 buildings were revalued for the first time resulting in a surplus of Rs.
60 million. Revaluation does not affect taxable profits.
(vi) On 1 January 2017 EL granted 5,000 share options each to 12 senior executives, conditional
upon the executives remaining in EL’s employment until 31 December 2018. The exercise
price is Rs. 20 per share. On grant date, EL estimated the fair value of the share options at
Rs. 180 per option.
As on 31 December 2017 it was estimated that 2 employees would leave EL before 31
December 2018. Fair value of each share as on 31 December 2017 was Rs. 150.
As per tax laws, intrinsic value of the share option on the exercise date is an admissible
expense.
(vii) On 1 January 2017 EL had issued 1.5 million 10% convertible Term Finance Certificates
(TFCs) of Rs. 100 each. Interest is payable annually on 31 December whereas the principal
is to be paid at the end of 2020. Two TFCs are convertible into one ordinary share at any
time prior to maturity. On the date of issue, the prevailing interest rate for similar debt without
conversion option was 12% per annum.
The tax authorities do not allow any deduction for the imputed discount on the liability
component of the convertible TFCs.
(viii) Net deferred tax liability as on 1 January 2017 arose on account of:
Rs. in million
Property, plant and equipment (Rs. 95 million × 35%) 33.25
Unused tax losses (Rs. 85 million × 35%) (29.75)
Deferred tax liability – net 3.50

(ix) The tax rate for 2017 is 30% while it was 35% in 2016 and prior periods.
Required:
Prepare notes on taxation and deferred tax liability/asset for inclusion in EL’s financial statements
for the year ended 31 December 2017, in accordance with the IFRSs.

© Emile Woolf International 86 The Institute of Chartered Accountants of Pakistan


Questions

CHAPTER 19 - PRESENTATION OF FINANCIAL STATEMENTS (IAS 34, IAS 24)


19.1. FAZAL LIMITED
Fazal Limited is engaged in the manufacturing of specialized spare parts for automobile
assemblers. During the year 2016, the company has undertaken the following transactions with its
related parties:
(i) Sales of Rs. 500 million were made to its only subsidiary M/s Sami Motors Limited
(SML). Being the subsidiary, a special discount of Rs. 25 million was allowed to SML.
(ii) SML returned spare parts worth Rs. 5.5 million.
(iii) Raw materials of Rs. 5 million were purchased from Jalal Enterprises, which is owned
by the wife of the CFO of Fazal Limited.
(iv) Equipment worth Rs. 3 million was purchased from Khan Limited (KL). The wife of the
Production Director of the company is a director in KL.
(v) The company awarded a contract for supply of two machines amounting to Rs. 7 million
per machine to an associated company.
(vi) In 2014, an advance of Rs. 2 million was given to the Chief Executive of the
company. During the year 2016, he repaid Rs.0.3 million. The balance outstanding as on
December 31, 2016 was Rs. 1,100,000.
Required
Prepare a note for inclusion in the company’s financial statements in accordance with the
requirement of IAS 24: Related Party Disclosures.
19.2. BABER LIMITED
During the year ended June 30, 2016, Baber Limited (BL) has carried out several
transactions with the following individuals/entities:
(i) AK Associates provides information technology services to BL. One of the directors of BL
is also the partner in AK Associates.
(ii) SS Bank Limited is the main lender. By virtue of an agreement it has appointed a nominee
director on the Board of BL.
(iii) Mr. Zee who supplies raw materials to BL, is the brother of the Chief Executive Officer of
the company.
(iv) JB Limited is the distributor of BL’s products and has exclusive distribution rights for
the province of Punjab.
(v) Mr. Tee is the General Manager-Marketing of BL and is responsible for all major decisions
made in respect of sales prices and discounts.
(vi) BL’s gratuity fund is administered by the Trustees appointed by the company.
(vii) MM Limited is the leading supplier of BL and supplies 60% of BL’s raw materials.
(viii) Ms. Vee who conducted various training programmes for the employees of the
company, is the wife of BL’s Chief Executive Officer.
Required
Comment as to whether the above individuals/entities are ‘related parties’ of the company or not.
Support your arguments with references from International Accounting Standards.
19.3. GOLDEN LIMITED
The following related party transactions were carried out by Golden Limited (GL) during the
first year of its operation i.e. year ended December 31, 2016.
(i) Inventory costing Rs. 15 million was sold for Rs. 18 million to Platinum Limited (PL) which
owns 60% shares in GL. It is GL’s policy to add 30% margin on cost. Outstanding liability
at year end, in respect of these purchases was Rs. 6.5 million.

© Emile Woolf International 87 The Institute of Chartered Accountants of Pakistan


Advanced accounting and financial reporting

(ii) PL provided administrative services to GL. The cost of these services, if billed in the
open market, would have amounted to Rs. 350,000. No entries were made to record these
transactions, as it was agreed that the services would be provided free of charge.
(iii) A property was sold to Silver Limited (SL), an associated company, at its fair market value
of Rs. 10 million. 50% of the amount was settled prior to year end. GL reimbursed Rs.
500,000 to SL on account of transfer and other incidental charges related to this
property.
(iv) An interest free loan of Rs. 2 million was granted to an executive director of the company
under the terms of employment. During the year, Rs. 200,000 were repaid by the executive
director.
(v) On July 1, 2016 GL obtained a short term loan of Rs. 25 million from one of its major
shareholder, at the prevailing annual interest rate of 12%. The principal as well as the
accrued mark-up were outstanding at the close of the year.
Required
Prepare a note on related party transactions for inclusion in GL’s financial statements for the year
ended December 31, 2016 showing disclosures as required under IAS - 24 (Related Party
Disclosures).
19.4. METAL LIMITED
On 1 July 2015, Metal Limited (ML) acquired 80% shareholdings in Copper Limited (CL), 90%
shareholdings in Zinc Limited (ZL) and 55% shareholdings in Steel Limited (SL). The following
transactions took place among these companies, during the period up to 30 June 2016:
(i) On 1 May 2015, ML sold a machine to CL at 20% above the carrying amount of Rs. 16
million. CL paid the entire amount on 15 July 2015. The useful life of the machine is 10
years.
(ii) On 1 July 2015, ZL awarded a contract of Rs. 15 million to Iron Builders and Developers
(IBD) for the extension of its existing factory. One of the directors of ML is also a partner in
IBD.
(iii) Since the date of acquisition, ML has been providing management services to CL and ZL.
ML did not charge management fee for its services during the first year. However, with
effect from 1 July 2015, management fee has been charged from each company at the
rate of Rs.0.5 million per month. Payment is made on the 10th day of the next month.
(iv) On 1 January 2016, ML sold goods amounting to Rs. 10 million to Gold Limited (GL). The
wife of chief financial officer of ZL is a major shareholder in GL.
Required
Prepare a note on related party disclosure including comparative figures, for inclusion in the
individual financial statements of ML, CL, ZL and SL, for the year ended 30 June 2016.
19.5. ENGINA
Engina, a foreign company has approached a partner in your firm to assist in obtaining local
stock exchange listing (or stock market registration) for the company. Engina is registered in a
country where transactions between related parties are considered to be normal but where such
transactions are not disclosed. The directors of Engina are reluctant to disclose the nature of
their related party transactions as they feel that although they are a normal feature of business in
their part of the world, it could cause significant problems politically and culturally to disclose
such transactions.
The partner in your firm has requested a list of all transactions with parties connected with the
company and the directors of Engina have produced the following summary:
(a) Every month, Engina sells Rs. 50,000 of goods per month to Mr Satay, the financial
director. The financial director has set up a small retailing business for his son and the
goods are purchased at cost price for him. The annual turnover of Engina is Rs. 300
million. Additionally, Mr Satay has purchased his company car from the company for Rs.
45,000 (market value Rs. 80,000). The director, Mr Satay, earns a salary of Rs. 500,000 a
year, and has a personal fortune of many millions of pounds.

© Emile Woolf International 88 The Institute of Chartered Accountants of Pakistan


Questions

(b) A hotel property had been sold to a brother of Mr Soy, the Managing Director of Engina, for
Rs. 4 million (net of selling cost of Rs.0.2 million). The market value of the property was
Rs. 4.3 million but prices have been falling rapidly. The carrying value of the hotel was Rs.
5 million and its value in use was Rs. 3.6 million. There was an over-supply of hotel
accommodation due to government subsidies in an attempt to encourage hotel
development and the tourist industry.
(c) Mr Satay owns several companies and the structure of the group is outlined below. Engina
earns 60% of its profits from transactions with Car and 40% of its profits from transactions
with Wheel. All of the above companies are incorporated in the same country.

Required
Write a report to the directors of Engina setting out the reasons why it is important to disclose
related party transactions and the nature of any disclosure required for the above transactions
under IAS 24 Related Party Disclosures.

© Emile Woolf International 89 The Institute of Chartered Accountants of Pakistan


Advanced accounting and financial reporting

CHAPTER 20 - IAS 33: EARNINGS PER SHARE


20.1. AIRCON LTD
Mr Hamad, currently owns 20 million shares in Aircon Ltd. He recently received the published
financial statements of Aircon Ltd for the year ended 31 March 2016. Mr Hamad is not sure how
the performance of the company during the year will affect the market value of the entity’s shares
but he is aware that the earnings per share statistics are often used by analysts in assessing the
performance of listed companies.
Extracts from these published financial statements and other relevant information are given
below.
Statement of profit or loss for the period ended 31 March 2016
2016 2015
Rs.’m Rs.’m
Revenue 18,000 15,300
Cost of sales (11,340) (9,180)
Gross profit 6,660 6,120
Operating expenses (3,420) (3,240)
Operating profit 3,240 2,880
Interest payable (540) (576)
Profit before tax 2,700 2,304
Taxation (846) (720)
Profit after tax 1,854 1,584
Statement of financial position as at 31 March 2016
2016 2015
Rs.’m Rs.’m Rs.’m Rs.’m
Intangible assets 5,400 -
Tangible assets 7,200 6,660
──── ────
12,600 6,660
Current Assets
Inventory 2,340 1,800
Receivables 2,700 2,160
Cash in bank 180 5,220 162 4,122
──── ──── ──── ────
17,820 10,782
──── ────
Capital and Reserves
Share Capital 2,700 900
Share Premium 4,860 900
Retained Earnings 1,620 1,206
──── ────
9,180 3,006
Current Liabilities
Trade Payables 3,060 2,160
Taxation 900 756
Bank Overdraft 1,080 1,260
──── ────
5,040 4,176
──── ────
14,220 7,182
15% Loan Note 3,600 3,600
──── ────
17,820 10,782
──── ────

© Emile Woolf International 90 The Institute of Chartered Accountants of Pakistan


Questions

The following information is also relevant:


(i) The share capital of the company comprises Rs. 1 equity shares only.
(ii) On 1 October 2015, the company made a rights issue to existing shareholders of two new
shares for every one share held at a price of Rs. 5.94 per share and paid issue cost of Rs.
180,000.
(iii) The market price of shares immediately before the rights issue was Rs. 6.30 per share.
(iv) No other changes took place in the equity capital of Aircon Ltd in the year ended 31 March
2016.
Required
(a) Compute EPS for the year and the comparative figures that will be included in the
published financial statements of Aircon Ltd for the year ended 31 March 2016.
(b) Using the extracts you have been provided with, write a report to Mr Hamad identifying the
key factors which led to the change in the EPS of Aircon Ltd since the year ended 31
March 2016.
(c) Comment on the relevance of the EPS statistics to shareholders.
20.2. CACHET LTD
The statement of profit or loss for the year ended 31 December 2016 relates to Cachet Ltd.

Rs. Rs.
Profit Before Tax 121,900
Less: Taxation 52,900

69,000
Less: Transfer to general reserve 5,750
Dividends:
Preference shares 1,380
Ordinary shares 2,070

(92,00)

Retained profit 59,800

1 January 2016, the issued share capital of Cachet Ltd was 23,000 6% preference shares of Rs.
1 each and 20,700 ordinary shares of Rs. 1 each.
Required
Calculate the basic and diluted earnings per share for the year ended 31 December, 2016 under
the following circumstances:
(i) No change in the issued share capital.
(ii) The company made a bonus issue of one ordinary share for every four shares in issue at
30 September, 2016.
(iii) The company made a rights issue of shares on 1 October 2016 in the proportion of 1 for
every 5 shares held at a price of Rs. 1.20. The middle market price for the shares on the
last day of quotation cum rights was Rs. 1.80 per share.
20.3. MARY
On 1 January Year 5, Mary had 5 million ordinary shares in issue. The following transactions in
shares took place during the next year.
1 February A 1 for 5 bonus issue

© Emile Woolf International 91 The Institute of Chartered Accountants of Pakistan


Advanced accounting and financial reporting

1 April A 1 for 2 rights issue at Rs. 1 per share. The market price of the shares prior to
the rights issue was Rs. 4.
1 June An issue at full market price of 800,000 shares.
In Year 5 Mary made a profit before tax of Rs. 3,362,000. It paid ordinary dividends of Rs.
1,200,000 and preference dividends of Rs. 800,000. Tax was Rs. 600,500. The reported EPS for
Year 4 was Rs.0.32.
Required
Calculate the EPS for Year 5, and the adjusted EPS for Year 4 for comparative purposes.
20.4. MANDY
Mandy has had 5 million shares in issue for many years. Earnings for the year ended 31
December Year 4 were Rs. 2,579,000. Earnings for the year ended 31 December Year 3 were
Rs. 1,979,000. Tax is at the rate of 30%.
Outstanding share options on 500,000 shares have also existed for a number of years. These
can be exercised at a future date at a price of Rs. 3 per share. The average market price of
shares in Year 3 was Rs. 4 and in Year 4 was Rs. 5.
On 1 April Year 3 Mandy issued Rs. 1,000,000 convertible 7% bonds. These are convertible into
ordinary shares at the following rates.
On 31 December Year 6 30 shares for every Rs. 100 of bonds
On 31 December Year 7 25 shares for every Rs. 100 of bonds
On 31 December Year 8 20 shares for every Rs. 100 of bonds
Required
Calculate the diluted EPS for Year 4 and the comparative diluted EPS for Year 3.
20.5. AAZ LIMITED
The profit after tax earned by AAZ Limited during the year ended December 31, 2016 amounted to
Rs. 127.83 million. The weighted average number of shares outstanding during the year were 85.22
million.
Details of potential ordinary shares as at December 31, 2016 are as follows:
 The company had issued debentures which are convertible into 3 million ordinary
shares. The debenture holders can exercise the option on December 31, 2018. If the
debentures are not converted into ordinary shares they shall be redeemed on December
31, 2018. The interest on debentures for the year 2016 amounted to Rs. 7.5 million.
 Preference shares issued in 2013 are convertible into 4 million ordinary shares at the
option of the preference shareholders. The conversion option is exercisable on December
31, 2020. The dividend paid on preference shares during the year 2016 amounted to Rs.
2.45 million.
 The company has issued options carrying the right to acquire 1.5 million ordinary
shares of the company on or after December 31, 2016 at a strike price of Rs. 9.90 per
share. During the year 2016, the average market price of the shares was Rs. 11 per
share.
The company is subject to income tax at the rate of 30%.
Required
(a) Compute basic and diluted earnings per share.
(b) Prepare a note for inclusion in the company’s financial statements for the year ended
December 31, 2016 in accordance with the requirements of International Accounting
Standards.

© Emile Woolf International 92 The Institute of Chartered Accountants of Pakistan


Questions

20.6. ABC LIMITED


The following information pertains to ABC Limited, in respect of year ended March 31, 2016.
Rs. in ‘000
Consolidated profit for the year (including non-controlling interest) 15,000
Profit attributable to non-controlling interest 2,000
Dividend paid during the year to ordinary shareholders 4,000
Dividend paid on 10% Cumulative preference shares for the year 2015 2,000
Dividend paid on 10% Cumulative preference shares for the year 2016 2,000
Dividend declared on 12% Non-cumulative preference shares for the year 2016 2,400
(i) The company had 10 million ordinary shares at March 31, 2015.
(ii) The cumulative preference shares were issued at the time of inception of the company.
(iii) The 12% non-cumulative preference shares are convertible into ordinary shares, on or
before December 31, 2017 at a premium of Rs. 2 per share. The conversion rights are not
adjusted for subsequent bonus issues.
(iv) 0.50 million non-cumulative preference shares were converted into ordinary shares on July
1, 2015.
(v) The dividend declared on the non-cumulative preference shares, as referred above, was
paid in April 2016.
(vi) 1.20 million right shares of Rs. 10 each were issued at a premium of Rs. 1.50 per share on
October 1, 2015. The market price on the date of issue was Rs. 12.50 per share.
(vii) 20% bonus shares were issued on January 1, 2016.
(viii) Due to insufficient profit no dividend was declared during the year ended March 31,
2015.
(ix) The average market price for the year ended March 31, 2016 was Rs. 15 per share.
Required
Compute the basic and diluted earnings per share and prepare a note for inclusion in the
consolidated financial statements for the year ended March 31, 2016.
20.7. ALPHA LIMITED
Alpha Limited (AL), a listed company, acquired 80% equity in Zee Limited (ZL) on 1 July 2010.
The following information has been extracted from their draft financial statements:
AL ZL
----- Rs. in '000 -----
Balance as at 1 January 2013:
Share capital (Rs. 100 each) 80,000 35,000
12% Convertible bonds (Rs. 100 each) 30,000 -
Profit for the year ended 31 December 2013 (after tax) 60,000 25,000
Following information is also available:
(i) The bonds were issued at par on 1 January 2011 and are convertible at any time
before the redemption date of 31 December 2015, at the rate of five ordinary shares
for every four bonds.
(ii) Cost and fair value information of ZL’s investment property is as under:
31-Dec-2013 31-Dec-2012
-------- Rs. in '000 --------
Cost 65,000 60,000
Fair value 67,000 59,000

© Emile Woolf International 93 The Institute of Chartered Accountants of Pakistan


Advanced accounting and financial reporting

ZL uses cost model while the group policy is to use the fair value model to account
for investment property.
(iii) AL operates a defined benefit gratuity scheme for its employees. The actuary’s report
has been received after the preparation of draft financial statements and provides the
following information pertaining to the year ended 31 December 2013:
Rs. in '000
Actuarial losses 150
Current service costs 8,000
Net interest income 3,000
(iv) On 1 August 2013, under employees’ share option scheme, 60,000 shares were
issued by AL to its employees at Rs. 150 per share against the average market price
of Rs. 250 per share.
(v) Dividend details are as under:
AL ZL
2013 (Interim) 2012 (Final) 2013 (Interim) 2012 (Final)
Cash 18% 10% 12% 15%
Bonus shares - 20% - 16%
At the time of payment of dividend, income tax at 10% was deducted by AL and ZL.
(vi) Applicable tax rate for business income is 35%.
Required:
Extracts from the consolidated profit and loss account of Alpha Limited (including earnings per
share) for the year ended 31 December 2013 in accordance with the International Financial
Reporting Standards.
(Note: Comparative figures and information for notes to the financial statements are not required)
20.8. SAJJAD LIMITED
(a) Following information pertains to Sajjad Limited (SL) for the year ended 31 December
2016:
(i) The share capital of SL comprises of:
Rs. in million
Ordinary share capital (Rs. 100 each) 1,000
9% Class A preference shares (Rs. 100 each) 200
6% Class B preference shares (Rs. 100 each) 300
(ii) Class A preference shares which were issued on 1 January 2014 are cumulative, non-
convertible and non-redeemable. These shares were issued at Rs. 77.22 per share
i.e. at a discount of Rs. 22.78 per share. These shareholders are entitled to annual
dividend of 9% with effect from 1 January 2017. At the time of issue, the market
dividend yield on such type of preference shares was 9% per annum.
(iii) Class B preference shares which were issued on 1 January 2016 are non-
cumulative, non-convertible and non-redeemable. The payment of dividend of these
shares was made on 29 December 2016. These shareholders are also entitled to
participate in any remaining profits after adjusting dividend to ordinary and preference
shareholders. Such remaining profits are allocated between the Class B shareholders
and the ordinary shareholders in such a manner that the profit per share of ordinary
shareholders is twice the profit per share of Class B shareholders.
(iv) SL earned profit after tax of Rs. 150 million during the year ended 31 December 2016
and paid interim dividend of Rs. 2.50 per share to ordinary shareholders.
Required:
Compute basic earnings per share for the ordinary shareholders for the year ended 31 December
2016.

© Emile Woolf International 94 The Institute of Chartered Accountants of Pakistan


Questions

20.9. TIGER LIMITED


Following information pertains to Tiger Limited (TL):

Quarter ended 31- Half year ended 31-


Dec-2017 Dec-2017
Profit after tax (Rs. in million) 140 239
Average market price per share (Rs.) 330 360

Ordinary shares
 20 million shares of Rs. 100 each were outstanding as at 1 July 2017.
 4 million shares were issued on 1 August 2017 at market price of Rs. 355 per share.
Convertible bonds
 On 1 November 2016 TL issued 0.8 million 7% convertible bonds at par value of Rs.
1,000 each. Each bond is convertible into 3 ordinary shares at any time prior to maturity
date of 31 October 2019. On inception the liability component was calculated as Rs. 760
million. On the date of issue, the prevailing interest rate for similar debt without conversion
option was 9% per annum.
 50% of these bonds were converted into ordinary shares on 1 November 2017.
Warrants
On 1 January 2016, TL issued share warrants giving the holders right to buy 6 million ordinary
shares at Rs. 340 per share. The warrants are exercisable within a period of 2 years.
Applicable tax rate is 30%.
Required:
Compute basic and diluted earnings per share to be disclosed in statement of profit or loss for
the following periods:
a) Quarter ended 31 December 2017
b) Half year ended 31 December 2017
(Show all relevant workings)

© Emile Woolf International 95 The Institute of Chartered Accountants of Pakistan


Advanced accounting and financial reporting

CHAPTER 21 - IAS 36: IMPAIRMENT OF ASSETS


21.1. CHARLOTTE
Charlotte Ltd is a company with a 31 December year-end.
The following is relevant to three tangible non-current assets held by Charlotte.
Machine 1
This was purchased on 1 January Year 1 for Rs. 420,000. It had an estimated residual value of
Rs. 50,000 and a useful life of ten years and was being depreciated on a straight-line basis.
On 1 January Year 6 Charlotte revalued this machine to Rs. 275,000 and reassessed its total
useful life as fifteen years with no residual value.
On 1 January Year 7 an impairment review showed machine 1’s recoverable amount to be Rs.
100,000 and its remaining useful life to be five years.
Machine 2
This was purchased on 1 January Year 1 for Rs. 500,000. It had an estimated residual value of
Rs. 60,000 and a useful life of ten years and was being depreciated on a straight-line basis.
On 1 January Year 7 this machine was classified as held for sale, at which time its fair value was
estimated at Rs. 200,000 and costs to sell at Rs. 5,000. On 31 March Year 7 the machine was
sold for Rs. 210,000.
Machine 3
This was purchased on 1 January Year 1 for Rs. 600,000. In Year 1 depreciation of Rs. 30,000
was charged. On 1 January Year 2 this machine was revalued to Rs. 800,000 and its remaining
useful life assessed as eight years.
On 1 January Year 7 this machine was classified as held for sale, at which time, its fair value was
estimated at Rs. 550,000 and costs to sell at Rs. 5,000.
On 31 March Year 7 the machine was sold for Rs. 550,000.
Tax is at the rate of 30%.
Required
Show the effect of the above on profit or loss and revaluation reserve of Charlotte in Year 7.
21.2. ABA LIMITED
Aba Limited conducts its activities from two properties, a head office in the city centre and a
property in the countryside where staff training is conducted. Both properties were acquired on 1
April 2013 and had estimated lives of 25 years with no residual value. The company has a policy
of carrying its land and buildings at current values. However, until recently property prices had
not changed for some years. On 1 October 2015 the properties were revalued by a firm of
surveyors. Details of this and the original costs are:

Land Buildings

Rs. Rs.
Head office – cost 1 April 2013 500,000 1,200,000
– revalued 1 October 2015 700,000 1,350,000
Training premises – cost 1 April 2013 300,000 900,000
– revalued 1 October 2015 350,000 600,000

The fall in the value of the training premises is due mainly to damage done by the use of heavy
equipment during training. The surveyors have also reported that the expected life of the training
property in its current use will only be a further 10 years from the date of valuation. The estimated
life of the head office remained unaltered.

© Emile Woolf International 96 The Institute of Chartered Accountants of Pakistan


Questions

Note: Aba Limited treats its land and its buildings as separate assets. Depreciation is based on
the straight-line method from the date of purchase or subsequent revaluation.
Required
Prepare extracts of the financial statements of Aba Limited in respect of the above properties for
the year to 31 March 2016.
21.3. HUSSAIN ASSOCIATES LTD
The assistant financial controller of the Hussain Associates Ltd group has identified the matters
below which she believes may indicate impairment of one or more assets:
(a) Hussain Associates Ltd owns and operates an item of plant that cost Rs. 640,000 and had
accumulated depreciation of Rs. 400,000 at 1 October 2015. It is being depreciated at
12½% on cost.
On 1 April 2016 (exactly half way through the year) the plant was damaged when a factory
vehicle collided into it. Due to the unavailability of replacement parts, it is not possible to
repair the plant, but it still operates, albeit at a reduced capacity. It is also expected that as
a result of the damage the remaining life of the plant from the date of the damage will be
only two years.
Based on its reduced capacity, the estimated present value of the plant in use is Rs.
150,000. The plant has a current disposal value of Rs. 20,000 (which will be nil in two
years’ time), but Hussain Associates Ltd has been offered a trade-in value of Rs. 180,000
against a replacement machine which has a cost of Rs. 1 million (there would be no
disposal costs for the replaced plant). Hussain Associates Ltd is reluctant to replace the
plant as it is worried about the long-term demand for the product produced by the plant.
The trade-in value is only available if the plant is replaced.
Required
Prepare extracts from the statement of financial position and statement of profit or loss of
Hussain Associates Ltd in respect of the plant for the year ended 30 September 2016.
Your answer should explain how you arrived at your figures.
(b) On 1 April 2015 Hussain Associates Ltd acquired 100% of the share capital of Sparkle
Limited, whose only activity is the extraction and sale of spa water. Sparkle Limited had
been profitable since its acquisition, but bad publicity resulting from several consumers
becoming ill due to a contamination of the spa water supply in April 2016 has led to
unexpected losses in the last six months. The carrying amounts of Sparkle Limited’s
assets at 30 September 2016 are:

Rs.000

Brand (Sparkle Spring – see below) 7,000


Land containing spa 12,000
Purifying and bottling plant 8,000
Inventories 5,000

32,000

The source of the contamination was found and it has now ceased.
The company originally sold the bottled water under the brand name of ‘Sparkle Spring’,
but because of the contamination it has re-branded its bottled water as ‘Refresh’. After a
large advertising campaign, sales are now starting to recover and are approaching
previous levels. The value of the brand in the balance sheet is the depreciated amount of
the original brand name of ‘Sparkle Spring’.
The directors have acknowledged that Rs. 1.5 million will have to be spent in the first three
months of the next accounting period to upgrade the purifying and bottling plant.

© Emile Woolf International 97 The Institute of Chartered Accountants of Pakistan


Advanced accounting and financial reporting

Inventories contain some old ‘Sparkle Spring’ bottled water at a cost of Rs. 2 million; the
remaining inventories are labelled with the new brand ‘Refresh’. Samples of all the bottled
water have been tested by the health authority and have been passed as fit to sell. The old
bottled water will have to be relabelled at a cost of Rs. 250,000, but is then expected to be
sold at the normal selling price of (normal) cost plus 50%.
Based on the estimated future cash flows, the directors have estimated that the value in
use of Sparkle Limited at 30 September 2016, calculated according to the guidance in IAS
36, is Rs. 20 million. There is no reliable estimate of the fair value less costs to sell of
Sparkle Limited.
Required
Calculate the amounts at which the assets of Sparkle Limited should appear in the
consolidated statement of financial position of Hussain Associates Ltd at 30 September
2016. Your answer should explain how you arrived at your figures.
21.4. IMPS
A division of IMPS has the following non-current assets, which are stated at their carrying values
at 31 December Year 4:
Rs. m Rs. m

Goodwill 70

Property, plant and equipment:


Land and buildings 320
Plant and machinery 110
430
500
Because these assets are used to produce a specific product, it is possible to identify the cash
flows arising from their use. The management of IMPS believes that the value of these assets
may have become impaired, because a major competitor has developed a superior version of the
same product and, as a result, sales are expected to fall.
The following additional information is relevant:
Forecast cash inflows arising from the use of the assets are as follows:
Year ended 31 December:
Rs. m
Year 5 185
Year 6 160
Year 7 130
(i) The directors are of the opinion that the market would expect a pre-tax return of 12% on an
investment in an entity that manufactures a product of this type.
(ii) The land and buildings are carried at valuation. The surplus relating to the revaluation of
the land and buildings that remains in the revaluation reserve at 31 December Year 4 is
Rs. 65 million. All other non-current assets are carried at historical cost.
(iii) The goodwill does not have a market value. It is estimated that the land and buildings
could be sold for Rs. 270 million and the plant and machinery could be sold for Rs. 50
million, net of direct selling costs.
Required
(a) Calculate the impairment loss that will be recognised in the accounts of IMPS.
(b) Explain how this loss will be treated in the financial statements for the year ended 31
December Year 4.

© Emile Woolf International 98 The Institute of Chartered Accountants of Pakistan


Questions

21.5. GYO MOVERS LIMITED


On 1 July 2013, GYO Movers Limited (GML) acquired a business engaged in providing
transportation service and recognized goodwill of Rs. 10 million. The business operates three
different bus routes namely Green, Yellow and Orange. The business had been running
exceptionally well. However, during the year ended 30 June 2016 entrance of new competitors
has affected its performance.
GML considers each route as a separate Cash-Generating Unit (CGU). As on 30 June 2016,
following information is available in respect of each CGU:

Green Yellow Orange

Number of buses* 80 50 40

Expected remaining useful life (in years) 20 15 10

------------ Rs. in million ------------

Carrying amount of buses 225 150 95

Other assets - carrying value 400 350 100

- fair value Not available

Fair values less cost to sell of the CGU 500 450 250

Expected net cash flows per annum 70 60 50

*Assume that all buses are of same make and model.


Carrying amount of corporate assets used interchangeably by all segments are as follows:

Carrying amount Fair value


Particulars
------------ Rs. in millions ------------

Head office building 100 Not available

Computer network 55 46

Equipment 45 60

For impairment testing of each CGU, following quotations were obtained from three different
showrooms located in different cities.

Showroom- Showroom- Showroom-


Particulars 1 2 3

---------------- Rs. in million ----------------

Average sale price for each bus 2.52 2.62 2.50

Estimated transaction cost for disposal of each bus 0.05 0.20 0.10

Pre-tax discount rate of GML is 12%.


Required:
Prepare relevant extracts from the statement of financial position as at 30 June 2016 in
accordance with International Financial Reporting Standards.

© Emile Woolf International 99 The Institute of Chartered Accountants of Pakistan


Advanced accounting and financial reporting

21.6. KHYBER LTD


The following details relate to a cash generating unit (CGU) of Khyber Ltd (KL) as at 30 June
2017:

Carrying Fair value less


value cost to sell
----------- Rs. in million -----------
Building (revaluation model)* 22 21.7
Machinery (cost model) 15 16
Equipment (cost model) 19 Not measurable
License (cost model) 20 18
Investment property (fair value model) 22 22
Investment property (cost model) 8 Not measurable
Goodwill 3 Not measurable
Inventory at NRV 8 8

*Balance of surplus on revaluation of building as on 30 June 2017 amounted to Rs. 3 million.


Value in use and fair value less cost to sell of the CGU at 30 June 2017 were Rs. 100 million and
Rs. 95 million respectively.
Required:
Compute the amount of impairment and allocate it to individual assets. Also calculate the amount
to be charged to profit or loss account for the year ended 30 June 2017 under each of the
following independent situations:
i. There has been a significant decline in budgeted net cash flows of the CGU.
ii. KL decided to dispose of the CGU as a group in a single transaction and classified it as
‘Held for sale’. Carrying value of all individual assets have been remeasured in accordance
with the applicable IFRSs

© Emile Woolf International 100 The Institute of Chartered Accountants of Pakistan


Questions

CHAPTER 22 - IAS 40: INVESTMENT PROPERTY


22.1. VICTORIA
Victoria owns several properties and has a year end of 31 December. Wherever possible,
Victoria carries investment properties under the fair value model.
Property 1 was acquired on 1 January Year 1. It had a cost of Rs. 1 million, comprising Rs.
500,000 for land and Rs. 500,000 for buildings. The buildings have a useful life of 40 years.
Victoria uses this property as its head office.
Property 2 was acquired many years ago for Rs. 1.5 million for its investment potential. On 31
December Year 7 it had a fair value of Rs. 2.3 million. By 31 December Year 8 its fair value had
risen to Rs. 2.7 million. This property has a useful life of 40 years.
Property 3 was acquired on 30 June Year 2 for Rs. 2 million for its investment potential. The
directors believe that the fair value of this property was Rs. 3 million on 31 December Year 7 and
Rs. 3.5 million on 31 December Year 8. However, due to the specialised nature of this property,
these figures cannot be corroborated. This property has a useful life of 50 years.
Required
(a) For each of the above properties briefly state how it would be treated in the financial
statements of Victoria for the year ended 31 December Year 8, identifying any impact on
profit or loss.
(b) Produce an analysis of property, plant and equipment for Victoria for the year ended 31
December Year 8, showing each of the above properties separately.

© Emile Woolf International 101 The Institute of Chartered Accountants of Pakistan


Advanced accounting and financial reporting

CHAPTER 24 - FIRST TIME ADOPTION OF IFRS


24.1. IFRS 1
A company which has always prepared its Financial Statements to 31 December each year,
prepared its first IFRS Financial Statements for the year ended 31 December 2016. These
statements show comparative figures for the year ended 31 December 2015.
Required
(a) Identify the first IFRS reporting period and state the date of transition to IFRS.
(b) Present the procedures which must be followed in order to prepare the financial
statements for the year ended 31 December 2016.
(c) Identify the reconciliations which the company must include in its financial statements for
the year ended 31 December 2016.
(d) State the contents of a typical statement of changes in equity.

© Emile Woolf International 102 The Institute of Chartered Accountants of Pakistan


Questions

CHAPTER 25 - SPECIALISED FINANCIAL STATEMENTS


25.1. LATEEF BANK LIMITED
Lateef Bank Limited (LBL) is listed on Karachi and Lahore Stock Exchanges and has 150
branches including 10 overseas branches. The LBL’s lending to financial institutions as of
September 30, 2016 comprised of the following:
(i) Call money lending at year end amounted to Rs. 850 million (2015: Rs. 1,200 million).
The markup on these unsecured lendings ranged between 15% to 17% (2015: 10% to
12%) and they matured on various dates, in October 2016.
(ii) Short term lending on account of repurchase agreement (reverse repo) amounted to Rs.
2,100 million (2015: Rs. 2,850 million). These carried markup ranging from 9.5% to
13.2% (2015: 8% to 10.5%) and matured on various dates, in October 2016. These
were secured against Market Treasury Bills of Rs. 1,650 million (2015: Rs. 1,850 million)
and Pakistan Investment Bonds of Rs. 450 million (2015: Rs. 1,000 million). The market
value of these securities held as collateral, on September 30, 2016, amounted to
Rs. 2,250 million (2015: Rs. 2,930 million).
The above amounts include lendings in foreign currencies amounting to Rs. 110
million (2015: Rs. 150 million).
Required
Prepare a note on lendings to financial institutions for inclusion in LBL’s financial statements for
the year ended September 30, 2016 giving appropriate disclosures in accordance with the
guidelines issued by State Bank of Pakistan.
25.2. AL-AMIN BANK LIMITED
Al-Amin Bank Limited is listed on all the stock exchanges in Pakistan. At year end, the
total advances amounted to Rs 75,000 million which include non-performing advances of
Rs. 5,000 million. The break-up of the non-performing advances and the provisions there-against
is as under:
2016
Classified Provisions
Category of classification
lending held
Rs. in million
Other assets especially mentioned 100 5
Sub-standard 660 120
Doubtful 840 530
Loss 3,400 3,345
5,000 4,000
The sub-standard category includes advances of Rs. 260 million pertaining to overseas
operations of the bank. The required provision of Rs. 50 million has been made against such
advances.
During the year the movement in the specific provision was as under:
Rs. in million
Opening balance 3,320
Charge for the year 802
Reversals (90)
Amounts written off (50)
Exchange rate adjustment 18
Total 4,000

© Emile Woolf International 103 The Institute of Chartered Accountants of Pakistan


Advanced accounting and financial reporting

In addition to the above specific provisions, it is the bank’s policy to make additional
general provision based on the judgment of the bank. The opening balance for general provision
was Rs. 65 million. During the year, the bank made provisions of Rs. 25 million and Rs. 15 million
against consumer and agriculture advances respectively.
Required
Prepare relevant notes on non-performing advances and provisions for inclusion in the
financial statements of Al-Amin Bank Limited giving appropriate disclosure in accordance with the
guidelines issued by the State Bank of Pakistan.
25.3. IAS 26
IAS 26: Accounting and Reporting by Retirement Benefit Plans and IAS 19: Employee Benefits
deal with employee benefits but there are differences between the two standards.
(a) Highlight the main differences between IAS 26 and IAS 19.
(b) What is a Defined Benefit Plan?
(c) What is a Defined Contribution Plan?
(d) Explain the meaning of the actuarial present value of promised retirement benefit.
25.4. SOGO LIMITED
SOGO Limited operates an approved funded gratuity scheme for all its employees.
Benefits under the scheme become vested after 5 years of service. No benefit is payable to an
employee if he leaves before 5 years of service. A total of 752 employees were eligible for the
benefits under the fund as of December 31, 2016.
Following is the trial balance of the Fund as of June 30, 2016:
Debit Credit
Amounts in Rupees
Cash at bank - current account 17,930,120
Receivable from SOGO Limited 1,147,150
Defence Savings Certificate 102,133,664
Term Finance Certificates 11,832,089
Term Deposits 6,414,058
Investment – SUN Limited 17,594,893
Investment – PEACE Company Limited 587,169
Investment - NIT Units 16,911,510
Due to outgoing members 4,301,017
Accrued expenses 3,822
Withholding tax payable 61,251
Members Fund 142,472,122
Profit on investments 23,389,251
Dividend income 2,696,399
Contribution for the year 10,623,106
Transferred / paid to outgoing members 12,432,973
Bank charges 3,342
Audit fee 10,000
Liabilities no more payable 3,450,000
186,996,968 186,996,968

© Emile Woolf International 104 The Institute of Chartered Accountants of Pakistan


Questions

Following are the details of investments and income thereon:

During the year 2016


Balance as
at July 01, Profit / Profit /
Principal
2015 Addition interest interest
realized
accrued realized
Government
Securities
Defence Savings
-
Certificate 87,812,855 21,376,809 (1,600,000) (5,456,000)
Unlisted Securities
and deposits
Term Finance (12,873,068
19,943,656 5,000,000 1,655,223 (1,893,722)
Certificates )
Term Deposits 11,584,631 - 357,219 (5,300,000) (227,792)
Listed Securities
SUN Limited 8,220,957 9,373,936 - - -
PEACE Limited 587,169 - - - -
NIT Units 16,911,510 - - - -

The following gains/(losses) on restatement of investments at their fair values, have not been
accounted for:

Rupees
SUN Limited (784,518)
PEACE Limited 317,728
NIT Units 4,026,551

Required
Prepare the following in accordance with the requirements of International Accounting
Standards:
(a) Statement of net assets available for benefits along with the note on investments.
(b) Statement of changes in net assets available for benefits.
25.5. JABBAR (PVT) LIMITED
Jabbar (Pvt) Ltd (JPL) was incorporated on 1 July 2016 and is preparing its financial statements
for the year ended 30 June 2017 in accordance with IFRS for Small and Medium-sized Entities
(SMEs). The following matters are under consideration:
(i) JPL has constructed an office building at a cost of Rs. 3.3 million, which was completed on
30 June 2017. The cost includes interest of Rs. 0.3 million relating to a loan specifically
obtained to finance the construction. At year end, recoverable amount of the building has
been estimated at Rs. 3.1 million.
(ii) On 1 January 2017 JPL had purchased two shops A and B for Rs. 5 million and Rs. 4
million respectively. Shop A is being used by JPL for marketing purposes and shop B was
rented out soon after its purchase. At year end, shops A and B have been:
 depreciated @ 5% per annum.
 revalued to Rs. 6 million and Rs. 5 million respectively.
Required:
Discuss how the above matters should be dealt with in the financial statements of JPL in
accordance with IFRS for SMEs. (Assume that cost to sell is negligible)

© Emile Woolf International 105 The Institute of Chartered Accountants of Pakistan


Advanced accounting and financial reporting

25.6. KARACHI BANK LIMITED


Following balances have been extracted from the trial balance of Karachi Bank Limited (KBL) for
the year ended 31 December 2016.

Rs. in million
Bills discounted and purchased 679
Call money lending 650
Cash in hand 9,100
Current account with Habib Bank Limited 412
Current account with State Bank of Pakistan 14,500
Current account with National Bank of Pakistan 2,300
Deposit account with Central Bank of Afghanistan 700
Deposit account with National Bank of Pakistan 1,400
Deposit account with United Bank Limited 311
Deposits and prepayments 3,189
Interest accrued 21,450
Loans, cash credits and running finances 114,200
Market treasury bills 24,500
National Prize Bonds 68
Net investment in finance lease 4,900
Operating fixed assets 24,700
Pakistan Investment Bonds (20% given as collateral) 1,800
Provision against non-performing advances (6,678)
Provision for diminution in value of investment (222)
Repurchase agreement lending 6,100
Sukuk Bonds 1,200

Required:
Prepare the asset side of the statement of financial position as at 31 December 2016 of KBL,
based on the above balances. (Notes to the financial statements are not required)
25.7. LEOPARD INCOME FUND
Following information is available from the records of Leopard Income Fund (an open ended
mutual fund) for the year ended 30 June 20X8:
i. Undistributed income as at 1 July 20X7 comprised of realised and unrealised income of
Rs. 97 million and Rs. 7 million respectively.
ii. Total net assets at 1 July 20X7 amounted to Rs. 9,752 million.
iii. Allocation of net income for the year is as follows:

Rs. in million
Total comprehensive income 214
Income already paid on units redeemed (50)
164

© Emile Woolf International 106 The Institute of Chartered Accountants of Pakistan


Questions

iv. Accounting income available for distribution relating to capital gains and other than capital
gains amounts to Rs. 3 million and Rs. 161 million respectively.
v. Distribution during the year amounted to Rs. 150 million.
vi. Details of issuance and redemption of units during the year are as follows:

Issuance Redemption
Units in million 388 441
----- Rs. in million -----
Capital value 7,372 (8,382)
Element of income /(loss) 70 (64)*
7,442 (8,446)

*including Rs. 50 million of income already paid on units redeemed


vii. Unrealized loss included in undistributed income as at 30 June 20X8 amounted to Rs. 4
million.
Required:
Prepare a statement of movement in unit holders’ fund for the year ended 30 June 20X8. (Ignore
disclosure of comparative figures and net assets value per unit)

© Emile Woolf International 107 The Institute of Chartered Accountants of Pakistan


Advanced accounting and financial reporting

CHAPTER 28 - ISLAMIC ACCOUNTING STANDARDS


28.1 SALE AND LEASE BACK TRANSACTIONS
In the light of Islamic Financial Accounting Standards issued by the Institute of Chartered
Accountants of Pakistan, discuss how the gain/loss on ‘sale and lease back transactions’ shall be
accounted for in the financial statements of a listed company if an asset is sold at:
 fair value
 above fair value
 below fair value

© Emile Woolf International 108 The Institute of Chartered Accountants of Pakistan


Certified Finance and Accounting Professional

B
Advanced accounting and financial reporting

SECTION
Answers

CHAPTER 1 - BUSINESS COMBINATIONS AND CONSOLIDATION


1.1. HELLO
Consolidated statement of financial position as at 31 December 2016
Rs.
Assets
Non-current assets
Property, plant and equipment (225 + 175 + 10 – 2) 408,000
Goodwill (W3) 8,000

Current assets (271 + 157) 428,000


———–
844,000
———–
Equity and liabilities
Shareholders’ equity
Called up share capital 100,000
Retained earnings (W5) 291,800
———–
391,800
Non-controlling interest (W4) 79,200
Current liabilities 373,000
———–
844,000
———–
WORKINGS
(1) Group structure
Hello

60%

Solong

© Emile Woolf International 109 The Institute of Chartered Accountants of Pakistan


Advanced accounting and financial reporting

(2) Net assets of Solong Inc


Reporting Date of Post-
date acquisition acquisition
Rs. Rs.
Share capital 100,000 100,000
Retained earnings
Per the question 90,000
Less: Fair value adjustment for depreciation
(2/10 × 10,000) (2,000)
88,000 60,000
Fair value adjustment 10,000 10,000
198,000 170,000
(3) Goodwill Rs.
Cost 110,000
Net assets acquired
60%  170,000 (W2) (102,000)
————
8,000
————

(4) Non-controlling interest Rs.


40%  198,000 (W2) 79,200

(5) Retained earnings Rs.


Hello 275,000
Solong (60%  (88,000 – 60,000 (W2)) 16,800
————–
291,800
————–

1.2. HASAN LIMITED


Hasan Limited
Consolidated statement of financial position as at 31 March 2016
Rs.000 Rs.000
Assets
Non-current assets
Property, plant and equipment (W1) 4,020
Goodwill (W4) 480
Software (W1) 1,440
Investments (65 + 210) 275
–––––––––––––
6,215
Current assets
Inventories (W2) 1,274
Trade receivables (524 + 328) 852
Cash and bank (20 + 55 cash in transit) 75
–––––––––––––
2,201
–––––––––––––

Total assets 8,416


–––––––––––––

© Emile Woolf International 110 The Institute of Chartered Accountants of Pakistan


Answers

Rs.000 Rs.000
Equity and liabilities
Capital and reserves
Equity capital 2,000
Reserves
Share premium 2,000
Retained earnings (W3) 2,420
–––––––––––––
4,420
–––––––––––––

6,420
Non-controlling interest (W5) 350

Government grants (230 + 40) 270


Current liabilities
Trade payables (475 + 472) 947
Operating overdraft 27
Income tax liability (228 + 174) 402
–––––––––––––
1,376
–––––––––––––

Total equity and liabilities 8,416


–––––––––––––

Workings
(W1) Property, plant and equipment

Rs.000
Balance from question – Hasan Limited 2,120
Balance from question – Shakeel Limited 1,990
Fair value adjustment on acquisition (see below) (120)
Over-depreciation re fair value adjustment year to 31 March 2016 30
–––––––––––––

4,020
–––––––––––––

A fair value of the leasehold based on the present value of the future rentals (receivable in
advance) would be the next (non-discounted) payment of the rental plus the final three
years as an annuity at 10%:

Rs.000
PV of rental receipts: Rs. 80,000 + (Rs. 80,000  2.50) 280
Carrying value on acquisition is (400)
–––––––––––––

Fair value reduction of leasehold (120)


–––––––––––––

The depreciation of the leasehold in Shakeel Limited’s accounts would be Rs. 100,000 per
annum. However in the consolidated accounts it should be Rs. 70,000 (Rs. 280,000/4).
This would require a reduction in depreciation of Rs. 30,000 in the consolidated accounts
for the next four years.

© Emile Woolf International 111 The Institute of Chartered Accountants of Pakistan


Advanced accounting and financial reporting

Software:
Shakeel Consolidated Difference
Limited’s figures
accounts
Rs.000 Rs.000
Capitalised amount 2,400 2,400
Depreciation to
31 March 2015 (300) 8 year life (480) 5 year life
––––– –––––
Value at date of 180 fair
acquisition 2,100 1,920 value adjustment
Depreciation to 180 additional
31 March 2016 (300) (480) amortisation
––––– –––––
Carrying value
31 March 2016 1,800 1,440
––––– –––––
(W2) Inventories
Rs.000
Amounts given in the question (719 + 560) 1,279
Unrealised profit in inventories (25  25/125) (5)
–––––––––––––

1,274
–––––––––––––

(W3) Retained earnings


Rs.000
Retained profits of Shakeel Limited, 31 March 2016 1,955
Adjustments:
Excess charge for leasehold depreciation 30
Insufficient charge for Software amortisation (180)
Unrealised profit in inventory (W2) (5)
–––––––––––––

Adjusted retained profits at 31 March 2016 1,800


Retained earnings of Shakeel Limited at 1 April 2015 2,200
–––––––––––––

Shakeel Limited: loss for the year (post-acquisition loss) (400)


–––––––––––––

Rs.000
Parent company share of post-acquisition loss (90%) (360)
Hasan Limited reserves at 31 March 2016 2,900
Goodwill impairment (120)
–––––––––––––

Consolidated retained profits at 31 March 2016 2,420


–––––––––––––

(W4) Goodwill
Rs.000
At acquisition date
Shares of Shakeel Limited 1,500
Share premium of Shakeel Limited 500
Retained earnings of Shakeel Limited 2,200

© Emile Woolf International 112 The Institute of Chartered Accountants of Pakistan


Answers

Rs.000
Fair value adjustments:
Leasehold (W1) (120)
Software (W1) (180)
–––––––––––––

3,900
–––––––––––––

Acquired by Hasan Limited (90%) 3,510


Cost of investment 4,110
–––––––––––––

Goodwill at acquisition 600


Impairment 120
–––––––––––––

Goodwill at 31 March 2016 480


–––––––––––––

(W5) Non-controlling interests


Rs.000
Share capital of Shakeel Limited 1,500
Share premium of Shakeel Limited 500
Adjusted retained earnings of Shakeel Limited, 31 March 2016 (W3) 1,800
Fair value adjustments:
Leasehold (120)
Software (180)
–––––––––––––

Total net assets at 31 March 2016 3,500


–––––––––––––

Non-controlling interests (10%) 350


–––––––––––––

(W6) Elimination of current accounts:


Rs.000
Shakeel Limited’s current account with Hasan Limited per question 75
Deduct cash in transit regarding this balance (15)
–––––––––––––

Adjusted figure to cancel 60


–––––––––––––

(W7) Elimination of intra-group loan:


Rs.000
Investment in Hasan Limited’s books 200
Deduct repayment in transit (40)
–––––––––––––

Non-current liability in Shakeel Limited’s books 160


–––––––––––––

1.3. FLAMSTEED LTD AND HALLEY LTD


(a) An impairment loss is the amount by which the carrying amount of an asset or a cash
generating unit exceeds its recoverable amount.
(b) The following external sources of information may indicate that an asset is impaired.
(i) There are observable indications that the assets value has declined during the
period significantly more than would be expected as a result of the passage of time
or normal use.
(ii) Significant changes with an adverse effect on the entity have taken place during the
period or will take place in the near future, in the technological, market, economic or
legal environment in which the entity operates or in the market to which the asset is
dedicated.

© Emile Woolf International 113 The Institute of Chartered Accountants of Pakistan


Advanced accounting and financial reporting

(iii) The carrying amount of the net asset of the entity is more than its market
capitalization.
(iv) The carrying amount of the investment in the separate financial statements exceeds
the carrying amount in the consolidated financial statement of the investee’s net
asset, including associated goodwill, or the dividend exceeds the total
comprehensive income of the subsidiary, joint venture or associates in the period
the dividend is declared.
(v) Market interest rate or other market rate of return on investment have increased
during the period and those increases are likely to affect the discount rate used in
calculating the asset value in use and decrease the assets recoverable amount
materially.
(c) Flamsteed Ltd group: extract of consolidated statement of financial position as at 30 June
2016
Rs.‘000
Assets
Non-Current Assets
Property, Plant and Equipment (100,000 + 80,000) 180,000
Goodwill (WK) 13,468
Intangible-brand name 10,000
203,468
Current Assets
Inventory (6,000 + 16,000) 22,000
Receivables (32,000 + 14,000) 46,000
Cash (4,000 + 0 + 4,000) 8,000
76,000

Total Assets 279,468


Workings
(i) Goodwill
Rs.‘000 Rs.‘000
Consideration transferred 77,468
Fair value of NCI 18,000
Net Asset acquired as represented by: 95,468
Ordinary share capital 50,000
Revaluation surplus on acquisition 10,000
Retained earnings on acquisition 12,000
Intangible assets (brand name) 10,000 (82,000)
13,468
Note
The deferred consideration has been discounted at 7% for 2 years (1 st July 2015 – 1st July
2017).

1.4. BRADLEY LTD


Consolidated statement of financial position as at 31 December 2016
Rs.’Million Rs.’Million
Non-current assets
Goodwill (working 1) 120
Land & building (630 + 556 + 140) 1,326
Machinery & equipment (570 + 440) 1,010
2,456

© Emile Woolf International 114 The Institute of Chartered Accountants of Pakistan


Answers

Rs.’Million Rs.’Million
Current assets
Inventory (714 +504 – 24) 1,194
Trade receivables (1,050 + 252 – 50) 1,252
Cash/Bank (316 + 60) 376 2,822
5,278
Ordinary shares of Rs. 1 each 3,000.0
Retained earnings (Working 3) 1,323.2
Non-controlling Int. (Working 4) 376.8
4,700
Current liabilities
Trade payables (440 + 188 - 50) 578
5,278
Workings:
Rs. million
1. Calculation of goodwill:
Fair value of consideration 1,320
Plus fair value of NCI at acquisition 330
Less net acquisition – fair value of
Assets acquired & liability:
Share capital 1,200
Retained Earning 190
Fair value adj at acquisition 140 (1,530)
Goodwill 120

2. Group structure
960 million
 100
1,200 million 80%
3. Retained earnings:
As per question 1,160 424
Adjustment (unrealised profit) (24)
Pre-acquisition retained earnings (190)
234
Group share of post-acquisition retained
earnings:
(80% x 234) 187.2
1,323.2
4. Non-controlling interest: Rs. million
Fair value of NCI at acquisition 330.0
Plus NCI’s share of post-acquisition
retained earnings (20% x 234) 46.8
376.8
Alternative workings:
(W1) Fair value adjustment:
Dr: Consolidated land & building Rs. 140 million
Cr: Revaluation reserve Rs. 140 million

© Emile Woolf International 115 The Institute of Chartered Accountants of Pakistan


Advanced accounting and financial reporting

(W2) Consolidation Schedule


Bliss Ltd Rs. ’M Bradley in NCI Post –
Bliss 80% 40% Acq
(W3)
Rupees in millions
Ordinary share capital 1,200 960 240
Revaluation Res. (W1) 140 112 28
Retained earnings 424 152 84.8 187
────
Net assets acquired 1,224
Cost of acquisition (1,320)
Goodwill (partial value) (96)
Goodwill attribute to NCI (W5) (24) 24
────
Goodwill (fair value) ( 120)
────
Unrealised profit on inventory - (24)
NCI (fair value) 376.8
────
Retained earnings of Bradley Ltd 1,160
Consolidated retained earnings 1,323
960𝑚 𝑠ℎ𝑎𝑟𝑒𝑠
(W3) Bradley in Bliss = × 100 = 80%
1,200𝑚 𝑠ℎ𝑎𝑟𝑒𝑠

(W4) Bradley’s share of Bliss’s pre-acquisition retained earnings = 80% x


= 80% x Rs. 190m = Rs. 152m
(W5) Goodwill attribute to NCI Rs. m
Fair value of NCI @ date of acquisition 330
Less: fair value of net assets attributable to
NCI (20% x (1,200 + 140 + Rs. 190m)) 306
Goodwill attributable to NCI 24

1.5. X LTD
Consolidated statement of financial position as at 31 December 2016
for the X Ltd Group
All workings in Rs.000
ASSETS Rs.000
Non-current assets
Property, plant and equipment (12,000 + 4,000 + 750(W1)) 16,750
Goodwill (W2) 208
Intangible asset (W1) 90
17,048
Current assets
Inventories (2,200 + 800 -30 (W3)) 2,970
Receivables (3,400 + 900) 4,300
Cash and cash equivalents (800 + 300) 1,100
8,370
Total assets 25,418

© Emile Woolf International 116 The Institute of Chartered Accountants of Pakistan


Answers

EQUITY AND LIABILITIES Rs.000


Equity
Share capital (Rs. 1 equity shares) 10,000
Retained earnings (W4) 7,893
Total equity attributable to parent 17,893
Non-controlling interest (W5) 1,741
Total equity 19,634
Non-current liabilities
Long term borrowings 2,700
Current liabilities (2,000 + 1,000 + 84) 3,084
Total liabilities 5,784
Total equity and liabilities 25,418

Workings
1. Fair value adjustments

At acquisition date Movement 31 Dec 2016


Rs.000 Rs.000 Rs.000
PPE 800 (50) 750
Inventories 200 (200) -
Intangible assets 150 (60) 90
Liabilities (210) 126 (84)
──── ──── ────
940 (184) 756
──── ──── ────
2. Goodwill

Rs.000 Rs.000
Consideration transferred 3,800
NCI at fair value 1,600
────
5,400
Net assets at fair value:
Share capital 1,000
Retained earnings 3,200
Fair value adjustments 940
────
(5,140)
────
Goodwill on acquisition 260
20% impairment (52)
────
Goodwill at 31 December 2016 208
────

© Emile Woolf International 117 The Institute of Chartered Accountants of Pakistan


Advanced accounting and financial reporting

3. Unrealised profit on inventories


Sales of Rs. 300k x 20% x 50% left in inventories at y/e = Rs. 30,000
4. Retained earnings
Rs.000 Rs.000
As per SOFP 7,500 4,000
Pre-acquisition reserves (3,200)
Adjustments arising from movement in
FV adjustments (184)
────
616
────
Group share 75% 462
Unrealised profit on inventory transfer (30)
Goodwill impairment (75% x 52)(W2) (39)
────
Consolidated reserves 7,893
────
5. Non-controlling interests
Rs.000
NCI at acquisition (at fair value) 1,600
25% x post-acquisition retained earnings Rs. 616,000 (W4) 154
Goodwill impairment (25% x 52)(W2) (13)
─────
1,741
─────

1.6. KHAN LIMITED


(a) Khan Limited
Consolidated statement of financial position as at December 31, 2016
Rupees in
ASSETS million
Non-current assets
Property plant & equipment (W- 1) 14,800
Goodwill (W - 2) 100
14,900

Current assets (1,069+ 1,316) 2,385

17,285

© Emile Woolf International 118 The Institute of Chartered Accountants of Pakistan


Answers

Rupees in
EQUITY AND LIABILITIES million
Equity
Share capital 6,800
General reserve (W5) 1,975
Retained earnings 3,844
12,619
Non-controlling interest (W8) 2,420

Total equity 15,039

Non-current liabilities
14% Term finance certificates (2,250-1,500) 750

Current liabilities
Accounts payable (445 + 190) 635
Dividend payable (W3) 861
17,285

(b) Khan Limited


Consolidated statement of profit or loss and other comprehensive income for the
year ended December 31, 2016
Rupees in
million
Profit before tax and interest (W4) 4,315
Interest expense (315 - 210) (105)

Profit before tax 4,210


Taxation expense (650 + 474) (1,124)

Profit for the period 3,086


Other comprehensive income -
Total comprehensive income 3,086

Attributable to:
Equity holders of parent Balancing 2,894
Non-controlling interest (W-7) 192
3,086

© Emile Woolf International 119 The Institute of Chartered Accountants of Pakistan


Advanced accounting and financial reporting

(c) Khan Limited


Consolidated statement of retained earnings for the year ended December 31, 2016
Rupees in
million
Balance as at January 1, 2016 (W- 6) 1,700
Total comprehensive income for the year 2016 2,894
Dividends (750)
Balance as at December 31, 2016 3,844

W1 – Property, plant & equipment


Rupees in
million
Cost – KL 16,250
Cost – GL 25,000
Acc. depreciation – KL (9,750)
Acc. depreciation – GL (17,000)
14,500
Fair value adjustment 1,000
Less: Depreciation on increased fair
value (Rs. 1,000 x 10% x 7) (700)

14,800

W2 – Goodwill
Purchase consideration 5,500
Less:
Share capital (75% of 5,000) (3,750)
Retained earnings (75% of 1,000) (750)
General reserve (75% of 200) (150)
FV increase in PPE (1,000 x 75%) (750)
100

W3 - Dividend payable
Ordinary dividend – KL 750
Ordinary dividend - GL (300 x 25%) 75
Preference dividend - GL (60 x 60%) 36
861

© Emile Woolf International 120 The Institute of Chartered Accountants of Pakistan


Answers

Rupees in
million
W4: Profit before tax and interest
KL 2,865
GL 1,550
Current year depreciation on increased value of PPE (1,000 x 10%) (100)
4,315
W5: General reserve
General reserve – KL 1,750
General reserve – GL (500 – 200) x 75% 225
1,975
W6: Retained earnings
Retained earnings – KL 2,000
Retained earnings – GL (1,200 - 1,000) x 75% 150
Less: Depreciation charge on increased FV (1,000 x 6 x 10% x 75%) (450)
1,700
W7: Non-controlling interest (for statement of comprehensive income)
Share from profit of GL (1,550+210-300-474—120) x 25% 217
Less: Current year depreciation on
increased of PPE (100 x 25%) (25)
192
W8: Non-controlling interest (for statement of financial position)
Share capital (5,000 x 25%) 1,250
Preference shares (1,000 x 60%) 600
General reserve (500 x 25%) 125
Opening retained earnings (1,200 x 25%) 300
Comprehensive income for the year (W-7) 192
Increase in FV of building (1,000 x 25%) 250
Less: Depreciation charge on increased FV (1,000 x 6 x 10% x 25%) (150)
Less: Dividend on ordinary shares (300 x 0.25) (75)
Less: Dividend on preference shares (120 x 0.6) (72)
2,420

© Emile Woolf International 121 The Institute of Chartered Accountants of Pakistan


Advanced accounting and financial reporting

1.7. WHITE LIMITED


White Limited
Consolidated Statement of Finance Position
As on 31 December 2016

Assets Rs. in million

Goodwill [56.7(W-1)×20] 1,134.00

Property, plant and equipment [14,900+3,000+(325×20)+800] 25,200.00

Investment property (800–800) -

Current assets [6,660+2,500+(305×20)] 15,260.00

Total assets 41,594.00

Equity & liabilities

Share capital 11,400.00

Retained earnings (W-2) 15,089.65

Exchange reserves (W-5) 1,270.65

27,760.30

Non controlling interest (W-3) 1,091.20

28,851.50

Revaluation surplus [150+650÷20] 182.50

Current liabilities [6,360+2,300+(195×20)] 12,560.00

Total equity and liabilities 41,594.00

W-1: Goodwill GL YL

Rs. in million T $ in million

Cash payment 4,200.00 243.00


(270×90%)

Fair value of previously held equity (4.5×23) 103.50

Total cash consideration and NCI 4,200.00 346.50

Less : Fair value of net assets acquired

Share capital 1,500.00 225.00

Retained earnings 3,500.00 90.00

5,000.00 315.00

WL's share in net assets 4,500.00 289.80


(GL:5,000×90%),(YL:315×92%)

Bargain purchase/Goodwill (300.00) 56.70

© Emile Woolf International 122 The Institute of Chartered Accountants of Pakistan


Answers

W-2: Consolidated retained earnings Rs. in million


WL (Given) 9,500.00
Post acquisition - GL [(7,900–3500)×90% 3,960.00
Post acquisition - YL (2,148.75(W-2.1)×92%) 1,976.85
Bargain purchase (W-1) 300.00
Gain on derecognition of associate (W-4) 484.50 484.50
Reversal of exchange gain on investment in YL by WL [75×(20–17)] (225.00)
Reversal of exchange gain on investment in YL by GL (270×(20–17)×92% (745.20)
Elimination of income from investment property (800–650)×90% (135.00)
Depreciation expense to be booked for the year by GL (650÷20) (32.50)
Rent expense to be reversed in WL books 60.00
Rent income to be reversed in GL books (60×90%) (54.00)
15,089.65

W-2.1: Post acquisition profit of YL


Profit for nine month [{(210-90)120+(225×10%)}×18] 2,565.00
Less: 10% interim dividend (22.5×18.5) (416.25)
2,148.75

W-3: Non-controlling interest


At acquisition (GL : (1500+3500)×10%)+(YL(225+90)×8%×17) 928.40
Post acquisition - GL [(7,900–3500)×10% 440.00
Post acquisition - YL (2,148.75×8%) 171.90
Reversal of exchange gain on investment in YL by GL [270×(20-17) × 8%] (64.80)
Elimination of income from investment property (800–650 )×10% (15.00)
Rent income to be reversed in GL books (60×10%) (6.00)
Exchange gain relating to year-end transactions (W-5) 95.70
Indirect holding adjustment (270×10%×17) (459)
1,091.20
W-4: Gain on derecognition of associate
Fair value of investments on 1 April 2016 (4.5×23×17) 1,759.50
Less: Cost of investment (75×17) (1,275.00)

Gain 484.50

W-5: Exchange reserves

Conversion Rs.
Relating to goodwill T$
rate in million

Balance on acquisition date i.e. 1 April 2016 56.70 17.00 963.90


Balance as on 31 December 2016 56.70 20.00 1,134.00

170.10

© Emile Woolf International 123 The Institute of Chartered Accountants of Pakistan


Advanced accounting and financial reporting

Relating to translation of FS foreign Conversion Rs.


operations T$
rate in million

Net assets as on 31 December 2016 435.00 20.00 8,700.00

Net assets on acquisition date 315.00 17.00 5,355.00

Profit since acquisition {120 + (225×10%)} 142.50 18.00 2,565.00


Dividend paid (22.50) 18.50 (416.25)

435.00 7,503.75

Exchange gain for the year ended 31


December 2016 - 1,196.25

Less: Exchange gain relating to NCI


(1,196.25×8%) 95.70

Exchange gain to parent 1,100.55

Total exchange reserves (170.10+1,100.55) 1,270.65

© Emile Woolf International 124 The Institute of Chartered Accountants of Pakistan


Answers

CHAPTER 2 - BUSINESS COMBINATIONS ACHIEVED IN STAGES


2.1. STEP ACQUISITION
The profits of AS since the investment was acquired (all retained) are Rs. 40 million (= Rs. 300m
– Rs. 260m). During this period, H held 25% of the equity of AS and it is assumed that AS is an
associate. Profits attributable to H for the year are therefore Rs. 10 million (= 25%  Rs. 40
million).
(i) Total gain or profit attributable to the investment in AS

Rs. m

Initial investment in associate at cost 80

Share of post-investment retained profits 10

90

Fair value of investment at 30 June 95

Gain recognised when step acquisition occurs 5

The total gain/profit recognised for the year from the investment in AS is therefore Rs. 10
million + Rs. 5 million = Rs. 15 million.

(ii) Total goodwill on acquisition

Fair value of shares that gave control (40%) 160

Fair value of previous investment (25%) 95

255

Fair value of NCI at acquisition 120

375

Net assets of AS at 30 June 300

Total goodwill 75

(iii) Goodwill attributable to NCI

Fair value of investment in 25% of AS (35%  300) 105

Goodwill attributable to NCI (balancing figure) 15

Total NCI 120

2.2. A LTD
(a) Treatment of B Ltd
IFRS 3 Business combinations requires goodwill on acquisition to be calculated at the date
control is gained. The second acquisition gives A Ltd a 75% holding and therefore control
over B Ltd. The simple investment of 15% will be derecognised and the 75% holding will
be fully consolidated as a subsidiary in the group financial statements. The goodwill will be
calculated as the cost of the 60% acquired in the year plus the fair value of the previously
held interest of 15%, compared with the fair value of the net assets at the date of
acquisition (1 April 2016).

© Emile Woolf International 125 The Institute of Chartered Accountants of Pakistan


Advanced accounting and financial reporting

(b) Consolidated statement of financial position for the A Ltd Group as at 31 September
2016.
A Ltd
ASSETS Rs.000
Non-current assets
Property, plant and equipment (22,000+5,000) 27,000
Goodwill (W orking 1) 405
27,405
Current assets
Inventories (6,200+800– 40 (Working 2)) 6,960
Receivables (6,600+1,900) 8,500
Cash and cash equivalents (1,200+300) 1,500
16,960
Total assets 44,365

EQUITY AND LIABILITIES


Equity
Share capital (Rs. 1 equity shares) 20,000
Retained earnings (W orking 3) 8,629
Other components of equity (W orking 6)
28,629
Non-controlling interest (Working 4) 1,604
Total equity 30,233

Non-current liabilities
5% Bonds 2015 (Working 5) 4,032
Current liabilities (8,100+2,000) 10,100
Total liabilities 14,132
Total equity and liabilities 44,365

Working 1 Goodwill Rs.000 Rs.000


Consideration transferred for the 60% 2,900
Fair value of 15% holding at 1 April 2016 800
Fair value of non-controlling interest 1,250
4,950
Net assets acquired:
Share capital 1,000
Retained earnings (5,000- 1,500) 3,500 (4,500)
450
Impaired by 10% (45)
Net value of goodwill 405

Working 2 Unrealised profit on inventories Rs.000


Sales from B Ltd to A Ltd 400
50% in inventories 200
Profit margin 20% - adjust Inventories and retained 40
earnings of B Ltd

© Emile Woolf International 126 The Institute of Chartered Accountants of Pakistan


Answers

Working 3 Retained earnings A Ltd B Ltd

Rs.000 Rs.000

As at 30 September 2016 7,500 5,000

Pre-acquisition (5,000 – (3,000 x 6/12)) (3,500)

Less unrealised profit of B Ltd (working 2) (40)

1,460

Group share 75% 1,095

Group share of impairment (75% x 45) (34)

Additional finance costs on bonds (working 5) (132)

Group profit on derecognition of AFS Investment


– gain to date of deemed disposal 1 April 2016 (800 - 600) 200

Consolidated retained reserves 8,629

Working 4 Non-controlling interest Rs.000

Fair value at 1 April 2016 1,250

Plus 25% adjusted post-acquisition reserves 1,460 (working 3) 365

Less NCI share of goodwill impairment (25% x 45) (11)

NCI at 30 September 2016 1,604

Working 5 Bonds – amortised


Rs.000 Rs.000 Rs.000 Rs.000
cost

Opening Effective Interest paid Value at 30


value rate 8.5% 5% x Rs. 4m September

To 30 September 2016 3,900 332 (200) 4,032

The difference of Rs. 132,000 must be added to the value of the bond liability and
deducted from
A Ltd’s retained earnings.
Working 6 Other reserves and AFS investment
IFRS 3 requires that the 15% simple investment be derecognised and on derecognition any
gain/loss would be considered realised. The gain of Rs. 200,000 (FV of Rs. 800,000 at
date of derecognition less the investment cost of Rs. 600,000) represents the group gain
and will be included in the consolidated reserves.
The balance on other reserves again relates to the treatment of the investment in the
parent’s own accounts and the gains on the AFS investment (B Ltd) and not relevant for
the group accounts – as the B Ltd has been fully consolidated.

© Emile Woolf International 127 The Institute of Chartered Accountants of Pakistan


Advanced accounting and financial reporting

2.3. X LTD GROUP


(a) (i) X Ltd group: Consolidated statement of profit or loss and other
comprehensive income for the year ended 31 December 2016

Rs.000
Revenue (1,200 + 290) 1,490
Cost of sales (810 + 110 + 4 (W1)) (924)
Gross profit 566
Operating expenses (100 + 40 + 9 (W2)) (149)
417
Investment income
(50 – intra group dividend 40 (80% x 50)) 10
Finance costs (45 + 10) (55)
Share of associate’s profit (40% x 30) 12
Profit before tax 384
Income tax expense (80 + 30) (110)
Profit for the year 274
Other comprehensive income
Revaluation of property, net of tax (60 + 20) 80
Share of associate’s OCI (40% x 10) 4
Other comprehensive income for the year, net of tax 84
Total comprehensive income 358

Profit for the year attributable to:


Owners of the parent (274 – 17 (W3)) 257
Non-controlling interest 17
274
Total comprehensive income attributable to:
Owners of the parent (358 – 21 (W3)) 337
Non-controlling interest 21
358
(ii) Consolidated statement of changes in equity for the year ended 31 December
2016
X Ltd NCI Total
group
Rs.000 Rs.000 Rs.000
Equity at 1 January 2016 (W4)/(W5) 1,868 216 2,084
Total comprehensive income for the year 337 21 358
Dividends (100) (100)
Dividend paid to NCI (20% x 50) (10) (10)
Equity at 31 December 2016 2,105 227 2,332

© Emile Woolf International 128 The Institute of Chartered Accountants of Pakistan


Answers

Workings
(W1) Net assets of subsidiary

Acquisition 1 Jan 31 Dec


1 Jan 2010 2016 2016
Rs.000 Rs.000 Rs.000
Share capital 200 200 200
Retained reserves 420 640 (bal) 710
620 840 910
Fair value adjustment 60 60 60
Accumulated additional depreciation
on FV
adjustment (60/15 yrs = 4 per yr) (12) (16)
Accumulated impairment of goodwill
(W2) (30) (39)
Adjusted net assets 680 858 915
Post-acquisition retained reserves to 1
Jan/31 Dec 178 235

(W2) Goodwill

Rs.000 Rs.000
Consideration transferred 620
NCI at fair value 180
800
Net assets acquired:
Share capital 200
Retained earnings 420
Fair value adjustment 60 (680)
120
Impairment 2015 (25%) (30)
90
Impairment 2016 (10% of carrying value) (9)

(W3) Non-controlling interest

PFY TCI
Rs.000 Rs.000
Profit for year/TCI of Y Ltd 100 120
Less impairment of goodwill in the year (W2) (9) (9)
Less depreciation on FV adjustment for the year
(W1) (4) (4)
87 107
20% NCI share 17 21

© Emile Woolf International 129 The Institute of Chartered Accountants of Pakistan


Advanced accounting and financial reporting

(W4) Group equity attributable to parent at 1 January 2016

Rs.000
Parent’s equity at 1 January 2016 as per SOCIE 1,700
Plus share of post-acquisition retained reserves of Y Ltd to 142
1 January 2016 (80% x 178 (W1))
Plus share of post-acquisition retained reserves of Z Ltd to
1 January 2016 (40% x(500-435)) 26
Equity attributable to parent at 1 January 2016 1,868

(W5) Group equity attributable to NCI at 1 January 2016

Rs.000
At acquisition 180
Plus share of post-acquisition retained reserves to 1 January 2016
(20% x 178 (W1)) 36
Equity attributable to NCI at 1 January 2016 216

(b) (i) Additional acquisition of shares


The purchase of the additional 10% of Y Ltd’s share capital is treated as a
transaction between owners of the entity, as NCI reduces and parent’s share
increases. No additional goodwill is calculated as X Ltd already controls Y Ltd and
goodwill is only calculated when control is attained. Any difference between the
consideration paid by X Ltd and the reduction in the NCI is adjusted through group
retained earnings.
(ii)

Adjustment to parent’s equity Rs.000


Consideration transferred 120
Reduction in NCI at 1 January 2017 (50% x Rs. 227,000) (114)
Adjustment to retained earnings – debit 6

(c) Additional investment in Z Ltd


The additional 20% investment will give X Ltd the majority holding of Z Ltd’s ordinary
shares. This gives the presumption of control, unless there is evidence to the contrary and
once control is attained Z Ltd will be treated as a subsidiary and fully consolidated.
Goodwill on acquisition is calculated at 1 January 2017 and the existing investment will be
restated to FV at the date of acquisition

2.4. PLAIN LTD


Consolidated statement of financial position as at 31 March 2016
Rs. m
Assets
Tangible non-current assets (W4) 1,745
Intangible non-current assets – goodwill 45 – 15 (W3) 30
Investment in associate (W5) 95
Held to maturity investment (W6) 50
Current assets (477 + 190 - 250) 417
2,337

© Emile Woolf International 130 The Institute of Chartered Accountants of Pakistan


Answers

Consolidated statement of financial position as at 31 March 2016


Rs. m
Equity and liabilities
Share capital of Rs. 1 800
Share premium 150
Revaluation reserve 90
Retained earnings (W8) 169
1,209
Non-controlling interest (W9) 99
1,308
Non-current liabilities (640 + 30) 670
Current liabilities (214 + 130) 344
Pension liability (W7) 15
2,337
Workings
(W1) Retained profits of Stripes
Rs. m
Fair value of net assets at 1 April 2015 460
Fair value adjustment for land (25)
Carrying value of net assets 435
Share capital plus share premium (260)
Therefore retained earnings at 1 April 2015 175

Rs. m
Carrying value of net assets at 1 April 2013 325
Share capital plus share premium (260)
Therefore retained earnings at 1 April 2013 65
(W2) Gain or loss on acquiring control of Stripes
1 April 2015 Rs. m Rs. m
Fair value of initial investment in Stripes at 1 April 2015 150
Initial cost of investment 120
Share of retained earnings 1 April 2013 – 1 April 2015 33
(= 30%  (175 – 65) –see W1
Carrying value of investment in associate 153
Loss recognised on gaining control of Stripes (3)
This loss has not yet been recognised in the individual financial statements of Plain; it must
therefore be included in the calculation of group reserves (see Working 8).
(W3) Goodwill in Stripes at acquisition
Rs. m
Fair value of initial investment at acquisition 150
Cost of additional shares 260
Total cost 410
Fair value of net assets acquired (80%  460) 368
Goodwill at acquisition attributable to Plain 42
Goodwill attributable to NCI 3
Total goodwill at acquisition date 45

© Emile Woolf International 131 The Institute of Chartered Accountants of Pakistan


Advanced accounting and financial reporting

Goodwill in statement of financial position: There has been impairment of Rs. 15 million in
goodwill. This is apportioned between the interests of the equity owners of Plain and NCI
in the ratio 80:20.
Impairment of goodwill attributable to parent = Rs. 15m  80% = Rs. 12 million
Impairment of goodwill attributable to NCI = Rs. 15m  20% = Rs. 3 million.
(W4) Tangible non-current assets
Rs. m
Plain 1,280
Stripes 440
Fair value adjustment 25
1,745
(W5) Investment in associate – Spots
Rs. m
Cost 60
Group share of post-acquisition profit
(324 – (200 – 16)) × 25% 35
95

Or Rs. m
Share of net assets (25% × 324) 81
Fair value adjustment (25% × 16) 4
Goodwill [60 – (200 × 25%)] 10
95
(W6) Held to maturity investment
Rs. m
Amortised cost
Cost 54
Less: Discount (20/5) (4)
50
Tutorial note: It is not correct to recognise interest on a straight line basis. It is used here
as a simplification. IAS 39 requires the recognition of interest using the effective rate.
(W7) Pension
Rs. m
Scheme assets
Cash 250
Expected return 26
Actuarial gain (bal fig) 26
Fair value of scheme assets 302
Scheme liabilities
Current service cost 276
Interest cost 41
Present value of obligation 317

Pension scheme liability (317 – 302) 15


Expense in profit or loss
Current service cost 276
Interest cost 41
Expected return (26)
Expense in profit or loss 291

© Emile Woolf International 132 The Institute of Chartered Accountants of Pakistan


Answers

(W8) Consolidated retained earnings


Rs. m
Plain (given) 390
Stripes post-acquisition retained earnings
(210 – 175 (W1)) × 80% 28
Loss on acquiring control (W2) (3)
Goodwill impairment attributable to parent (W3) (12)
Share of post-acquisition profits of associate (W5) 35
Discount on investment (W6) (4)
Pension cost (W7) (291)
Actuarial gain (W7) 26
169
(W9) Non-controlling interest in Stripes
Rs. m
Book value (20% × 470) 94
Fair value adjustment (20% × 25) 5
Goodwill (3 – impairment 3) (W3) 0
99

2.5. MANGO LTD


Mango Ltd
Consolidated statement of financial position as at 31 March 2016
Rs. m
Non-current assets
Non-current assets (3,295 + 2,000 + 226 + 2) 5,523.0
Goodwill (W2) 89.0
Current assets (1,685 + 861) 2,546.0
Total assets 8,158.0

Equity and liabilities


Share capital 850.0
Retained earnings (W4) 3,405.9
Other components of equity (W5) 257.0
4,512.9
Non-controlling interest (W3) 649.1
5,162.0
Non-current liabilities (1,895 + 675) 2,570.0
Current liabilities (320 + 106) 426.0
Total equity and liabilities 8,158.0

© Emile Woolf International 133 The Institute of Chartered Accountants of Pakistan


Advanced accounting and financial reporting

Workings
(W1a) Net assets in subsidiary at acquisition – before measurement period
adjustments
At end of reporting At
period acquisition
Rs. m Rs. m
Share capital 1,020 1,020
Retained earnings 980 900
Other components of equity 80 70
1,990

Fair value adjustments:


Contingent liability (6)
Property 266
260
Fair value of net assets (given) 2,250

The total fair value adjustment of Rs. 260 million above is taken as a bal ancing
figure as is the fair value adjustment that relates to property.
The amount in respect of the contingent liability and an amount within the property
adjustments is subsequently found to be incorrect. This information came to light in
the measurement period. Therefore, they retrospectively adjust the carrying amount
of goodwill. In this case the easier approach is to calculate goodwill using the
corrected figures.
(W1b) Net assets in subsidiary at acquisition – after measurement period
adjustments
At At acquisition
consolidation
Rs. m Rs. m
Share capital 1,020 1,020
Retained earnings 980 900
Reduction of depreciation recognised on the
buildings (Rs. 40/20 years) 2
Adjustment re recognition of the provision 5
987 900
Other components of equity 80 70
1,067 1,990
Fair value adjustments:
Contingent liability (6 – 1) (5) (5)
Property (266 – 40) 226 226
Fair value of net assets 2,308 2,211

The contingent liability at acquisition is recognised for consolidation purposes as a


deduction from the net assets of the subsidiary. It is not recognised in the
subsidiary’s own accounts as it does not pass the IAS 37 recognition criteria.

© Emile Woolf International 134 The Institute of Chartered Accountants of Pakistan


Answers

The contingent liability has evolved into a provision by the date of consolidation.
This means that it is recognised as a liability and the amount has been expensed in
the subsidiary’s own financial statements. The adjustment made above (Dr Fair
value adjustment and Cr Retained earnings) is made because the expense which
has been recognised by the subsidiary since the date of acquisition relates to an
amount that has already been recognised in the consolidation workings at
acquisition.
(W2) Goodwill
Rs. m
Cost of investment 975
Fair value of initial holding 705
Fair value of NCI at date of acquisition 620
2,300
Net assets acquired (W1b) (2,211)
89
(W3) Non-controlling interest
Rs. m
Fair value of NCI at date of acquisition 620.0
NCI’s share of post-acquisition growth in:
Retained earnings (30% of (987 – 900)) 26.1
Other components in equity (30% of (80 – 70)) 3.0
649.1
(W4) Retained earnings
Rs. m
Mango Ltd’s balance as per the question 3,340.0
fair value adjustment re initial holding (705 – 700) 5.0
Share of post-acquisition growth (70% of (987 – 900)) 60.9
3,405.9
(W5) Other components of equity
Rs. m
Mango Ltd’s balance as per the question 250.0
Share of post-acquisition growth (70% of (80 – 70)) 7.0
257.0

© Emile Woolf International 135 The Institute of Chartered Accountants of Pakistan


Advanced accounting and financial reporting

CHAPTER 3 - CONSOLIDATED STATEMENTS OF PROFIT OR LOSS AND OTHER


COMPREHENSIVE INCOME
3.1. MILLARD LTD
Millard Ltd: Consolidated profit and loss account for the year ended 31 December 2016
NOTE Rs.’000
Revenue (1) 425,000
Cost of sales (2) (162,600)
Gross profit 262,400
Distribution costs (35,000)
Impairment losses (4) (7,000)
Administrative expenses (28,000)
Operating profit 192,400
Investment income (5) 3,750

Debenture interest (6) (58,750)


Profit before tax 137,400
Tax (52,500)
Profit after tax 84,900
Non-controlling Interests (7) (8,380)
Group profit 76,520
Preference dividends (13,750)
Ordinary dividends (20,000)
Retained profit 42,770
Retained profit 1 January 2016 (8) 72,350
Retained profit 31 December 2016 115,120
Workings:
1 Revenue: Rs.’000
Millard 312,500
Fillmore 125,000
437,500
Less: inter-company group sales 12,500
425,000

2 Cost of Sales: Rs.’000


Millard 125,000
Fillmore 50,000
Inter-company sales (12,500)
Unrealized profits (3) 100
162,600

3 Provision for unrealised profits on inventory Rs.’000


25 2000 400
2016 
125 1
25 1500
2015  (300)
125 1
Charged to profit and loss a/c 100

© Emile Woolf International 136 The Institute of Chartered Accountants of Pakistan


Answers

4 Goodwill on acquisition: Rs.’000 Rs.’000


Cost of investment 67,000
Net assets acquired:
- On ordinary share capital 62,500
- Profit and loss account 12,500
75,000
Percentage acquired 50/62.5 x 100 = 80%
80% of Rs. 75,000 (60,000)
Goodwill (written-off in 2016) 7,000

5 Investment income: Rs.’000


As per draft account 7,950
Inter-company dividend 80% x Rs. 5,250 (4,200)
Investment income 3,750

6 Debenture interest Rs.’000


Parent’s 47,500
Subsidiary 15,000
Inter-company amount 25% x Rs. 15,000 (3,750)
Investment income 58,750

7 Non-controlling interest:
Rs.’000 Rs.’000
Profit after tax 24,500
Less: Preference dividend (4,375) 4,375
20,125 x 20% 4,025
8,400
Non-Controlling Interest share of unrealized profit 20% x Rs. 100 (20)
8,380

8 Retained Profit b/f Rs.’000 Rs.’000


Parent’s 66,750
Subsidiary 19,500
Pre-acquisition (12,500)
Parent’s share 7,000 x
80% 5,600
72,350

© Emile Woolf International 137 The Institute of Chartered Accountants of Pakistan


Advanced accounting and financial reporting

3.2. SHERLOCK
Sherlock Ltd: Consolidated statement of profit or loss and other comprehensive income for the year
ended 31 December 2016.
Rs. m
Revenue 538.0
Cost of sales (383.0)
Gross profit 155.0
Other income 29.0
Administrative costs (30.0)
Other expenses (72.6)
Operating profit 81.4
Finance costs (10.0)
Finance income 15.0
Profit before tax 86.4
Income tax expense (31.0)
Profit for the year 55.4
Other comprehensive income
Revaluation surplus 7.8
Remeasurement 2.0
Loss on cash flow hedge (3.0)
6.8
Total comprehensive income for year 62.2
Profit attributable to:
Rs. m
Owners of the parent (balancing figure) 43.8
Non-controlling interest (W1a) 11.6
55.4
Total comprehensive income attributable to:
Rs. m
Owners of the parent (balancing figure) 51.8
Non-controlling interest (W1a) 10.4
62.2
Workings
W1 Balances for inclusion in the consolidated statement of profit or loss and other
comprehensive income
Sherlock Katie Ltd
Mycroft Ltd Adjustment Total
Ltd (6/12)
Rs. m Rs. m Rs. m Rs. m Rs. m
Revenue 400 115 35 (12) W3 538
Cost of sales (312) (65) (18) 12 W3 (383)
Gross profit
Other income 21 7 1 29
Administrative costs (15) (9) (6) (30)
Other expenses (35) (19) (4)
Goodwill impairment W2 (3)
Pension cost W4 (7.2)
Revaluation W5 (2.4)
Share Katie Ltd W6 (2.0) (72.6)
Operating profit

© Emile Woolf International 138 The Institute of Chartered Accountants of Pakistan


Answers

Sherlock Katie Ltd


Mycroft Ltd Adjustment Total
Ltd (6/12)
Rs. m Rs. m Rs. m Rs. m Rs. m
Finance costs (5) (6) (2)
Cash flow hedge W7 3.0 (10)
Finance income 6 5 4 15
Profit before tax
Income tax expense (19) (9) (3) (31)
Profit for the year 41 22 7
OCI
Revaluation surplus 10
Revaluation W5 (2.2) 7.8
Remeasurement W4 2 2.0
Cash flow hedge W7 (3.0) (3.0)
Total CI for year 50.8 19 7 6.8
W1a Non-controlling interests
Mycroft Ltd Katie Ltd Total
Profit for the year 22 7
NCI percentage holding 40% 40%
NCI 8.8 2.8 11.6

Mycroft Ltd Katie Ltd Total


Total CI for the year W1 19 7
NCI percentage holding 40% 40%
NCI 7.6 2.8 10.4
W2 Goodwill write off
Rs. m
Cost of investment 80
FV of NCI at acquisition 45
125
Fair value of identifiable net assets (110)
Goodwill on acquisition 15
Write off in previous year (20%) (3)

W3 Inter-company trading
The inter-company trading amounts must be eliminated (ie sales and purchases).
There is no adjustment in respect of the loss. The question states that the sale is at fair value.
Therefore the loss is realised. Only unrealised amounts are adjusted on consolidation.
W4 Pension scheme
Amounts charged to profit or loss: Rs. m
Interest (10% of (Rs. 50m – Rs,48m)) 0.2
Current service cost 4.0
Past service cost 3.0
7.2
Amount charged to OCI
Remeasurement 2.0

© Emile Woolf International 139 The Institute of Chartered Accountants of Pakistan


Advanced accounting and financial reporting

W5 Revaluation of plant
Rs. m
Original cost (1 January 2015) 12.0
Depreciation in year ended 31 December 2015 (1.2)
Carrying amount before revaluation at 31 December 2015 10.8
Revaluation recognised in year ended 31 December 2015 2.2
Value at 31 December 2015 13.0
Depreciation in year ended 31 December 2016 (÷9) (1.4)
Carrying amount before revaluation at 31 December 2016 11.6
Fall in value to be recognised 4.6
Value at 31 December 2016 7.0
Dr Cr
Rs. m Rs. m
Plant 4.6
Statement of profit or loss 2.4
Other comprehensive income 2.2
W6 Share Katie Ltd expense
Rs. m
Balance recognised in year ended 31 December 2015
8,000 Katie Ltds  Rs. 100  4 directors  1/4 0.8

Balance recognised in year ended 31 December 2015


8,000 Katie Ltds  Rs. 100  7 directors  2/4 2.8
Charge for the year ended 31 December 2016 2.0
Dr Cr
Rs. m Rs. m
Statement of profit or loss 2.0
Equity 2.0
W7 Cash flow hedge
Mycroft Ltd’s loss on the effective cash flow hedge has been treated incorrectly. The effective part
of any gain or loss on a cash flow hedge should be recognised in other comprehensive income
and accumulated in a cash flow hedge reserve in equity.
The following corrective journal is necessary:
Dr Cr
Rs. m Rs. m
Loss on cash flow hedge
(in other comprehensive income) 3
Finance cost (profit or loss) 3

© Emile Woolf International 140 The Institute of Chartered Accountants of Pakistan


Answers

3.3. FAISAL LIMITED


Faisal Limited
Consolidated statement of financial position as at 31 December 2016
Rs. in million
Non-current assets
Property, plant and equipment (W1) 31,926.00
Accumulated depreciation (W1) (7,491.00)
24,435.00
Goodwill (W6) 1,380.00
Other investments 11,100.00
36,915.00
Current assets
Inventory (W3) 23,740.00
Accounts receivable (6,240 + 2,460 + 6,580  800) 14,480.00
Cash and bank balances (4,920 + 660 + 2,700) 8,280.00
46,500.00
Total assets 83,415.00

Equity and liabilities


Share capital 30,000.00
Retained earnings (W8) 42,379.75
72,384.75
Non-controlling interest (W7) 5,655.25
78,035.00
Current liabilities
Accounts payable (2,760 + 1,980 + 1,440  800) 5,380.00
Total equity and liabilities 83,415.00

Faisal Limited
Consolidated statement of profit or loss
for the year ended 31 December 2016
Rs. in million
Sales (W4) 100,100.00
Cost of sales (W4) (80,991.00)
Gross profit 19,109.00
Operating expenses (3,600 + 2,100 + 5,400) (11,100.00)
8,009.00
Gain on sale of non-current assets (540 – 54) 486.00
Dividend income (1,080 – (80%  600)) 600.00
Profit for the year 9,095.00
Attributable to:
Ordinary shareholders of parent 8,599.75
Non-controlling interest (W9) 495.25
9,095.00

© Emile Woolf International 141 The Institute of Chartered Accountants of Pakistan


Advanced accounting and financial reporting

Workings (all figures in millions of rupees)


(W1) Unrealised profit adjustments; Transfer of non-current asset
Figures if no
Figures in the Adjustment
transfer had
accounts required
been made
Against SL’s figures:
Cost 144 150 6 Dr
Accumulated depreciation
(144/3 years)  6/12 (24)
150  3 years/5 years (75)
(24) (75) 51 Cr
Carrying amount 120 75
Charge for the year 24 15 9 Cr

Against FL’s figures:


Profit on disposal 54 54 Dr

Double entry in consolidated financial statements Dr Cr


Profit on disposal 54
Depreciation charge for year 9
Property, plant and equipment (cost) 6
Accumulated depreciation 51

NCI in the statement of profit or loss 2.25


NCI in the statement of financial position 2.25
Being the NCI share of the depreciation adjustment (20%  9)

Accumulated
Consolidated balances PP and E depreciation
Rs. millions Rs. millions
FL 22,500 5,760
SL 3,480 420
AIL 5,940 1,260
Adjustments for inter-company transfer 6 51
31,926 7,491
(W2) Unrealised profit adjustments: Inter-company trading
FL to AIL SL to AIL AIL to FL Total
Sales 2,400 1,800 3,600 7,800

Inventory held 900 600 1,200


Gross profit percentage on sale 20% 10% 30%
Unrealised profit 180 60 360 600
NCI’s share (based on selling company’s
NCI)
25%  60 15
20%  360 72

© Emile Woolf International 142 The Institute of Chartered Accountants of Pakistan


Answers

Double entry in consolidated financial statements Dr Cr


Cost of sales (closing inventory) 600
Closing inventory in statement of financial position 600

NCI in the statement of financial position 87


NCI in the statement of comprehensive income 87

Impact on consolidated retained earnings (600 – 97) 513

(W3) Consolidated inventory


Rs. millions
FL 14,460
SL 4,200
AIL 5,680
(600)
23,740

(W4) Consolidated sales and cost of sales


Sales Cost of sales
Rs. millions Rs. millions
FL 57,600 49,200
SL 16,500 18,000
AIL 33,800 21,000
Inter-company sales (7,800) (7,800)
Unrealised profit 600
Depreciation adjustment on inter-company transfer of
non-current assets (9)
100,100 80,991

(W5) Net assets of SL


At end of
At
reporting
acquisition
period
Rs. millions Rs. millions
Share capital 12,000 12,000
Retained earnings (3,600) -
8,400 12,000

Net assets of AIL


At end of
At
reporting
acquisition
period
Rs. millions Rs. millions
Share capital 6,000 6,000
Retained earnings (12,200) 5,400
18,200 11,400

© Emile Woolf International 143 The Institute of Chartered Accountants of Pakistan


Advanced accounting and financial reporting

(W6) Goodwill
SL AIL
Rs. millions Rs. millions
Cost of investment 9,000 10,500
NCI at acquisition
25%  12,000 (W5) 3,000
20%  11,400 (W5) 2,280
12,000 12,780
Net assets acquired (12,000) (11,400)
 1,380

(W7) Non-controlling interest in statement of financial position


SL AIL
Rs. millions Rs. millions
NCI at acquisition
25%  12,000 (W5) 3,000
20%  11,400 (W5) 2,280
NCI’s share of post-acquisition profit / (loss)
25%  (3,600) (W5) (900)
20%  (12,200 – 5,400) (W0) 1,360
2,100 3,640
Total (2,100 + 3,640) 5,740.00
Unrealised profit adjustments
on inter-company sale of inventory (W2) (87.00)
on inter-company sale of non-current asset (W1) 2.25
5,655.25

(W8) Retained earnings


Rs. millions
FL 40,200.00
Share of SL (75%  (3,600)) (2,700.00)
Share of AIL (80%  (12,200 – 5,400) (W5)) 5,440.00
42,940.00
Unrealised profit adjustments
Inter-company sale of inventory (W2) (600.00)
NCI share 87.00
(513.00)

Inter-company sale of non-current asset (W0)


Unrealised profit (W1) (54.00)
Depreciation adjustment (W1) 9.00
NCI share of depreciation adjustment (W1) (2.25)
(47.25)
42,379.25

© Emile Woolf International 144 The Institute of Chartered Accountants of Pakistan


Answers

(W9) Non-controlling interest in statement of profit or loss


SL AIL
Rs. millions Rs. millions
Sales 16,500 33,800
Cost of sales (18,000) (21,000)
Operating expenses (2,100) (5,400)
(3600) 7400
25% 20%
Non-controlling interest (900) 1,480
Total (1,480 – 900) 580.00
Unrealised profit adjustments
on inter-company sale of inventory (W2) (87.00)
on inter-company sale of non-current asset (W1) 2.25
495.25

3.4. GOLDEN LIMITED


Golden Limited
Consolidated statement of profit or loss for the year ended June 30, 2016
Rs. in million
Sales (875 + 350 - 40) 1,185.00
Cost of sales (567 + 206 - 33.6 (W1)) (739.40)
Gross profit 445.60
Selling expenses (33 + 11) (44.00)
Administrative expenses (63 + 40) (103.00)
Interest expenses (30 + 22) (52.00)
Other income (65 - 36) [20 x Rs. 2 x 90%) 29.00
Impairment losses
Goodwill (W2) (9.18)
Investment in associates (W3) (25.80)
Share of loss from associates [(Rs. 82 x 40%)+0.6] (33.40)
Profit before tax 207.22
Income tax expense (73 + 15) (88.00)
Profit for the year 119.22
Attributable to:
Ordinary shareholders of parent 114.26
Non-controlling interest (W4) 4.96
119.22

W1: Adjustment in cost of sales Rs. in million


Intra-group purchases (40.00)
Additional depreciation on machines 4.00
Unrealized profit in inventories 2.40
(33.60)

© Emile Woolf International 145 The Institute of Chartered Accountants of Pakistan


Advanced accounting and financial reporting

W2: Impairment on goodwill Rs. in million


Shares issued (18 x 4/5 x Rs. 20) 288.00
Less: Net assets acquired:
Share capital 200
Pre-acquisition reserves 24
Fair value adjustment (22 + 20 + 3) 45
269
Holding % 90% 242.10
Goodwill 45.90
20% Impairment in goodwill 9.18

W3: Impairment in the value of investment in associates Rs. in million


Cash paid (6 x 12) 72.00
Less: Post-acquisition losses:
Reserves on acquisition 40
Reserves at June 30, 2016 (108-82) 26
(14)
% holding 40% (5.60)
Elimination of unrealized gain to the extent of GL's share
(Rs. 11.5 x 0.15 / 1.15 x 40%) (0.60)
65.80
Fair value as per impairment testing 40.00
Impairment losses 25.80

W4: Non-controlling interests


Profit of YL 56.00
Less: Additional depreciation (4.00)
Unrealized profit in inventories (2.40)
49.60
Non-controlling interest % 10%
4.96

© Emile Woolf International 146 The Institute of Chartered Accountants of Pakistan


Answers

CHAPTER 4 - COMPLEX GROUPS


4.1. PARVEZ LTD
(a) Parvez Ltd: Consolidated statement of profit or loss for the year ended 31 December
2016
Rs. 000
Revenue (W4) 92,360.0

Cost of sales (W4) (28,123.0)


Gross profit 64,237.0
Distribution costs (3,325 + 2,137 + 1,900) (7,362.0)
Administrative expenses (3,475 + 950 + 1,900) (6,325.0)
Operating profit 50,550.0
Interest paid (325.0)
Profit before tax 50,225.0
Tax (17,931.0)
Profit after tax 32,294.0

Attributable to:
Ordinary shareholders of parent 28,580.8
Non-controlling interest (W9) 3,713.2
32,294.0

(b) Parvez Ltd: Consolidated statement of financial position as at 31 December 2016


Rs. 000
Goodwill (W6) 3,963.5
Property, plant and equipment
(35,483 + 24,273 + 13,063) 72,819.0
Current assets (W3) 19,446.0
96,228.5
Share capital 8,000.0
Retained earnings (W8) 56,641.3
Non-controlling interest (W7) 8,453.2
Current liabilities (13,063 + 10,023 + 48) 23,134.0
96,228.5

(W1) Group structure

© Emile Woolf International 147 The Institute of Chartered Accountants of Pakistan


Advanced accounting and financial reporting

(W2) Unrealised profit adjustments: Inter-company trading


Vazir to Saad to Total
Saad Parvez
Sales 480,000 260,000 740,000

Inventory held 75,000 60,000


Unrealised profit adjustment  25/125  33.3/133.3
Unrealised profit 15,000 15,000 30,000

NCI’s share (based on selling


company’s NCI)
10%  15 1,500
28%  15 4,200

Double entry in consolidated financial statements Dr Cr


Cost of sales (closing inventory) 30,000
Closing inventory in statement of financial position 30,000

NCI in the statement of financial position 5,700


NCI in the statement of comprehensive income 5,700

Impact on consolidated retained earnings (30 – 5.7) 24,300


(W3) Consolidated current assets
Rs. 000
Parvez 1,568
Saad 9,025
Vazir 8,883
Unrealised profit (W2) (30)
19,446
(W4) Consolidated sales and cost of sales
Cost of
Sales sales
Rs. 000 Rs. 000
Parvez 45,600 18,050
Saad 24,700 5,463
Vazir 22,800 5,320
Inter-company sales (740) (740)
Unrealised profit 30
92,360 28,123
(W5) Net assets summary
At date of At date of Post-acquisition
Saad Ltd consolidation acquisition
Rs. 000 Rs. 000
Share capital 3,000 3,000
Retained earnings 24,075 1,425 22,650
27,068 4,425

© Emile Woolf International 148 The Institute of Chartered Accountants of Pakistan


Answers

At date of At date of
Vazir Ltd Post-acquisition
consolidation acquisition
Rs. 000 Rs. 000
Share capital 2,000 2,000
Retained earnings 19,898 950 18,948
21,898 2,950
(W6) Goodwill
In Saad Ltd In Vazir Ltd
Rs. 000 Rs. 000
Cost 6,650.0
90% × 3,800 3,420.0
NCI at acquisition
10% × 4,425 W5 442.5
28% × 2,950 W5 826.0
7,092.5 4,426.0
Net assets at acquisition (4,425.0) (2,950.0)
2,667.5 1,296.0
Total 3,963.5
(W7) Non-controlling interest (statement of financial position)
Rs. 000 Rs. 000
NCI at acquisition
Saad Ltd: 10% × 4,425 (W5) 442.5
Vazir Ltd: 28% × 2,950 (W5) 826.0
NCI’s share of post-acquisition profits
10% × 22,650 (W5) 2,265.0
28% × 18,948 (W5) 5,305.4
NCI in Saad Ltd ’s share of net assets of Vazir Ltd
10% × 3,800 (380.0)
2,327.5 6,131.4
Total 8,458.9
Unrealised profit (W2) (5.7)
8,453.2
(W8) Consolidated retained earnings carried forward
Rs. 000
All of Parvez Ltd
Per the question 22,638.0
Share of Saad Ltd
90%  22,650 (W5) 20,385.0
Share of Vazir Ltd
72% × 18,948 (W5) 13,642.6
Unrealised profit (W2) (24.3)
56,641.3
(W9) Non-controlling interest (statement of profit or loss)
Saad Ltd Vazir Ltd
Rs. 000 Rs. 000
Profit after tax 10,760 9,439
10% 28%
Non-controlling interest 1,076 2,642.9

© Emile Woolf International 149 The Institute of Chartered Accountants of Pakistan


Advanced accounting and financial reporting

Total (1,076 + 2,642.9) 3,718.9


Unrealised profit (W2) (5.7)
3,713.2

4.2. HASAN, RIAZ AND SIDDIQ


Hasan Limited: Consolidated statement of financial position as at 31 December 2016

Rs.
Goodwill (W6) 26,250
Property, plant and equipment (1,102,500 + 271,950 + 122,550) 1,497,000

Inventories (W4) 783,520


Receivables (241,920  8,000 W2) + 129,680 + 29,750 – 17,500 W2) 375,850
Cash and bank balances ((88,200 + 8,000 W2) + 4,725 + 8,105) 109,030
2,791,650
Share capital 1,750,000
Retained earnings (W8) 181,795
Other reserves (W9) 402,500
Non-controlling interest (W7) 191,625
Payables (95,480 + (86,645 + 12,500 W2) + 88,605 – 17,500 W2) 265,730
2,791,650
(W1) Group structure

Non-controlling
Hasan’s
interest
interest
(balance)
In Riaz Ltd 75% 25%
In Siddiq Ltd (40% + (75%  20%)) 55% 45%

(W2) Individual company adjustments: Transaction before the year-end not yet
accounted for
Books of Riaz
Purchase of inventory from Hasan Dr Cr
Closing inventory 12,500
Payable (to Hasan) 12,500

© Emile Woolf International 150 The Institute of Chartered Accountants of Pakistan


Answers

Books of Hasan
Cash received from Riaz Dr Cr
Cash 8,000
Receivable (from Hasan) 8,000
The inter-company balances can be reconciled as follows after these adjustments
have been processed:
Hasan’s Riaz’s
financial financial
statements statements
Receivable Payable
Given in the question
Receivable from Riaz (note 5 in the question) 25,500
Payable to Hasan (note 7 in the question) 5,000
Cash from Riaz (8,000)
Purchase from Hasan 12,500
17,500 17,500

These balances must be cancelled out on consolidation as follows:


Dr Cr
Consolidated payables 17,500
Consolidated receivables 17,500
(W3) Unrealised profit adjustments: Inter-company trading

Inventories held by Riaz purchased from Hasan


Rs.
Purchased 30 December 12,500
Purchased previously 10,400
Purchased previously 22,900
Mark up adjustment  25/125
Unrealised profit 4,580

Double entry in consolidated financial


statements Dr Cr
Cost of sales (closing inventory) 4,580
Closing inventory in statement of financial
position 4,580

There is no double entry for the NCI as all sales were from the parent.
(W4) Consolidated inventories
Rs.
Hasan 526,610
Riaz (163,290 + 12,500 (W2) 175,790
Vazir 85,700
Unrealised profit (W3) (4,580)
783,520

© Emile Woolf International 151 The Institute of Chartered Accountants of Pakistan


Advanced accounting and financial reporting

(W5) Net assets summary


At date of At date of Post-
Riaz Ltd consolidation acquisition acquisition
Rs. Rs.
Share capital 420,000 420,000
Accumulated profits 17,500 35,000 (17,500)
Other reserves 70,000 nil 70,000
437,500 455,000

At date of At date of Post-acquisition


Siddiq Ltd consolidation acquisition
Rs. Rs.
Share capital 175,000 175,000
Accumulated losses (17,500) (35,000) 17,500
157,500 140,000
(W6) Goodwill
In Riaz In Siddiq Ltd
Ltd
Rs. Rs.
Cost 367,500 49,000
75% × 24,500 18,375
67,375
NCI at acquisition
25% × 455,000 W5 113,750
45% × 140,000 W5 63,000
481,250 130,375
Net assets at acquisition (455,000) (140,000)
26,250 (9,625)

The balance for Siddiq is a gain on a bargain purchase.


(W7) Non-controlling interest (statement of financial position)
Rs. Rs.
NCI at acquisition
25% × 455,000 W5 113,750
45% × 140,000 W5 63,000
NCI’s share of post-acquisition profits
25% × (17,500) (W5) (4,375)
45% × 17,500 (W5) 7,875
NCI’s share of other reserves
25% × 70,000 (W5) 17,500
NCI in Riaz Ltd ’s share of net assets of
Siddiq Ltd
25% × 24,500 (6,125)
120,750 70,875
Total 191,625

© Emile Woolf International 152 The Institute of Chartered Accountants of Pakistan


Answers

(W8) Consolidated retained earnings carried forward


All of Hasan Ltd Rs.
Per the question 180,250
Unrealised profit (W2) (4,580)
175,670
Share of Riaz Ltd
75% × (17,500) (W5) (13,125)
Share of Siddiq Ltd
55% × 17,500 (W5) 9,625
Gain on bargain purchase (W6) 9,625
181,795
(W9) Consolidated other reserves
Rs.
All of Hasan Ltd
Per the question 350,000
Share of Riaz Ltd
75% × 70,000 (W5) 52,500
402,500

4.3. LALIWALA GROUP


Computation of goodwill

Cash consideration 2,000.00


Market based measure of replacement awards 140.00
Total consideration transferred 2,140.00

Net assets acquired


Fair value of assets 3,618.00
Fair value of liabilities (1,888.00)
Development expenditure 153.00
Contingent liability (25.00)
Adjustment in deferred tax (W-1) (70.85)
1,787.15
Goodwill 352.85

Impact on deferred tax as on 31 December 2016: Timing Tax Deferred


difference rate tax
Effect of deferred tax on acquisition (W-1) (70.85)
Unrealized profit on closing stock held by PAL (80×20%) (16.00) 35.0% (5.60)
Unrealized profit on closing stock held by LG (140×15%) (21.00) 25.0% (5.25)
Undistributed profit of NAL – Associate (to be realized
through dividend) [(50–20)×30%×60%)] (5.40) 12.5% (0.68)
Undistributed profit of NAL – Associate (Realized through
sale) [(50–30)×30%×40%)] (3.60) 17.5% (0.63)
Increase in intrinsic value (150–90) (60.00) 25.0% (15.00)
(98.01)

© Emile Woolf International 153 The Institute of Chartered Accountants of Pakistan


Advanced accounting and financial reporting

W-1: Adjustment for deferred tax on 1 January 2016

Impact on
Deferred
Taxable
Carrying Tax tax
Fair value /(deductible)
value rate liability/
time
(assets)
difference
Property, plant and equipment 1,532 1,259 273.00 35% 95.55
Investments 490 367 123.00 35% 43.05
Retirement benefit obligations 60 17 (43.00) 35% (15.05)
Development expenditure 153 - 153.00 35% 53.55
Contingent liability 25 - (25.00) 35% (8.75)
Unused tax losses 300 - (300.00) 25% (75.00)
Intrinsic value of share options 90 - (90.00) 25% (22.50)
Net adjustment in deferred tax 70.85

4.4. SHAKIR LIMITED


Shakir Limited
Consolidated Statement of Financial Position
As on 30 June 2017
Rs. in million
Assets:
Property, plant & equipment (W-1) 28,233.0
Investment in ML (W-5) 1,980.0
Stock-in-trade [2,414 + 1,750 – 4(W-7)] 4,160.0
Trade & other receivables [2,200+1,800+120(W-8) ] 4,120.0
Cash and bank (1,600 + 1,900) 3,500.0
41,993.0

Equity & Liabilities


Share capital 20,000.0
Group reserves (W-2) 10,031.4
Non controlling interest (16,229 (W-7) × 40%) 6,491.6
Trade and other payables [4,400 + 1,070] 5,470.0
41,993.0

W-1: Property plant & Equipment:


SL 16,500.0
BL 11,000.0
Power generation plant [620-62 (620÷10)] 558.0
Fair value adjustment [200-25 (200x2÷16)] 175.0
28,233.0

© Emile Woolf International 154 The Institute of Chartered Accountants of Pakistan


Answers

W-2 : Group reserves


SL’s retained earnings 6,189.0
SL’s Share premium 1000.0
Impairment of ML’s goodwill [800 (W-4) × 20%] (160.0)
Post acquisition profit of ML
 Till last year [{5,600 – 4,500}(W-3) × 80%] 880.0
 For the year [700 × 60%] 420.0
Equity adjustment on sale of 20% shares of ML [1,188 – (5,600 (W-3) × 68.0
20%)]
Gain on further 35% disposal (W-5) 486.0
Reversal of gain on disposal of ML (W-6) (1,089.0)
Post acquisition profit – BL [3,429(W-7) × 60%] 2,057.4
Bargain purchase 180.0
10,031.4

W- 3: Net Assets of ML At reporting At 1 July 2016 At acquisition


------------------ Rs. in million ------------------
Share capital 2,200.0 2,200.0 2,200.0
Share Premium 900.0 900.0 900.0
Retained Earnings 3,200.0 2,500.0 1,400.0
6,300.0 5,600.0 4,500.0

W-4: Computation of Goodwill on acquisition of ML Rs. in million


Cash consideration 4,400.0
Less: Net assets acquired [4,500(W-3)×80%] (3,600.0)
Goodwill 800.0

W-5: Gain on part disposal of ML with losing control Rs. in million


Consideration received 2,926.0
Fair value of residual investment [220×25%×36] 1,980.0
Net assets derecognized [6,300(W-3)×60%] 3,780.0
Goodwill derecognized (800–160) 640.0
Net assets sold (4,420.0)
Gain on disposal 486.0

W-6: Gain on sale of ML's shares in SL's books Rs. in million


20% disposal [1,188 – (4,400 × 20÷80)] 88.0
35% disposal [2,926 – (4,400 × 35÷80)] 1,001.0
1,089.0

© Emile Woolf International 155 The Institute of Chartered Accountants of Pakistan


Advanced accounting and financial reporting

At reporting At acquisition
W-7: Net assets of BL --------- Rs. in million ---------
Share capital 10,000.0 10,000.0
Retained earnings 6,000.0 (Bal.) 2,600.0
Increase in fair value of building 175.0 200.0
(200×14÷16)
Share of profit from joint operation (W-8) 58.0 -
Unrealized profit of BL in SL's closing stock [44– (4.0) -
(50×80%)]
16,229.0 12,800.0
(7,500+180)÷0.6

Post acquisition profit 3,429.0

W-8: Joint operation Rs. In million


Receivable from Joint operator (1100–670–130 ) × 40% 120.0
Depreciation expense (62.0)
BL’s share of profit of joint operation 58.0

4.5. ANT, BEE AND FLY


Ant Limited
Consolidated Statement of Financial Position
As on 31 December 2017

Assets: Rs. in million


Property, plant and equipment [3,510+2,835+ 2,200– (20 – 6(W-1))] 8,531.00
Goodwill [175 (W-2) + 108 (W-4)] 283.00
Investment property (130 + 45 + 5(W-1)+ 8(W-1)) 188.00
Current assets (2,120 + 1,420 + 2,800) 6,340.00
Total Assets 15,342.00

Equity and liabilities


Share capital 5,500.00
Group reserves (W-5) 2,476.75
NCI (W-7) 2,631.25
Gratuity [25 + 8 (W-9)] 33.00
Current liabilities (1,775 + 1,386+ 1,500+ 40(W-3)) 4,701.00
Total equity and liabilities 15,342.00

© Emile Woolf International 156 The Institute of Chartered Accountants of Pakistan


Answers

Acquisition Reporting
1-Apr-17
W-l: Net Assets - BL date date
------------------ Rs. in million ------------------
Share capital 4,000.00 4,000.00 4,000.00
Retained earnings 520.00 815.00 1,314.00
Decrease in FV of machine (20.00) (20.00) (20.00)
Depreciation expense
-
(20 x l0% x 2.25), (20 x l0% x 3) 4.50 6.00
Adjustment for uniform accounting policy
- - 13.00
[58-45]
4,500.00 4,799.50 5,313.00

{
{
Post acquisition profit 299.50 513.50

W-2: Goodwill - BL Rs. in million


Cost 3,100
Net assets (4,500 (W-l) x 65%) (2,925)
175

W-3: Net Assets - FL Acquisition date Reporting date


Rs. in million
Share capital 2,500 2,500
Retained earnings 1,150 1,000
Contingent liability (50) (40)
3,600 3,460

Post-acquisition loss

W-4: Goodwill – FL
Cost (2,400×75%)
{ (140)

Rs. in million
1,800
Net assets [3,600 × 45% (60%×75%)] (1,620)
On acquisition 180
Impairment (W-8) (72)
On reporting date 108

W-5: Group reserves Rs. in million


AL 2,000.00
Post acquisition - BL (Up to Mar 2017) - [(299.5 (W-l) x 65%) 194.68
(Apr to Dec 2017) (513.5 (W-l) x 75%)] 385.12
Post acquisition - FL (140 (W-3) x 45%) (63.00)
Equity adjustment on further holding of 10% (W-6) 39.95
Gratuity expense (W-9) (8.00)
Impairment of goodwill of FL (W-8) (72.00)
2,476.75

© Emile Woolf International 157 The Institute of Chartered Accountants of Pakistan


Advanced accounting and financial reporting

W-6: Equity adjustment on further holding of 10% Rs. in million


Net assets acquired (4,799.5 (W-l) x 10%) 479.95
Cost (440.00)
Increase in equity 39.95

W-7: NCI Rs. in million


Acquisition - BL (4,500 × 35%) 1,575.00
Post acquisition (Up to Mar 2017) - BL [(299.5 (W-1) × 35%) 104.82
(Apr to Dec 2017) (513.5 (W-1) × 25%)] 128.38
10% further acquisition (4,799.5 (W-1) × 10%) (479.95)
Acquisition - FL (3,600 × 55%) 1,980.00
Post acquisition - FL (140 (W-3) × 55%) (77.00)
Indirect holding (2,400 × 25%) (600.00)
2,631.25

W-8: Impairment of Goodwill- FL Rs. in million


Grossing up of goodwill (180/0.45) 400
Net assets on 31 December 2017 (W-3) 3,460
3,860
Recoverable amount (3,700)
Notional write off 160
Impairment to be recorded (160x45%) 72

W-9: Gratuity scheme Rs. in million


Charge for the year (P&L and OCI)
Current service cost 85
Interest cost (25×12%) 3
Re-measurement gain (10)
78
Already charged to P&L
Contribution paid (70)
Net increase 8

© Emile Woolf International 158 The Institute of Chartered Accountants of Pakistan


Answers

CHAPTER 5 - ASSOCIATES AND JOINT VENTURES


5.1. JOINT ARRANGEMENTS
(a) (i) Joint Operations
A joint operation is a joint arrangement whereby the parties that have joint control of
the arrangement have right to the assets and obligations for the liabilities relating to
the arrangement. Those parties are called joint operators.
(ii) Joint venture
A joint venture is a joint arrangement whereby the parties that have joint control of
the arrangement have right to the net asset of the arrangement. These parties are
referred to as joint venturers.
(b) Elements to be recognised by a joint operator
(i) Its assets and share of any assets held jointly
(ii) Its liabilities and share of any liabilities incurred jointly
(iii) Its revenue from the sale of its share of the output arising from the joint operation
(iv) Its share of the revenue from the sale of the output by the joint operations
(v) Its expenses and share of any expenses incurred jointly.
(c) Characteristics of joint arrangements
(i) The parties are bound by a contractual arrangement
(ii) The contractual arrangement gives two or more of those parties joint control of the
arrangement.

5.2. HELIUM
Consolidated statement of financial position as at 31 December 2016
Rs.000
Assets
Non-current assets
Property, plant and equipment 500
Interest in associate (W6) 51
Goodwill 15
Current assets 605
———
Total assets 1,171
———
Equity and liabilities
Capital and reserves
Share capital 100
Retained earnings (W5) 737
———
837
Non-controlling interest 84
Long-term liabilities 250
———
Total equity and liabilities 1,171
———

© Emile Woolf International 159 The Institute of Chartered Accountants of Pakistan


Advanced accounting and financial reporting

Workings
(1) Group structure

Helium

30%
60%

Arsenic
Sulphur

(2) Net assets


Sulphur
Balance Post
sheet date Acquisition acquisition
Rs.000 Rs.000 Rs.000
Share capital 30 30 –
Retained earnings 180 70 110
—— ——
210 100
—— ——
(3) Goodwill
Sulphuric
Rs.000

Cost of investment 75
Share of net assets acquired
(60%  100 (W2)) (60)
——
15
——
(4) Non-controlling interest
Rs.000

Sulphur (40%  210) 84


——
(5) Retained earnings
Rs.000
Helium 650
Sulphur (60%  110 (W2)) 66
Arsenic (30%  (100 − 30)) 21
——
737
——
(6) Investment in associate
Rs.000
Cost 30
Share of post-acquisition profit (30%  (100 − 30)) 21
——
51
——

© Emile Woolf International 160 The Institute of Chartered Accountants of Pakistan


Answers

5.3. HAMACHI LTD


(a)
Hamachi Ltd
Consolidated statement of financial position as at 31 March 2016
Rs.000 Rs.000
Non-current assets
Property, plant and equipment (8,050 + 3,600) 11,650
Goodwill (W2) 702
Licence (180 – 60) (W3) 120
12,472
Investments
Associate (W6) 717
Others (4,000 + 910 – 3,240 – 630 + 120 FV) 1,160
1,877
14,349
Current assets
Inventory (830 + 340) 1,170
Accounts receivable (520 + 290 – 40) 770
Bank (240 + 40) 280
2,220
Total assets 16,569
Equity and liabilities
Equity attributable to equity holders of the parent:
Ordinary shares of Rs. 1 each 5,000
Retained earnings (W5) 8,415
13,415
Non-controlling interest (W4) 374
13,789
Non-current liabilities
10% Loan notes (500 + 240) 740
Current liabilities
Accounts payable (420 + 960) 1,380
Taxation (220 + 250) 470
Overdraft 190
2,040
Total equity and liabilities 16,569
Workings
(W1) Net assets in subsidiary
At end of
At reporting
acquisition period
Rs.000 Rs.000
Share capital 1,200 1,200
Retained earnings 800 2,300
Fair value adjustment:
Investment property 120 120
Licence 180 180
Amortisation of licence 180/6 x 2yrs (60)
2,300 3,740

© Emile Woolf International 161 The Institute of Chartered Accountants of Pakistan


Advanced accounting and financial reporting

(W2) Goodwill
Rs.000
Cost of investment (Rs. 3  1,200  90%) 3,240
Net assets acquired (90%  2,300) (W1) 2,070
Goodwill 1,170
Less impairment (468)
702
(W3) Unrealised profit in inventory
((2/3 × 65,000) × 30/130) × 30% = Rs. 3,000
Parent sells to associate, therefore reduce group retained earnings and Investment
in associate
(W4) Non-controlling interest
10%  3,740 = Rs. 374
(W5) Retained earnings
Rs.000
Hamachi Ltd 7,500
Saba Ltd – group share post-acquisition
90%  (3,740 – 2,300) 1,296
Anogo Ltd – group share post-acquisition
30%  (600  6/12) 90
Unrealised profit (W3) (3)
Less impairment (468)
8,415
(W6) Investment in associate
Rs.000
Investment at cost 630
Post-acquisition profit (30%  (600  1/2)) 90
Unrealised profit in inventory (3)
717
(b) IAS 28 Investments in Associates and Joint Ventures defines associates. In order for an
investment to be classified as an investment in an associate the investor must have
‘significant influence’ over the investee. Significant influence is presumed to exist where
there is a holding of 20% or more of the voting power unless the investor can clearly
demonstrate that this is not the case. Conversely a holding of less than 20% is presumed
not to be an associate, unless it can be clearly demonstrated that the investor can exercise
significant influence. The voting rights can be held directly or through subsidiaries.
IAS 28 says that a majority holding by one investor does not preclude another investor
having significant influence. An investing company owning a majority holding in another
company normally has control over the investee and would thus class it as a subsidiary. In
normal circumstances it is difficult to see how a company could be controlled by one entity
and be significantly influenced by a different entity unless ‘control’ was passive. The 20%
test is not definitive and the following other evidence should be considered.
Does the investing company:
 have representation on the Board of the investee?
 participate in the policy making processes (operational and financial); have material
transactions with the investee?
 interchange managerial personnel with the investee; or provide technical expertise to
the investee?

© Emile Woolf International 162 The Institute of Chartered Accountants of Pakistan


Answers

5.4. HIDE
Hide
Consolidated statement of profit or loss for the year ended 30 June 2016
Rs.000
Revenue 15,131
Cost of sales and expenses (13,580)
———
Operating profit before tax 1,551
Tax (736)
———
Profit after tax 815
Share of profit of associate (30% of 594) 178
———
Profit for the year 993
———

Profit for the year attributable to members of Hide 963


Non-controlling interest (W2) 30
———
Profit for the year 993
———
Workings
(1) Group structure

Hide

30%
80%
Arrive

Seek

(2) Consolidation schedule


Hide Seek (5/12) Adjustment Total
Rs. 000 Rs. 000 Rs. 000 Rs. 000
Revenue 12,614 2,567 (50) 15,131
Cost of sales
Per question (11,318) (2,302) 50
Unrealised profit
50  (25/125) (10) (13,580)

Tax (621) (115) (736)


Profit for the year 150
Non-controlling interest (%) 20%
Non-controlling interest (Rs. 000) 30

© Emile Woolf International 163 The Institute of Chartered Accountants of Pakistan


Advanced accounting and financial reporting

5.5. HARK, SPARK AND ARK


Hark Group
Consolidated statement of financial position as at 31 March 2016
Rs.000 Rs.000
Non-current assets
Property, plant and equipment (working 1) 90,200
Goodwill (working 4) 23,000
Investment in associate (working 6) 9,500
Other investments 650
123,350
Current assets (working 5) 24,300
Total assets 147,650
Equity and liabilities
Equity shares of Rs. 1 each (working 3) 21,000
Share premium (working 3) 42,000
Retained earnings (working 8) 43,730
85,730
106,730
Non-controlling interests (working 7) 7,420
Total equity 114,150
Non-current liabilities
Deferred consideration for Spark shares 5,500
6% loan notes 10,000
7% loan notes 6,000
21,500
Current liabilities: 7,000 + 5,000 12,000
Total equity and liabilities 147,650
Workings
1 Property, plant and equipment (PPE)
Rs.000 Rs.000
Hark 60,000
Spark 31,000
Profit on transfer of machines (3 million – 2 million) 1,000
Less: Depreciation on this amount in accounts of Spark
(1,000/5 years) (200)
Unrealised profit in machines (800)
PPE in consolidated statement of financial position 90,200

2 Deferred consideration
The present value of the deferred consideration at 1 April 2015 is Rs. 6.05 million 
1/(1.10)2 = Rs. 5 million.
During the year to 31 March 2016 there is a finance charge of 10% (= Rs. 500,000) on this
amount, reducing the parent’s share of the consolidated profit.
The deferred consideration at 31 March 2016 is Rs. 5 million + Rs. 500,000 = Rs.
5,500,000. This is payable in just over 12 months and is included in the consolidated
statement of financial position as a non-current liability.

© Emile Woolf International 164 The Institute of Chartered Accountants of Pakistan


Answers

3 Share issues
The share issues to acquire the shares in Spark and Ark are not recorded in the summary
statement of financial position of Hark (as stated in the question).
Share Share
Total capital premium
To acquire the shares in Spark Rs.000 Rs.000 Rs.000
Hark shares issued: (4 million at Rs. 9) 36,000 4,000 32,000
To acquire the shares in Ark
Hark shares issued: (1 million at Rs. 9) 9,000 1,000 8,000
Increase in share capital and share premium
of Hark 5,000 40,000
In summary statement of financial position 16,000 2,000
In consolidated statement of financial
position 21,000 42,000

4 Goodwill
Hark has acquired 4 million/5 million = 80% of the shares of Spark.
At 1 April 2015 the fair value of the net assets of Spark was (share capital plus reserves) =
Rs.(5 + 4 + 16) million = Rs. 25 million
Rs.000
Purchase consideration paid by the parent company
Issue of 4 million shares at Rs. 9 36,000
Deferred consideration 5,000
41,000

Fair value of parent company share of net assets


(80%  Rs. 25 million) 20,000
Purchased goodwill attributable to parent 21,000

Rs.000
Fair value of NCI at acquisition date (1 million shares  Rs. 7) 7,000
NCI share of net assets at this date (20%  Rs. 25 million) 5,000
Purchased goodwill attributable to NCI 2,000

There has been no impairment of goodwill during the year.


Rs.000
Purchased goodwill attributable to parent 21,000
Goodwill attributable to NCI 2,000
Total goodwill in consolidated statement of financial position 23,000
Alternatively, total goodwill could be calculated as follows:
Rs.000
Purchase consideration paid by the parent company 41,000
(see above)
Fair value of NCI at acquisition date 7,000
48,000
Net assets of the subsidiary at the acquisition date 25,000
(at fair value)
Total goodwill (parent and NCI) 23,000

© Emile Woolf International 165 The Institute of Chartered Accountants of Pakistan


Advanced accounting and financial reporting

5 Current assets
The cost of the goods sold by Spark to Hark was Rs. 3,600,000  100/150 = Rs. 2,400,000
and the profit was Rs. 1,200,000.
Since 75% of these goods are in closing inventory, the unrealised profit on intra-group
sales is 75%  Rs. 1,200,000 = Rs. 900,000. Current assets in the consolidated statement
of financial position (inventory) should be reduced by this amount.
The question states that the transaction costs of the acquisition of Spark have not yet been
recorded. These costs reduce the consolidated profit, and also (presumably) reduce the
current assets of Hark.
Current assets on consolidation Rs.000
Hark 18,200
Spark 8,000
Less: unrealised profit in closing inventory (900)
Less: expenses of acquisition of Spark (1,000)
Current assets in consolidated statement of financial position 24,300

6 Investment in associate (Ark)


Since Hark owns 25% of the equity of Ark, it is assumed that Ark is an associated entity.
Rs.000
Cost of investment: 25%  6 million shares  Rs. 6 9,000
Share of post-acquisition retained profit: 25%  Rs. 2 million 500
9,500
7 Non-controlling interests
Rs.000
Share of net assets of Spark at 31 March 2016 (20%  Rs. 28 million) 5,600
Goodwill attributable to NCI (working 4) 2,000
7,600
NCI share of unrealised profit in inventory (20%  Rs. 900,000) (180)
NCI at 31 March 2016: fair value method 7,420

8 Consolidated retained earnings


Rs.000 Rs.000
Hark retained earnings (36,000 + 8,000) 44,000
Spark
Profit for year ended 31 March 2016 3,000
Unrealised profit in closing inventory (900)
2,100
Parent company share (80%) 1,680
Share of post-acquisition retained profits of Ark 500
(25%  Rs. 2 million)
Costs of acquisition of Spark (expensed) (1,000)
Additional finance costs: deferred consideration (500)
Unrealised profit in machines (working 1) (800)
Loss on other (800 – 650) (150)
Consolidated retained earnings at 31 March 2016 43,730

© Emile Woolf International 166 The Institute of Chartered Accountants of Pakistan


Answers

5.6. P, S AND A
P Group
Consolidated statement of financial position as at 31 December Year 5
Assets
Non-current assets Rs.
Property, plant and equipment (450,000 + 240,000) 690,000
Goodwill (W3) 45,000
Investment in associates (W5) 168,800
903,800
Current assets
Inventory (70,000 + 90,000 – 10,000) 150,000
Other current assets (20,000 + 110,000 + 130,000) 260,000
Total assets 1,313,800
Equity and liabilities
Equity
Share capital 100,000
Share premium 160,000
Consolidated accumulated profits (W6) 711,300
Attributable to equity holders of the parent 971,300
Non-controlling interest in S (W4) 102,500
Total equity 1,073,800
Long-term liabilities (40,000 + 20,000) 60,000
Current liabilities (100,000 + 80,000) 180,000
Total equity and liabilities 1,313,800

Workings
P owns 75% of the equity of S and 30% of the equity of A. Therefore S is a subsidiary and A is
an associate.
W1: Net assets summary
Calculate the net assets of S and A at the acquisition date and at the end of the reporting period.
At this stage, make any fair value adjustments and eliminate the unrealised profit in inventory.
At date of At date of Post-
Net assets of S consolidation acquisition acquisition
Rs. Rs. Rs.
Equity shares 200,000 200,000 -
Share premium 80,000 80,000 -
Accumulated profits (per question) 140,000 60,000 80,000
410,000 340,000

W2: Unrealised profit on inter-company trading


Sale by S to P: Rs. 40,000 × 33.33/133.33 = Rs. 10,000.
Dr Cr
Consolidated accumulated profits (75%) 7,500
Non-controlling interest (25%) 2,500
Consolidated inventory 10,000

© Emile Woolf International 167 The Institute of Chartered Accountants of Pakistan


Advanced accounting and financial reporting

Sale by P to A: Rs. 16,000 × 33.33/133.33 = Rs. 4,000.


P’s share: Rs. 4,000  30% = Rs. 1,200
Dr Cr
Consolidated accumulated profits (75%) 1,200
Investment in associate 1,200
W3: Goodwill on acquisition of S
Rs.
Cost of the acquisition 320,000
Non-controlling interest at acquisition (25%  340,000 (W1)) 85,000
405,000
Less: Fair value of identifiable net assets at acquisition (W1) (340,000)
Goodwill at acquisition 65,000
Minus: Impairment to date (20,000)
Balance carried forward 45,000

W4: Non-controlling interest in S

Rs.
Non-controlling interest at acquisition (25%  340,000 (W1)) 85,000
Share of post-acquisition profits (25%  80,000 (W1)) 20,000
Unrealised profit (W2) (2,500)
102,500

W5: Investment in associate

Rs.
Investment at cost 140,000
P’s share of post-acquisition accumulated profits 30,000
(30%  (250,000 – 150,000)
Unrealised profit (W2) (1,200)
168,800

W6: Consolidated accumulated profits.

Rs.
Accumulated profits of P 650,000
P’s share of post-acquisition profits of S
(75% × Rs. 70,000 (W2)) 60,000
Unrealised profit (sale by S to P (W3)) (7,500)
P’s share of post-acquisition accumulated profits (W5) 30,000
Unrealised profit (W2) (1,200)
Impairment of goodwill (20,000)
Consolidated accumulated profits 711,300

© Emile Woolf International 168 The Institute of Chartered Accountants of Pakistan


Answers

5.7. H LTD GROUP


Consolidated statement of profit or loss and other comprehensive income for the H Ltd group for
the year ended 31 May 2016
Rs.000
Revenue (6,000 + 3,000) 9,000
Cost of sales (4,800 + 2,400) (7,200)
Gross profit 1,800
Distribution costs (64 + 32) (96)
Administrative expenses (336 + 168) (504)
Finance costs (30 + 15) (45)
Share of profit of associate (30% x 100) 30
Profit before tax 1,185
Income tax expense (204 + 102) (306)
PROFIT FOR THE YEAR 879
Other comprehensive income:
Revaluation of PPE (200 + 100) 300
Actuarial gain on pension plan assets 40
Actuarial loss on pension plan liabilities (52)
Gain on AFS investment 14
Tax effect of other comprehensive income (42 + 21) (63)
Share of OCI of associate (net of tax) (30% x 24) 7
Other comprehensive income for the year, net of tax 246
TOTAL COMPREHENSIVE INCOME FOR THE YEAR 1,125

Profit for the period attributable to: Rs.000


Owners of the parent entity 822.4
Non-controlling interests (20% x 283) 56.6
879

Total comprehensive income attributable to:


Owners of the parent entity 1052.6
Non-controlling interests (20% x 362) 72.4
1,125

5.8. ALPHA AND BETA


Alpha Limited
Statement of financial position
As on 30 June 2016

Rs. in million
Property, plant and equipment [2,650+(750×0.8)] 3,250.00
Goodwill (W-1) 11.00
Stock in hand [695+(250×0.8) – 56.45(W-2)] 838.55
Other assets [570 + (180 × 0.8) – (320 × 0.8) – (150 × 0.8)] 338.00
Investment in SV-2 (200+305)× 0.5- 11(W-2) 11 (W-2) OR
443-200 -140+[305×50%]-3 -11(W-2) 241.50
4,679.05

© Emile Woolf International 169 The Institute of Chartered Accountants of Pakistan


Advanced accounting and financial reporting

Capital 2,000.00 2,000.00


Accumulated profit (W3) 1,310.05
10% bank loan [500 + (320 × 0.8)] 756.00
Current liabilities [665 + (405 × 0.8) – (320 × 0.8) – (150 × 0.8)] 613.00
4,679.05

Alpha Limited
Statement of comprehensive income
For the year ended 30 June 2016

Rs. in million
Sales [4250 + (650×0.8) –502(W-2)] 4,268.00
Less: Cost of sales [2,993 + (480 × 0.8) – 437.4(W-2)] (2,939.60)
Gross profit 1,328.40
Less: Expenses [657 + (145×0.8) + 3] (776.00)
Add: Share of profit in SV-2 [(50 × 0.5) – 2.85(W-2)] 22.15
Net profit 574.55

W-1: Computation of goodwill on further investment in SV-1

Rs. in million
Consideration paid (excluding acquisition related costs) 140.00
Less: Further share of BL acquired [400 + (55 – 25)] × [50% × 60%] 129.00
Goodwill 11.00

W-2: Adjustments to be made due to intercompany transactions

Share of
Cost of Investment profit Cost of sales
Sales
sales in SV – 2 from adjustments
associate
………………… Rs. In million……………
Joint venture
AL to CV – 2 - 11.00 (11.00)
(110 x 0.2 x 0.5)
SV-2 to AL - (2.85) (2.85)
(38 x 0.15 x 0.5)

Joint operator
AL to SV – 1 (350.00) (350.00) (17.60)
17.60 (220x0.10x0.8)
SV-1 to AL (152.00) (152.00) (36.00)
(190x 0.8) 36.00 (150 x0.3 x0.8)
(502.00) (437.40) (11.00) (2.85) (56.45)

© Emile Woolf International 170 The Institute of Chartered Accountants of Pakistan


Answers

W3 Accumulated Profit

Parent Reserves Rs. In million


Parent Reserves opening (1,193-600) 593.00
Post acquisition income from SV1 (55-25)*0.5 15.00
Post acquisition share of profit from SV2 (305 – 50)*0.5 127.50
Current year income 574.55
1,310.05

5.9. SNAKE LIMITED

Total Total
Total assets comprehensive
liabilities income
--------------- Rs. in million ---------------
Given 2,500.00 1,610.00 659.00

Investment in associate
Share of profit during 2017 (W-1) 45.00 45.00
Disposal (W-1) (302.60) (302.60)
(257.60) (257.60)
Revised amounts 2,242.4 1,610.00 401.4

W-1: Disposal of associate Rs. in million


Disposal proceeds 290.00
FV of investment retained (0.8 × 128) 102.40
392.40
Carrying amount as at 31 December 2017 (W-2) (405.00)
Loss on disposal of associates 12.60
Gain already recorded to be reversed 290.00
302.60

W-2: Carrying amount of associate Rs. in million


Cost (3 × 200) 600.00
Share of loss till 31 December 2016 [(1200 – 1700 ) × 30%] (150.00)
Impairment (90.00)
Carrying amount as at 31 December 2016 360.00
Share of profit for 2017 [(1350 – 1200) × 30%] 45.00
Carrying amount as at 31 December 2017 405.00

© Emile Woolf International 171 The Institute of Chartered Accountants of Pakistan


Advanced accounting and financial reporting

CHAPTER 7 - IAS 21: FOREIGN CURRENCY


7.1. DND LIMITED
Date Description Dr. Cr.
Rs. Rs.
1-Jul-16 Advance to suppliers 2,060,000
Cash 2,060,000
(Amount paid on signing the contract. Exchange
rate was Rs. 103/US$)
30-Sep-16 Advance to suppliers 5,175,000
Cash 5,175,000
(Amount paid on delivery. Exchange rate was Rs.
103.5/US$)
30-Sep-16 PPE in transit/ CWIP 10,340,000
Advance to suppliers 7,235,000
Payable to suppliers 3,105,000
(Recording of asset on the delivery date as risk
and rewards are transferred to the company)
31-Dec-16 Exchange loss 15,000
Payable to suppliers 15,000
(Adjustment of exchange rate as of reporting date.
Exchange rate was Rs.104/US$)
31-Jan-17 Property, plant and Equipment 10,340,000
PPE (In transit/ in progress) 10,340,000
(Transfer the new plants and machineries to
Property, Plant and Equipment)
31-Jan-17 Payable to suppliers 3,120,000
Exchange loss (Bal.) 15,000
Cash 3,135,000
(Final payment to supplier. Exchange rate was
Rs. 104.5/US$1)

7.2. STARLIGHT LIMITED


(a) Starlight Limited. Translated Profit and Loss Account
QR Rate Rs.’000
Turnover 344,880 32 11,036
Cost of Sales (249,710) 32 (7,991)
Gross Profit 95,170 3,045
Expenses (29,490) 32 (944)
Profit Before Tax 65,680 2,102
Taxation (17,325) 32 (554)
Profit After Tax 48,355 1,547
Interim dividend (16,300) 32 (522)
Retained profit for the year 32,055 1,026

© Emile Woolf International 172 The Institute of Chartered Accountants of Pakistan


Answers

(b) (i) Calculation of goodwill


Cost of investment: QR'000 QR'000
(Rs. 2,500,000 x 30 QR) 75,000
Less net assets acquired:
Share capital 20,250
Pre–acquisition reserves 49,300
69,550
Group share 80% thereon (55,640)
Goodwill in QR 19,360
Closing conversion rate to PKR 33
Goodwill in PKR 638,880
(ii) Non–controlling Interest in statement of profit or loss:
20% x Rs. 1,547,000 = Rs. 309,400

7.3. PERCEPT LTD


Translation of financial position of Trint Ltd as at 31 December 2016
(a) YEN YEN RATE
Rs.’000 Rs.’000 Rs.’000
Property plants and equipment 12,375 0.8 9,900
Financial assets 1,250 0.8 1,000
Current assets 8,250 0.8 6,600
21,875 17,500
Share capital 5,000 0.9 4,500
Retained earnings 4,500 0.9 4,050
Fair value adjustment 2,875 0.9 2,588
Post-acquisition (Balancing figure) 3,000 1,163
Non-current liabilities 4,000 0.8 3,200
Current liabilities 2,500 0.8 2,000
Note: Fair value of property,
plant and equipment: = 12,375 Yen – 5,000 yen (equity) - 4,500
Yen (pre-acquisition profit) = 2,875 Yen
(b) Goodwill
Yen Rate Rs.
‘000 ‘000
Cost of acquisition 6,900 0.9 6,210
Fair value (NCI) 6,250 0.9 (5,625)
13,150 11,835
Net asset acquired 12,375 0.9 (11,138)
775 0.9 698
Retranslated at the closing rate 775 0.8 620
Exchange gain on retranslation of goodwill 78
Parent’s share (70% x 78) 54.6
Non-controlling interest share (30% x 78) 23.4
78

© Emile Woolf International 173 The Institute of Chartered Accountants of Pakistan


Advanced accounting and financial reporting

(c) Exchange rate difference arising on re-translation of Trint Ltd’s net assets

Difference from translation of opening net assets

Rs.’000
Opening rate: 12,375,000 Yen at 0.9 11,137.5
Closing rate: 12,375,000 Yen at 0.8 9,900
Exchange gain 1,237.5
Difference arising from translation of profit
Average rate (2,000 Yen at 0.85) 1,700
Closing rate (2,000 Yen at 0.8) 1,600
Exchange gain 100
Total exchange gain 1,337.5
Parent’s share of the exchange gain (70% of 1,337.5) 401.25
Non-controlling interest share of the exchange gain (30% x 1,337.5) 936.25
1,337.50

7.4. ORLANDO
(a) Year to June Year 4
The revenue and the receivable for the sale of €96,000 should be translated at the spot
rate of 0.8 = $120,000
The capital expenditure of €1m should also be translated at the spot rate of 0.8:
Debit Property, plant and equipment $1,250,000
Credit: Payables $1,250,000.
The receipt on 12 June relating to the receivable is translated at the rate at that date of 0.9.
This generates cash of $106,667 to settle a receivable of $120,000. Hence an exchange
loss of $13,333 is recognised in profit or loss.
The non-current asset is not re-translated at the year end, but the outstanding payable (a
monetary item) must be re-stated to the year end exchange rate of 0.7. This gives a year-
end payable balance of $1,428,571. This has increased from the initial $1,250,000;
therefore an exchange loss of $178,571 will be recognised in profit or loss.

(b) Year to June Year 5


When the payable is settled after the year end at the spot rate of 0.8, it results in a
payment of $1,250,000. There is an exchange gain of $178,571 compared with the
carrying value at the end of Year 4.

7.5. MANCASTER AND STOCKPOT


Part A
(1) (a) Functional currency
Functional currency is the currency of the primary economic environment in which
the entity operates. It reflects the underlying transactions, events and conditions that
affect the company. It is not simply the currency of the country where the company
is based. For example, if a company is incorporated and listed in Pakistan but
operates in the South African mine fields, then its functional currency will be the
South African rand, not rupees, because the rand is the currency in which it
generates and spends the most cash.

© Emile Woolf International 174 The Institute of Chartered Accountants of Pakistan


Answers

(b) Presentation currency


Presentation currency is the currency in which the financial statements are
presented. Continuing the example of the company referred to above, since the
company is listed in Pakistan it will present and file its accounts in rupees.
Transactions and balances measured in a currency other than the presentation
currency will have to be translated into the presentation currency for reporting
purposes. So the sales and operating costs incurred in South Africa will need to be
translated from Rand into Sterling.
(2) Factors to be considered when determining the functional currency of an overseas
subsidiary
The primary economic environment in which an entity operates is defined in IAS 21 as the
one in which it primarily generates and expends cash.
Primary indicators
An entity must consider the following factors in determining its functional currency:
 the currency that mainly influences sales prices for goods and services
 the country whose competitive forces and regulations mainly determined the sales prices of its
goods and services
 the currency that mainly influences labour, material and other costs of goods or services.
Additional indicators
The following factors may also provide evidence:
 the currency in which funds from financing activities are generated (i.e. currency used for issuing
debt and equity)
 the currency in which surplus cash is invested.
Part B
(a) Translation: Statement of financial position of Stockpot at 31 March Year 4

EU000 Rate $000


Property, plant and equipment 30,000 2.2 13,636
Inventories 18,000 2.2 8,182
Trade receivables 15,000 2.2 6,819
Trade payables (10,400) 2.2 (4,727)
Bank overdraft (7,600) 2.2 (3,455)
Non-current liabilities (20,000) 2.2 (9,091)
25,000 11,364
Issued capital 15,000 3.0 5,000
Pre-acquisition reserves 5,000 3.0 1,667
20,000 6,667
Post-acquisition reserves 5,000 balancing figure 4,697
25,000 11,364

© Emile Woolf International 175 The Institute of Chartered Accountants of Pakistan


Advanced accounting and financial reporting

Mancaster Group: Consolidated statement of financial position at


31 March Year 4

$000 $000
Non-current assets
Goodwill (see workings) 682
Property, plant and equipment (20,000 + 13,636) 33,636
$000 $000
Current assets:
Inventories (10,000 + 8,182) 18,182
Trade receivables (10,000 + 6,819) 16,819
35,001
69,319
$000
Capital and reserves:
Issued capital 9,000
Accumulated profits (see workings) 16,205
25,205
Non-controlling interest (see workings) 2,841
28,046
Non-current liabilities:
Loans (10,000 + 9,091) 19,091
Current liabilities:
Bank overdraft (6,100 + 3,455) 9,555
Trade payables (7,900+4,727) 12,627
22,182
69,319

(b) Translation: Statement of profit or loss of Stockpot for year ended 31 March Year 4

EU000 Rate $000


Revenue 60,000 2.3 26,087
Cost of sales (30,000) 2.3 (13,043)
Gross profit 30,000 13,044
Operating expenses (16,000) 2.3 (6,957)
Operating profit 14,000 6,087
Interest payable (2,000) 2.3 (870)
Profit before tax 12,000 5,217
Tax (4,200) 2.3 (1,826)
Profit after tax 7,800 3,391

The statement of profit or loss has been translated at the average rate as an
approximation to the actual (historical) rate. The closing rate is not allowed under IAS 21.

© Emile Woolf International 176 The Institute of Chartered Accountants of Pakistan


Answers

Mancaster Group: Consolidated statement of profit or loss for the year ended 31
March Year 4
$000
Revenue (50,000 + 26,087) 76,087
Cost of sales (25,000 + 13,043) (38,043)
Gross profit 38,044
Operating expenses (15,000 + 6,957) (21,957)
Operating profit 16,087
Interest payable (1,000 + 870) (1,870)
Profit before tax 14,217
Tax (3,600 + 1,826) (5,426)
Profit after tax 8,791
Attributable to
Equity holders of the parent 7,943
Non-controlling interest (25% × 3,391) – see translation 848
8,791

Workings
(1) Goodwill at date of acquisition

$000 Rate EU000


Cost of investment 5,500 3 16,500
Minus: Share of net assets acquired:
Share capital (translated at 3.0) 5,000
Accumulated profits (translated at 3.0) 1,667
6,667
Group share (75%) 5,000 3 (15,000)
Goodwill 500 1,500
Re-stated to closing rate: (1,500/2.2) 682
Translation gain on goodwill – to group reserves 182

(2) Consolidated accumulated profits

$000
Mancaster: 12,500
Stockpot: group share of post-acquisition profits (75% ×
4,697) – see translation of statement of financial position 3,523
Translation gain on goodwill 182
16,205

(3) Non-controlling interest

$000
Non-controlling share of net assets at 31 March Year 4 :
(25% × 11,364) – see translation of Stockpot statement of
financial position 2,841

© Emile Woolf International 177 The Institute of Chartered Accountants of Pakistan


Advanced accounting and financial reporting

7.6. A, B AND C
A group: Summarised consolidated statement of profit or loss and other comprehensive income
for the year ended 30 September 2016

Rs.000
Revenue (4,600 +3,385(W1)) 7,985
Costs and expenses (3,700+2,462(W1)) (6,162)
Share of associate’s profit (W3) 160
Profit before tax 1,983
Income tax expense (200+231(W1)) (431)
Profit for the year 1,552

Other comprehensive income


Revaluation gains net of tax (200+185(W1)) 385
Share of associate’s OCI (W3) 28
Forex gain in year (W4) 803
Total other comprehensive income 1,216
Total comprehensive income 2,768

Profit for year attributable to:


Equity holders of the parent 1,414
Non-controlling interest (W5) 138
1,552

Total comprehensive income attributable to:


Equity holders of the parent 2,432
Non-controlling interest (W5) 336
2,768

Consolidated statement of financial position for the A group as at 30 September 2016


Rs.000
Assets
Non-current assets
Property, plant and equipment (7,000 + 6,349 (W1)) 13,349
Goodwill (W2) 635
Investment in associate (W6) 1,220
15,204
Current assets (3,000 + 3,175 (W1)) 6,175
Total assets 21,379
Equity and liabilities
Equity attributable to the parent
Share capital 2,000
Retained reserves (W8) 13,522

© Emile Woolf International 178 The Institute of Chartered Accountants of Pakistan


Answers

15,522
Non-controlling interest (W7) 1,476
Total equity 16,998
Current liabilities (2,000 + 2,381(W1)) 4,381
Total equity and liabilities 21,379

Rate @ avge
W1 Translation of B
A$000 rate Rs.000
Statement of profit or loss and other
comprehensive income
Revenue 2,200 Rs./A$0.65 3,385
Cost of sales and expenses (1,600) Rs./A$0.65 (2,462)
Profit before tax 600 923
Income tax (150) Rs./A$0.65 (231)
Profit for year 450 692
Other comprehensive income:
Revaluation gains on PPE Total OCI 120 Rs./A$0.65 185
120 185
Total comprehensive income 570 877
Statement of financial position
Non-current assets
Property, plant and equipment 4000 @CR A$0.63 6349
Current assets 2,000 @CR A$0.63 3,175
Total assets 6,000 9,524
Share capital 1000 @HR A$0.50 2000
Pre-acquisition reserves 1800 @HR A$0.50 3600
Post-acquisition reserves 1,700 Bal fig 1,543
Total equity 4,500 7143
Current liabilities 1,500 @CR A$0.63 2,381
Equity and liabilities 6,000 9524

W2 Goodwill A$000 Rate Rs.000


Consideration transferred 2,600 Rs./A$0.50 5,200
NCI @ FV 600 Rs./A$0.50 1,200

Net assets acquired:


Share capital (1,000) Rs./A$0.50 (2,000)
Retained earnings (1,800) Rs./A$0.50 (3,600)
Goodwill at 1 October 2013 400 800
Forex loss (balancing figure) (237)
Goodwill at 30 September 2015 400 Rs./A$0.71 563
Forex gain (balancing figure) 72
Goodwill at 30 September 2016 400 Rs./A$0.63 635

© Emile Woolf International 179 The Institute of Chartered Accountants of Pakistan


Advanced accounting and financial reporting

W3 Share of associate’s profit/OCI Rs.000


Share of associate’s PFY (40% x Rs. 400,000) 160
Share of associate’s other comprehensive income (40% x Rs. 70,000) 28

W4 FOREX gains/losses in the year Rs.000


Closing net assets @ CR (A$4,500,000/0.63) or from W1 7,143
Less opening net assets @ OR ((A$4,500,000 less TCI A$570,000)/0.71) (5,535)
Less TCI for year @ average rate (A$570,000/0.65) (877)
Forex gain on translation of subsidiary’s net assets 731
Plus Forex gain on translation of goodwill 72
Total Forex gains on translation of subsidiary 803

W5 NCI share of Profit/Total comp income PFY TCI


Rs.000 Rs.000
Subsidiary’s PFY/TCI (W1) 692 877
20% share 138 175
Forex gain on translation of subsidiary (20% x Rs. 803,000) 161
138 336

W6 Investment in associate Rs.000


Investment at cost 900
Plus share of post-acquisition reserves 40% x (Rs. 1,500,000 -Rs. 700,000) 320
1,220

W7 Non-controlling interest Rs.000


NCI on acquisition (W2) 1,200
NCI share of post-acquisition reserves of subsidiary
(20%xRs. 1,543,000(W1)) 309
NCI share of net FOREX losses on translation of goodwill
(20% x Rs.(237,000-72,000)) (33)
NCI at 30 September 2016 1,476

W8 Reserves A B
Rs.000 Rs.000
As per SOFP 12,100 5,143

Less pre-acquisition reserves (W1) (3,600)


1,543
Group share 80% x Rs. 1,543,000 1,234
Group share of associate’s post-acquisition reserves 320
(W6)
Group share of net FOREX losses on translation of
goodwill (80% x Rs.(237,000-72,000)) (132)
Group reserves 13,522

© Emile Woolf International 180 The Institute of Chartered Accountants of Pakistan


Answers

7.7. OMEGA LIMITED


Omega Limited
Extract from Statement of comprehensive income for the year ended 31 December 2013
 Profit for the year: Rupees
Dividend received from AWL (IFRS 9,B5.7.5.1) (20,000*10*15%*26.5) 795,000
Transfer of FV gain reserve of 31-12-2012, on derecognition of AWL
investment W.1 500,000
FV / exchange gains on valuation of AWL shares on 1-6-2013 W.1 2,124,000
Loss on de-recognition of AWL' shares W.1 ( 308,000)
 Other comprehensive income:
FV gain/(loss) on investment available for sale W.1 693,000
Exchange gain on investment available for sale W.1 225,225

W-1 FV per Gain /


No. of Investment
Date share (loss) Remarks
shares
AED AED Conv.@ Rupees Rupees
1-May-2012 20,000 12.00 240,000 25.00 6,000,000
31-Dec-2012 20,000 13.00 260,000 25.00 6,500,000 500,000 FV gain
1-Jun-2013 22,000 14.00 308,000 28.00 8,624,000 2,124,000 Gain on
(20,000x1.1) valuation of
AWL on its
acquisition
by HL
1-Jun-2013 16,500 18.00 297,000 28.00 8,316,000 Loss on de-
(22,000/4*3) (308,000) recognition
of
AWL
shares
31-Dec-2013 16,500 19.50 321,750 28.00 9,009,000 693,000 FV gain
31-Dec-2013 16,500 19.50 321,750 28.70 9,234,225 225,225 Exchange
gain
3,234,225

7.8. PARENT COMPANY LIMITED


(i) Parent Company Limited
Consolidated statement of financial position as at 30 June 2014
Assets Rs. in million
Non-current assets
Property, plant and equipment 4,200+3,500+250×17.3 12,025.00
Intangible assets (W-1) 796+1,730 2,526.00
Current assets 3,500+4,000+450×17.3 15,285.00
29,836.00

© Emile Woolf International 181 The Institute of Chartered Accountants of Pakistan


Advanced accounting and financial reporting

Equity and liabilities

Equity attributable to owners of PL:

Ordinary shares capital 6,000.00

Retained earnings (W-4) 5,565.15

Exchange reserve [(W-1) 253+(W-2) 813.20]] × 75% 799.65

12,364.80
Non-controlling interest W-4 (731.20+2,050) 2,781.20

15,146.00

Current liabilities 4,700+4,800+300×17.3 14,690.00

29,836.00

(ii) Parent Company Limited


Consolidated statement of other comprehensive income

For the year ended 30 June 2014

Other comprehensive income: Rs. in million

Items that may be translated to profit or loss:

 Exchange gain on translation of goodwill W-1 55.00

 Exchange gain on translating of foreign operations W-2 195.80

250.80

W-1: Goodwill and exchange gain thereon LS FS

---------- Rs. in million ----------

Purchase consideration 2,000 (300 × 15) 4,500

NCI fair value on acquisition date 540 (90 × 15) 1,350

2,540 5,850

Net assets on acquisition date (1,800/1.2+250) (1,750) (120+260) × 15 (4,200)

Estimated liability for a pending claim 6 -

Goodwill on acquisition date i.e. 1 July 2012 1,650

Impairment on 30 June 2014 - (10 × 17.3) (173)

Goodwill as at 30 June 2014 1,477

Goodwill as at 30 June 2014 @ Rs. 17.30 796 (100×17.3) 1,730

Exchange gain reserve as 30 June 2014 253

Exchange gain reserve as 30 June 2013 (148.5 × 75%) (198)

Exchange gain for the year 55

© Emile Woolf International 182 The Institute of Chartered Accountants of Pakistan


Answers

W-2: Exchange reserve on translation of FS foreign operations


CU in million Conversion @ Rs. in million
Net assets as at 30 June 2014 400.00 17.30 6,920.00
Net assets as at 30 June 2013
(400-30+18) 388.00 16.80 6,518.40
Profit for the year 30.00 17.00 510.00
Dividend paid during the year (120×15%) (18.00) 16.90 (304.20)
400.00 6,724.20
Exchange gain for the year ended 30
Jun 2014 195.80
Exchange reserve as at 30 June 2013 (463.05/75%) 617.40
Exchange gain on foreign operations
as at 30 June 2014 813.20

W-3: FS retained earnings CU in million Conversion @ Rs. in million


Net assets as at 30 June 2014 400.00 17.30 6,920.00
Net assets on acquisition date (280.00) 15.00 (4,200.00)
Post-acquisition retained earnings as at
30 June 2014 including OCI item of
exchange gain 120.00 2,720.00
Exchange gain to be classified to OCI W-2 (813.20)
Post-acquisition retained earnings as at
30 June 2014 1,906.80

W-4: Consolidated retained earnings Retained


NCI - LS NCI - FS
and NCI earnings
----- Rs. in million -----
Balance as at 30 June 2014 3,500.00 - -
NCI fair value on acquisition date - 540.00 1,350.00
Post-acquisition profit – LS (650×80%) 520.00 130.00 -
Post-acquisition profit – FS
(1,906.8×75%) 1,430.10 - 476.70
LS earnings used for bonus issue
240.00
(300×80% /20%) 60.00 -
Liability paid in May 2014 booked on acq.
4.80
(6×80%/20%) 1.20 -
Exchange gain on translation of FS
- 266.55
(W-2) 1,066.20×25% -
Goodwill impairment (173×75%/25%) (129.75) - (43.25)
5,565.15 731.20 2,050.00

© Emile Woolf International 183 The Institute of Chartered Accountants of Pakistan


Advanced accounting and financial reporting

7.9. KANGAROO LIMITED


Statement of Financial Position
As on 31 December 2017
Assets Rs. in million
Investment property (W-1) 290.00
Investments (105 + 130) (W-2) 235.00
Liabilities
Unearned rent (0.24 × 8 ÷12 × 110) 17.60
Statement of comprehensive income
For the year ended 31 December 2017
Profit and Loss account Rs. in million
Exchange loss on 20% payment (2.6 × 20% × (105 – 108) (1.56)
Increase in fair value of investment property (W-1) 18.30
Rent income (0.24 × 4÷12 × 110) 8.80
Transaction cost – Investment-A (2.00)
Dividend income (12 + 9) 21.00
Realised gain on investment-A [(23 × 0.98 – (100 × 20%)] 2.54
Unrealised Gain – Investment-A (W-2) 25.00

Other comprehensive income


Unrealized gain- Investment-B (W-2) 22.90
Realised gain on investment-B [(50 × 0.98 – (153 × 0.3)] 3.10

W-1: Investment company Rs. in milllion


Advance payment (2.6 × 10% × 100) 26.00
Initial recognition (2.6 × 70% × 105) 191.10
(2.6 × 70% × 105) 54.60
Total cost 271.70
Fair value (2.5 x 116) 290.00
Gain (P & L) 18.30

W-2: Investments Investment A Investment B


--------- Rs. in million ---------
Purchase price 100.00 150.00
Transaction cost 3.00
Total cost 100.00 153.00
Cost of shares held at 31 Dec 2017 (100×80%) 80.00 (153×70%)107.10
Fair value - 31 Dec 2017 105.00 130.00
Gain 25.00 22.90

© Emile Woolf International 184 The Institute of Chartered Accountants of Pakistan


Answers

CHAPTER 8 - IAS 7: STATEMENTS OF CASH FLOWS


8.1. EVERNEW LTD
Consolidated statement of cash flows for the year ended 31 December 2016

Rs.’000 Rs.’000
Profit before taxation 138,960
Adjustment for non-cash items:
Depreciation charges 72,720
Profit on disposal of subsidiary (W.1) (5,040)
Interest expenses (payable) 10,080
Operating profit before working
Capital changes 216,720
Changes in working capital
Increase in inventory (W2) (28,800)
Increase in Receivables (W2) (32,400)
Increase in Creditors (W2) 25,200
(36,000)
Cash generated from operations 180,720
Income tax paid (W.3) (37,080)
Net cash flow from operating activities 143,640
Cash flow from investing activities:
Purchases of non-current assets (W4) (111,240)
Sales of Pastit Limited (W5) 41,040
Net cash used in investing activities (70,200)
Cash flow from financing activities:
Redemption of 10% debenture (W6) (18,000)
Dividend paid to non-controlling interest (W7) (3,600)
Interest paid (10,080)
Net cash used in financing activities (31,680)
Net increase in cash & cash equivalent 41,760
Cash & cash equivalent b/f (14,400 – 36,000) (21,600)
Cash & cash equivalent c/f 20,160
Cash & cash equivalent c/f is represented by:
Cash in hand 63,360
Bank overdraft (43,200)
20,160

© Emile Woolf International 185 The Institute of Chartered Accountants of Pakistan


Advanced accounting and financial reporting

Workings
(W1) Profit on disposal of subsidiary:
The entire 80% shareholding was sold.
Rs.’000
Net asset of subsidiary sold (shown in the question) 43,200
Sales proceeds 39,600
Less Net asset sold x 80% = (80% x Rs. 43,200) 34,560
Profit on disposal of subsidiary 5,040

(W2) Movement in Working Capital

Add Less Bal. Cash flow


31/12/16
disposal 31/12/15 statement
Rs.’000
Rs.’000 Rs.’000 Rs.’000
Inventory 180,000 14,400 165,600 28,800
Receivables 151,200 18,000 136,800 32,400
Trade creditors (108,000) (10,800) (93,600) (25,200)
(W3) Income Tax Paid
Taxation
Rs.’000 Rs.’000
Tax on disposal 2,160 Balance b/f 39,240
Cash/Bank 37,080 Tax for the year – P & L 46,800
Balance c/f 46,800
86,040 86,040
(W4) Non-current assets
Non-current assets
Rs.’000 Rs.’000
Balance b/f 360,000 Disposal 28,800
Cash/Bank 111,240 Depreciation – (P & L) 72,720
Balance c/f 369,720
471,240 471,240
W5 Cash flow from sale of Pastit Limited
Rs.’000
As per question 39,600
Add Bank overdraft of Pastit Limited on disposal 1,440
41,040

(W6) Movement on debenture


Rs.’000
Balance b/f at 01/01/2015 90,000
Disposal of subsidiary (3,600)
Cash paid (bal. figure) (18,000)
Balance c/f at 31/12/2016 68,400

© Emile Woolf International 186 The Institute of Chartered Accountants of Pakistan


Answers

(W7) Non-controlling interest

Rs.’000 Rs.’000
Disposal 8,640 B/d 41,400
Dividend paid to NCI 3,600 P&L 7,200
B/d 36,360
48,600 48,600

8.2. BISHOP GROUP


(a)
Statement of cash flows for year ended 31 December 20X2
Rs.000 Rs.000
Cash flows from operating activities (Note 1) 2,282
Interest paid (120 + 205) (325)
Dividends received 90
Taxation paid (W2) (117)
–––––––
Net cash flows from operating activities 1,930

Cash flows from investing activities:


Payments to acquire tangible non-current assets (W3) (4,996)
Receipts from sale of tangible non-current assets 810
Purchase of investments (300)
–––––––
Net cash used in investing activities (4,486)

Cash flows from financing activities:


Proceeds of share issue 3,824
Additional loans (1,200 – 800 – 25) 375
Capital payments under finance leases (W4) (150)
Dividends paid to NCI (W1) (295)
Equity dividends paid (600)
–––––––
Net cash provided by financing activities 3,154
–––––––
Net increase in cash and cash equivalents 598
Effect of exchange rate movements 53
–––––––
651
Cash and cash equivalents brought forward 169
–––––––
Cash and cash equivalents carried forward 820
–––––––
Notes to the statement of cash flows
Reconciliation of operating profit to net cash inflow from operating activities
Rs.000
Operating profit 2,849
Depreciation 1,200
Profit on sale of non-current assets (810 – 720) (90)
Increase in inventories (6,135 – 5,740 – 117) (278)
Increase in receivables (5,720 – 4,380 – 339) (1,001)
Decrease in payables (1,420 – 1,760 – 58) (398)
Net cash flows from operating activities 2,282

© Emile Woolf International 187 The Institute of Chartered Accountants of Pakistan


Advanced accounting and financial reporting

Workings
(1)
Non-controlling interest
Rs.000 Rs.000
Dividend paid to NCI 295 Balance b/fwd 2,500
Balance c/fwd 2,800 Statement of profit or 420
loss
Exchange gain
(20% × 875) 175
3,095 3,095

(2)
Tax
Rs.000 Rs.000
Tax paid 117 B/fwd current tax 167
C/fwd current tax 700 B/fwd deferred tax 400
C/fwd deferred tax 550 Statement of profit or loss 800
1,367 1,367

(3) Non-current assets


Rs.
Opening NBV 7,520
Depreciation (1,200)
Disposals at NBV (720)
New finance leases 700
Exchange rate gains 424
Purchase for cash 4,996
Closing NBV 11,720

(4)
Obligations under finance leases
Rs.000 Rs.000
Cash paid 355 Balance b/f < 1 year 50
Balance c/f < 1 year 110 Balance b/f > 1 year 250
Balance c/f > 1 year 740 Finance charge in profit or loss 205
Non current asset additions 700
1,205 1,205

The payment of Rs. 355,000 is split as Rs. 205,000 interest and Rs. 150,000 capital as
payments are made in arrears and hence the year end payment pays off the year’s finance
cost.
(b) The statement of profit or loss and statement of financial position are based on the
accruals concept whereas the statement of cash flows is based on the cash concept. Cash
is the 'life blood' of the company and is therefore critical to an entity’s survival. Without
cash to pay suppliers, the work force and other payables, the company will cease to
operate, irrespective of how profitable it is.
Shareholders need to know that a company is viable and has the resources to continue,
and perhaps expand, operations. Suppliers need to know they will be paid and customers
need to know the company is in a position to continue operations.

© Emile Woolf International 188 The Institute of Chartered Accountants of Pakistan


Answers

Profit may be significantly affected by the choice of accounting policies made by a


company. This means it is more subjective than cash and more open to manipulation.
However, the statement of cash flows itself may be subject to window dressing, for
example by delaying payment of suppliers until after year end. The auditor needs to be
involved in this respect to ensure the shareholders and other users receive meaningful
information.
The statement of cash flows gives additional information not provided by the other financial
statements.

8.3. THE GRAPE GROUP


Group statement of cash flows for the year ended 31 March Year 4.

Rs.000
Cash flows from operating activities
Net profit before taxation 9,550
Adjustments for:
Depreciation (Note 1) 1,176
Loss on sale of assets 18
Income from associate (139)
Interest expense 552
Operating profit before working capital changes 11,157
Increase in inventories (1,127 – 139) (988)
Increase in receivables (273 – 85) (188)
Increase in payables (203 – 68) 135
Cash generated from operations 10,116
Interest paid (552)
Income taxes paid (W3) (2,400)
Net cash from operating activities 7,164

Cash flows from investing activities

Acquisition of subsidiary net of cash acquired (346 – 3) (343)


Purchase of property, plant and equipment (1,875 – 315) (1,560)
Proceeds from sale of property, plant and equipment (W1) 156
Dividends received from associate (W2) 93
Net cash used in investing activities (1,654)
Cash flows from financing activities Rs.000
Proceeds from issuance of share capital (675 + 519 - 152) 1,042
Repayment of loan notes (990)
Dividends paid (2,100)
Net cash used in financing activities (2,048)
Net increase in cash and cash equivalent 3,462
Cash and cash equivalents at beginning of period 1,728
Cash and cash equivalents at end of period 5,190

© Emile Woolf International 189 The Institute of Chartered Accountants of Pakistan


Advanced accounting and financial reporting

Notes to the statement of cash flows


(1) Major non-cash transactions
During the year the group purchased a subsidiary undertaking. Part of the consideration
for the acquisition was in the form of shares. Further details of the acquisition are given
below.
(2) Purchase of subsidiary undertaking
Rs.000
Net assets acquired:
Property, plant and equipment 315
Inventories 139
Receivables 85
Cash at bank and in hand 3
Payables (68)
474
Goodwill 24
498
Satisfied by:
Shares allotted 152
Cash 346
498

Workings
(1) Proceeds from sale of property, plant and equipment
Rs.000
Cost of assets sold 429
Accumulated depreciation (255)
Loss on sale (18)
Proceeds 156

(2) Dividends received from associate


Interest in associate
Rs.000 Rs.000
Balance b/d 1,920 Dividends received from 93
associates
Share of associates' profit 139 Balance c/d 1,966
after tax
2,059 2,059

(3) Taxation
Taxation
Rs.000 Rs.000
Cash paid 2,400 Balance b/d 2,400
Balance c/d 2,950 Statement of profit or loss 2,950
5,350 5,350

© Emile Woolf International 190 The Institute of Chartered Accountants of Pakistan


Answers

CHAPTER 9 - IFRS 9: FINANCIAL INSTRUMENTS: RECOGNITION AND


MEASUREMENT
9.1. AJI PANCA LTD
Capital and reserves Rs.
Share capital (Rs. 1 ordinary shares) (W2) 1,625,000
Share premium (W3) -
Retained earnings 6,116,812
7,741,812
Liabilities (W5) 164,751

Workings
(1) Profit for the year
Rs.
Original 508,500
Minus: Finance charges (W5) (14,988)
493,512
(2) Ordinary share capital
Rs.
At 1 January 1,000,000
Issue at full price on 31 March 300,000
1,300,000
Bonus issue on 30 June (1,300,000 ÷ 4) 325,000
1,625,000
(3) Share premium
Rs.
At 1 January 200,000
Issue at full price on 31 March ((300,000  0.30) – 20,000) 70,000
270,000
Bonus issue on 30 June (270,000)
NIL
(4) Retained earnings
Rs.
At 1 January 5,670,300
Minus: Bonus issue on 30 June (325,000 (W2) – 270,000 (W3) (55,000)
Add: Profit for the year (W1) 493,512
Add back: Preference dividends charged to retained earnings
(W5) 8,000
6,116,812
(5) Redeemable preference shares
Rs.
Liability at beginning of year
Year 1 ((100,000  Rs. 1.60) – 2,237)) 157,763
Finance charge at 9.5% 14,988
Interest paid at 4% (8,000)
Liability at end of year 164,751

© Emile Woolf International 191 The Institute of Chartered Accountants of Pakistan


Advanced accounting and financial reporting

9.2. PASSILA LTD


(a) The face value of the debentures
Rs. 100 X 20,000 = Rs. 2,000,000
The amount accrued to the company as proceeds (MV) =
Rs. 97.5 X 20,000 = Rs. 1,950,000
(b) The difference between the face value and the market value of the debentures is
Rs. 50,000. This is as a result of discount allowed on the issue on the debentures.
Discount on debentures attracts investors.
(c) Nominal interest rate is the rate based specifically on the face value of the loan capital.
In case of Passila Ltd., the nominal interest rate on the debentures is 8% per annum on
Rs. 2,000,000.
The effective interest is the rate based on the market value. This is the actual value
collected on issue which can be at par, discount or premium. For Passila Ltd., the
effective interest rate will be 8% of Rs. 1,950,000
(d) The nominal interest payable
Rs. 2,000,000 X 8% X 6 months ÷ 12 months
= Rs. 80,000
(e) (i) The face value of Rs. 2,000,000 will be the most appropriate valuation to be
disclosed in the Statement of financial position. The management may be
interested in the quoted market value or the proceeds, but for the sake of outside
investors who would only be interested in the company having good reputations
devoid of trading losses, it is advisable that the face value be adopted.
(ii) Disclosing the debentures’ liability at face value plus interest payment for five
years may seem proper in the eyes of external investors and credit institutions,
but principally, it would be wrong to credit debentures’ account with both the face
value and the interest payments. An interest payment on debentures is a
revenue item which is debited to the profit and loss account.
(iii) Disclosing debentures’ liability at market value on the Statement of financial
position will amount to disclosure at replacement value. The market value
should be disclosed.

9.3. FINANCIAL INSTRUMENTS


(a) IFRS 9 requires that all financial assets and financial liabilities are recognised at fair value
at initial recognition.
A company must classify financial assets as subsequently measured at:
 amortised cost;
 fair value through other comprehensive income; or
 fair value through profit or loss
Amortised cost
Amortised cost is calculated as:
Initial cost recognised
Plus: Interest at the effective rate
Minus: Cash received/paid
Fair value
Fair value is defined as “the price that would be received to sell an asset or paid to transfer
a liability in an orderly transaction between market participants at the measurement date”.

© Emile Woolf International 192 The Institute of Chartered Accountants of Pakistan


Answers

Financial assets
A financial asset must be measured at amortised cost if both of the following conditions are
met:
 the asset is held within a business model whose objective is to hold assets in order to collect
contractual cash flows; and
 the contractual terms of the financial asset give rise on specified dates to cash flows that are
solely payments of principal and interest on the principal amount outstanding.
A financial asset is measured at fair value through other comprehensive income (“FVOCI”) if
both of the following criteria are met:
 The objective of the business model is achieved both by collecting contractual cash flows and
selling financial assets; and
 The asset’s contractual cash flows represent solely payments of principal and interest.
Financial assets included within the FVOCI category are initially recognized and subsequently
measured at fair value. Movements in the carrying amount should be recorded through OCI,
except for the recognition of impairment gains or losses, interest revenue and foreign exchange
gains and losses which are recognized in profit and loss.
Any asset which is not measured at amortised cost or other comprehensive income must be
measured at fair value through profit or loss (FVTPL). FVTPL is the residual category.
Regardless of the business model assessment, an entity can elect to classify a financial asset
at FVTPL if doing so reduces or eliminates a measurement or recognition inconsistency
(“accounting mismatch”).
Financial liabilities
The classification and measurement of financial liabilities under IFRS 9 remains the same as in
IAS 39, except where an entity has chosen to measure a financial liability at FVPL. For such
liabilities, changes in fair value related to changes in own credit risk, are presented separately
in OCI.
A financial liability must be measured at amortised cost with specific exceptions including:
 Derivatives that are liabilities at the reporting date; and
 Financial liabilities that might arise when a financial asset is transferred but this transfer does not
satisfy the derecognition criteria.
A company is allowed to designate a financial liability as measured at fair value through profit or
loss. This designation can only be made if:
 it eliminates or significantly reduces a measurement or recognition inconsistency; or
 this would allow the company to reflect a documented risk management strategy.
Any such designation is irrevocable.
(b) (i) 3% Bond
The bond must initially be recorded at its purchase price of Rs. 250,000. The bond
seems to satisfy the amortised cost criteria. The company seem to operate a
business model whose objective is to hold financial assets in order to collect
contractual cash flows and it seems that the cash it will collect will be solely payment
of interest and principle. The market value is not relevant.
Interest will be credited to profit or loss using the effective interest rate, resulting in
finance income of Rs. 24,250 (9.7% × 250,000). The effective rate reflects the total
return received by the investor over the duration of the bond – being the coupon +
Rs. 50,000 premium on redemption. The coupon recorded in the statement of cash
flows is Rs. 9,000 (3% × 300,000).
The difference between the effective interest and the actual coupon is added to the
investment to give an amortised cost at the end of Year 3 of Rs. 265,250 (250,000 +
24,250 – 9,000).

© Emile Woolf International 193 The Institute of Chartered Accountants of Pakistan


Advanced accounting and financial reporting

(ii) Equity shares in XYZ


The shares must be classified as FVTPL. They will initially be recorded at their cost
of Rs. 30,000. As they have been classed as ‘fair value through profit or loss’ the
transaction costs must be expensed to profit or loss immediately. At the end of each
reporting period, the shares must be re-measured to their market value, with the
resulting gain or loss being taken to profit or loss.
At 1 January Year 3, the investment has a carrying value of Rs. 34,000. By the 31
December Year 3 this value is now Rs. 35,000. A Rs. 1,000 gain will therefore be
recognised in profit or loss for the year.

(iii) Convertible bond


A convertible bond is a compound instrument. In essence, issuing a convertible
bond is equivalent to issuing a non-convertible bond plus a call option on the entity’s
shares. Therefore, the bond should be divided into a liability portion and an equity
portion in accordance with the rules in IAS 32.
Note that the investor would have to do something similar
To establish the liability (debt) element, the future cash flows from the bond are
discounted at the normal market rate to establish the value of an equivalent but
redeemable bond.
Using a rate of 7% this gives a net present value of:
20,000 20,000 520,000
   $460,635
1.07 1.072 1.073
As the bond was issued for Rs. 500,000, it implies that the call option embedded
within the bond was sold for Rs. 39,365 (Rs. 500,000 – 460,635).
 The liability component is measured at amortised cost after initial recognition.
Interest will be recognised at the effective rate of 7%. The difference between the
cash interest paid (4%) and the interest expense recognised will increase the
amortised cost of the liability year on year until the bond is redeemed.

9.4. CASCABEL LTD


(a) At 31 July 2016 this instrument meets the definition of a derivative:
Small or no initial investment.
Its value is dependent on an underlying economic item; exchange rate.
Its settlement will take place at some future date.
As a derivative it should be accounted for as an “asset or liability held at fair value through
profit or loss”. The value of the derivative instrument will be the difference between the
value of the contract when settled compared with the cost of A$2m being purchased at the
spot rate at the year-end date.
Cost of A$2m at a contracted rate of A$0.64 = Rs. 3,125,000
Cost of A$2m at the forward rate of A$0.70 = Rs. 2,857,143
The derivative results in a liability at the year-end date of Rs. 267,857 (Rs. 3,125,000 - Rs.
2,857,143) as the contract has unfavourable terms when compared to the spot rate. The
loss on derivative would be charged to the statement of profit or loss in the year to 31
August 2016.
Recorded as:
Dr Statement of profit or loss (loss on derivative) Rs. 267,857
Cr Liabilities – derivatives Rs. 267,857

© Emile Woolf International 194 The Institute of Chartered Accountants of Pakistan


Answers

(b) If the derivative was designated as a hedging instrument in a cash flow hedge then the
loss of Rs. 267,857 would be recognised in other comprehensive income until the related
cash flow (hedged item) occurred, and shown as a loss in other comprehensive income in
the year ended 31 August 2016. This ensures that the movements in the hedged item and
the hedging item can be offset in the same accounting period.

9.5. FAIR VALUE HEDGE ACCOUNTING


(a) Journals at 31 December 2015

Debit Credit
Forward contract asset 95,000
P&L account – fair value gain 95,000
Being mark-to-market for the derivative

Debit Credit
P&L account – fair value loss on inventory 100,000
Inventory 100,000
Being fair value loss on inventory, attributable to the risk being hedged

(b) Journals at 31 March 2016

Debit Credit
Forward contract asset (142000 – 95000) 47,000
P&L account – fair value gain 47,000
Being mark-to-market for the derivative

Debit Credit
P&L account – further fair value loss on inventory 50,000
Inventory 50,000
Being fair value loss on inventory, attributable to the risk being hedged

Debit Credit
Bank 1,150,000
P&L account 1,150,000
Being sales proceeds

Debit Credit
Bank 142,000
Forward contract 142,000
Being forward contract closed out for cash

Debit Credit
P&L account – cost of sales 850,000
Inventory 850,000
Being inventory carrying value now derecognised upon sale

© Emile Woolf International 195 The Institute of Chartered Accountants of Pakistan


Advanced accounting and financial reporting

The results of these journals can be summarised as:

Debit/(Credit)
Cash Derivative Inventory P&L
Inventory brought forward 1,000,000
December:
Change in fair value (FV) of forward
95,000 (95,000)
Change in FV of inventory (100,000) 100,000
March:
Change in FV of forward 47,000 (47,000)
Change in FV of inventory (50,000) 50,000
Revenue 1,150,000 (1,150,000)
Close out derivative 142,000 (142,000)
Cost of sale (850,000) 850,000
TOTALS 1,292,000 NIL NIL 292,000

9.6. CASH FLOW HEDGE ACCOUNTING


Fair value of the derivative contract at 31 December:

31 Dec 28 Feb
The cash flow under the contract will be 400,000 * 0.7 = Rs. 280,000 Rs. 280,000
The cash flow available in the market is 400,000 * 0.75 = Rs. 300,000
The cash flow available in the market is 400,000 * 0.80 = Rs. 320,000
Therefore the fair value of the derivative (an asset) is Rs. 20,000 Rs. 40,000

(a) Journals at 31 December

Debit Credit
Forward contract 20,000
Equity – cash flow hedge reserve 20,000
Being fair value change, deferred to equity as an effective cash flow hedge

(b) Journals at 28 February

Debit Credit
Forward contract 20,000
Equity – cash flow hedge reserve 20,000
Being fair value change January and February 2016

Debit Credit
Bank 40,000
Forward contract 40,000
Contract closed with payment from the FX dealer of 400,000 * (0.70-0.80) = Rs. 40,000

© Emile Woolf International 196 The Institute of Chartered Accountants of Pakistan


Answers

Debit Credit
Property, Plant and Equipment 320,000
Bank 320,000
Being initial recognition of purchase price of machine: 400,000 * 0.80 = Rs. 320,000

Debit Credit
Equity – cash flow hedge reserve 40,000
Property, Plant and Equipment 40,000
Being transfer of deferred gains/losses on closure of a cash flow hedge

The machine will therefore be recorded at 320,000 – 40,000 = Rs. 280,000


The results of these journals can be summarised as:

Debit/(Credit)
Cash Derivative Equity PP&E
December:
Change in (FV) of forward 20,000 (20,000)
February:
Change in FV of forward 20,000 (20,000)
Purchase of drilling rig (320,000) 320,000
Basis adjustment 40,000 (40,000)
Close out of derivative 40,000 (40,000)
TOTALS (280,000) - - 280,000

9.7. WATERS LTD


1 Investment in 7% treasury stock 2022
As there is not an intention to hold the investment to maturity, the investment should be
classified as at fair value (with gains and losses recognised in profit or loss).
At initial recognition it will be measured at fair value which is the consideration given of Rs.
208,200.
There is no interest received up to year end (first payment will be received on 31 October
2017)
The market value of the stocks at the reporting date is Rs. 196,140 and the revaluation
loss of Rs. 12,060 will be recognised in profit or loss.
Alternative:
Waters Ltd could choose to recognise finance income in the statement of profit or loss at
6.3%  208,200  4/12 = Rs. 4,372.
The financial asset will then have a carrying amount of Rs. 212,572 (208,200 + 4,372) prior
to remeasuring to the fair value. The market value of the stocks at the reporting date is Rs.
196,140 and the revaluation loss of Rs. 16,432 will be recognised in profit or loss.
Notice that the overall profit impact is the same in each case as *4,372 + (16,432) =
12,060.

© Emile Woolf International 197 The Institute of Chartered Accountants of Pakistan


Advanced accounting and financial reporting

2 Futures Prif contract


The derivative will be classified as at fair value through profit or loss.
Initial transaction costs cannot be included as part of the carrying amount and therefore
the fee of Rs. 750 will be immediately charged to profit or loss.
At the reporting date the contract is valued at the fair value of PR1.99/Rs. 1 so the loss is
Rs. 1,269 to be included in profit or loss and as a liability on the statement of financial
position
3 Investment in Gilmour Ltd
This would normally be classified as at fair value (with gains and losses recognised in
profit or loss).
On initial recognition it would be valued at fair value which would be the cost of Rs.
1,212,500. The directly attributable transaction costs (Rs. 35,000) would be expensed to
profit or loss.
At the reporting date the shares will be valued at fair value (Rs. 5.20 per share) ignoring
selling costs = Rs. 1,300,000.
The revaluation gain of Rs. 87,500 will be recognised in profit or loss.
Alternative:
Waters Ltd could have made an irrevocable election at initial recognition to recognise
gains and losses in other comprehensive income. If this election had been made the
shares would have been measured on initial recognition at the cost of Rs. 1,212,500 plus
directly attributable transaction costs Rs. 35,000 = Rs. 1,247,500.
At the reporting date the shares would then have been be valued at fair value with the
revaluation gain of Rs. 52,500 recognised in other comprehensive income.
4 Amount receivable from Mason
On recording the sale, the revenue needs to be discounted at the imputed rate of interest
of 11%. Revenue recognised on 1 July is therefore Rs. 450,450 (500,000  1.11).
The receivable on the statement of financial position will include the accrued interest
element of Rs. 24,775 (Rs. 450,450 x 0.11 x 6/12) and so will be Rs. 475,225 (Rs. 450,450
+ Rs. 24,775) in total. The accrued interest of Rs. 24,775 will be recognised as finance
income.
The receivable would not be adjusted for any change in interest rates.
5 Investment in 8.5% treasury stock 2018
This would be classified as to be subsequently measured at amortised cost.
On initial recognition, it will be recorded at fair value, the cost of Rs. 107,100.
Finance income will be credited to profit or loss using the gross redemption yield of 5.9%.
Interest recognised in profit or loss will be Rs. 4,213 (Rs. 107,100  5.9%  8/12).
The investment in the statement of financial position at 31 December 2016 will be at Rs.
107,100 plus Rs. 4,213 = Rs. 111,313. (No interest will have been received to date as it is
paid annually in arrears).
The market value is not reflected in the statement of financial position at 31 December
2016 but it would be disclosed in accordance with IFRS 7.
6 Investment in loan notes
The investment has been classified as held for trading so it is accounted for as a financial
asset at fair value through profit or loss.
On acquisition it will be recorded at its cost of Rs. 25,000.
At the reporting date the notes will be revalued to their fair value of Rs. 25,500 with the
Rs. 500 uplift being recognised in profit or loss

© Emile Woolf International 198 The Institute of Chartered Accountants of Pakistan


Answers

7 Selling shares short


On initial recognition, the journal would be:

Rs. Rs.
Dr Cash (10,000  Rs. 3.60) 36,000

Cr Financial liability 36,000

At the reporting date the financial liability must be revalued to its fair value of Rs. 33,000:

Rs. Rs.
Dr Financial liability 3,000
Cr Statement of profit or loss 3,000

9.8. ARIF INDUSTRIES LIMITED


(a) Amortisation table
Interest income @
Opening Expected cash Closing
15.5%
balance flow balance
(Cr P&L)
A B = A x 15.5% C E=A+B+C
2016 104,641,483 16,219,430 (15,000,000) 105,860,913
2017 105,860,913 16,408,441 (15,000,000) 107,269,354
2018 107,269,354 16,626,750 (20,000,000) 103,896,104
2019 103,896,104 16,103,896 (120,000,000) -
(b) Amortisation table with adjustment for future income estimate
Effect of
Interest income
Opening Expected change in Closing
@ 15.5%
balance cash flow estimate balance
(Cr P&L)
(Cr P&L)
A B=A x 15.5% C D E=A+B+C+D
2016 104,641,483 16,219,430 (15,000,000) - 105,860,913
2017 105,860,913 16,408,441 (15,000,000) 1,622,535 108,891,889
2018 108,891,888 16,878,243 (20,000,000) - 105,770,132
2019 105,770,131 16,394,370 (20,000,000) - 102,164,502
2020 102,164,501 15,835,498 (118,000,000) - -

Computation of effect of change in estimate:


Expected cash flow Discounted rate (effective
Discounted
(revised) rate = 15.5%)
2018 (20,000,000) 0.8658 17,316,017
2019 (20,000,000) 0.7496 14,992,223
2020 (118,000,000) 0.6490 76,583,649
Revised present value 108,891,889
Existing present value (as at end of 2017) 107,269,354
Effect of change in estimate 1,622,535

© Emile Woolf International 199 The Institute of Chartered Accountants of Pakistan


Advanced accounting and financial reporting

9.9. QASMI INVESTMENT LIMITED


Qasmi Investment Limited
Journal entries for 31 December 2011 and 2012
Debit Credit
Date Description
Rs. in million
31-Dec-2011 Accrued Interest written off (P&L) 12.00
Accrued Interest - 2010 12.00
(Accrued interest on 12%TFCs for 2010 is no
more receivable, now written off.)
Financial assets (12% TFCs) W.3 (16.89–12.00) 4.89
Interest income (P&L) 4.89
(Interest income on 12% TFCs at 4.426% for
2011)
Impairment loss (P&L) W.1 19.16
Financial assets (12% TFCs) 19.16
(Impairment of financial assets (12% TFCs) as
interest for 2010 to 2013 is no more
receivable)
Financial assets (12% TFCs) W.1
31-Dec-2012
(88.53×16.426%) 14.54
Interest income (P&L) 14.54
(Interest income for 2012)
Financial assets (12% TFCs) W.2 10.31
Impairment reversal (P&L) 10.31

(Reversal of impairment of financial assets on


rescheduling of payments for TFCs)
W.1 Impairment
Carrying value of 12% TFCs on 31-12-2011 W.3 107.69
PV of future cash flows on 31-12-2011 120×[(1.16426)–2 ] 88.53

Impairment loss 19.16

W.2 Impairment Reversal


Revised carrying amount on rescheduling at lower of (A) and (B) below 113.38
(A) PV of the future cash flow as per the agreed revised schedule 115.00
(B) Amortised cost on impairment reversal date of 31-12-2012 would have
been had the impairment not been recognised. W.3 113.38
Existing carrying amount at 31-12-2012 88.53×1.16426 (103.07)

Impairment reversal 10.31

© Emile Woolf International 200 The Institute of Chartered Accountants of Pakistan


Answers

W.3 Original amortisation schedule

Effective interest @ Cash flow


Cash flow dates Amortised cost
16.426% (Interest @ 12%)

-------------------------------- Rs. in million --------------------------------


01-Jan-2009 (100×95%) 95.00
31-Dec-2009 15.60 (12.00) 98.60
31-Dec-2010 16.20 (12.00) 102.80
31-Dec-2011 16.89 (12.00) 107.69
31-Dec-2012 17.69 (12.00) 113.38

9.10. RASHID INDUSTRIES LIMITED


Investment in KL
Initial measurement
According to IFRS 9, at initial recognition, RIL may make irrevocable election to present
subsequent changes in fair value in equity investment in other comprehensive income instead of
profit or loss account.
If RIL opted as above, investment in KL would initially be recognized at fair value plus transaction
costs i.e. Rs. 20 million.
However, if RIL opted to measure the investment at fair value through profit and loss (FVTPL),
investment should initially be measured at Rs. 19.96 million (20/1.002) and transaction costs of
Rs. 0.04 million (20–19.96) should be charged to profit and loss account.
Subsequent measurement
On 31 December 2016, if fair value through other comprehensive income has been opted,
investment in KL should be measured at fair value of Rs. 12.4 million and a loss of Rs. 7.6 million
[20–12.4(155,000×80)] (instead of Rs. 5 million) should be booked through other comprehensive
income.
According to IFRS 9, amount presented in other comprehensive income shall not be
subsequently transferred to profit or loss. However, the entity may transfer the cumulative gain /
(loss) within equity.
If fair value through profit or loss has been opted, then RIL should account for the loss of Rs.
7.56 million (20–0.04(transaction cost)–12.4) through profit and loss account.
Investment in BL
Initial measurement
The investment in BL should be recognized as held for trading at fair value of Rs. 64.87 million
(65÷1.002) and transaction cost of Rs. 0.13 million should be charged to profit and loss account.
Subsequent measurement
As at 30 November 2016, the investment should be re-measured to fair value at the market price
of Rs. 83.835 million (135,000×621) and a gain of Rs. 18.965 million (83.835–64.87) shall be
booked in the profit and loss account.
Reclassification of asset
On 30 November 2016 when RIL decided to hold the shares for a longer period, investment in BL
should be reclassified from held for trading to non-trading investment. Further, RIL may make
irrevocable election that investment in BL would be re-measured at fair value through other
comprehensive income, as discussed in the case of KL above. Similarly, treatment on 31
December 2016 would depend on whether RIL opted to re-measure at fair value through OCI or
not.

© Emile Woolf International 201 The Institute of Chartered Accountants of Pakistan


Advanced accounting and financial reporting

9.11. LAHORE STEEL LIMITED


LSL
General Journal
Debit Credit
Date Particulars
Rs. in million
1-Jan-2016 Debentures (W-1) 9.63
Equity (W-3) 1.43
Cash (0.1×107) 10.70
Debt settlement gain (Balancing) 0.36
31-Dec-2016 Finance cost (W-1) 6.06
Cash (W-1) 5.40
Debentures (Balancing) 0.66

W-1: Movement of liability Rs. in million


Initial amount (W-2) 95.57
Finance cost accrued 2015 (95.57 × 7%) 6.69
Finance cost paid 2015 (100 × 6%) (6.00)
Liability at the end of 2015 96.26
10% redeemed (96.26 × 10%) (9.63)
Liability after redemption 86.63
Finance cost accrued 2016 (86.63 × 7%) 6.06
Finance cost paid 2016 (90 × 6%) (5.40)
Liability at the end 2016 87.29
W-2: Liability component (1 January 2015)
PV at 7% of
 interest payments for 2015-2019 (100 × 6% × 4.1001) 24.60
 principal payment at end of 2019 (50 × 0.7130) 35.65
 interest payments for 2020 (50 × 6% × 0.6663) 2.00
 principal payment at end of 2020 ((50 × 0.6663) 33.32
Liability component 95.57
W-3: Equity component repurchased
Total payment (0.1 × 107) 10.70
Fair value of liability repurchased [92.69 (W-4) × 10%] (9.27)
1.43
W-4: Fair value of liability component (1 January 2016)
PV at 8% of
 interest payments for 2016-2019 ((100 × 6% × 3.3121) 19.87
 principal payment at end of 2019 (50 × 0.7350) 36.75
 interest payments for 2020 (50 × 6% × 0.6806) 2.04
 principal payment at end of 2020 (50 × 0.6806) 34.03
92.69

© Emile Woolf International 202 The Institute of Chartered Accountants of Pakistan


Answers

CHAPTER 10 - FINANCIAL INSTRUMENTS: PRESENTATION AND DISCLOSURE


10.1. SERRANO LTD
IAS 32 Financial Instruments: Disclosure and Presentation says that the issuer of a compound
(hybrid) instrument (i.e. one that contains both a liability debt and an equity element) should
classify the instrument’s components separately. Thus the advice of Ancho Services is wrong;
convertible loan stock cannot be classified as pure equity. The proceeds of the issue have to be
split between the amount attributable to the conversion rights, which is then classed as equity,
and the balance of the proceeds being classed a liability/debt. There are several methods of
obtaining these amounts, but from the information given in the question these can only be
calculated on a ‘residual value of equity’ basis:

Cash Factor at Present


flows 10% value
Rs.000 Rs.000
Year 1 interest 600 0.91 546
Year 2 interest 600 0.83 498
Year 3 interest 600 0.75 450
Year 4 interest and capital 10,600 0.68 7,208
––––––
Total value of debt component 8,702
Proceeds of the issue 10,000
––––––
Equity component (residual amount) 1,298
––––––

Statement of profit or loss: Rs.


Interest paid (6% of Rs. 10 million) 600,000
Provision for additional finance costs 270,000
((10%  8.702m) – 0.6m) 870,000
Statement of financial position:
Non-current liabilities:
6% Convertible Loan Stock (from above) 8,702,000
Provision for additional finance costs 270,000
8,972,000

Capital and reserves:


Option to convert to equity (from above) 1,298,000

10.2. POBLANO LTD


In the financial statements of Poblano Ltd for the year to 30 September 2016.
In the statement of profit or loss, the finance cost relating to the loan notes is Rs. 640,000.
In the statement of financial position:
Non-current liability for the loan notes = Rs. 9,384,000
Equity component of loan notes = Rs. 856,000.

© Emile Woolf International 203 The Institute of Chartered Accountants of Pakistan


Advanced accounting and financial reporting

Workings: Rs. 10 million of loan notes


Discount
Annual factor at Present
Year Cash flows cash flow 7% value
Rs.000 Rs.000
1 Interest 400 0.93 372
2 Interest 400 0.87 348
3 Interest + Redemption 10,400 0.81 8,424
Value as straight loan notes 9,144
Issue price 10,000
Equity component (residual amount) 856

Finance cost: year to 30 September 2016


Rs.000
Total finance cost: 9,144  7% 640
Interest payable on 30 September 2016 (Rs. 10 million  4%) 400
Accrual to add to carrying value of debt 240

Carrying value of loan notes: 30 September 2016


Rs.000
Initial valuation of debt element 9,144
Add accrued interest 240
Carrying amount at 30 September 2016 9,384

10.3. PIQUIN LTD


(a) Convertible instrument
A convertible instrument is considered part liability and part equity. IAS 32 requires that
each part is measured separately on initial recognition. The liability element is measured
by estimating the present value of the future cash flows from the instrument (interest and
potential redemption) using a discount rate equivalent to the market rate of interest for a
similar instrument with no conversion terms. The equity element is then the balance,
calculated as follows:
Rs.
PV of the principal amount Rs. 10m at 7% redeemable in 5 yrs 7,130,000
Rs. 10m x 0.713
PV of the interest annuity at 7% for 5 yrs 2,050,000
(5% x Rs. 10m) x 4.100
Total value of liability element 9,180,000
Equity element (balancing figure) 820,000
Total proceeds raised 10,000,000
The equity will not be remeasured, however the liability element will be subsequently
remeasured at amortised cost using the effective interest rate of 7%. The total finance cost
for the year ended 31 December 2016 is Rs. 642,600 (7% x 9,180,000). The coupon rate
of interest of 5% has already been charged to profit or loss in the year so a further Rs.
142,600 should be recorded:
Dr Finance costs Rs. 142,600
Cr Non-current liability Rs. 142,600

© Emile Woolf International 204 The Institute of Chartered Accountants of Pakistan


Answers

(b) Preference shares


The substance of the instrument is a debt instrument. IAS 32 requires that any instrument
that contains an obligation to transfer economic benefit be classified as a liability. The
cumulative nature of the returns on the preference shares means that the outflow of benefit
is inevitable. The preference shares would then be classified as debt and would in fact
increase the gearing of the entity.

10.4. AJI LTD


(a) IAS 32 requires that the equity and liability elements within convertible instruments be
initially recognised separately. The initial carrying amount of the liability is estimated by
measuring the fair value of a similar liability that has no equity element. This is achieved by
calculating the present value of the future cash flows associated with the instrument
assuming that it is not converted on redemption (ie: the interest and principal repayment
cash flows) discounted at the prevailing market rate for a similar instrument without
conversion rights. The difference between this amount and the proceeds (ie: the residual)
is recognised as equity.
The bonds are initially recognised as:

Rs.000 Rs.000
Dr Bank (proceeds of issue) 6,000
Cr Liability (W1) 5,609
Cr Equity (W2) 391

Working 1

Liability element Rs.000


PV of the principal (at 9% after 4 years) = (Rs. 6m x 0.708) 4,248
PV of interest of 7% on Rs. 6m for 4 years = (Rs. 6m x 0.07 x 3.24) 1,361
Total value of liability element 5,609

Working 2

Equity element Rs.000


Total proceeds raised on issue 6,000
Total value of liability element (5,609)
Value attributable to equity 391

(b) (i) In accordance with IAS 39, the liability element will be subsequently measured at
amortised cost using the effective interest rate (which in this case is the interest rate
used to discount the principal to PV, ie 9%). The equity element is not subsequently
re-measured.
The interest of Rs. 420,000 (7% x Rs. 6m) has already been paid and recorded. The
additional finance cost is recorded as:
Rs.000 Rs.000
Dr Finance costs (W1) 85
Cr Liability element of bonds 85

(ii) Extracts from statement of financial position Rs.000


Equity and Liabilities
Equity - Other component of equity 391

Liabilities (W1) 5,694

© Emile Woolf International 205 The Institute of Chartered Accountants of Pakistan


Advanced accounting and financial reporting

Working 1
Opening balance Finance cost at 9% Interest paid 7% Closing balance
Rs.000 Rs.000 Rs.000 Rs.000
5,609 505 (420) 5,694
Tutorial note:
The total finance cost for the year ended 31 December 2016 is Rs. 505K, however
the interest paid of Rs. 420K has already been recorded so only the difference of
Rs. 85K is recognised.

10.5. CHILTEPIN LTD


The convertible bonds on issue will be recorded as:
Dr Bank Rs. 3,900,000
Cr Liability Rs. 3,729,400
Cr Equity Rs. 170,600
Workings:
Time Narrative Cash flow Discount Present
factor (7%) value
1 to 4 Interest 5% x Rs. 4m x 3.387 3.387 677,400
4 Repayment Rs. 4,000,000 0.763 3,052,000
Fair value of the liability 3,729,400
Fair value of equity component (balancing figure) 170,600
Total fair value (amount raised) 3,900,000

The liability will then be accounted for in accordance with IAS 39, i.e. at amortised cost using the
effective interest rate of 7%.

Opening Finance cost Interest paid Closing


carrying value @ 7% carrying value
Rs. Rs. Rs. Rs.
y/e 30/9/2016 3,729,400 261,058 (200,000) 3,790,458

The interest paid of Rs. 200,000 has already been posted, so the additional Rs. 61,058 is
recorded as:
Dr Finance costs Rs. 61,058
Cr Liability Rs. 61,058

10.6. HABENERO LTD


(a) The preference shares will be classified as a liability despite being called “shares”. IAS 32
requires us to consider the substance of the instrument in order to determine whether it
should be classified as debt or equity. In this case the 5% dividend payable on the shares
is cumulative which will eventually result in an outflow of economic benefit for Habenero
Ltd and hence represents an obligation. It therefore meets the definition of a liability. Once
the principal amount is classed as a liability, it follows then that any payment associated
with this instrument (in this case the 5% dividend) will be presented as a finance cost and
be charged in arriving at profit for the year.
The ordinary shares have no inherent obligation as they will not be repaid, nor do they
provide any fixed return to the shareholder. Indeed ordinary shares contain only a residual
interest in the profits of the entity (i.e.: after all obligations have been settled) and hence
will be classified as equity. The associated dividend, when paid, will be presented in the
statement of changes in equity as a reduction in retained earnings.

© Emile Woolf International 206 The Institute of Chartered Accountants of Pakistan


Answers

(b) (i) Initial recognition of the HFT investment is at cost and transaction costs are
charged to the statement of profit or loss:
Dr HFT Investment Rs. 1,400,000
Cr Bank Rs. 1,400,000
Being recognition of investment (where Rs. 1,400,000 = Rs. 2.80 x 500,000 shares)
Dr Statement of profit or loss Rs. 7,000
Cr Bank Rs. 7,000
Being write off of transaction costs (where Rs. 7,000 = Rs. 1,400,000 x 0.5%), with
the costs taken to profit or loss rather than included as part of the initial investment
(because of being classified as HFT).
(ii) Subsequent measurement is at fair value with the gain or loss taken to profit or loss:
Dr HFT Investment Rs. 310,000
Cr Statement of profit or loss Rs. 310,000
Being the gain on HFT investment (where Rs. 310,000 = Rs.(3.42 – 2.80) x 500,000
shares), with the gain being recognised in profit for the year.

10.7. PASHAM TELECOM LIMITED


On 1 October 2016
As the company entered into the swap agreement with the purpose of hedging the fair value of
the company’s own debt, therefore this is a fair value hedge.
The loan should initially be recognized at fair value. Swap has to be recorded initially at its fair
value. Since the swap was entered at ‘market rates’, its fair value is zero at the agreement date
and therefore no accounting entry is required on that date.
Brokerage of Rs. 1 million with respect to swap arrangement should be charged to profit and loss
account.
On 31 December 2016
PTL should record net interest expense of Rs. 17.483 million for the quarter ended 31
December 2016.

Rs. in million
Interest expense on TFC (900 × 8% × 3 ÷ 12) 18.000
Interest expense on SWAP (900 × 6.27% × 3 ÷ 12) 14.108
Interest income on SWAP (900 × 6.5% × 3 ÷ 12) (14.625)
17.483

At 31 December 2016, the hedge is required to be assessed and effectiveness of hedge is


required to be determined, to decide whether hedge accounting is to be continued or not.
Being ‘receive fixed’ and ‘pay variable’ interest rate swap, fair value hedge accounting rules are
to be applied.

Rs. in million
TFCs issued at par 900.00
Fair value at 31 December 2016 (992×0.9) 892.80
Gain in TFCs – Other income 7.20

The swap is deemed effective and hedge accounting shall continue to be used. By considering
this, swap liability of Rs. 7.29 million should be recorded through profit and loss account and
debenture liability should be reduced by Rs. 7.2 million. (changes being reported in profit and
loss account)

© Emile Woolf International 207 The Institute of Chartered Accountants of Pakistan


Advanced accounting and financial reporting

CHAPTER 11 - IAS 19: EMPLOYEE BENEFITS


11.1. LABURNUM LIMITED
Calculation of the actuarial gain/losses in year to 31 December 2016
FV of PV of
plan plan
assets liabilities
Rs.000 Rs.000
Opening balance 2,600 2,900
Service cost 450
Interest cost (8% x Rs. 2,900,000) 232
Expected return (8% x Rs. 2,600,000) 208
Past service cost 90
Benefits paid (240) (240)
Contributions 730
3,298 3,432
Actuarial gain on assets 102 -
Actuarial loss on liabilities - 68
Closing balance 3,400 3,500

11.2. JABEL LIMITED


Statement of profit or loss expense Rs.000
Service cost 300
Interest cost (7% x Rs. 1,400,000) 98
Expected return (7% x Rs. 1,200,000) (84)
Net expense 314
The net expense in profit or loss will be Rs. 314,000.
Actuarial gains and losses FV of plan PV of plan
assets liabilities
Rs.000 Rs.000
Opening balance 1,200 1,400
Service cost 300
Interest cost (7% x Rs. 1,400,000) 98
Expected return (7% x Rs. 1,200,000) 84
Benefits paid (220) (220)
Contributions 400
1,464 1,578
Actuarial loss on plan assets (64) -
Actuarial loss on plan liabilities - 22
Closing balance 1,400 1,600

Within other comprehensive income there will be an actuarial loss on plan assets of Rs. 64,000
and an actuarial loss on plan liabilities of Rs. 22,000.

© Emile Woolf International 208 The Institute of Chartered Accountants of Pakistan


Answers

11.3. KAGHZI LIMITED


Calculation of the actuarial gains/losses in the year to 31 December 2016
FV of plan PV of plan
assets liabilities
Rs.000 Rs.000
Opening balance 1,400 1,700
Service cost 320
Interest cost (7% x Rs. 1,700,000) 119
Expected return (7% x Rs. 1,400,000) 98
Benefits paid (170) (170)
Contributions 580
1,908 1,969
Actuarial gain on plan assets 192 -
Actuarial loss on plan liabilities - 431
Closing balance 2,100 2,400

11.4. LASURA LTD


(i) Statement of profit or loss expense
Rs.000
Service cost 500
Interest cost (8% x Rs. 2,400,000) 192
Expected return (8% x Rs. 2,200,000) (176)
Net expense in profit or loss 516

(ii) Other comprehensive income


Rs.000
Actuarial gain on plan assets (W1) (74)
Actuarial loss on plan liabilities (W1) 58
Net actuarial gain in OCI (16)

Working 1 FV of assets PV of
liabilities
Rs.000 Rs.000
Opening balance 2,200 2,400
Service cost 500
Interest cost (8% x Rs. 2,400,000) 192
Expected return (8% x Rs. 2,200,000) 176
Contributions paid in 300
Paid to retired members (450) (450)
2,226 2,642
Actuarial gain on plan assets 74
Actuarial loss on plan liabilities 58
Closing balance 2,300 2,700

© Emile Woolf International 209 The Institute of Chartered Accountants of Pakistan


Advanced accounting and financial reporting

11.5. UNIVERSAL SOLUTIONS


(a) (i) Defined benefit pension scheme
The employees of a defined benefit scheme will be guaranteed a pension based on
their final salary and their number of years of service. Accordingly, the higher paid
the employee is on retirement and the longer the length of service:
the greater the employee’s pension entitlement and
the greater the liability of the pension fund.
An actuary will advise the company of the cash contributions to be paid into the plan
each year in order to provide the promised pensions. This is a complicated
calculation involving many estimates such as employee mortality, future increases in
salary and expected future investment returns.
The employer has an open-ended liability to make additional contributions should
there be a deficit in the defined benefit pension fund. A deficit may arise, for
example, if salary levels rise more than expected or staff turnover reduces,
increasing service years.
It will be necessary for the actuary to regularly re-value the pension fund’s assets
and liabilities to assess the surplus or deficit position and revise the company’s
contributions.
(ii) The basis to be adopted in measuring scheme assets
Assets should be measured at their fair value. For quoted securities, for example,
this means their market price.
(iii) The basis to be adopted in measuring scheme liabilities
Liabilities should be measured on an actuarial basis (i.e. discounted cash flow),
using the projected unit method.
The projected unit method is an accrued benefits valuation method in which the
scheme’s liabilities reflect projected future earnings. To derive the scheme liabilities,
the expected future pension payments should be discounted at a rate that reflects
the time value of money, for example, using an AA (high quality) corporate bond
rate.
(iv) Actuarial gains and losses
Actuarial gains and losses are deficits or surpluses that arise because:
 events have not coincided with the actuarial assumptions made at the last
valuation (experience gains and losses) or
 the actuarial assumptions have changed.
For example, if the actuary forecast that investment returns were going to be 7% in
a year, but in fact the return actually achieved was only 5%, this would give rise to
an actuarial deficit.
(b)
(i) Statement of financial position - extract
Year 3 Year 4
Defined benefit net liability 200 300
(ii) Journal
Dr Cr
P&L
Interest cost 10
Current service cost 100
110
OCI 130
Cash 140
Defined benefit net liability 100

© Emile Woolf International 210 The Institute of Chartered Accountants of Pakistan


Answers

Workings
Company
Pension fund
position
Statement of
Liabilities Assets financial
position
Rs. Rs. Rs.
Opening balance 1 January Year 4 (1,200) 1,000 (200)
Interest cost (5%) (60) 50 (10)
Current service cost (100) (100)
Contributions to the pension fund 140 140
Benefits paid out 95 (95)
Amounts recorded by company (1,265) 1,095 (170)
Actuarial difference (balance) (135) 5 (130)
Closing balance 31 Dec Year 4 (1,400) 1,100 (300)

11.6. DHA INTERIORS LTD


Report to the Directors of DHA Interiors Ltd
Terms of Reference
This report sets out the differences between a defined contribution and defined benefit plan, and
the accounting treatment of the company’s pension plans. It also discusses the principles
involved in accounting for warranty claims, and the accounting treatment of those claims.
(a) Pension plans – IAS 19
A defined contribution plan is a pension plan whereby an employer pays fixed contributions
into a separate fund and has no legal or constructive obligation to pay further contributions
(IAS 19 paragraph 7). Payments or benefits provided to employees may be a simple
distribution of total fund assets, or a third party (an insurance company) may, for example,
agree to provide an agreed level of payments or benefits. Any actuarial and investment
risks of defined contribution plans are assumed by the employee or the third party. The
employer is not required to make up any shortfall in assets and all plans that are not
defined contribution plans are deemed to be defined benefit plans.
A defined benefit plan is any plan other than a defined contribution plan. It is the residual
category. An employer’s obligation under a defined benefit plan is to provide the agreed
amount of benefits to current and former employees. The differentiating factor between
defined benefit and defined contribution schemes is in determining where the risks lie. If an
employer cannot demonstrate that all actuarial and investment risk has been shifted to
another party and that its obligations are limited to contributions made during the period,
then the plan is a defined benefit plan. Any benefit formula that is not solely based on the
amount of contributions, or that includes a guarantee from the entity or a specified return,
means that elements of risk remain with the employer and must be accounted for as a
defined benefit plan.
For defined contribution plans, the cost recognised in the period is the contribution payable
in exchange for service rendered by employees during the period. Accounting for a defined
contribution plan is straightforward because the employer’s obligation for each period is
determined by the amount to be contributed for that period. Often, contributions are a
percentage of employee salary in the period as its base. No actuarial assumptions are
required to measure the obligation or the expense.
The employer should account for the contribution payable at the end of each period based
on employee services rendered during that period, reduced by any payments made during
the period. If the employer has made payments in excess of those required, the excess is
a prepaid expense to the extent that the excess will lead to a reduction in future
contributions or a cash refund.

© Emile Woolf International 211 The Institute of Chartered Accountants of Pakistan


Advanced accounting and financial reporting

For defined benefit plans, the amount recognised in the statement of financial position is
the present value of the defined benefit obligation (that is, the present value of expected
future payments required to settle the obligation resulting from employee service in the
current and prior periods), as reduced by the fair value of plan assets at the reporting date.
If the balance is an asset, the amount recognised may be limited under IAS 19
Pension Plan 1 is a defined benefit plan as the employer has the investment risk as the
company is guaranteeing a pension based on the service lives of the employees in the
scheme. The employer’s liability is not limited to the amount of the contributions. There is a
risk that if the investment returns fall short the employer will have to make good the
shortfall in the scheme. Pension Plan 2 is a defined contribution scheme because the
employer’s liability is limited to the contributions paid.
(b) Accounting for the two plans
Pension Plan 1
The accounting for the defined benefit plan results in a liability of Rs. 20.5 million as at 31
October 2016, an expense in the statement of profit or loss of Rs. 20.5 million and a
charge in other comprehensive income of Rs. 1.5 million for the year (see Appendix 1).
Pension Plan 2
The company does not recognise any assets or liabilities for the defined contribution
scheme but charges the contributions payable for the period (Rs. 10 million) to operating
profit. The contributions paid by the employees will be part of the wages and salaries cost
and when paid will reduce cash.
Appendix 1
The accounting for the defined benefit plan is as follows:
31 October 2016 1 November 2015
Rs. m Rs. m
Present value of obligation 240 200
Fair value of plan assets (225) (190)
––––– –––––
Liability recognised 15 10
––––– –––––
Expense in Statement of profit or loss year ended 31 October 2016:
Rs. m
Current service cost 20.0
Net interest expense 0.5
–––––
Expense 20·5
–––––
Analysis of amount in statement of other comprehensive income (OCI):
Rs. m
Actuarial loss on obligation (w2) 29
Actuarial gain on plan assets (w2) (27·5)
–––––
Actuarial loss on obligation (net) 1·5
–––––
Working 1
Movement in net liability in statement of financial position at 31 October 2016:
Rs. m
Opening liability 10.0
Expense 20·5
Contributions (17.0)
Actuarial loss 1.5
–––––
Closing liability 15.0
–––––

© Emile Woolf International 212 The Institute of Chartered Accountants of Pakistan


Answers

Working 2 – Change in present value of the obligation and fair value of plan assets
Fair value
PV of of plan Net
obligation assets liability
Rs.000 Rs.000 Rs.000
At start of year (200.0) 190.0 (10.0)
Interest expense (5% × 200) (10.0) (10.0)
Interest earned (5% × 270) 9.5 9.5
Net interest (0.5)
Current service cost (20.0) (20.0)
Contributions paid 17.0 17.0
Benefits paid out (given) 19.0 (19.0) 0
Expected year end position (211.0) 197.5 (13.5)
Remeasurement (balancing figure) (29.0) 27.5 (1.5)
Actual year end position (240.0) 225.0 (15.0)

© Emile Woolf International 213 The Institute of Chartered Accountants of Pakistan


Advanced accounting and financial reporting

CHAPTER 12 - IFRS 2: SHARE BASED PAYMENTS


12.1. TOSHACK LTD
(i) Liability as at 31 December 2016:
Appreciation in price between 1/1/2013 – 31/12/2016
Rs. 31 – Rs. 22 = Rs. 9
No of employees on which rights vest = 75%  1000 = 750
No of rights granted per employee = 50
Liability Rs. 9  750  50 = Rs. 337,500
(ii) Gain between 1/1/2016 – 31/12/2017
(Rs. 28 – Rs. 22)  750  50 = Rs. 225,000
Payment to employees Rs. 112,500
Comment: The transaction would be accounted for as a cash settled share base payment
if the entity has incurred a liability to settle in cash or other asset.

12.2. IFRS 2
(a) (i) The need for accounting standard regulation
Share options are often granted to employees at an exercise price that is higher
than the market price of the shares. Therefore, the options have no intrinsic value to
the company and, prior to the issue of IFRS 2, these transactions were not generally
recognised until such time as the shares were issued. This approach could be seen
as resulting in a distortion of reported results between accounting periods and
leaving liabilities unrecorded.
In addition, the subject of accounting for share-based payments contains a number
of other contentious issues, notably relating to the measurement principles to be
applied in recognising the transactions. If employees agree to stay until their options
vest, the organisation must recognise the service they will provide in return, but how
should this be valued?
IFRS 2 was therefore issued in February 2004 to provide comprehensive guidance
on these matters.
(ii) The three types of share based payments
These can be summarised as follows:
Category Features
Equity-settled share-based The entity pays for goods or services by issuing
payment transactions equity instruments in the form of shares or share
options.
Cash-settled share-based The entity incurs a liability for goods or services
payment transactions and the settlement amount is based on the price
(or value) of the entity’s shares or other equity
instruments.
Share based payments with Transactions where an entity acquires goods or
cash alternatives receives services and either the entity or the
supplier can choose payment to be a cash amount
based on the price (or value) of the entity’s shares
or other equity instruments, or equity instruments
of the entity.

© Emile Woolf International 214 The Institute of Chartered Accountants of Pakistan


Answers

(b) (i) Assuming all options vest


31 December Year 5 Profit and Equity
loss
Expected outcome
(at grant date value)
500 employees × 100 options 750,000
× Rs. 15 fair value
3 years to vest ×1/3
Year 1 charge 250,000
Balance carried forward 250,000

31 December Year 6 Rs.


Expected outcome (at grant date value)
500 × 100 × Rs. 15 750,000
×2/3
Recognised by the year end 500,000
Minus expense previously recognised (250,000)
Year 2 charge 250,000
Balance carried forward 500,000
31 December Year 7 Rs.
Actual outcome (at grant date value)
500 × 100 × Rs. 15 750,000
Minus expense previously recognised (500,000)
Year 3 charge 250,000
Balance at end of year 3 750,000

(ii) Reflecting revised vesting assumptions


Profit Equity
or loss
31 December Year 5 Rs.
Expected outcome (at grant date value)
85% × 500 × 100 × Rs. 15 637,500
×1/3
Year 1 charge 212,500
Balance carried forward 212,500

31 December Year 6
Expected outcome (at grant date value)
88% × 500 × 100 × Rs. 15 660,000
×2/3
440,000
Minus expense previously recognised (212,500)
Year 2 charge 227,500
Balance carried forward 440,000

31 December Year 7
Actual outcome (at grant date value)
44,300 × Rs. 15 664,500
Minus expense previously recognised (440,000)
Year 3 charge 224,500
Balance at the end 664,500

© Emile Woolf International 215 The Institute of Chartered Accountants of Pakistan


Advanced accounting and financial reporting

12.3. SAVAGE LTD


This is an equity-settled share-based transaction and in accordance with IFRS 2 the fair value of
the share options is used to estimate the fair value of the services provided by the employees.
The total fair value is allocated over the three year vesting period and is based on the fair value
at the grant date.
2015
Total expected expense:
1,000 options x Rs. 11 x (300 – 10 – 30) Rs. 2,860,000
Fraction of vesting period by the year end 1/3
Expense in 2015 Rs. 953,333

Dr Statement of profit or loss Rs. 953,333


Cr Equity Rs. 953,333
2016
Total expected expense:
1,000 options x Rs. 11 x (300 -10 – 20 – 15) Rs. 2,805,000
Fraction of vesting period by the year end 2/3
Expense recognised by the year end Rs. 1,870,000
Recognised in 2015 Rs. 953,333
To be recognised in 2016 Rs. 916,667

Dr Statement of profit or loss Rs. 916,667


Cr Equity Rs. 916,667

12.4. YORATH LTD


(a) 2016 equity balance required:
Total expected expense (at end of 2016)
500 options x Rs. 148 x 520 (600 – 20 – 25 – 15 – 20) Rs. 38,480,000
Fraction of vesting period by the year end ¾
Expense recognised by the year end Rs. 28,860,000
Total expected expense (at end of 2015)
500 options x Rs. 148 x 515 (600 – 20 – 25 – 40) Rs. 38,110,000
Fraction of vesting period by the year end 2/
4

Expense recognised by the last year end Rs. 19,055,000


To be recognised in 2016 Rs. 9,805,000
Recorded in 2016 financial statements:
Dr Statement of profit or loss – staff costs Rs. 9,805,000
Cr Equity – other reserves Rs. 9,805,000
(b) The sales director is incorrect, despite no cash changing hands, the share options are
issued in exchange for employees providing services to Yorath Ltd. Possibly the options
have been given as a reward for service provided or in lieu of a pay rise or bonus which
would otherwise have been paid in cash. As there is no direct wage cost, we instead must
calculate an equivalent cost of receiving staff services and match this with the revenue that
the staff helps to generate. We do this by estimating the value inherent in the options and
allocating that over the period in which employees must remain with Yorath Ltd, in this
case 4 years.

© Emile Woolf International 216 The Institute of Chartered Accountants of Pakistan


Answers

The amount chargeable to the statement of profit or loss is based on the fair value of the
share options at the grant date. This is not subsequently remeasured as these share
options represent an equity-settled share-based payment. The equivalent cost will be
updated each year for those employees that are still eligible or expected to be eligible at
the year end to ensure that the amount charged reflects the amount that is expected to
vest.

12.5. QUALTECH LTD


(i) Share-based payment

Total expected expense (at end of 2016)


1,000 options x Rs. 122 x 240 (300 – 25 – 15 – 20) Rs. 29,280,000
Fraction of vesting period by the year end 2/
3

Expense recognised by the year end Rs. 19,520,000


Total expected expense (at end of 2015)
1,000 options x Rs. 122 x 235 (300 – 25 – 40) Rs. 28,670,000
Fraction of vesting period by the year end 1/
3

Expense recognised by the last year end Rs. 9,556,667


To be recognised in 2016 Rs. 9,963,333

Double entry in 2016:


Dr Statement of profit or loss – staff costs Rs. 9,963,333
Cr Other reserves (equity) Rs. 9,963,333
Being the charge for share-based payment for the year ended 31 December 2016
(ii) Share-based payments that are to be settled in cash would be credited instead to liabilities
in the statement of financial position and the liability would be remeasured using the fair
value of the shares at each year-end date until the end of the vesting period.

12.6. BRIDGE LTD


(i) Statement of profit or loss charge

Total expected expense (at end of 2016)


1,000 options x Rs. 50 x 213 (300 – 20 – 23 – 44) Rs. 10,650,000
Fraction of vesting period by the year end 2/
4

Expense recognised by the year end Rs. 5,325,000


Total expected expense (at end of 2015)
1,000 options x Rs. 50 x 215 (300 – 20 – 65) Rs. 10,750,000
Fraction of vesting period by the year end ¼
Expense recognised by the last year end Rs. 2,687,500
To be recognised in 2016 Rs. 2,637,500

Double entry in 2016:


Dr Statement of profit or loss – staff costs Rs. 2,637,500
Cr Other reserves (equity) Rs. 2,637,500
Being the charge for share-based payment for the year ended 31 December 2016

© Emile Woolf International 217 The Institute of Chartered Accountants of Pakistan


Advanced accounting and financial reporting

(ii) Share options, such as those granted by Bridge Ltd, are given by an entity in return for
services provided by its employees. In effect the share options are given to the employees
as a form of bonus or reward for these services and are therefore part of the employee’s
remuneration package. The value of these options (or relevant part thereof) must then be
reflected in the staff costs included within the statement of profit or loss.

12.7. CAPSTAN LTD


SARs are an example of a cash-settled share-based transaction and, in accordance with IFRS 2
Share-based payments, are initially measured at fair value at the grant date and subsequently
remeasured to fair value at each year-end. The liability is remeasured and any difference is
charged to the statement of profit or loss as an expense.
2015

Total expected expense (at end of 2015)


1,000 SARs x Rs. 80 x 233 (300 – 32 – 35) Rs. 18,640,000
Fraction of vesting period by the year end 1/
3

Liability to be recognised by the year end Rs. 6,213,333


Double entry in 2015:
Dr Statement of profit or loss – staff costs Rs. 6,213,333
Cr Liability Rs. 6,213,333
2016

Total expected expense (at end of 2016)


1,000 SARs x Rs. 120 x 230 (300 – 32 – 28 – 10) Rs. 27,600,000
Fraction of vesting period by the year end 2/
3

Liability to be recognised by the year end Rs. 18,400,000


Less opening liability (see above) (Rs. 6,213,333)
To be recognised in 2016 Rs. 12,186,667

Double entry in 2016:


Dr Statement of profit or loss – staff costs Rs. 12,186,667
Cr Liability Rs. 12,186,667

12.8. NEWTOWN LTD


(i) 2015

Total expected expense (at end of 2016)


1,000 SARs x Rs. 110 x 405 (500 – 42 – 28 – 25) Rs. 44,550,000
Fraction of vesting period by the year end 2/
3

Liability to be recognised by the year end Rs. 29,700,000


Less opening liability:
Total expected expense (at end of 2015)
1,000 SARs x Rs. 90 x 383 (500 – 42 – 75) Rs. 34,470,000
Fraction of vesting period by the year end 1/
3

Liability recognised by the end of 2015 Rs. 11,490,000


To be recognised in 2016 Rs. 18,210,000

© Emile Woolf International 218 The Institute of Chartered Accountants of Pakistan


Answers

Double entry in 2016:


Dr Statement of profit or loss – staff costs Rs. 18,210,000
Cr Liability Rs. 18,210,000
(ii) If a share-based payment was settled in equity rather than cash the implications would be:
Recognition:
There would be a credit to other reserves within equity in the statement of financial
position, rather than a liability. However the debit would still be to staff costs.
Measurement:
The amount would be initially and subsequently measured using the fair value of the rights
at the grant date rather than re-measured at each year end.

12.9. SINDH TRANSIT LTD


(a) Accounting entries
Accounting entries for year ended 31 December 2016:
Share options
Dr Staff costs (statement of profit or loss) (W1)Rs. 17,820,000
Cr Equity Rs. 17,820,000
Share appreciation rights (SARs)
Dr Staff costs (statement of profit or loss) (W2)Rs. 6,495,000
Cr Liabilities (non-current) Rs. 6,495,000
Working 1: Options
Total expected expense (at end of 2016)
1,000 options x Rs. 220 x 327 (400 – 15 – 22 – 36) Rs. 71,940,000
Fraction of vesting period by the year end 2/
4

Expense recognised by the year end Rs. 35,970,000


Total expected expense (at end of 2015)
1,000 options x Rs. 220 x 330 (400 – 15 – 55) Rs. 72,600,000
Fraction of vesting period by the year end 1/
4

Expense recognised by the last year end Rs. 18,150,000


To be recognised in 2016 Rs. 17,820,000

Working 2: SARs
Total expected expense (at end of 2016)
500 SARs x Rs. 140 x 327 (400 – 15 – 22 – 36) Rs. 22,890,000
Fraction of vesting period by the year end 2/
4

Liability to be recognised by the year end Rs. 11,445,000


Less opening liability:
Total expected expense (at end of 2015)
500 SARs x Rs. 120 x 330 (400 – 15 – 55) Rs. 19,800,000
Fraction of vesting period by the year end 1/
4

Liability recognised by the end of 2015 Rs. 4,950,000


To be recognised in 2016 Rs. 6,495,000

© Emile Woolf International 219 The Institute of Chartered Accountants of Pakistan


Advanced accounting and financial reporting

(b) In accordance with IFRS 2, the share options and the share appreciation rights are
recognised as an expense in the statement of profit or loss as they are awarded in return
for employee service.
The treatment of each of above stated however is different in the statement of financial
position. The share appreciation rights will result in a future outflow of cash and therefore
represent an obligation and are presented as a liability. The liability should reflect the most
reliable measurement at each balance sheet date and so the total amount payable that is
estimated at each year-end date is estimated using the updated fair values.
The options represent an equity-settled share-based payment and do not meet the
definition of obligation, and so instead the entry is to equity. The equity element is
measured initially and subsequently at the fair value at the grant date.

12.10. XYZ LIMITED


(a) Treatment of share options issued to Marketing Managers
Since options granted to back office managers are granted under non-market condition,
any subsequent change in the non-market condition from previous estimates should be
taken in to account in estimating the expense to be recognized.
Since XYZ is not able to achieve the average 10% profit during the three years, the
expenses booked till the previous year should be reversed i.e. Rs. 853,333 [(8×(50-
18)×5000×2÷3]
Treatment of share options issued to Back Office Managers
Since options granted to back office managers are based on market conditions under
which the probability of meeting the condition was taken into account in fair value of share
option at the grant date, any subsequent changes in the probability of meeting the
condition has no effect on the expense recognition.
Furthermore, when a modification occurs during the vesting period, the incremental fair
value of the option is recognized over the period from the modification date until the date
when equity instruments vest.
By considering the above, XYZ should record the following expenses at 30 June 2016 and
taking the credit effect to the equity:
 In order to record modification impact, the incremental fair value of the option should
be recognized as expense i.e. Rs. 405,000 [(14-5)×5,000×9]
 When options is settled, XYZ should recognize the following expense:

Rupees
Expense to be recorded at settlement date (9×30×5,000) 1,350,000
Expense already recorded till last year (8×30×5,000×2÷3) (800,000)
550,000

(b) Investment in bonds


Since the investment in bonds was made with the intention to hold them till maturity, these
should be recorded at amortized cost and transaction costs which are directly attributable
to the acquisition of the financial assets should be capitalized at initial recognition. The
interest revenue should be calculated by using effective interest method.
In the case of modification in cash flows, the entity should recalculate the gross carrying
amount of the bonds using original effective interest rate and recognise the difference in
P&L in either case, gain or loss.
In accordance with the requirement of impairment, an entity should recognize a loss
allowance equal to life time expected credit losses if the credit risk on that asset has
increased significantly.

© Emile Woolf International 220 The Institute of Chartered Accountants of Pakistan


Answers

Based on the above, the accounting treatment of investment in bonds in the books of XYZ
at 30 June 2016 should be as follows:
 The interest revenue and premium amortization should be recorded at Rs. 4.7 million
(W-1) and Rs. 1.3 million (W-1) respectively.
 The life time expectancy loss of Rs. 5 million should be charged to P & L account.
 Investment in bonds should be disclosed at Rs. 99.99 million (W-1) in statement of
financial position
W-1: Amortization table

Interest Expected
Opening cashflows Closing Premium
income @ Change in
Balance balance amortization
Year @ 6% estimate
4.5186%

----------------------------------- Rs. in million -----------------------------------

2014 106.50 4.81 (6.00) 105.31 (1.19)

2015 105.31 4.76 (6.00) 104.07 (1.24)

2016 104.07 4.70 (6.00) 2.22 (W-2) 104.99 (1.30)

Less: Provision for loss allowance (5.00)

99.99
W-2 : Effect of change in estimate

Expected cash flows Discounted factor Present value


Year @ 6.25% & 6.5% (2019) (Rs. in million)
@ 4.5186%

2017 6.25 0.957 5.98

2018 6.25 0.915 5.72

2019 106.50 0.876 93.29

Revised present value 104.99

Present value without the effect of change (104.07 + 4.7 – 6) 102.77

Effect of change in estimate 2.22

12.11. RAVI LIMITED


Amount to be charged to the profit or loss in respect of the share option scheme is as follows:
1,000 Note-1 × (500 × 85%) Note-2 × 38 Note-3 × (1÷5) Note-4 = 3,230,000

Note-1: Vesting conditions, other than market conditions, shall be taken into account by
adjusting the number of equity instruments included in the measurement of the
transaction amount. Average sales would be Rs. 312.55 million (W-1) over five
years which is more than the minimum average sales of Rs. 300 million.

© Emile Woolf International 221 The Institute of Chartered Accountants of Pakistan


Advanced accounting and financial reporting

Note-2: Service condition shall be taken into account by adjusting the number of equity
instruments included in the measurement of the transaction amount. In respect of
service condition, management estimates that 15% of the employees would leave
the organization over the vesting period of five years so provision would be made
for 85% of employees i.e. 425 (500 × 85%)

Note-3: Only market condition shall be taken into account when estimating the fair value of
the share options at the measurement date. Subsequent changes in the probability
of meeting the condition have no impact and are ignored.

Note-4: The expense will be spread over the vesting period of 5 years.

In light of above, Rs. 3.23 million should be debited to P & L account and credited to equity
account.
W1: Average sales:

Year Sales

2017 210.00

2018 252.00

2019 302.40

2020 362.88

2021 435.46

Average 312.55

© Emile Woolf International 222 The Institute of Chartered Accountants of Pakistan


Answers

CHAPTER 13 - DISPOSAL OF SUBSIDIARIES


13.1. PATCHE LTD
(a) (i) Statement of profit or loss and other comprehensive incomes for the year ended 30
June 2016
Rs.’m
Operating profit 390
Profit on disposal of shares (W1) 210
Profit before tax 600
Income tax (120)
Profit for the year 480
Other comprehensive income 60
540

(ii) Statement of changes in equity for the year ended 30 June 2016
Share Retained Total
capital earning
Rs.’m Rs.’m Rs.’m
Balance b/f (opening Balance) 1,500 660 2,160
Profit for the year - 480 480
Balance c/d 1,500 1,140 2,640

(b) Consolidated statement of profit or loss and other comprehensive incomes for the
year ended 30 June 2016
Rs.’m
Profit before tax (390 + 180) 570
Income tax expenses (120 + 60) (180)
Profit for the year 390
Other comprehensive income (60 + 30) 90
Total comprehensive income 480
Attributable to
Non-controlling interest (W2) 30
Members of the parent (480 – 30) 450
480

(c) Consolidated statement of financial position as at 30 June 2016


Non-current assets: Rs.’m

Property plant and equipment (1605 + 534) 2,139


Goodwill (W3) 240
2,379
Current assets:
Inventories (960 + 570) 1,530
Trade receivables (750 + 525) 1,275
Cash and bank (240 + 267) 507
3,312
5,691

© Emile Woolf International 223 The Institute of Chartered Accountants of Pakistan


Advanced accounting and financial reporting

Equity attributable to owners of the parent


Share capital 1,500.0
Reserves (W4) 1,432.5
2,932.5
Non-controlling interest (W6) 472.5
3,405
Current liabilities:
Trade payables (885 + 513) 1,398
Income tax payables (240 + 180 + 90 (W1)) 510
Provisions (285 + 93) 378
2,286
5,691
Workings:
1. Gain on disposal of shares in parent’s separate financial statement:-
Rs.’m
Fair value of consideration received 480
Less: Original cost of shares (765 x 20%/85%) (180)
Parent gain 300
Less tax on parent’s gain (30%) (90)
210
2. Non-controlling interest (NCI)
Profit for the year: Rs.’m
Pre-disposal periods = 9/12 x 120m x 15% = 13.5
Post-disposal periods = 3/12 x 120m x 35% = 10.5 24
Other comprehensive income
Pre-disposal periods = 9/12 x 30m x 15% = 3.375
Post-disposal periods = 3/ x 30m x 35% = 2.625 6
12

30
3. Goodwill
Rs.’m
Consideration transferred (285 + 480) 765
Non-Controlling interest at fair value 135
Less:
Fair value of identifiable net assets
at acquisition:
Share capital 600
Pre-acquisition reserve 60 (660)
240
4. Consolidated reserves
Rs.’m
All of Patche
Per question at year-end 930.00
Adj. to equity on disposal (W5) 217.50
Tax on parent gain (W1) (90.00)
1,057.50

© Emile Woolf International 224 The Institute of Chartered Accountants of Pakistan


Answers

Groups’ share of post-acquisition reserve of Somers:


85% (412.5 (see below) x 85%) 350.63
65% (37.5 (see below) x 65%) 24.37
1,432.50
Post-acquisition profits of Somers
Rs.’m
Per question at year-end 510.0
Profit since the date of disposal (when Somers is a 65%
subsidiary): Rs. 150m  3/12 (37.5)
Balance on reserves at the date of disposal 472.5
Balance on reserves at the date of acquisition (60.0)
Post-acquisition reserves up to the date of disposal (for
which period Somers was an 85% subsidiary) 412.5
5 Adjustment to equity on disposal of shares in group financial statement
Rs.’m
Fair value on consideration received 480.00
Increase in NCI in net asset and goodwill at disposal
(196.8 x 20%/15%) (262.50)
217.50
6 Non-controlling interest
Rs.’m
NCI @ acquisition 135.00
NCI share of post-acquisition reserve:
Somers (412.5 x 15%) 61.88
Somers (37.5 x 35%) 13.12
Increase in NCI (W5) (214.5 + 48) 262.50
472.50

13.2. DISPOSAL
Rs. Rs.
million million
Consideration from sale of shares 960
Fair value of retained shares in Spool 100
1,060
Net assets of Spool at carrying value 800
Minus: non-controlling interest de-recognised (10%  800) (80)
720
Gain on sale of shares 340

None of the assets of Spool have been re-valued, therefore there is no balance on a revaluation
reserve; therefore none of this gain should be transferred directly to retained earnings and not
reported in profit or loss.
There is no information to suggest that a reclassification adjustment is required to reclassify
income previously reported as other comprehensive income as profit or loss.
The total gain of Rs. 340 million on disposal of the shares should therefore be recognised in profit
or loss for the period.
Hoo will recognise an investment in Spool in its statement of financial position in accordance with
the requirements of IAS 39. On initial recognition, this investment should be valued at Rs. 100
million.

© Emile Woolf International 225 The Institute of Chartered Accountants of Pakistan


Advanced accounting and financial reporting

13.3. PART DISPOSAL


The disposal of 10% of the shares in S leaves P with a controlling interest; therefore the disposal
of the shares should be accounted for as an equity transaction between owners of the group. No
gain or loss is recognised in the consolidated financial statements of P.
It is assumed that the profits of S for the year were Rs. 200 million (all retained; therefore Rs. 900
million - Rs. 700 million). At 30 June it is assumed that profits for the year to date were Rs. 100
million (= Rs. 200 million  6/12); therefore the net assets of S at this date were Rs. 800 million.
P NCI
Rs. m Rs. m
Before the share sale (80%  800) 640 (20%) 160
After the share sale (70%) 560 (30%) 240
Change in interest in S - 80 + 80
The shares were sold for Rs. 94 million adding to the assets in P’s statement of financial position.
The transaction should therefore be accounted for in equity as follows:
Debit: Cash Rs. 94 million
Credit: NCI Rs. 80 million
Credit: Reserves attributable to P (= gain = balance) Rs. 14 million
Rs. million Rs. million
Post-acquisition profit attributable to S (see above) 200
Less: Impairment of goodwill (8)
Recognised profit 192
Attributable to equity owners of P
1 January – 30 June (80%  200  6/12) 80
1 July – 31 December (70%  200  6/12) 70
Goodwill impairment (8)
Attributable to NCI 142
1 January – 30 June (20%  200  6/12) 20
1 July – 31 December (30%  200  6/12) 30
50
192

13.4. THE A GROUP


A‘s original investment in C was 90% of C’s 400,000 shares (360,000 shares). During the year A
has disposed of 350,000 of these shares, which reduces the investment from subsidiary status to
that of a ‘simple’ investment.
A Group
Consolidated statement of financial position as at 31 December Year 4
Rs.000
Goodwill (W2, B only) 428
Investment in C at fair value 44
Other net assets (W4) 6,661
7,133
Equity
Share capital 1,500
Accumulated profits attributable to owners of A (working 1) 5,347
Equity attributable to owners of A 6,847
Non-controlling interest: 20%  (1,260 + 170) 286
Total equity 7,133

© Emile Woolf International 226 The Institute of Chartered Accountants of Pakistan


Answers

Statement of profit or loss for year ended 31 December Year 4


Working
A B Group
Rs.000 Rs.000 Rs.000
Operating profit 1,200 250 1,450
Minus: Dividend from B (16) nil (16)
1,184 250 1,434
Gain on disposal of C (W2) 237 237
Profit before tax 1,421 250 1,671
Tax (360) (60) (420)
Profit after tax 1,061 190 1,251

Attributable to:
Equity owners of A (1,061 + 80%  190) 1,213
Non-controlling interest: 20% × 190 38
1,251
Workings
(1) Movement on consolidated reserves attributable to owners of parent
A B C Group
Rs.000 Rs.000 Rs.000 Rs.000
At 31 December Year 3 (W5) 3,300 272 612 4,184
Profit for year attributable to A 1,213
Dividends paid by A (50)
At 31 December Year 4 5,347
(2) Disposal of shares in C, with loss of control
Gain to parent Rs.000 Rs.000
Net assets of C at date of disposal: de-recognised 1,400
Purchased goodwill in C de-recognised
(see working 3) 472
1,872
Minus: Non-controlling interest de-recognised
(10%  1,400) (140)
Assets attributable to A de-recognised 1,732
Fair value of investment retained 44
Sale proceeds 1,925
1,969
Total gain on disposal of shares 237
Since there has been no revaluation of non-current assets and there is no information
about any reclassification adjustments that might be required, it is assumed that this entire
gain should be included in profit or loss for the year.
(3) Calculation of goodwill
B C
Rs.000 Rs.000
Cost of Investment 1,164 1,120
Less: Group share of the fair value of the net assets at
acquisition
80% × (500 + 420) (736)
90% × (400 + 320) (648)
428 472

© Emile Woolf International 227 The Institute of Chartered Accountants of Pakistan


Advanced accounting and financial reporting

(4) Other net assets

Rs.000
A’s net assets as 1 January Year 4 2,516
B’s net assets at 1 January Year 4 1,260
A’s retained profit year ended 31 December Year 4 790
B’s retained profit year ended 31 December Year 4 170
Proceeds of disposal of C 1,925

6,661

(5) Calculation of post-acquisition retained profits b/f attributable to A

Rs.000
A As given in the question 3,300
B and C Group share of post-acquisition

B 80%  (760 - 420) 272

C 90%  (1,000 - 320) 612

Total 4,184

13.5. BARTLETT LTD


Consolidated statement of profit or loss for the year ended 31 December 2016

Total
Rs.
Revenue (1,926,500 + 396,200 + 260,800) 2,583,500
Cost of sales (1,207,200 + 202,950 + 193,100) (1,603,250)

Gross profit 980,250


Net operating expenses (400,100 + 152,650 + 52,650) (605,400)

Operating profit 374,850


Profit on disposal of operations (W1) 66,360

Profit on ordinary activities before taxation 441,210


Tax on profit on ordinary activities (110,000 + 4,750 + 13,750) (128,500)

Profit for the year 312,710

Profit attributable to: Rs.


Owners of the parent 286,162
Non-controlling interests (W2) (11,458)

Profit for the year 312,710

© Emile Woolf International 228 The Institute of Chartered Accountants of Pakistan


Answers

Workings
(1) Profit on disposal of Lymon
Recognise: Rs.
Proceeds 212,000

Derecognise: Rs.
Net assets of subsidiary

Net assets at January 2016 140,000


Profit to 1 July 2016 (6/12 x 20,600) 10,300

150,300
Non-controlling interest (20%) (30,060)

(120,240)
Unimpaired goodwill (25,400)

Profit on disposal 66,360

(2) Non-controlling interests


In Lymon Inc 20% x (6/12 x Rs. 20,600) 2,060
In Zeigler Inc 35% x (6/12 x Rs. 53,700) 9,398

11,458

© Emile Woolf International 229 The Institute of Chartered Accountants of Pakistan


Advanced accounting and financial reporting

CHAPTER 14 - IFRS-5: NON-CURRENT ASSETS HELD FOR SALE AND


DISCONTINUED OPERATIONS
14.1. SAUL
Statement of profit or loss for the year ended 31 December Year 1
Rs.000
Continuing operations
Revenue 3,315
Cost of sales (2,125)
––––––
Gross profit 1,190
Distribution costs (255)
Administrative expenses (680)
––––––
Profit before tax 255
Income tax expense (90)
––––––
Profit for the period from continuing operations 165
Discontinued operations
Loss for the period from discontinued operations (W) (15)
––––––
Profit for the period 150
––––––
Statement of financial position as at 31 December Year 1
Rs.000 Rs.000
Assets
Non-current assets
Property, plant and equipment (1,900 – 510) 1,390
Intangible assets 40
––––––
1,430
Current assets
Inventories 350
Trade and other receivables 190
Cash 90
––––––
630
––––––
2,060
Non-current assets classified as held for sale 450
––––––
Total assets 2,510
––––––
Equity and liabilities
Equity
Share capital 600
Retained earnings (1,700 – 60) 1,640
––––––
2,240
Current liabilities
Trade and other payables (195 – 10) 185
Current tax payable 75
Liabilities classified as held for sale 10
––––––
270
––––––
Total equity and liabilities 2,510
––––––

© Emile Woolf International 230 The Institute of Chartered Accountants of Pakistan


Answers

Tutorial note
Division A is classified as discontinued in Year 1 because, although it has not been sold during
the period it meets the IFRS 5 criteria for classification as ‘held for sale’.
Working: Discontinued operation

Continuing Discontinued
operations operations Total
Rs.000 Rs.000 Rs.000
Revenue 3,315 585 3,900
Cost of sales (2,125) (375) (2,500)
–––––– –––– ––––––
Gross profit 1,190 210 1,400
Distribution costs (255) (45) (300)
Administrative expenses (680) (120) (800)
Impairment loss (510 – 450) - (60) (60)
–––––– –––– ––––––
Profit before tax 255 (15) 240
Income tax expense (90) - (90)
–––––– –––– ––––––
Profit/(loss) for the period 165 (15) 150
–––––– –––– ––––––

14.2. SHAHID HOLDINGS


(a) IFRS 5 Non-current assets held for sale and discontinued operations defines non-current
assets held for sale as those assets (or a group of assets) whose carrying amounts will be
recovered principally through a sale transaction rather than through continuing use. A
discontinued operation is a component of an entity that has either been disposed of, or is
classified as ‘held for sale’ and:
(i) represents a separate major line of business or geographical area of operations
(ii) is part of a single co-ordinated plan to dispose of such, or
(iii) is a subsidiary acquired exclusively for sale.
IFRS 5 says that a ‘component of an entity’ must have operations and cash flows that can
be clearly distinguished from the rest of the entity and will in all probability have been a
cash-generating unit (or group of such units) whilst held for use. This definition also means
that a discontinued operation will also fall to be treated as a ‘disposal group’ as defined in
IFRS 5. A disposal group is a group of assets (possibly with associated liabilities) that it is
intended will be disposed of in a single transaction by sale or otherwise (closure or
abandonment). Assets held for disposal (but not those being abandoned) must be
presented separately (at the lower of cost or fair value less costs to sell) from other assets
and included as current assets (rather than as non-current assets) and any associated
liabilities must be separately presented under liabilities. The results of a discontinued
operation should be disclosed separately as a single figure (as a minimum) on the face of
the statement of profit or loss with more detailed figures disclosed either also on the face
of the statement of profit or loss or in the notes.
The intention of this requirement is to improve the usefulness of the financial statements
by improving the predictive value of the (historical) statement of profit or loss. Clearly the
results from discontinued operations should have little impact on future operating results.
Thus users can focus on the continuing activities in any assessment of future income and
profit.

© Emile Woolf International 231 The Institute of Chartered Accountants of Pakistan


Advanced accounting and financial reporting

(b) The timing of the board meeting and consequent actions and notifications is within the
accounting period ended 31 October 2016. The notification of staff, suppliers and the press
seems to indicate that the sale will be highly probable and the directors are committed to a
plan to sell the assets and are actively locating a buyer. From the financial and other
information given in the question it appears that the travel agencies’ operations and cash
flows can be clearly distinguished from its other operations. The assets of the travel
agencies appear to meet the definition of non-current assets held for sale; however the
main difficulty is whether their sale and closure also represent a discontinued operation.
The main issue is with the wording of ‘a separate major line of business’ in part (i) of the
above definition of a discontinued operation. The company is still operating in the holiday
business, but only through Internet selling. The selling of holidays through the Internet
compared with through high-street travel agencies requires very different assets, staff
knowledge and training and has a different cost structure. It could therefore be argued that
although the company is still selling holidays the travel agencies do represent a separate
line of business. If this is the case, it seems the announced closure of the travel agencies
appears to meet the definition of a discontinued operation.
(c) Shahid Holdings statement of profit or loss year ended 31 October:

2016 2015
Rs.’000 Rs.’000
Continuing operations
Revenue 25,000 22,000
Cost of sales (19,500) (17,000)
 
Gross profit 5,500 5,000
Operating expenses (1,100) (500)
 
Profit/(loss) from continuing operations 4,400 4,500
Discontinued operations
Profit/(loss) from discontinued operations (4,000) 1,500
 
Profit for the period 400 6,000
 
Analysis of discontinued operations
Revenue 14,000 18,000
Cost of sales (16,500) (15,000)
 
Gross profit/(loss) (2,500) 3,000
Operating expenses (1,500) (1,500)
 
Profit/(loss) from discontinued operations (4,000) 1,500
 

Note: other presentations may be acceptable.

© Emile Woolf International 232 The Institute of Chartered Accountants of Pakistan


Answers

14.3. PRIMA
Holiday villas
IAS 16 allows property, plant and equipment to be re-valued or left at historical cost. Revaluation
should be based on the fair value (the open market value in an arm’s length transaction).
Revaluation is not required every year, but must be conducted when it is believed that the fair
value differs materially from the carrying value.
The method of accounting for the villa that is to be sold is covered by IFRS 5 which requires that
where, at the end of a reporting period, an asset is held for sale it should be reclassified, re-
measured and no longer depreciated. An asset is only classified as held for sale where the
following conditions are all met:
 The asset is available for sale in its present condition.
 The sale is believed to be highly probable:
 Appropriate level of management is committed to the sale;
 There is an active programme underway to find a buyer;
 The asset is marketed at a realistic price.
 Completion of sale expected within 12 months of classification.
From the limited information provided it appears that these conditions have been met and
therefore, under the rules of IFRS 5, the villa should be re-measured to the lower of its carrying
value and its fair value minus costs to sell.
Therefore, the villas should be valued at 31 December Year 4 as follows:
Fair Carrying
value value
Rs. Rs.
All villas 25.00 20.00
Property held for sale (1.00) (1.25)
Properties to be retained 24.00 18.75

The villas to be retained should be re-valued to Rs. 24m, resulting in an increase in the
revaluation reserve of Rs. 5.25m (24-18.75).
The villa to be sold should be written down from its carrying value to its fair value minus costs to
sell of Rs.0.95m (Rs. 1m – 50,000). This impairment of Rs. 300,000 (1.25m – 0.95m) will be
charged against the revaluation reserve for this asset. If there is insufficient revaluation reserve,
then the write down must be charged to profit or loss.
The villa held for sale must be re-classified from ‘Non-current assets’ to ‘Current assets’ as a
separate line item.
Depreciation should not be charged when an asset has been classified as held for sale.
However, the other villas should be depreciated. IAS 16 states that expenditure on repairs and
maintenance does not remove the need to depreciate an asset. The villas have a finite useful life
and therefore must be depreciated. If the residual value of these assets is greater than the
carrying value then the depreciation charge will be zero. It is not acceptable therefore to have a
policy of non-depreciation on such assets, and a prior year adjustment should be made to correct
the error if the error is material.
Head office
The head office should be recorded under property, plant and equipment at cost. IAS 23 (revised
2009) requires that borrowing costs should be capitalised as part of the cost of an asset if they
are directly attributable to the acquisition, construction or production of a ‘qualifying asset’. A
qualifying asset is an asset that necessarily takes a long period of time to get ready for its
intended use or sale.

© Emile Woolf International 233 The Institute of Chartered Accountants of Pakistan


Advanced accounting and financial reporting

In this situation the company is therefore required to capitalise the borrowing costs as part of the
asset cost. Capitalisation must cease when the asset is substantially complete. Construction
finished on 31 May Year 4 and, although minor modifications continued for a further three
months, the standard states that minor modifications indicate that the asset is substantially
complete.
Cost at 30 June Year 4 Rs.000 Rs.000
Land 1,000
Building: Construction cost 8,000
Interest 9% × 5million × (20/12) years
(1 October Year 2 to 1 June Year 4) 750
8,750
Total 9,750

Prima is to receive a government grant. IAS 20 requires that the grant be recognised when there
is reasonable assurance that the entity will meet any conditions and receive the grant. As the
grant has not been received, a receivable will be recorded under current assets. The credit can
be treated in one of two ways:
Option 1: Record as deferred income and release to profit or loss over the useful life of the asset
Option 2: Deduct the grant from the carrying amount of the asset.
If the second option is taken, the asset will be carried at Rs. 8.25m rather than at Rs. 9.75m. The
effect on profit or loss will be the same in both cases.
Land should not normally be depreciated, because land has an indefinite useful life in most
situations. However, as buildings have a limited useful life, a residual value must be allocated to
the building and the depreciable amount must then be written off over the 50 year useful life.
Depreciation will be charged in Year 4 for the four months from 1 September to 31 December.
The estimates of residual value and useful life must be revised each year and the depreciation
amended prospectively.
Yachts
It is important to note that the yachts are held for rental purposes, so they are non-current assets,
not inventory.
The yachts cost Rs. 20m to build, but the recoverable amount on completion (higher of value in
use and net selling price) is only Rs. 18m, and so the assets must be initially recognised at their
recoverable amount. The impairment write down of Rs. 2m will be charged to profit or loss in
Year 4 in accordance with IAS 36.
Recove
Cost rable
amount
Rs. m Rs. m
Engines (15%) 3 2.7
Interior (25%) 5 4.5
Remainder (60%) 12 10.8
20 18
IAS 16 requires that each part of the asset that has a cost that is significant in relation to the total
cost must be depreciated separately. Therefore, in the first year the depreciation charge will be
as follows:
Rs. m
Engines Rs. 2.7m × 1/3 × 9/12 = 0.675
Interior Rs. 4.5m × 1/2 × 9/12 = 1.688
Remainder Rs. 10.8m × 1/5 × 9/12 = 1.620
Charge to profit or loss in Year 4 3.983

© Emile Woolf International 234 The Institute of Chartered Accountants of Pakistan


Answers

CHAPTER 15 - IFRS 13: FAIR VALUE MEASUREMENT


15.1. MONIBA LIMITED
Principal market
Market A is the principal market because it has highest market share
Most advantageous market
Market C is the most advantageous market because it has highest exit price - net of
transportation cost and transaction cost of Rs. 28,200 per unit(W-1)
W-1 : Net proceeds to determine most advantageous market

Market A Market B Market C


Exit price (Rs. per unit) 29,500 30,500 29,600
Transport cost (Rs. per unit) (800) (1,000) (400)
Transaction cost (Rs. per unit) (700) (1,500) (1,000)
Net proceeds 28,000 28,000 28,200

Fair value of the asset in:

- Principal market (29,500 - 800) Rs. 28,700

- Most advantageous market (29,600 - 400) Rs. 29,200

© Emile Woolf International 235 The Institute of Chartered Accountants of Pakistan


Advanced accounting and financial reporting

CHAPTER 16 - IFRS 15: REVENUE FROM CONTRACTS WITH CUSTOMERS


16.1. PARVEZ LIMITED
(1) Sale and repurchase agreement
The transaction is in the nature of sale and repurchase agreement therefore the economic
phenomenon of the transaction is that of a loan for which the goods have been given as
security. Therefore no contract of sale of goods or services is identified.
The difference between the sale price of Rs.18m and the repurchase price of Rs.19m
represents the interest on the loan for a period of four months.
To account for the transaction in accordance with its substance:
 The goods should remain in inventories of PL at the lower of cost and net realisable
value.
 No sale should be recorded.
 The amount once received from the bank should be treated as a current loan liability
of Rs.18m.
 Interest should be charged applying implicit rate to profit or loss for each reporting
period.
(2) Consignment inventories
There is a contract for sale of cars between Pervez Limited (PL) and dealer containing
confirmation of respective right and obligation, payment term, commercial substance and
probability of collection of price.
There is only one performance obligation, namely, the transfer of cars to the dealer.
As per contract, the transaction price would be list price on the date of sale to third parties
during the six month period. Thereafter, though not specifically mentioned, after the lapse
of fifteen days the list price applicable on sixteenth day would be the transaction price of
the unsold cars not returned.
Since there is only one performance obligation, the question of allocation of transaction
price does not arise till the time of sale to third parties.
PL will recognize revenue upon satisfaction of performance obligation. Performance
obligation would be satisfied once the dealer has sold any cars to third parties during the
six month period. Thereafter, if the dealer does not return the unsold cars within fifteen
days, the performance obligation would be considered as satisfied on sixteenth day.
On 31 March 2017 the vehicles should remain in inventories in PL books of accounts.

16.2. SACHAL LIMITED


International Financial Reporting Standard (IFRS 15) provides that the revenue is recognized:
(a) when the performance obligation is satisfied by the entity by transferring a promised
good or service (ie an asset) to the customer; and
(b) the asset is transferred when the customer obtains the control of that asset.
Based on this principle, the following is the considerations to be taken into account in determining
accounting for revenue:
a) Restaurant management software
There exists a contract for sale of Restaurant management software between SL and
customers containing confirmation of respective right and obligation, payment term,
commercial substance and price is collected in advance.

© Emile Woolf International 236 The Institute of Chartered Accountants of Pakistan


Answers

There are two performance obligations, namely:


 Explicit: delivery of software and
 Implicit: six month on-site support
As per contract, the transaction price is Rs.1.5 million for both performance obligations.
Based on stand-alone selling price approach, software will be priced as Rs.1.35 million
(i-e. 1.50 m – 0.15) and six month on-site support services will be priced as Rs.0.15
million (i-e. 0.30 million x 6/12).
PL will recognize revenue from sale of software upon delivery if SL can objectively
conclude that the software meets the requirements of the customer. The term of full
payment of transaction price in advance is a reasonable evidence of clarity of
specification between SL and customer. The agreed thirty days trial time will be
considered as a formality of the contract.
PL will recognize revenue from on-site support services over six months period on
straight-line basis.
b) Maintenance support for the standard software package
Such service is provided under a written contract that contains confirmation of respective
right and obligation, payment term, commercial substance. SL will assess the
collectability of the price if not received in advance.
The performance obligation is to provide maintenance and support services.
The price of the service is Rs.0.30 million for one year term.
Since there is only one performance obligation, the question of allocation of transaction
price does not arise.
PL will recognize revenue over one year period on straight-line basis, as in this case
input method is appropriate. The pattern of resources consumed by SL is evenly spread
over the period of contract.
c) Customized software
Such service is provided under a written contract that contains confirmation of respective
right and obligation, payment term, commercial substance. SL will assess the
collectability of the price.
The performance obligations are:
 Designing and development of customized software, and
 Maintenance and support services of the said software
The price of the service will be determined on the basis of terms of contract.
The price will be allocated between the two performance obligations. Price of
maintenance services for the first year is included in the total contract price. The
allocated price would be 10% of the contract price, which is the stand-alone price of the
said services.
Satisfaction of performance obligation:
Revenue from design and development - PL will recognize revenue from design and
development over time, because the software at every stage is expected to be customer
specific and would have no alternative use for SL. The terms of payment at different
stages of project also confirms that SL would have an enforceable right to receive
payment if the contract is terminated before completion. In this case output method would
be appropriate, as the resources applied on different stages vary. Therefore, the amount
of recognized revenue would correspond to the development stage of the software at the
end of reporting period.
Revenue from Maintenance and support services - PL will recognize revenue over one
year period on straight-line basis, as in this case, input method is appropriate. The
pattern of resources consumed by SL is evenly spread over the period of contract.

© Emile Woolf International 237 The Institute of Chartered Accountants of Pakistan


Advanced accounting and financial reporting

16.3. BRILLIANT LIMITED


Identification of performance obligations
There are three performance obligations:
1. Transfer of 15 Plastic card printing machines and its software
2. Transfer of 8 Laminators
3. Transfer of 100,000 plastic cards
Although the software is distinct from printing machine, but both are highly dependable to each
other and inter-related. In the context of this contract, these are providing a combined output to
PL. Therefore, software is not a separate performance obligation.
The total transaction price as per the contract is Rs.9.2 million.
On the basis of available information the stand-alone prices of each item will be estimated using
the following approaches:
Plastic card printing machines and its software:
In the absence of observable stand-alone price, we may use ‘adjusted market assessment’
approach. The competitor’s machine is sold at Rs.750,000 which is similar (not identical) to BL’s
machine. As per given information, we may use customers’ rating for adjustment of competitors’
price that worked out as follows:
Rupees
Competitors’ price 750,000
Adjusted price of BL machine (7/9*750,000) 583,000
Total price (15*583,000) 8,745,000
Laminators:
There is neither observable stand-alone price nor any comparable competitors’ product available
in the market in which BL operates. In this case, we may use ‘expected cost plus a margin
approach’. The estimated stand-alone price is worked out as follows:
Rupees
Expected cost to BL 200,000
Margin estimated (800,000 - 600,000)/600,000 = 33% 66,000
266,000
Total price (8*266,000) 2,128,000
Plastic cards:
Observable stand-alone price is available
Total price (100,000*12) 1,200,000

Total of stand-alone prices is:


Plastic card printing machines and its software 8,745,000
Laminators 2,128,000
Plastic cards 1,200,000
Total 12,073,000

Allocation of Rs.9.2 million (transaction price) will be based on relative stand-alone prices, as the
difference of Rs.2.873 million between stand-alone price and transaction price is not specific to
any performance obligation.

© Emile Woolf International 238 The Institute of Chartered Accountants of Pakistan


Answers

Rupees
Plastic card printing machines and its software 6,663,961
(9,200,000*8,745,000/12,073,000)
Laminators 1,621,602
(9,200,000*2,128,000/12,073,000)
Plastic cards 914, 437
(9,200,000*1,200,000/12,073,000)
Total 9,200,000

16.4. WAQAS LIMITED


The following is the available data of the original project:

Transaction price Rs.20 million

Cost of the project Rs.12 million

At the signing of the contract only one performance obligation is identified. Therefore, the
question of allocation the transaction price of Rs.20 million would not arise.

The revenue would be recognized over time because the installation and construction will be
done on the land of ACL and control of asset will be transferred progressively and will create right
of payment for WL. Amount of revenue recognized would correspond to the progress of the
project. The progress will be measured using input method, that is, cost incurred plus margin.

At the end of seventh month:

Additional Reservoir:

a) is distinct from original RO plant project

b) increased the price of the contract by Rs.2.5 million which reflected WL’s stand-alone price
of similar construction work. The following working explains it further:

Cost estimated Rs.1.8 million

Usual margin (8/12*100=67%) Rs.1.2 million

Normal price Rs.3.0 million

Agreed consideration Rs.2.5 million

The reduced price is reasonable due to less administrative resources is to be applied for
additional work.

The contract of additional reservoir will be treated as separate contract and its revenue will be
recognized separate from original contract. The revenue from this contract will be recognized
over time, as construction of reservoir will be done on the land of ACL and control of asset will be
transferred progressively and will create right of payment for WL.

At this stage the revenue from RO plant project will be recognized as follows:

Percentage of work completed

(4.2/12.0*100) 35%

Revenue to be recognized (35%*20) Rs.7.0 million

© Emile Woolf International 239 The Institute of Chartered Accountants of Pakistan


Advanced accounting and financial reporting

At the end of tenth month:


Increasing the size of reservoir will increase the scope of the contract, but it cannot be
considered as a distinct work already agreed. Increased contract price also does not reflect WL’s
stand-alone price of similar work because it is equal to the cost of work. Therefore, WL should
account for this modification as part of single performance obligation that is partially satisfied on
the date of modification. A cumulative catch-up adjustment will be done, which is worked out as
follows:
Original contract Modified
Contract price 20.0 21.0
Total contract cost 12.0 13.0
Cost incurred so far 7.2 7.2
% of completion 60% 55%
Cumulative Revenue recognition 12.0 11.55
Difference between the two amounts of cumulative revenue will be the adjustment to the revenue
account.
Revenue from additional reservoir
% of completion (0.72/1.8 * 100) 40%
Revenue to be recognized (40% * 2.5) Rs.1.0 million
At the end of sixteenth month:
Additional work of pumping and piping facility increased the scope of the contract. It is also
distinct from the RO plant project. However, the increased price of the contract does not reflect
WL’s stand-alone price of similar work because it provides nominal margin to WL. Therefore, this
contract cannot be accounted for as separate contract. This contract will terminate the existing
contract and create a new contract. There will be two performance obligations (a) Transfer of RO
plant; and (b) transfer of pumping and piping facility.
The price of new contract is worked out as follows:
% of completion of existing contract (11.70/13.0*100) 90%
Rupees
Revenue recognized (21.0 * 90%) 18.9m
Remaining promised consideration (21.0 – 18.9) 2.1m
Consideration of modification 3.0m
New contract price 5.1m
Allocation of new contract price on the basis of cost plus margin approach
Total estimated cost of new modified contract (13.0+2.8) 15.8m
Less: Already incurred cost 11.7m
Cost to be incurred 4.1m
Allocation
RO plant project (1.3/4.1*5.1) 1.62m
Pumping and piping facility (2.8/4.1*5.1) 3.48m
The revenue will be recognized over time.
Revenue from additional reservoir
% of completion (1.35/1.8 * 100) 75%
Revenue to be recognized (75% * Rs. 2.5m) 1.875m

© Emile Woolf International 240 The Institute of Chartered Accountants of Pakistan


Answers

16.5. ZEBRA LIMITED


i. Since ZL has granted the supplier the right to choose whether the share-based transaction is
settled in cash or by issuing equity instruments, the entity has granted a compound financial
instrument.
Since the fair value of land is available so the Land will be recorded at Rs. 230 million and
corresponding effect will be taken to liability to the extent of Rs. 210 million (fair value of the
debt component on 1 October 2017 i.e. 7,000 shares × 3,000 per share) and remaining Rs.
20 million to the equity.
On 31 December 2017 the liability will be remeasured in accordance with the prevailing fair
value of HL’s share to Rs. 203 million (i.e. 7,000 × 2,900) and the resulting decrease of Rs. 7
million will be credited to Profit and loss account.
ii. Since a part of the payment for the license has been deferred beyond normal credit terms so
the license will be initially recognised at cash price equivalent of Rs. 80 million i.e. Rs. 50
million plus Rs. 30 million (i.e. present value of Rs. 36.3 million discounted at 10% for 2
years.)
The advertisement cost of Rs. 10 million incurred on launching of the channel cannot be
included in the cost of the license and will be charged to Profit and loss account.
Since the renewal cost is significant so the useful life of the license will be restricted to the
original 5 years only.
The residual value of the license will be assumed to be zero since there is no active market
for the license and there is no commitment by 3rd party to purchase the license at the end of
useful life.
The amortization for the year will be Rs. 12 million [(80 – 0) × 1/5 ×9/12] calculated from 1
April 2017 when the license was available for use:
Unwinding of interest expense of Rs. 2.25 million (30 × 10% × 9/12) shall be recorded with
increasing the liability of payable for license with same amount.

16.6. HAWKS LIMITED


Total
Total
Total assets comprehensive
liabilities
income
--------------- Rs. in million ---------------
Given 2,500.00 1,610.00 659.00

(i) Revenue
Revenue to be booked (W-1) 9.00 (40.00) 49.00
Contract cost (W-2.1) 14.77 14.77
23.77 (40.00) 63.77
Revised amounts 2,523.77 1,570.00 722.77

W-1: Revenue to be recognized

Bonus for Total revenue to


Revenue to be booked higher be booked
rating
----------------------- Rs. in million -----------------------
Advertisement (at point of time) [3×6.01(W-2)] 18.03 4.00 22.03
Broadcasting (over the time) [(2.7×9.99(W-2)] 26.97 26.97
49.00

© Emile Woolf International 241 The Institute of Chartered Accountants of Pakistan


Advanced accounting and financial reporting

W-2: Allocation of transaction price

Standalone Contract price Revenue per


price (Rs. in Proportion (Rs. in million) unit (Rs. in
million) million)
Advertisement (5×1.3×5)32.5 37.57% 30.06 OR 6.01
Broadcasting (9×1.2×5)54.0 62.43% 49.94 9.99
86.5 100% 80.00

W-2.1: Contract cost

Advertisement Broadcasting Total


------------------------------ Rs. in million ------------------------------
Total 18.20 35.60 53.80
Charged to P & L (15.10) (23.93) (39.03)
(8.5+9.2)+(8.9×0.7)

3.1 11.67 14.77


[9+(8.9×0.3)]

© Emile Woolf International 242 The Institute of Chartered Accountants of Pakistan


Answers

CHAPTER 17 - IFRS 16: LEASES


17.1. X LTD
(a)
A B C D E
Period Opening Fin. Charge Rentals Closing Balance
Balance at 15% of B (B – (D - C)
Rs.’000 Rs.’000 Rs.’000 Rs.’000
2016 11,420 1,713 4,000 9,133
2017 9,133 1,370 4,000 6,503
2018 6,503 975 4,000 3,478
2019 3,478 522 4,000
──── ────
4,580 16,000
──── ────
(b)
Statement of Financial Position (Extract) as at 31 December 2016

Rs.’000
Non-Current assets
(Rs.11,420,000 – Rs.2,855,00) 8,565
Non-Current Liabilities
(Obligation under lease) 6,503
Current Liabilities
Obligation under lease
(Rs.9,133,000 – Rs.6,503,000) 2,630

Note: Annual Depreciation


11,420,000
= = Rs.2,855,000
4

17.2. PROGRESS LTD


(a) Annuity method

Year 1 Year 2 Year 3


Rs. Rs. Rs.
Cash flow 3,200,000 - -
Outstanding - 1,920,000 1,350,400
Capital repayment 1,280,000 569,600 637,952
Balance 1,920,000 1,350,400 712,448
Interest @ 12% of balance 230,400 162,048 85,494
Capital repayment 569,600 637,952 714,506
800,000 800,000 800,000

© Emile Woolf International 243 The Institute of Chartered Accountants of Pakistan


Advanced accounting and financial reporting

(b) Journal entries

Dr Cr
Rs. Rs.
2016
Jan. 3 Right of use - Plant and machinery 3,200,000
Fine Rentals Limited 3,200,000
Initial recognition of machine
Jan. 3 Fine Rentals Limited 1,280,000
Bank 1,280,000
Payment of initial deposit under lease
Dec. 31 Fine Rentals Limited 569,600
Interest expense 230,400
Bank 800,000
Apportionment of annual installment
between Principal repayment and interest

Dec. 31 Profit and Loss Account 230,400


Interest Expense 230,400
Write-off of FL interest expense to Profit
and loss account

2017
Dec. 31 Fine Rentals Ltd 637,952
Interest expense 162,048
Bank 800,000
Apportionment of annual installment for
the year between Principal repayment
and interest
Dec. 31 Profit and Loss Account 162,048
Interest Expense 162,048
Write-off of FL interest expense to Profit
and loss account
2018
Dec. 31 Fine Rentals Limited 714,506
Interest expense 85,494
Bank 800,000
Apportionment of annual installment for
the year between Principal repayment and
interest
Dec. 31 Profit and Loss Account 85,494
Interest Expense 85,494
Write-off of FL interest expense to Profit
and loss account

© Emile Woolf International 244 The Institute of Chartered Accountants of Pakistan


Answers

17.3. MIRACLE TEXTILE LIMITED


Miracle Textile Limited
Statement of financial position (extracts) as at 30 June 2016
Note 2016 2015
ASSETS Rs. Rs.
Non-current assets
Right of use - Machinery 4 16,000,000 18,000,000

LIABILITIES
Non-current liabilities
Obligation under lease 9 6,505,219 10,633,074

Current liabilities
Current portion of obligation 9 4,127,856 3,566,925

Miracle Textile Limited


Notes to the financial statements (extracts) for the year ended 30 June 2016

4- Property, plant and equipment 2016 2015

Right of Use Assets


Cost
Opening balance 20,000,000 -
Addition during the year - 20,000,000
20,000,000 20,000,000
Accumulated depreciation
Opening balance (2,000,000) -
Depreciation for the year (2,000,000) (2,000,000)
(4,000,000) (2,000,000)
Balance as at 30 June 16,000,000 18,000,000

9- Obligations under lease (W1)


30-Jun-16 30-Jun-15
Financial Financial
Lease charges Present Lease charges for Present
payment for future Value payment future Value
periods periods
Rs. Rs. Rs. Rs. Rs. Rs.
Not later than
one year 5,800,000 - 5,800,000 5,800,000 - 5,800,000
Later than
one year but
not later than
five years 7,800,000 1,294,781 6,505,219 13,600,000 2,966,925 10,633,075
Later than
five years - - - - - -
13,600,000 1,294,781 12,305,219 19,400,000 2,966,925 16,433,075

© Emile Woolf International 245 The Institute of Chartered Accountants of Pakistan


Advanced accounting and financial reporting

9.1 The Company has entered into a lease agreement with a bank in respect of a machine.
The lease liability bears interest at the rate of 15.725879% per annum. The company
has the option to purchase the machine by paying an amount of Rs.2 million at the end
of the lease term. The lease rentals are payable in annual instalments ending in June
2016. There are no financial restrictions in the lease agreement.

W1: Lease Schedule

Payment Opening Principal Interest @ Closing


Instalment
date principal repayment 15.73% principal

01-Jul-14 20,000,000 5,800,000 5,800,000 - 14,200,000

01-Jul-15 14,200,000 5,800,000 3,566,925 2,233,075 10,633,075

01-Jul-16 10,633,075 5,800,000 4,127,856 1,672,144 6,505,219

01-Jul-17 6,505,219 5,800,000 4,776,997 1,023,003 1,728,222

30-Jun-18 1,728,222 2,000,000 1,728,222 271,778 -

20,000,000 5,200,000

17.4. ACACIA LTD


Relevant extracts
Statements of profit or loss for the year ended 31 March 2016 (extracts)
Rs.
Depreciation (272,850 ÷ 6) 45,475
Lease payments 6,000*
Finance costs (W) 19,460

* Considering low value item as described in IFRS16


Statement of financial position as at 31 March 2016 (extracts)

Rs.
Non-current assets
Right of use(272,850 – 45,475) 227,375
Non-current liabilities
Lease liabilities (Note 1) 135,810
Current liabilities
Lease liabilities (Note 1) 78,250

Statement of cash flows for the year ended 31 March 2016 (extracts)

Cash flows from financing activities


Payment of lease liabilities (78,250)

© Emile Woolf International 246 The Institute of Chartered Accountants of Pakistan


Answers

Notes to the financial statements (extracts)


(1) Analysis of lease liabilities
Gross basis
Rs.
Lease liabilities include the following:
Gross payments due within
One year 78,250
Two to five years (2 × 78,250) 156,500
234,750
Less: Finance charges allocated to future periods
((78,250 × 4) – 272,850 – 19,460) (20,690)
214,060
(Alternatively) Net basis
Rs.
Lease liabilities include the following:
Amounts due within
One year 78,250
Two to five years 135,810
214,060
WORKING:
Lease of plant

Year to 31 March B/f Payment Capital Interest @ 10% C/f

Rs. Rs. Rs. Rs. Rs.

2016 272,850 (78,250) 194,600 19,460 214,060

2017 214,060 (78,250) 135,810

17.5. SHOAIB LEASING LIMITED


(a) Entries in the books of Lessor

Date Particulars Dr. Cr.

1-Jul-16 Lease payments receivable (W1) 2,680,000


Machine 2,100,000
Unearned finance income (W1) 580,000

30-Jun-17 Bank 860,000


Lease payments receivable 860,000

30-Jun-17 Unearned finance income 272,941


Finance income (W2) 272,941

30-Jun-18 Bank 860,000


Lease payments receivable 860,000

30-Jun-18 Unearned finance income 196,640


Finance income (W2) 196,640

© Emile Woolf International 247 The Institute of Chartered Accountants of Pakistan


Advanced accounting and financial reporting

30-Jun-19 Bank 960,000


Lease payments receivable 960,000

30-Jun-19 Unearned finance income 110,419


Finance income (W2) 110,419

W1: Total finance income Rs.


Total future lease payments (Rs.860,000 x 3) 2,580,000
Add: Purchase bargain option 100,000
Gross investment in finance lease 2,680,000
Less: Cost of assets 2,100,000
Total finance income 580,000

W2: Amortization schedule


Principal Principal
Instalment Interest Principal
Date Opening Closing
Rs.
30-Jun-17 2,100,000 860,000 272,941 587,059 1,512,941
30-Jun-18 1,512,941 860,000 196,640 663,360 849,581
30-Jun-19 849,581 960,000 110,419 849,581 nil
580,000 2,100,000

(b) Shoaib Leasing Limited


Extracts from the statement of financial position as at June 30, 2017
Note 2017
Rs.
Non-current Assets
Net investment in leases 10 849,578
Current Assets
Current portion of net Investment in leases 663,360
10 Net investment in leases
Lease payments receivables 10.1 1,720,000
Add: Residual value of leased assets (part
of LP) 100,000
Gross Investments in leases 1,820,000
Less: Unearned lease income (307,062)
Net investment in leases 10.2 1,512,938
Less: Current portion of net investment in leases (663,360)
849,578

© Emile Woolf International 248 The Institute of Chartered Accountants of Pakistan


Answers

10.1 Lease payments


Less than one year 860,000
More than one year and less than 5 years 960,000
1,820,000
10.2 Net investment in leases
Less than one year 663,360
More than one year and less than 5 years 849,578
1,512,938

17.6. AKBAR LTD.


a) Right-of-use retained by AL
Financing
Since the consideration (Rs.850,000) exceeds the fair value (Rs.550,000) of the machine, the
agreement contains a financing transaction.
AL initially recognises a right-of-use asset as the proportion of the carrying amount that
reflects the right of use retained. The proportion is calculated by dividing the present value of
the lease payment by fair value
=> 440,000 CV ÷ 550,000 FV × 314,457 (W-1) = Rs.251,565
W-1
Fair value of Rs.614,456 less the part of the lease payments that is just a repayment of the
financing granted to the seller-lessee (Rs.300,000) = Rs.314,456
b) Gain / loss on rights transferred
Gain (refer below) = Rs.47,109
Rs.
Consideration received 850,000
Less: Financial liability
Financing 300,000
PV of lease liability 314,456
(A) 235,544
Less: Carrying value of machine transferred
Total carrying value 440,000
Less: Right-of-use asset 251,565
(B) 188,435
Gain on rights transferred (A-B) 47,109

Accounting Entry by Akbar Ltd.

Dr. Cr.
Rs. Rs.
Cash 850,000
Right-of-use asset 251,565
Machine 440,000
Financial liability 614,456
Gain on rights transferred to lessor 47,109

© Emile Woolf International 249 The Institute of Chartered Accountants of Pakistan


Advanced accounting and financial reporting

17.7. ALI LIMITED


Since transfer of an asset does not satisfy the requirements of IFRS 15 therefore Ali Ltd. treats
the transaction as a financing arrangement.
The sale proceeds have been incorrectly credited to operating income, and the operating costs
have been incorrectly debited with the lease payment. Both amounts should be reversed.
Therefore, Ali Ltd. is required to adjust its books by passing the following accounting entries:
Dr. Cr.
Rs. Rs.
Operating income 1,440,000
Financial liability 1,440,000

Operating expense of Rs.360,000 booked erroneously is rectified by reversing it and debiting:


 Interest expense of Rs.115,200 (i-e. Rs.1,440,000 x 8%)
 Financial liability of Rs.244,800 (i-e. principal portion)
Rs.360,000
The accounting entry would be:
Dr. Cr.
Rs. Rs.
Interest expense 115,200
Financial liability 244,800
Operating Expense 360,000

The remaining liability of Rs.1,195,200 should be shown as Rs.931,200 non-current and


Rs.264,000 as current.

17.8. MOAZZAM TEXTILE MILLS LIMITED


Generator A
The ratio between the carrying value (Rs.7,500,000) and fair value
(Rs.6,000,000) will determine the value of right-of-use as against PV of lease payments.
Lease liability
The PV of lease payments is computed by the following formula:
PV = R[1-(1+i)^-n]/i
R = Yearly payment; i = rate per annum; n = number of years
PV = 1,000,000x[1-(1+4.5%)^-5}/4.5%
PV = Rs.4,389,977
Right-of-use
ROU = CV/FV*PV
ROU => 7,500,000/6,000,000*4,389,977 = Rs.5,487,471
Loss on sale
Loss (refer working) = Rs.402,506
Working
Consideration received 6,000,000
Less: PV of lease liability (4,389,977)

© Emile Woolf International 250 The Institute of Chartered Accountants of Pakistan


Answers

Less: Carrying value of machine transferred


Total carrying value 7,500,000
Less: Right-of-use asset (5,487,471) (2,012,529)
Loss on sale = 402,506
Particulars Debit Credit
Rs. Rs.
Cash / Bank 6,000,000
Right-of-use 5,487,471
Loss on sale 402,506
Generator – Carrying value 7,500,000
Lease Liability 4,389,977
Generator B
Financing transaction
Since the consideration received (Rs.6,000,000) exceeds the fair value (Rs.5,000,000) of the
power generator, the agreement contains a financing transaction.
Sale and lease back
The ratio between the carrying value (Rs.6,000,000) and fair value (Rs.5,000,000) will determine
the value of right-of-use as against PV of lease payments.
Lease liability
The PV of lease payments is computed by the following formula:
PV = R[1-(1+i)^-n]/i
R = Yearly payment; i = rate per annum; n = number of years
PV = 1,000,000x[1-(1+4.5%)^-5}/4.5%
= Rs.4,389,977
Less: Financing = Rs.1,000,000
PV = Rs.3,389,977
Right-of-use
ROU = CV/FV*PV
ROU = 6,000,000/5,000,000*3,389,977
ROU = Rs.4,067,972
Loss on sale
Loss (refer W1) = Rs.322,005
W1
Consideration received 6,000,000
Less:
PV of lease liability (3,389,977)
Financing (1,000,000)
1,610,023
Less: Carrying value of machine transferred
Total carrying value 6,000,000
Less: Right-of-use asset (4,067,972) 1,932,028
Loss = Rs.322,005

© Emile Woolf International 251 The Institute of Chartered Accountants of Pakistan


Advanced accounting and financial reporting

Particulars Debit Credit


Rs. Rs.
Cash / Bank 6,000,000
Right-of-use 4,067,972
Loss 322,005
Generator – Carrying value 6,000,000
Lease Liability 4,389,977
Generator C
The ratio between the carrying value (Rs.7,000,000) and fair value (Rs.10,000,000) will
determine the value of right-of-use as against PV of lease payments.
Lease liability
The PV of lease payments is computed by the following formula:
PV = R[1-(1+i)^-n]/i
R = Yearly payment; i = rate per annum; n = number of years
PV = 1,500,000x[1-(1+4.5%)^-5}/4.5%
PV = Rs.6,584,965
Right-of-use
ROU = CV/FV*PV
ROU => 7,000,000/10,000,000*6,584,965 = Rs.4,609,475
Gain on sale
Gain (refer W2) = Rs.1,024,510
W2
Consideration received 10,000,000
Less: PV of lease liability 6,584,965
Less: Carrying value of machine transferred
Total carrying value 7,000,000
Less: Right-of-use asset (4,609,475) 2,309,525
Gain = Rs.1,024,510
Particulars Debit Credit
Rs. Rs.
Cash / Bank 10,000,000
Right-of-use 4,609,475
Generator – Carrying value 7,000,000
Lease Liability 6,584,965
Gain on sale 1,024,510

17.9. MODIFICATION THAT DECREASES THE SCOPE OF THE LEASE (IFRS 16,
ILLUSTRATIVE EXAMPLE 17)
At the effective date of the modification (at the beginning of Year 6), Lessee remeasures the
lease liability based on:
(a) a five-year remaining lease term,
(b) annual payments of CU30,000 and
(c) Lessee’s incremental borrowing rate of 5 per cent per annum. This equals CU129,884.

© Emile Woolf International 252 The Institute of Chartered Accountants of Pakistan


Answers

Lessee determines the proportionate decrease in the carrying amount of the right-of-use asset
on the basis of the remaining right-of-use asset (ie 2,500 square metres corresponding to 50
per cent of the original right-of-use asset).
50 per cent of the pre-modification right-of-use asset (CU184,002) is CU92,001. Fifty per cent
of the pre-modification lease liability (CU210,618) is CU105,309. Consequently, Lessee
reduces the carrying amount of the right-of-use asset by CU92,001 and the carrying amount of
the lease liability by CU105,309.
Lessee recognises the difference between the decrease in the lease liability and the decrease
in the right-of-use asset (CU105,309 – CU92,001 = CU13,308) as a gain in profit or loss at the
effective date of the modification (at the beginning of Year 6).
Lessee recognises the difference between the remaining lease liability of CU105,309 and the
modified lease liability of CU129,884 (which equals CU24,575) as an adjustment to the right-
of-use asset reflecting the change in the consideration paid for the lease and the revised
discount rate.

17.10. MODIFICATION THAT BOTH INCREASES AND DECREASES THE SCOPE OF THE LEASE
(IFRS 16, ILLUSTRATIVE EXAMPLE 18)
a) The consideration for the increase in scope of 1,500 square metres of space is not
commensurate with the stand-alone price for that increase adjusted to reflect the
circumstances of the contract. Consequently, Lessee does not account for the increase in
scope that adds the right to use an additional 1,500 square metres of space as a separate
lease.
The pre-modification right-of-use asset and the pre-modification lease liability in relation to
the lease are as follows.

Lease liability Right-of-use asset

Beginning 6% Lease Ending Beginning Deprecia- Ending


balance interest payment balance balance tion balance
expense charge
Year CU CU CU CU CU CU CU

1 736,009 44,160 (100,000) 680,169 736,009 (73,601) 662,408


2 680,169 40,810 (100,000) 620,979 662,408 (73,601) 588,807
3 620,979 37,259 (100,000) 558,238 588,807 (73,601) 515,206
4 558,238 33,494 (100,000) 491,732 515,206 (73,601) 441,605
5 491,732 29,504 (100,000) 421,236 441,605 (73,601) 368,004
6 421,236 368,004

b) At the effective date of the modification (at the beginning of Year 6), Lessee remeasures
the lease liability on the basis of: (a) a three-year remaining lease term, (b) annual
payments of CU150,000 and (c) Lessee’s incremental borrowing rate of 7 per cent per
annum. The modified liability equals CU393,647, of which (a) CU131,216 relates to the
increase of CU50,000 in the annual lease payments from Year 6 to Year 8 and (b)
CU262,431 relates to the remaining three annual lease payments of CU100,000 from Year
6 to Year 8.
c) Decrease in the lease term
At the effective date of the modification (at the beginning of Year 6), the pre-modification
right-of-use asset is CU368,004. Lessee determines the proportionate decrease in the
carrying amount of the right-of-use asset based on the remaining right-of-use asset for the
original 2,000 square metres of office space (ie a remaining three-year lease term rather
than the original five-year lease term). The remaining right-of-use asset for the original
2,000 square metres of office space is CU220,802 (ie CU368,004 ÷ 5 × 3 years).

© Emile Woolf International 253 The Institute of Chartered Accountants of Pakistan


Advanced accounting and financial reporting

At the effective date of the modification (at the beginning of Year 6), the pre-modification
lease liability is CU421,236. The remaining lease liability for the original 2,000 square
metres of office space is CU267,301 (ie present value of three annual lease payments of
CU100,000, discounted at the original discount rate of 6 per cent per annum).
Consequently, Lessee reduces the carrying amount of the right-of-use asset by
CU147,202 (CU368,004 – CU220,802), and the carrying amount of the lease liability by
CU153,935 (CU421,236 – CU267,301). Lessee recognises the difference between the
decrease in the lease liability and the decrease in the right-of-use asset (CU153,935 –
CU147,202 = CU6,733) as a gain in profit or loss at the effective date of the modification
(at the beginning of Year 6).

Lease liability CU153,935


Right-of-use asset CU147,202
Gain CU6,733

At the effective date of the modification (at the beginning of Year 6), Lessee recognises the
effect of the remeasurement of the remaining lease liability reflecting the revised discount
rate of 7 per cent per annum, which is CU4,870 (CU267,301 – CU262,431), as an
adjustment to the right-of-use asset.

Lease liability CU4,870


Right-of-use asset CU4,870

d) Increase in the leased space


At the commencement date of the lease for the additional 1,500 square metres of space
(at the beginning of Year 6), Lessee recognises the increase in the lease liability related to
the increase in scope of CU131,216 (ie present value of three annual lease payments of
CU50,000, discounted at the revised interest rate of 7 per cent per annum) as an
adjustment to the right-of-use asset.

Right-of-use asset CU131,216


Lease liability CU131,216

The modified right-of-use asset and the modified lease liability in relation to the modified
lease are as follows.

Lease liability Right-of-use asset

Beginning 7% Lease Ending Beginning Deprecia- Ending

balance interest payment balance balance tion balance

expense charge

Year CU CU CU CU CU CU CU

6 393,647 27,556 (150,000) 271,203 347,148 (115,716) 231,432

7 271,203 18,984 (150,000) 140,187 231,432 (115,716) 115,716

8 140,187 9,813 (150,000) - 115,716 (115,716) -

© Emile Woolf International 254 The Institute of Chartered Accountants of Pakistan


Answers

17.11. SUBLEASE CLASSIFIED AS A FINANCE LEASE (IFRS 16, ILLUSTRATIVE EXAMPLE 20)
The intermediate lessor classifies the sublease by reference to the right-of-use asset arising
from the head lease. The intermediate lessor classifies the sublease as a finance lease,
having considered the requirements in paragraphs 61–66 of IFRS 16.
When the intermediate lessor enters into the sublease, the intermediate lessor:
i. derecognises the right-of-use asset relating to the head lease that it transfers to the
sublessee and recognises the net investment in the sublease;
ii. recognises any difference between the right-of-use asset and the net investment in the
sublease in profit or loss; and
iii. retains the lease liability relating to the head lease in its statement of financial position, which
represents the lease payments owed to the head lessor.
During the term of the sublease, the intermediate lessor recognises both finance income on
the sublease and interest expense on the head lease.

17.12. SUBLEASE CLASSIFIED AS AN OPERATING LEASE (IFRS 16, ILLUSTRATIVE


EXAMPLE 21)
The intermediate lessor classifies the sublease by reference to the right-of-use asset arising
from the head lease. The intermediate lessor classifies the sublease as an operating lease,
having considered the requirements in paragraphs 61–66 of IFRS 16.
When the intermediate lessor enters into the sublease, the intermediate lessor retains the
lease liability and the right-of-use asset relating to the head lease in its statement of financial
position.
During the term of the sublease, the intermediate lessor:
(a) recognises a depreciation charge for the right-of-use asset and interest on the lease liability;
and
(b) recognises lease income from the sublease.

17.13. TRACK LIMITED


(a) Accounting entries for the year ended 30 June 2015

Debit Credit
Date Description
Rs. in million
01-07-2014 Cash 600
Financial liability 600
(Recognition of sale proceeds as financial
liability)
30-06-2015 Interest expense (W-1) 66.31
Financial liability 66.31
(Recognition of interest expense)
30-06-2015 Financial liability 90
Strong Bank Limited / Bank 90
(Payment of rentals)
30-06-2015 Depreciation expense (240 ÷15) 16
Accumulated deprecation 16
(Recording of depreciation of the property)

© Emile Woolf International 255 The Institute of Chartered Accountants of Pakistan


Advanced accounting and financial reporting

Debit Credit
Date Description
Rs. in million
30-06-2015 Deferred tax asset (W-2) 105.70
Deferred tax income 105.70
(Creation of deferred tax asset)
Brief explanation of the accounting treatment:
In the given situation the asset has been sold but the right to use has been retained along with
the right to repurchase the asset (call option).
In such situation the transaction can either be treated as a lease or as a financing transaction.
However, since in the given situation the present value of outflows (rentals and repurchased
price) i.e. Rs. 610.45 million (W-3) is higher than original selling price, the transaction is to be
treated as a financing transaction.
(b) Accounting entries for the year ended 30 June 2016

Debit Credit
Date Description
Rs. in million

30-06-2016 Interest expense (W-1) 63.69

Financial liability 63.69

(Recognition of interest expense)

30-06-2016 Financial liability 90.00

Strong Bank Limited / Bank 90.00

(Recognition of interest expense)

30-06-2016 Depreciation expense (240 ÷ 15) 16.00

Accumulated deprecation 16.00

(Recording of depreciation of the


property)

30-06-2016 Financial liability (W-1) 550

Property (240-16-16) 208

Gain on sale of property 342

(De-recognition of financial liability and


property)

30-06-2016 Deferred tax expense (W-2) 105.70

Deferred tax assets 105.70

(Derecognition of deferred tax asset)

W-1 Repayment schedule


Opening Interest @ Closing
Date Payment
principal 11.052% principal
1-Jul-14 600.00
30-Jun-15 600.00 66.31 (90.00) 576.31
30-Jun-16 576.31 63.69 (90.00) 550.00

© Emile Woolf International 256 The Institute of Chartered Accountants of Pakistan


Answers

W-2 Computation of deferred tax


Accounting Tax expense/ Deferred tax
Temp diff
expense (income) @ 30%
Interest 66.32 90 (23.68) (7.10)
Depreciation 16.00 - 16.00 4.80
Gain on sale of
property - (360) 360 108
105.70

W-3 PV of rentals and repurchase price


1-jul-14 30-jun-15 30-jun-16
Rent payment (net of tax) - 90 90
Repurchase price - 550
- 90 640
Discount factor @ 10% 1 0.909 0.826
Cash flows - 81.81 528.64
Net Cash flows 610.45

17.14. PATEL LIMITED


Patel Limited
Statement of financial position
As on 30 June 2017

Assets Rs. in million


Non current assets
Net investment in lease (W-2) 51.32
Right of use asset (W-4) 98.10

Current assets
Current portion of net investment in lease [21– 7.17 (W-2)] 13.83

Non current liabilities


Lease liabilities [32.50 (W-1) + 86.77 (W-3)] 119.27

Current liabilities
Lease liabilities [13.83 (18 – 4.17)(W-1) + 37.57 (50 – 12.43) (W-3)] 51.40
Patel Limited
Statement of profit or loss
For the year ended 30 June 2017

Rs. in million
Gain on sub-lease (W-6) 18.73
Depreciation (W-4) 32.70
Finance charges [5.31(W-1) + 15.85 (W-3)] 21.16
Finance income (W-2) 8.54
Loss on decrease in lease term of building (W-5) 8.40

© Emile Woolf International 257 The Institute of Chartered Accountants of Pakistan


Advanced accounting and financial reporting

W-1: Amortization schedule of lease - plant


Interest Instalment Principal o/s
Date
-------------------------- Rs. in million --------------------------
1-Jul-15 *170.66

30-Jun-16 6.36 18.00 59.02


30-Jun-17 5.31 18.00 46.34
30-Jun-18 4.17 18.00 32.50
*1[{1 - (1.09)-5 ÷ 0.09} X 18] – [1 x (1.09) -5]

W-2: Amortization schedule of sub lease - plant


Interest Instalment Principal o/s
Date
-------------------------- Rs. in million --------------------------
30-Jun-16 *277.61

30-Jun-17 8.54 21.00 65.15


30-Jun-18 7.17 21.00 51.32
*2[{1 - (l.ll) -5÷ 0.11} x 21]

W -3 : Amortization schedule of lease - Building (After modification)

Interest Instalment Principal o/s


Date
-------------------- Rs. in million --------------------

1-Jul-16 158.49

30-Jun-17 15.85 50 124.34

30-Jun-18 12.43 50 86.77

W-4 : Computation of right of use (ROU) asset (after modification)


Rs. in million
ROU assets – 1 July 2014 [50 × 4.9676 [{1– (1.12)-8÷0.12}] 248.38
Depreciation for two years (248.38 ÷ 8 × 2) (62.10)
ROU (before modification) – 1 July 2016 186.28
ROU derecognized due to reduction in lease term (186.29 ÷ 6 × 2) (62.10)
124.18
Increase in ROU due to decrease in borrowing rate
 PV of liability for remaining 4 years at 10% (50 × 3.1699) 158.49
 PV of liability for remaining 4 years at 12% (50 × 3.0373) (151.87)
6.62
ROU after modification – 1 July 2016 130.80
Depreciation for the year – 2016-17 (130.80 ÷ 4) 32.70
98.10

© Emile Woolf International 258 The Institute of Chartered Accountants of Pakistan


Answers

W-5 : Computation of loss on decrease in lease term of building

Decrease in lease liability [205.57(50 × 4.1114) – 151.87(W-4)] 53.70


ROU derecognized (186.29 ÷ 6 × 2) (62.10)

Loss on decrease in lease term (8.40)

W-6 : Gain on sub lease

Net investment in sub lease [{1– (1.11)-5÷0.11}]× 21 77.61


Carrying value of ROU derecognized (70.66 ÷ 6 × 5) (58.88)

Gain on sub lease 18.73

© Emile Woolf International 259 The Institute of Chartered Accountants of Pakistan


Advanced accounting and financial reporting

CHAPTER 18 - IAS 12: INCOME TAXES


18.1. SHAKIR INDUSTRIES
COMPUTATION OF TAX EXPENSE
FOR THE YEAR ENDED DECEMBER 31, 2016
2016
Rs.in
million
Profit before tax 15.80
Add: Inadmissible expenses
Accounting depreciation (Rs. 1.1 million + Rs.0.7 million) 1.80
Financial charges on finance lease 0.15
Penalty paid to SECP 0.70
Provision for gratuity 2.40
5.05

Less: Admissible expenses Rs. m


Tax depreciation 1.65
Lease payments 0.65
Payment of gratuity 1.60
Borrowing cost capitalised 2.30
6.20
Taxable profit for the year 14.65
Current tax expense @ 35% 5.13

COMPUTATION OF DEFERRED TAX EXPENSE


FOR THE YEAR ENDED DECEMBER 31, 2016
Carrying Tax Temp
amount base difference
Rs. m Rs. m Rs. m
Fixed assets – Owned 16.70 13.85 2.85
Fixed assets – Leased 1.80 - 1.80
Capital work in progress 2.30 - 2.30
Provision for gratuity (0.7 + 2.4 – 1.6) (1.50) - (1.50)
Obligation against assets subject to finance
lease (1.20) - (1.20)
Total 4.25
Deferred tax expense @ 35% 1.49

Rs.in
million
Deferred tax liability (Opening) 0.55
Deferred tax expense for the year (balancing figure) 0.94
Deferred tax liability as at December 31, 2016 (Rs. 4.25 million x 35%) 1.49

© Emile Woolf International 260 The Institute of Chartered Accountants of Pakistan


Answers

18.2. DWAYNE LTD (PART 1)


(a)
Carrying Tax Temporary To OCI
value base difference
Buildings 45,500 17,500
Revaluation 14,500  14,500
60,000 17,500 42,500

Plant 68,000 26,000 42,000


Investments 72,000 65,000 7,000 (1,000)
Dividend income 150 150 -
Loan 20,500 21,000 500
Defined Benefits 1,000  (1,000)
liability
91,000 13,500
@ 28% @28%
25,480 3,780

(b) Deferred taxation


liability
Rs. 000
Balance B/F 13,500
Due to change in rate (13,500 × 2/30) (900)
13,500 x 28/30 12,600
To OCI (28% x 13,500) 3,780
To statement of profit or loss (as a balancing figure) 9,100
25,480
(c) Journal
Debit Credit
Deferred tax liability 11,980
Movement due to rate change:
OCI ((24,000 × 30%) × 2/30) 480
P&L (balancing figure) 420
Movement due temporary differences arising
during 2013
OCI 3,780
P&L 9,100

18.3. DWAYNE LTD (PART 2)


(a) Rs. 000
DWAYNE 25,480
LARRY FV Tax base
Buildings 600 300 300
Plant 56 25 31
Inventory 152 144 8
Retirement (100)  (100)
benefit
239
@28% 67
(b) 25,547

© Emile Woolf International 261 The Institute of Chartered Accountants of Pakistan


Advanced accounting and financial reporting

Deferred
taxation
liability
Rs. 000
(c) Balance B/F 13,500
Due to change in rate (13,500 × 2/30) (900)
13,500 x 28/30 12,600
To OCI (28% x 13,500) 3,780
To statement of profit or loss (as a balancing figure) 9,100
Due to introduction of a new subsidiary 67
25,547
(d) Goodwill
Rs. 000
Cost 750
Less share of net assets
80% x (778 – 67) (569)
Goodwill arising 181

18.4. COHORT
Note for presentation to partner
Subject: Deferred Taxation
The calculation and presentation of deferred tax is considered by IAS 12 Income taxes. A
company is required to provide deferred tax on all material temporary differences using the full
provision method. Temporary differences arise because there is a difference in timing between
transactions being reflected in the financial statements and the item being taxed.
In light of the recent acquisitions of Legion and Air, deferred tax must be considered for the group
accounts. Additional tax issues arise at the group level that will not have been reflected in the
individual entity’s accounts and these points are outlined below.
Once the temporary differences have been identified, deferred tax must be provided at the tax
rate expected to be effective at the date when the tax is settled. Given this rate is not known
when the differences arise, a provision is made using the rates enacted at that time and the
estimate is then confirmed as tax changes arise.
Air
(a) The acquisition of air mid-year gives rise to a number of issues:
(1) Intangible asset
There is some concern that the acquisition of the database of key customers may
not be allowed for tax purposes but it has nevertheless been included in the tax
calculation on the assumption that a deduction will be allowed by the tax authorities.
If this deduction is not allowed, then an additional tax payment will need to be made
to the authorities, hence it would be prudent to recognise a liability for this amount
(probably classified as current taxation, rather than deferred taxation).
(2) Inter-company sales
When goods are sold between group members, the profits made are seen as
unrealised in the group accounts until the items are sold outside of the group.
However, the tax authorities tax the individual entities, not the group, and so the
profit will be subject to tax at the time of the inter-company sale. The unrealised
profit represents the temporary difference on which deferred tax must be provided.
The goods were sold at a margin of 33⅓%. Goods sold for Rs. 1.8m remain in
inventory at the year end, and hence the unrealised profit, and therefore temporary
difference, is Rs.0.6m.

© Emile Woolf International 262 The Institute of Chartered Accountants of Pakistan


Answers

(3) Unremitted profits


Un-remitted earnings represent a temporary difference in the group accounts. This
is because the tax base is the cost of the investment, yet in the consolidated
accounts, this cost is uplifted by the post-acquisition un-remitted profits. IAS 12
requires a provision to be made on this timing difference unless the parent controls
the timing of the reversal and it is probable that the difference will not reverse for the
foreseeable future. The payment of dividends is under the control of Cohort, but we
understand that the intention is to realise these un-remitted earnings through future
dividends and hence a provision must be made.
(b) Legion
The acquisition of Legion at the start of the year brings further deferred tax issues in the
group accounts as outlined below.
(1) Fair value through the profit or loss investments
The fair value adjustment has been reflected in the financial statements, yet the gain
is not taxed until the investments are sold. Hence the fair value adjustment of Rs.
4m gives rise to a temporary difference upon which deferred tax must be provided.
As the gain has been taken to profit or loss rather than other comprehensive income
or reserves, the deferred tax must also be expensed to profit or loss.
(2) General allowance
The allowance against the loan portfolio has reduced the carrying value of the loans
but the tax relief is not available until the loan is written off, and hence a temporary
difference is created on the provision. As the tax relief will reduce future tax charges,
the temporary difference of Rs. 2m gives rise to a deferred tax asset. The temporary
difference is accounted for even though there is no expectation that the difference
will reverse in the immediate future. However, a deferred tax asset can only be
reflected to the extent that it is probable that there will be future taxable profits
against which the temporary difference can be relieved.
(3) Unrelieved tax losses
When Legion was acquired, it had unused tax losses brought forward which could,
in principle, give rise to a deferred tax asset. However, it can only be recognised to
the extent that it is believed that the loss can be recovered. Given your belief that
there will not be sufficient future profits, the deferred tax can only be partially
recognised. If the fortunes of Legion change in the future, the deferred tax asset
should then be recognised, leading to a compensating amendment to goodwill.

18.5. MODEL TOWN GROUP


Adjustment to
financial Tax Temporary
statements base difference
Rs.000 Rs.000 Rs.000 Rs.000
Property plant, and equipment 10,000 2,400 7,600
Goodwill 6,000 6,000
Other intangible assets 5,000 0 5,000
Financial assets (cost) 9,000 1,500 9,000 1,500
──────
Total non-current assets 30,000
──────
Trade receivables 7,000 7,500 (500)
Other receivables 4,600 5,000 (400)
Cash and cash equivalents 6,700 6,700 –
────── ──────
Total current assets 18,300
──────
Total assets 48,300
──────
Long term borrowings 10,000 (400) 10,000 400

© Emile Woolf International 263 The Institute of Chartered Accountants of Pakistan


Advanced accounting and financial reporting

Deferred tax liability 3,600 3,600 –


Employee benefits 4,000 5,000 1,000
Current tax liability 3,070 3,070 –
Trade and other payables 5,000 4,000 (1,000)
────── ──────
Total liabilities 25,670 13,600
────── ──────
Share capital 9,000 –
Other reserves 4,500 1,500
400
Retained earnings 9,130
──────
Total equity 22,630
──────
Rs.000
Deferred tax liability 15,500 @ 30% 4,650
Deferred tax asset (1,900) @ 30% (570)
──────
Net deferred tax liability 13,600 @ 30% 4,080
Less existing liability (3,600)
──────
Adjustment to deferred tax 480
──────
(i) The financial assets should be valued at fair value with the increase going to OCI (Rs. 1.5
million).
(ii) The bond should be split into its equity and liability elements as per IAS 32.
(iii) As the development costs have been allowed for tax already, it will have a tax base of
zero. Goodwill is measured as a residual and, therefore, the impact is not measured under
IAS 12.
(iv) The accrual for compensation will not be allowed until a later period and, therefore, will
reduce the tax base relating to trade and other payables.

18.6. ELEPHANT LIMITED


Notes to the financial statement
For the year ended 31 December 2017

Taxation Rs. in million


Current tax (W-1) 22.12
Deferred tax 8.45
30.57

Reconciliation between tax expense and


Rs. in million Alternate
accounting profit
Accounting profit 103
Tax at applicable rate / applicable tax rate 30.90 30.00%
Donations not allowable (12 × 30%) 3.60 3.50%
Exempt grant (10 × 30%) (3.00) (2.91%)
Low rate on dividend (4 × 20%) (0.80) (0.78%)
Share scheme expense not allowed
[4.5 – 3.25{(150-20)×5,000×10}÷2] × 30% 0.37 0.36%
Effect of decrease in tax rate on opening
deferred tax liability [(3.5/0.35) × (0.35 – 0.3)] (0.50) (0.49%)
Tax expense / Average effective tax rate 30.57 29.68%

© Emile Woolf International 264 The Institute of Chartered Accountants of Pakistan


Answers

Movement in deferred tax liability/asset


Recognised in
Closing
Opening P&L
Equity OCI
(Bal.)
---------------------------- Rupees in million ----------------------------
Arising in respect of:
PPE 33.25 18.00 (10.75) 40.50
(60X30%) (95-20+60) x 30%

Unused tax losses (29.75) 29.75 -


Unpaid expense - (9.00) (9.00)
(30 x 30%)

Share scheme - (0.98) (0.98)


[(150-20) x 5,000 x 10]/ 2 x
30%

TFCs - 2.73 (0.57) 2.16


[9.11(150- [150-(140.89+1.91)] x 30%
140.89) x 30%] OR (9.11-1.91) x 30%

3.50 2.73 18.00 8.45 32.68

W-1: Computation of current tax Rs. in million


Accounting profit 103.00
Donations not allowable 12.00
Unpaid expenses allowable upon payment 30.00
Exempt government grant (10.00)
Dividend income taxable at lower rate (4.00)
Excess accounting depreciation 20.00
Shares scheme allowable on exercise (180 × 5,000 × 10) / 2 4.50
Finance cost on TFC (140.89 (W-2) × 12%) 16.91
Interest payment (150×10%) (15.00)
Taxable income 157.41
Unused tax losses (85.00)
72.41
Tax @ 30% 21.72
Tax @ 10% on dividend 0.40
Current tax 22.12

W-2: Computation of liability component Rs. in million


PV of interest amount (15 × 3.0373) 45.56
PV of principal (150 × 0.6355) 95.33
Liability component 140.89

© Emile Woolf International 265 The Institute of Chartered Accountants of Pakistan


Advanced accounting and financial reporting

CHAPTER 19 - PRESENTATION OF FINANCIAL STATEMENTS (IAS 34, IAS 24)


19.1. FAZAL LIMITED
The related parties comprise a subsidiary, an associated undertaking/an entity having significant
influence, director and key management personnel.
Aggregate transactions with related parties are as follows:
Entity
Key
having
Subsidiary Associate Director Management
significant
Personnel
influence
Rupees Rupees Rupees Rupees
Transactions
Sales 500,000,000
Sales discount 25,000,000
Sales return 5,500,000
Purchase of raw
5,000,000
material
Purchase of equipment 3,000,000
Purchase of machinery 14,000,000
Balances
Advances
At beginning of the
1,400,000
year
Repaid during the year 300,000
At the end of the year 1,100,000
(i) Sales discount represents a special discount which is not usually allowed to other
customers.
(ii) All transactions with related parties have been carried out on commercial terms and
conditions.

19.2. BABER LIMITED


(i) AK Associates will not be treated as related party merely on the ground that both entities
have a director in common.
However, if it can be proved that an entity has some influence on other entity; they will
be treated as related parties.
(ii) Provider of finance is not necessarily a related party. However, SS Bank has power to
appoint its nominee director in the Board and therefore enjoys significant influence; it will
be treated as related party.
(iii) Mr. Zee will not be treated as related party unless it can be proved that he has significant
influence over the CEO. Further, IAS-24 does not explicitly include ‘Brother’ in the
definition of close family member of an individual.
(iv) A distributor with whom an entity transacts a significant volume of business will not be
treated as related party, merely by virtue of the resulting economic dependence.
(v) Since Mr. Tee is the key management personnel of the company, he will be treated as
related party.
(vi) A post-employment benefit plan for the benefits of the employee is treated as related
party.
(vii) A supplier with whom an entity transacts a significant volume of business cannot be
termed a related party, merely by virtue of the resulting economic dependence.
(viii) Ms. Vee will be treated as related party as she is a close family member of CEO, and
hence in a position to influence her husband.

© Emile Woolf International 266 The Institute of Chartered Accountants of Pakistan


Answers

19.3. GOLDEN LIMITED


Notes to the Financial Statements
For the year ended December 31, 2011
Platinum Limited is the parent company which holds majority shares of the company.
Related party transactions
The transaction with related parties are carried out in the ordinary course of business at
commercial rates except stated otherwise.

Associated Key Major


Parent
Under- Management Share-
Company
takings Personnel holders

Rupees in '000

Transactions:

Sales 18,000

Sales discount 1,500

Sale of property 10,000

Reimbursement of expenses on
sale of property 500

Interest free loans granted 2,000

Short term borrowings acquired 25,000

Interest on short term borrowings 1,500


Balances:

Accounts receivable 6,500 5,000

Loans to staff 1,800

Loans payable 25,000

Interest payable on the loan at


12% 1,500

20.1 Sales to related parties have been made at 20% mark-up as against GL's policy to sell at
a markup of 30%.

20.2 Administrative services are provided by the parent company free of cost as per the
agreement. Market value of these services is Rs. 350,000.

20.3 In respect of sale of property, a buyer is required to bear all costs incurred on transfer. But
in this case the company has reimbursed the costs to SL

20.4 The interest free loan has been granted to the executive director as per the terms of
employment.

© Emile Woolf International 267 The Institute of Chartered Accountants of Pakistan


Advanced accounting and financial reporting

19.4. METAL LIMITED


IN THE BOOKS OF METAL LIMITED
Transactions with Related Parties
Related parties comprises of the company’s subsidiaries. Transactions with related parties are
as follows:
2013 2012
Rupees
Subsidiaries
Sale of machine (at carrying amount plus 20%) - 19,200,000
Management fees income (Note 23.1) 12,000,000 -
Management fee receivable 1,000,000
Other receivables - Sale of machine - 19,200,000
No management fee was charged during the year ended 30 June 2012. Except for this, all
transactions have been carried out on arm’s length basis, as approved by the board of
directors of the company.
IN THE BOOKS OF COPPER LIMITED
Transactions with Related Parties
Related parties comprise of Metal Limited (parent company) and its subsidiaries. Transaction
with related parties can be summarized as follows:
2013 2012
Rupees
Parent Company
Purchase of machine - 19,200,000
Management fees (Note 23.1) 6,000,000 -
Management fee payable 500,000 -
Other payables - Sale of machine 19,200,000
23.1 No management fee was charged for the year ended 30 June 2012. Except for this, all
transactions have been carried out on arm’s length basis, as approved by the board of
directors of the company.
IN THE BOOKS OF ZINC LIMITED
23 – Transactions with Related Parties
Related parties comprise of Metal Limited (parent company) and its subsidiaries. Transaction
with related parties can be summarized as follows:
2013 2012
Rupees
Parent Company
Contract for factory extension project (Note 23.1) 15,000,000 -
Management fees (Note 23.2) 6,000,000 -
Management fee payable 500,000 -
23.1 The contract has been awarded to Iron Builders and Developers in which one of the
directors of the parent company is a partner.

© Emile Woolf International 268 The Institute of Chartered Accountants of Pakistan


Answers

23.2 No management fee was charged for the year ended 30 June 2012. Except for this, all
transactions have been carried out on arm’s length basis, as approved by the board of
directors of the company.

IN THE BOOKS OF STEEL LIMITED

Related parties comprise of Metal Limited (parent company) and its subsidiaries. However,
there was no related party transaction during the year.

19.5. ENGINA
Report to: The Board of Directors of Engina
From: XXXXXXXX
Date:
Subject: Related party transactions
Related party transactions
This report addresses the disclosure requirements of IAS 24 Related Party Disclosures with regard to
Engina. IAS 24 requires that all entities, listed or otherwise, provide disclosure of such transactions as
they may affect the assessments made by users of an entity’s operations, risks and opportunities.
It is understood that Engina is reluctant to disclose related party transactions because they are
believed to be both politically and culturally sensitive, however the following advice must be
followed in order to secure a listing/stock exchange registration.
IAS 24: Scope and purpose
IAS 24 does not provide any exclusion from its scope, and so disclosure must be made. Related
party transactions are a normal feature of business, but an entity’s ability to succeed in business
is often affected by the strength of its relationship with other entities and individuals. The results
of the entity may be affected if these relationships were to be terminated. For example, the ability
of an entity to trade in a particular country may only be possible because of the presence of its
subsidiary in that local market. Similarly, prices and terms of trade may be preferential because
of the strength of the relationship. Therefore IAS 24 requires knowledge of these transactions to
be provided to the reader of the financial statements.
The results of an entity may be affected even if the related party transactions do not occur. A
parent may cease trading with a business partner upon acquisition of a subsidiary that can
supply similar products.
Disclosure must be given irrespective of whether the transactions took place at an arm’s length
value, as such transactions may still be lost if the relationship is terminated. Hence the
knowledge of such transactions provides valuable information to investors and regulators.
Disclosure requirements
IAS 24 requires that, at a minimum, the following disclosures must be given:
 The amount of the transaction
 The amount of any outstanding balance and the terms, conditions and guarantees
attached
 Allowance for any irrecoverable debts or amounts written off in the period
 Disclosure that transactions were at an arm’s length value can only be given if this
information can be substantiated.
Disclosures relevant to Engina
The following outlines the related party disclosure requirements for the three transactions you
have specifically requested comment on. It is your responsibility to bring any further related party
transactions to our attention in order that they can also be incorporated into your financial
statement disclosures.

© Emile Woolf International 269 The Institute of Chartered Accountants of Pakistan


Advanced accounting and financial reporting

(a) Sale of goods to directors


The sale of goods and a company car to Mr Satay both constitute related party
transactions, due to Mr Satay’s position as a director of Engina. IAS 24 requires disclosure
of all related party transactions with key management personnel. However, accounting
standards only apply to material transactions. An item is considered material where
knowledge of that transaction might influence the decisions of a user of the financial
statements. Materiality is not just a matter of size, as small transactions with a director may
still be of relevance to an investor if the transaction is material to the director, despite not
being material to the entity.
In the situation described, the transactions amount to Rs. 600,000 of sales and the sale of
a company car for Rs. 45,000 (market value Rs. 80,000). In terms of value these
transactions appear not to be material to Engina and neither do they appear to be material
in value to Mr Satay. However, given the sensitive nature of transactions with directors,
and especially senior directors like Mr Satay, the transactions should be disclosed in the
financial statements in accordance with good corporate governance practice.
Significant contracts with directors, such as these with Mr Satay, may also require
disclosure by the local Stock Exchange.
(b) Hotel property
The sale of the hotel to the brother of Mr Soy, constitutes a related party transaction
because of Mr Soy’s status as Managing Director. The property seems to have been sold
at below market price and IAS 24 requires disclosure of any information surrounding a
transaction which will allow the reader to understand its impact on the financial statements.
The hotel had a carrying value of Rs. 5m, but given the fall in market values it should have
been written down to its recoverable amount in accordance with IAS 36 Impairment.
Recoverable amount is measured at the higher of value in use (Rs. 3.6m) and fair value
minus costs of sale (Rs. 4.3 - 0.2m). Hence the property should have been recorded in the
statement of financial position at Rs. 4.1m.
As the property was sold at Rs. 100,000 less than this impaired value, disclosure of this
fact should be made, together with any other information relevant to the reader, such as
the reason for the sale in light of the expected decline in prices in the future.
(c) Mr Satay
Mr Satay has investments in 100% of the equity of Car and 80% of the equity of Wheel. In
turn, Wheel owns 100% of Engina. Engina and Wheel are related because of their parent-
subsidiary relationship. In addition, because all three entities are under the common
control of Mr Satay, IAS 24 also considers Engina and Car to be related. Therefore, the
transactions between Engina and both Wheel and Car are related party transactions.
The transactions will need to be disclosed in the individual financial statements of all three
entities. In the group accounts, all intra-group transactions are cancelled on consolidation,
and so disclosure need not be made at this level.
Further disclosure requirements of director’s interests in the equity of Engina may be
necessary under local Companies Acts requirements and Stock Exchange rules.

© Emile Woolf International 270 The Institute of Chartered Accountants of Pakistan


Answers

CHAPTER 20 - IAS 33: EARNINGS PER SHARE


20.1. AIRCON LTD
(a) Earnings Per Share
𝑃𝑟𝑜𝑓𝑖𝑡 𝑎𝑓𝑡𝑒𝑟 𝑡𝑎𝑥
=
𝑁𝑜.𝑜𝑓 𝑠ℎ𝑎𝑟𝑒𝑠 𝑖𝑠𝑠𝑢𝑒𝑑

1,854
2016 = Rs. = Rs. 1.01
1,818
1,584 6.06
2015 = x = Rs. 1.69
900 6.30
Workings
1. Calculation of theoretical ex-rights price
1 share at Rs. 6.30 each 6.30
2 rights issue for every 1 at Rs. 5.94 11.88
3 shares for 18.18
18.18
Price per share = = Rs. 6.06
3

2. Weighted average number of shares


6.30
1 April – 30 Sept. 2015 = 900m x 6/12 x = 467.8
6.06
1 Oct. – 31 March 2016 = 2,700 x 6/12 = 1,350
1,818

(b) Report
To: Mr Hamad

From: Management Accountant

Date: 15 April 2016

Subject: Evaluating the changes in EPS of Aircon Ltd


The key factors which has led to changes in the EPS of Aircon Ltd. are as follows:
Revenue and profitability. Revenue increased by Rs. 2,700 million (18%) last year, but
the gross profit and net profit ratios have not increased proportionately.
The gross profit percentage fell from 40% to 37% in 2016, while the net profit
percentage remained constant at 10%.
Factors responsible for the decline might be due to the inability of the entity to maintain
good profit margin coupled with the failure to also maintain good control over operating
expenses.
The more funds realised from the rights issue did not lead to any significant increase in
return on capital employed which fell from 43% (2,880/6,606) in 2015 to 25%
(3,240/12,780) in 2016.
Capital employed: raising over Rs. 5,760 million of new finance was largely used to
acquire intangible assets.
It is hoped that this asset will start generating substantial returns in the near future.
EPS has therefore fallen from Rs. 1.69 in 2015 to Rs. 1.01 in 2016.

© Emile Woolf International 271 The Institute of Chartered Accountants of Pakistan


Advanced accounting and financial reporting

Signed
Management Accountant
APPENDIX TO THE REPORT
The ratios that are relevant to discussion and evaluation of changes in EPS of Aircon Ltd
are those that relate to profitability and return on capital employed.
The effect of the rights issue should also be considered in the discussion in relation to how
the funds raised through the shares were employed.
TABLE OF RATIOS
(i) Change in revenue 18,000  15,300
= x 100 = 18% Increase
18,000
2016 2015
(ii) Costs of sales/revenue 11,340 6,120
= 63% = 40%
18,000 15,300
(iii) Gross profit % 6,600 6,120
= 37% = 40%
18,000 15,300
(iv) Net profit % 1,854
= 10% = 10%
18,000
(v) Operating expenses % 3,420 3,420
= 19% = 22%
18,000 15,300
(vi) Interest payable/sales 540 576
= 3% = 4%
18,000 15,300
(vii) Taxation/sales 846 720
= 5% = 5%
18,000 15,300
(viii) Capital employed 3,240 2,880
=25% = 43%
9,180  3,600 3,006  3,600
(ix) Assets/turnover 18,000 15,300
= 1.41 = 2.32
12,780 6,606
Relevance of EPS to shareholders
(i) The EPS is used to compute the price earning (P/E) ratio, a major market indicator
to determine how successful a company has been operating.
(ii) The price earning figure is a multiple of the EPS, where the multiple represents the
number of years’ earnings required to recoup the price paid for the share.
(iii) Rising trend in EPS is a more accurate performance indicator than rising trend in
profit after tax. The investor should consider the future economic conditions of an
entity with some other ratios such as dividend cover and ROCE.
(iv) EPS is a measure of performance from the existing and potential investors’
perspective.
(v) EPS show the amount available to each ordinary shareholder thereby indicating the
potential returns on individual investment.
(vi) EPS is used to compare the activities of two entities in the same industry.

© Emile Woolf International 272 The Institute of Chartered Accountants of Pakistan


Answers

20.2. CACHET LTD


BASIC EPS DILUTED
EPS
i. No change in share capital
PAT - Pref Div 100 69,000 - 1,380
= × 20,700
No. of shares
= 3.27 N/A
───── ────
ii. Bonus issue on 30 Sept. 2016:
No. of shares before bonus issue 20,700
Bonus (1 for 4) 5,175
─────
No. of shares after bonus issue 25,875
─────
=
PAT - Pref Div
× 100 69,000 - 1,380
No. of shares
25,875
= 2.61 N/A
───── ────
iii. Rights issue on 1 Oct. 2016
Before rights issue 5 shares 1.80 20,700 9.00 37,260
Rights issue (1 for 5) 1 share 1.20 4,140 1.20 4,968
─── ───── ──── ─────
After rights issue 6 shares 3.00 24,840 10.20 42,228
─── ───── ──── ─────
Theoretical ex-right price (Rs. 10.20/6) 1.70 1.70
──── ─────

Bonus element of issue increases shares to 20,000  1.8/


1.7 = 21,176
Full price element of issue increases shares to 20,700  6/5 = 24,840
Weighted average number of shares in issue
21,176  9/12 15,882
24,840  3/12 6,210
22,092

EPS

=
PAT - Pref Div
× 100 69,000 - 1,380
No. of shares = 3.06/share
22,092

20.3. MARY
Rs.
2 existing shares have a cum rights value of (2  Rs. 4) 8
1 new share is issued for 1
––
3 new shares have a theoretical value of 9
––
Theoretical ex-rights prices = Rs. 9/3 = Rs. 3

© Emile Woolf International 273 The Institute of Chartered Accountants of Pakistan


Advanced accounting and financial reporting

Weighted
Number Time Bonus Rights average
of shares factor fraction fraction number of
Date
shares
1 January Brought
forward 5,000,000 1/12 6/5 4/3 666,667
1 February Bonus issue
(1 for 5) 1,000,000
–––––––––
6,000,000 2/12 4/3 1,333,333
1 April Rights issue
(1 for 2) 3,000,000
–––––––––
9,000,000 2/12 1,500,000
1 June Issue at full
market price
800,000
–––––––––
31 December Carried 9,800,000
forward 7/12 5,716,667
––––––––––
9,216,667
––––––––––

Earnings for Year 5 are (3,362,000 – 600,500 – 800,000) Rs. 1,961,500


EPS Year 5 = 1,961,500/9,216,667 = Rs.0.21 or 21 paisa
EPS Year 4 (adjusted) = Rs.0.32 × 3/4 × 5/6 = Rs.0.20 or 20 paisa

20.4. MANDY
Adjusted total earnings

Rs. Rs.
Reported earnings 2,579,000 1,979,000
Add back interest saved
(1,000,000  7%) (1,000,000  7%  9/12) 70,000 52,500
Minus tax at 30% (21,000) (15,750)
49,000 36,750
Adjusted total earnings 2,628,000 2,015,750
Number of shares
Year 4 Number of
shares
1 January Brought forward 5,000,000
Dilutions:
Share options (W) 200,000
Convertible shares (1,000,000 ÷ 100  30) 300,000
––––––––––
31 December 5,500,000
––––––––––

© Emile Woolf International 274 The Institute of Chartered Accountants of Pakistan


Answers

Year 3 Weighted
Number of Time average
shares factor number of
Date shares
1 January Brought forward 5,000,000
Share options: dilution (W) 125,000
5,125,000 3/12 1,281,250
1 April Convertibles: dilution 300,000
5,425,000 9/12 4,068,750
5,350,000
Diluted EPS
Year 4 = 2,628,000/5,500,000 = Rs.0.48 or 48 paisa
Year 3 = 2,015,750/5,350,000 = Rs.0.38 or 38 paisa
Working
Cash receivable on exercise of all the options = 500,000 × Rs. 3 = Rs. 1,500,000

Year 4
Number of shares this would buy at full market price in Year 4 = Rs. 1,500,000/5 = 300,000
shares
Shares
Options 500,000
Minus number of shares at fair value (300,000)
––––––––
Net dilution 200,000
––––––––

Year 3
Number of shares this would buy at full market price in Year 3 = Rs. 1,500,000/4 = 375,000
shares
Shares
Options 500,000
Minus number of shares at fair value (375,000)
––––––––
Net dilution 125,000
––––––––

20.5. AAZ LIMITED


a) Step 1: Ranking in order of dilution
Increase Increase in no. Earnings per
in of ordinary incremental Rank
earnings shares shares
Rs. Rs.
Convertible Debentures
Increase in earnings
(Rs. 7.5m x 70%) 5,250,000 1.75 3
Increase in shares 3,000,000
Convertible Preference Shares
Increase in earnings
2,450,000 0.61 2
Increase in shares 4,000,000

© Emile Woolf International 275 The Institute of Chartered Accountants of Pakistan


Advanced accounting and financial reporting

Increase in no. Earnings per


Increase in
of ordinary incremental Rank
earnings
shares shares
Options
Increase in earnings -
Increase in shares - 1
(1.5m x 1.1 / 11) 150,000

Step 2: Testing for dilutive effect


Profit from
operations
attributable to
ordinary Ordinary
shareholders Shares EPS Effect
Rs. Rs.
Basic Earnings per share *125,380,000 85,220,000 1.471 -
Options (Rank 1) - 150,000
125,380,000 85,370,000 1.469 Dilutive

Convertible preference shares


(Rank 2) 2,450,000 4,000,000
127,830,000 89,370,000 1.430 Dilutive

Convertible debentures (Rank 3) 5,250,000 3,000,000


Anti-
133,080,000 92,370,000 1.44 Dilutive
*Rs. 127,830,000 – Rs. 2,450,000 = Rs. 125,380,000

b) AAZ Limited
Notes to the financial statements for the year ended December 31, 2016

EARNINGS PER SHARE


2016
Basic alternative to ordinary share holders
Profit (Rupees) 125,383,000
Weighted average number of ordinary shares outstanding during the
year 85,220,000
Earnings per share - basic (Rupees) 1.47

Diluted
Profit after taxation (Rupees) 127,833,000
Weighted average number of ordinary shares, options and
convertible preference shares outstanding during the year 89,370,000
Earnings per share - diluted (Rupees) 1.430
Because diluted earnings per share is increased when taking the convertible preference
shares into account (from Rs. 1.430 to Rs. 1.44), the convertible debentures are anti-
dilutive and are ignored in the calculation of diluted earnings per share.

© Emile Woolf International 276 The Institute of Chartered Accountants of Pakistan


Answers

20.6. ABC LIMITED


ABC Limited
Notes to consolidated financial statements for the year ended March 31, 2016
2016
Rs. in '000
Earnings per share basic
Profit after tax and non-controlling interest (15,000-2,000) 13,000
Dividend paid during the year to ordinary shareholders
-
(Rs. 4,000)
10% Cumulative preference dividend for 2015 (Rs. 2,000) -
10% Cumulative preference dividend for 2016 (2,000)
Dividend declared on 12% non-cumulative preference shares for 2016 (2,400)
Profit available for distribution to ordinary share holders 8,600
Diluted earnings per share
Profit available for distribution to ordinary share holders 8,600
Effect of dividend declared on 12% non-cumulative preference shares
convertible into ordinary shares on or before December 31, 2017 2,400
11,000
Weighted average number of ordinary shares W1 13,146
12% Non-cumulative preference shares convertible to ordinary shares
on or before December 31, 2017 W3 1,771
Weighted average number of ordinary shares - diluted 14,917
Antidiluted earning per share Rs. 0.74

W1: Weighted average ordinary shares outstanding for "Basic EPS"

Weighted
Number of Time Bonus average
Date
shares factor fractions (W3) number of
shares
1 April 2015 to 30 June 2015 10,000,000 × 3/12 6/51.00833 3,024,990
1 July
Conversion of cumulative prefs
at a premium of Rs. 2 per share
(500,000  10/12) 416,667
1 July to 30 September 10,416,667 × 3/12 6/51.00833 3,151,031
1 October
Rights issue 1,200,000

30 September to 31 December 11,616,667 × 3/12 6/5 3,485,000


1 January
Bonus issue (20%) 2,323,333
1 January to 31 March 13,940,000 × 3/12 3,485,000
Weighted average 13,146,021

© Emile Woolf International 277 The Institute of Chartered Accountants of Pakistan


Advanced accounting and financial reporting

W2: Calculation of bonus adjustment factor


No. of
@ Rs. Rs. in '000
shares
Bonus element with right issue
Outstanding shares before the exercise of rights at fair
value 10,417 12.50 130,213
Rights issued at a premium of Rs. 1.5 1,200 11.50 13,800
11,617 144,013

Rs.
Actual cum rights price per share 12.5000
Theoretical ex-right value per share (144,013/11,617) ÷ 12.3967
Adjusting factor 1.00833

Bonus issued on January 01, 2016 (20%)


Adjusting factor (6 shares for 5 shares) 1.2
W3: Diluted EPS
Number of Earnings
shares (Rs.) EPS (Rs.)
Basic EPS 13,146,021 8,600,000 0.65
Dilution:
Non-cumulative prefs in issue for the year
(W4)at a premium of Rs. 2 per share (for the
whole year)
2,000,000  10/12  12/12 1,666,667
Add back dividend paid to non-cumulative
prefs in issue at the year-end 2,400,000

Non-cumulative prefs actually converted in the


year (for the part of the year before
conversion)
(500,000  10/12)  3/12
i.e. 416,667  3/12 104,167
1,770,834
Adjusted figures 14,916,855 11,000,000 0.74
Diluted EPS: Rs. 11,000,000/14.917 million = Rs.0.74 per share
The non-cumulative preference shares are anti-dilutive

W4: Non-cumulative prefs in issue at the year-end


This can be found from the information about the dividend.
Rs. 2,400,000 is 12% of the nominal value of the shares.
Therefore, the nominal value is Rs. 20,000,000 (Rs. 2,400,000/0.12).
Therefore the number of shares (at Rs. 10 per share) is 2,000,000

© Emile Woolf International 278 The Institute of Chartered Accountants of Pakistan


Answers

20.7. ALPHA LIMITED


Alpha Limited
Extracts from consolidated profit and loss account for the year ended 31 December 2013
Rs. in '000
Profit for the year W.1 (49,462.16+26,950) 76,412.12
Profit attributable to
• Owners of Alpha Limited 76,412.12-5,390 71,022.12
• Non-controlling interest 26,950*20% 5,390
76,142.12
Earnings per share: Rupees
• Basic W.2 72.10
• Diluted W.2 53.39

W-1 Profit for the year AL ZL


(Rs. in '000)
Profit after tax 60,000.00 25,000.00
Cash dividend received from ZL (net of tax)
• Final dividend for 2012 (35,000*15%*80%)
*90% (3,780.00)
• Interim dividend for 2013 (35,000*1.16*12%*80%)
*90% (3,507.84)
FV gain on ZL's investment [67,000-(59,000+5,000)]
property (40.35) *65% 1,950
Cost of defined benefit (8,000-3,000)
gratuity sch. (19.120) *65% (3,250.00)
49,462.16 26,950

W-2 Basic / diluted EPS:


Weighted Basic
Basic/
average /Diluted
Diluted EPS
shares in earnings
(Rs.)
'000 (Rs. in '000)
Weighted average No. of shares:
1-Jan-2013 Balance
80,000/100, 800×7/12 800
1-Jan-2013 Bonus issue at 20%
(800*20%), 160×7/12 160
960
1-Aug-2013 Shares issued under
employees' share option
scheme
(60*5/12) 25
(960+60)×5/12
Basic earnings per share (EPS) 985 71,022 72.10

© Emile Woolf International 279 The Institute of Chartered Accountants of Pakistan


Advanced accounting and financial reporting

Weighted Basic
Basic/
average /Diluted
Diluted EPS
shares in earnings
(Rs.)
'000 (Rs. in '000)
Shares from assumed conversions:
1-Aug-2013 Convertible 12% bonds (5
shares for 4 bonds)
(30,000/100*5/4),
(30,000*0.12*0.65) 375 2,340
1-Aug-2013 Shares for no
consideration issued under
employees' share option.
(250-150)/250*60*7/12
(IAS 33.45) 14 -
Diluted earnings per share (EPS) 1,374 73,362 53.39

20.8. SAJJAD LIMITED

Rs. in million

Profit for the year 150.00

Less: Dividend

Class A Preference shareholders (9÷1.09×2) 16.51

Class B Preference shareholders (300×6%) 18.00

Profit attributable to class B preference shareholders


[90.49(W-1)×3÷(20+3)(W-2)] 11.80

46.31

Profit available for ordinary shareholders 103.69

Earnings per share (103.16÷10) 10.37

W-1 Undistributed earnings


Profit after tax 150.00
Less: Imputed dividend (16.51)
Dividend to class B preference shares (18.00)
Dividend to ordinary shareholders (25.00)
Undistributed earnings 90.49

W-2: Determination of ratio for distribution of undistributed earnings between ordinary and class
B preference shareholders

No. of outstanding
Weight Product
shares (in million)

Ordinary shareholder 10 2 20

Class B preference shareholder 3 1 3

23

© Emile Woolf International 280 The Institute of Chartered Accountants of Pakistan


Answers

20.9. TIGER LIMITED


a) Tiger Limited
EPS for quarter ended 31 December 2017

Numerator Denominator EPS Effect


Rs. in million Shares in million Rs. / share
Basic EPS 140.00 24.80 (W-1) 5.65
Warrant - - No effect
140.00 24.80 5.65
Bonds 8.05 1.60 5.03
(W-3) 0.8(2.4 ×1/3)+0.8(1.2×2/3)
OR 1.2+0.4(1.2÷3)
148.05 26.4 5.61 Dilutive

Rs. per share


Basic EPS 5.65
Diluted EPS 5.61

b) EPS for half year ended 31 December 2017

Numerator Denominator EPS Effect


Rs. in million Shares in million Rs. / share
Basic EPS 239.00 23.73 (W-2) 10.07
Warrant - 0.333
[6-(340÷360×6)]
239.00 24.06 9.93 Dilutive
Bonds 20.02 2.00 10.01
(W-3) 1.6(2.4 ×4/6)+0.4(1.2×2/6)
OR 1.2+0.8(1.2×4÷6)
259.02 26.067 9.94 Anti-dilutive

Rs. per share


Basic EPS 10.07
Diluted EPS 9.93

W-1: Weighted average shares for quarter ended 31 December 2017

Date Shares Period Total Alternate


1-Oct (20+4) 24.0 1÷3 8.00 24
1-Nov (0.8×3×50%) 1.20 0.8
(1.2×2÷3)
25.2 2÷3 16.80

24.80 24.80

© Emile Woolf International 281 The Institute of Chartered Accountants of Pakistan


Advanced accounting and financial reporting

W-2: Weighted average shares for half year ended 31 December 2017

Date Shares Period Total Alternate


1-Jul 20 1÷6 3.33 20.00
1-Aug 4 3.33
(4×5÷6)
24 3÷6 12.00
1-Nov 1.20 0.4
25.20 2÷6 8.40 (1.2×2÷6)
23.73 23.73

W-3: Interest on Bonds for half year (net of tax):

First quarter Rs. in million


July to Sep [(760×(9%×3/12×70%) ] 11.97
Second quarter
Oct [(760×(9%×1/12×70%)] 3.99
Nov to Dec [386.20(W-4)×(9%×2/12×70%) 4.06
8.05
20.02

W-4: Carrying value of bonds after conversion

Rs. in million
Initial recognition 760.00
Interest for the year (760×9%) 68.40
Interest paid (800×7%) (56.00)
772.40
Conversion (772.40×50%) (386.20)
386.20

© Emile Woolf International 282 The Institute of Chartered Accountants of Pakistan


Answers

CHAPTER 21 - IAS 36: IMPAIRMENT OF ASSETS


21.1. CHARLOTTE
Effect on Year 7 profit or loss

Rs.
Impairment loss
Machine 1 (W1) 122,300
Machine 2 (W2) 41,000
163,300
Depreciation charge
Machine 1: (100,000 ÷ 5) 20,000

Gain on disposal
Machine 2: (W2) 10,000
Machine 3: (210,000 - 195,000 (W2)) 245,000
255,000

Workings
(1) Machine 1

Rs.
Cost on 1 January Year 1 420,000
Depreciation to 1 January Year 6
5 years  ((420,000 – 50,000)/10 years)) (185,000)
Carrying amount on 1 January Year 6 235,000
Revalued to: 275,000
Revaluation gain before tax 40,000

In the year to 31 December Year 6 (on 1 January), the asset is revalued upwards by Rs.
40,000. Of this, Rs. 28,000 is taken to the revaluation reserve and Rs. 12,000 (Rs. 40,000
 30%) to deferred tax as a liability.

Dr (Rs.) Cr (Rs.)
Property, plant and equipment 145,000
Accumulated depreciation 185,000
Net effect on non-current assets 40,000
Revaluation surplus 28,000
Deferred tax liability 12,000

The total useful life of the asset was assessed as 15 years on 1 January Year 6. The asset
has already been owned for 5 years and depreciation in year 6 is based on the remaining
useful life of 10 years.
The company must also recognise incremental depreciation in accordance with section
235 of the Companies’ Act, 2017. An amount equal to the incremental depreciation net of
deferred taxation must be transferred to retained earnings through the statement of
changes in equity.

© Emile Woolf International 283 The Institute of Chartered Accountants of Pakistan


Advanced accounting and financial reporting

Dr (Rs.) Cr (Rs.)
Depreciation charge for the year
(275,000/10 years) 27,500
Accumulated depreciation 27,500
Revaluation surplus
(Rs. 28,000/10 years) 2,800
Retained earnings 2,800
Impairment loss:
Rs.
Carrying amount on 1 January Year 6 275,000
Depreciation to 1 January Year 7 (275,000 ÷ (15 – 5)) (27,500)
Carrying amount at 1 January Year 7 247,500
Recoverable amount (100,000)
Impairment loss 147,500

In the year to 31 December Year 7, the impairment loss is Rs. 147,500. Of this, Rs. 40,000
reverses the gain in the previous year. The revaluation reserve is reduced by Rs. 25,200
(Rs. 28,000 – Rs. 2,800). The remaining impairment loss of Rs. 122,300 is written off as a
loss in Year 7.
Also in the year to 31 December Year 7 the asset would be depreciated based on the
estimate of its remaining useful life of 5 years giving a charge of Rs. 20,000 (Rs. 100,000/
5 years).
(2) Machine 2
Rs.
Cost on 1 January Year 1 500,000
Depreciation to 1 January Year 7
6 years  ((500,000 – 60,000)/10 years)) (264,000)
Carrying amount on 1 January Year 7 236,000
Fair value minus cost to sell (200,000 – 5,000) (195,000)
Impairment loss 41,000

On 31 March Year 7 the machine is sold for Rs. 210,000 giving a gain on sale as follows:
Rs.
Proceeds 210,000
Selling costs (assumed to be as forecast) (5,000)
205,000
Carrying amount (195,000)
10,000
(3) Machine 3
Rs.
1 January Year 1 Cost 600,000
Depreciation to 1 January Year 2 (30,000)
Carrying amount on 1 January Year 2 570,000
Revalued to 800,000
Taken to revaluation reserve/deferred tax 230,000

© Emile Woolf International 284 The Institute of Chartered Accountants of Pakistan


Answers

The revaluation would have been accounted for as follows at 1 January Year 2
Dr (Rs.) Cr (Rs.)
Property, plant and equipment 200,000
Accumulated depreciation 30,000
Net effect on non-current assets 230,000
Revaluation surplus 161,000
Deferred tax liability 69,000
Depreciation and incremental depreciation would have been recognised in Year 2 to Year
6 inclusive as follows:
Dr (Rs.) Cr (Rs.)
Depreciation charge for the year
(800,000/8 years) 100,000
Accumulated depreciation 100,000

Revaluation surplus
(Rs. 161,000/8 years) 20,125
Retained earnings 20,125
This would result in balances for machine 3 and the revaluation surplus in respect of
machine 3 as follows:
Revaluation
Machine 3
surplus
Rs. Rs.
Carrying amount on1 January Year 2 800,000 230,000
Depreciation (5 years) (500,000)
Incremental depreciation (5 years) (100,625)
Balance at 1 January Year 7 300,000 129,375

Fair value on classification as held for sale 550,000


Costs to sell (5,000)
Fair value less costs to sell 545,000
Value at lower of carrying amount and fair value
less costs to sell: 300,000
On 31 March Year 7 the machine is sold for Rs. 550,000 giving a gain on sale as follows:
Rs.
Proceeds 550,000
Selling costs (assumed to be as forecast) (5,000)
545,000
Carrying amount (300,000)
245,000

The balance on the revalution reserve is transferred to retained earnings on the disposal of
the asset.
Dr (Rs.) Cr (Rs.)
Revaluation surplus 129,375
Retained earnings 129,375

© Emile Woolf International 285 The Institute of Chartered Accountants of Pakistan


Advanced accounting and financial reporting

21.2. ABA LIMITED


Aba Limited statement of profit or loss (extracts) – year to 31 March 2016

Note: workings in brackets are in Rs.000 Rs. Rs.


Depreciation: head office – 6 months to 1 October 2015
(1,200/25  6/12) 24,000
– 6 months to 31 March 2016
(1,350/22.5 (W1)  6/12) 30,000
–––––––
54,000
–––––––
Depreciation: training premises
– 6 months to 1 October 2016
(900/25  6/12) 18,000
– 6 months to 31 March 2016
(600/10  6/12) 30,000
––––––––
48,000
––––––––
Impairment loss (W2) 210,000
––––––––
258,000
––––––––
Statement of financial position (extracts) as at
31 March 2016 Rs. Rs.
Non-current assets
Land and buildings – head office (700 + 1,350 – 30) 2,020,000
– training premises (350 + 600 – 30) 920,000
––––––––
2,940,000
––––––––
Revaluation reserve
Head office land (700 – 500) 200,000
Building (1,350 – 1,080 (W1)) 270,000
Training premises land (350 – 300) 50,000
––––––––
520,000
Transfer to realised profit (270/22.5 (W1)  6/12
re depreciation of buildings) (6,000)
––––––––
514,000
––––––––
Workings
(W1) The date of the revaluation is two and a half years after acquisition. This means the
remaining life of the head office would be 22.5 years. The carrying value of the head
office building at the date of revaluation is Rs. 1,080,000 i.e. its cost less two and a half
years at Rs. 48,000 per annum (Rs. 1,200,000 – Rs. 120,000).
(W2) Impairment loss: the carrying value of training premises at date of revaluation is Rs.
810,000 i.e. its cost less two and a half years at Rs. 36,000 per annum (Rs. 900,000 – Rs.
90,000). It is revalued down to Rs. 600,000 giving a loss of Rs. 210,000. As the land and the
buildings are treated as separate assets the gain on the land cannot be used to offset the
loss on the buildings.

© Emile Woolf International 286 The Institute of Chartered Accountants of Pakistan


Answers

21.3. HUSSAIN ASSOCIATES LTD


(a) Impairment of plant
The plant had a carrying amount of Rs. 240,000 on 1 October 2015. The accident that may
have caused impairment occurred on 1 April 2016 and an impairment test would be done
at this date. The depreciation on the plant from 1 October 2015 to 1 April 2016 would be
Rs. 40,000 (640,000 x 121/2% x 6/12) giving a carrying amount of Rs. 200,000 at the date
of impairment. An impairment test requires the plant’s carrying amount to be compared
with its recoverable amount. The recoverable amount of the plant is the higher of its value
in use of Rs. 150,000 or its fair value less costs to sell. If Hussain Associates Ltd trades in
the plant it would receive Rs. 180,000 by way of a part exchange, but this is conditional on
buying new plant which Hussain Associates Ltd. is reluctant to do. A more realistic amount
of the fair value of the plant is its current disposal value of only Rs. 20,000. Thus the
recoverable amount would be its value in use of Rs. 150,000 giving an impairment loss of
Rs. 50,000 (Rs. 200,000 – Rs. 150,000).
The remaining effect on income would be that a depreciation charge for the last six months
of the year would be required. As the damage has reduced the remaining life to only two
years (from the date of the impairment) the remaining depreciation would be Rs. 37,500
(Rs. 150,000/ 2 years  6/12).Thus extracts from the financial statements for the year
ended 30 September 2016 would be:

Statement of financial position


Non-current assets Rs.
Plant (150,000 – 37,500) 112,500

Statement of profit or loss


Plant depreciation (40,000 + 37,500) 77,500
Plant impairment loss 50,000

(b) Purchase of Sparkle


There are a number of issues relating to the carrying amount of the assets of Sparkle
Limited that have to be considered. It appears the value of the brand is based on the
original purchase of the ‘Sparkle Spring’ brand. The company no longer uses this brand
name; it has been renamed ‘Refresh’. Thus it would appear the purchased brand of
‘Sparkle Spring’ is now worthless. Sparkle Limited cannot transfer the value of the old
brand to the new brand, because this would be the recognition of an internally developed
intangible asset and the brand of ‘Refresh’ does not appear to meet the recognition criteria
in IAS 38. Thus prior to the allocation of the impairment loss the value of the brand should
be written off as it no longer exists.
The inventories are valued at cost and contain Rs. 2 million worth of old bottled water
(Sparkle Spring) that can be sold, but will have to be relabelled at a cost of Rs. 250,000.
However, as the expected selling price of these bottles will be Rs. 3 million (Rs. 2 million 
150%), their net realisable value is Rs. 2,750,000. Thus it is correct to carry them at cost
i.e. they are not impaired. The future expenditure on the plant is a matter for the following
year’s financial statements.
Applying this, the revised carrying amount of the net assets of Sparkle Limited’s cash-
generating unit (CGU) would be Rs. 25 million (Rs. 32 million – Rs. 7 million re the brand).
The CGU has a recoverable amount of Rs. 20 million, thus there is an impairment loss of
Rs. 5 million. This would be applied first to goodwill (of which there is none) then to the
remaining assets pro rata. However under IAS2 the inventories should not be reduced as
their net realisable value is in excess of their cost. This would give revised carrying
amounts at 30 September 2016 of:

© Emile Woolf International 287 The Institute of Chartered Accountants of Pakistan


Advanced accounting and financial reporting

Rs.000
Brand nil
Land containing spa: 12,000 – [(12,000/20,000)  5,000] 9,000
Purifying and bottling plant:
8,000 – [(8,000/20,000)  5,000] 6,000
Inventories 5,000
20,000

21.4. IMPS
(a) Impairment loss

Rs. m
Carrying value 500
Recoverable amount (385)
Impairment loss 115

Recoverable amount is value in use (Working 1) as this is higher than the fair value less
costs of disposal (Working 2).
Workings
(1) Value in use:
Forecast cash flows discounted at 12%:
Rs. m
Year 1 (185 × 0.893) 165.2
Year 2 (160 × 0.797) 127.5
Year 3 (130 × 0.712) 92.6
Total 385.3

(2) The fair value less costs of disposal:

Rs. m
Goodwill 0
Freehold 270
Freehold land and buildings 50
320

(b) Treatment of impairment loss


IAS 36 requires the impairment loss to be allocated to the various non-current assets in the
following order: firstly, goodwill, secondly, to other assets, either pro-rata or on another
more appropriate basis.
Before Impairment After
impairment loss (W1) impairment
Rs. m Rs. m Rs. m
Goodwill 70 (70) -
Land and buildings 320 (33) 287
Plant and machinery 110 (12) 98
500 (115) 385

© Emile Woolf International 288 The Institute of Chartered Accountants of Pakistan


Answers

Because the land and buildings have been re-valued, the impairment is treated as a
revaluation decrease until the carrying amount of the asset reaches its depreciated
historical cost. The revaluation reserve relating to the asset is Rs. 65 million and so is
adequate to cover the full impairment of Rs. 33m. The impairment must be separately
disclosed and the notes to the accounts must specify by class of asset the impairment
recognised directly to equity.
The impairment loss on the goodwill and plant (Rs. 82 million) must be recognised in profit
or loss for the year. The notes to the accounts must specify the line item in which the
impairment loss has been included.
Where the impairment write-down is material, information must also be provided as to the
events and circumstances that led to the loss, the nature of the assets affected, the
segment to which the asset belongs, that recoverable amount was based on value in use
and the discount rate used to calculate this.
Workings
Loss on the various non-current assets
After the impairment loss has been recognised on the goodwill there is still 115 - 70 = 45
loss to be allocated to the other noncurrent assets, on a pro-rata basis.

Loss on land and buildings:


320
x 45  33
320  110
Loss on plant and machinery:
110
x 45  12
320  110

21.5. GYO MOVERS LIMITED


GYO Movers Ltd.
Extracts from statement of financial position As on 30 June 2016
Fixed Asset Rs. in million
Property, plant & equipment (W-1) 1,186.55
W-1: Determination of carrying amount after impairment
Green Yellow Orange Corporate Assets
Other Other Other HO Computer Total
Description Buses Buses Buses Goodwill Equipment
assets assets assets Building Network
---------------------------------------- Rs. in million ----------------------------------------
Carrying amount 225.00 400.00 150.00 350.00 95.00 100.00 10.00 100.00 55.00 45.00 1,530.00
Round 1 : Allocation of impairment
determined in (W-2)
-First, allocate the impairment amount to
goodwill (10.00) (10.00)
- Second, allocate the remaining impairment
amount i.e. Rs. 333.44 (343.44-10) to all other
assets proportionately subject to limiting to FV (27.40) *1(104.20) (26.50) *2(91.18) - - - *3(26.05) (9.00) - (284.33)
fair value 197.60 123.50 98.80
(80×2.47) (50×2.47) (40×2.47) 46.00 60.00
Carrying amount after 1st round of
impairment 197.60 295.80 123.50 258.82 95.00 100.00 - 73.95 46.00 45.00 1,235.67
Round 2 : Allocation of remaining impairment
proportionately 49.12 (343.44 - 10- 284.33) - *4(23.12) *5(20.22) - *6(5.78) - - (49.12)
Carrying value after impairment 197.60 272.68 123.50 238.60 95.00 100.00 - 68.17 46.00 45.00 1,186.55

© Emile Woolf International 289 The Institute of Chartered Accountants of Pakistan


Advanced accounting and financial reporting

Green Yellow Orange Corporate Assets


Other Other Other HO Computer Total
Description Buses Buses assets Buses assets Goodwill Equipment
assets Building Network
---------------------------------------- Rs. in million ----------------------------------------
*1[400÷(225+400+150+350+100+55)]×333.44 *2 [350÷(225+400+150+350+100+55)]× 333.44 [100÷(225+400+150+350+100+55)]× 333.44
*4[295.80÷(295.80+258.82+73.95)]×49.12 *5[258.83÷(295.80+258.83+73.95)]×49.12 [73.95÷(295.80+258.83+73.95)]×49.12

W-2: Determination of impairment amount


Green Yellow Orange Total
……….. Rs. In million ……..
Carrying value of buses 225.0 150.0 95.0
Carrying value of other assets 400.0 350.0 100.0
Carrying value of all each CGU (A) 625.00 500.00 195.0 1,320.00
Useful life (in years) 20 15 10
Weighting based on useful life (B) 2 1.5 1
Carrying amount after weighting (A×B) 1250 750 195 2,195
Pro-rata allocation of total corporate assets and 119.59 71.75 18.66 210
goodwill[i.e. 10+ (100 + 55 + 45) = 210] (c)
Carrying amount after allocation (A+C) 744.59 571.75 213.66 1,530.00
Less: Recoverable amount (W-3) 522.90 450.00 282.50
Impairment loss 221.69 121.75 - 343.44

W-3: Determination of recoverable amount

Green Yellow Orange


--------------- Rs. in million ---------------
Net cash flows 70 60 50
Annuity factor at 12% 7.47 6.81 5.65
Value in use of each CGU (i) 522.9 408.6 282.5

Fair value less cost to sell (ii) 500.0 450.0 250.0

Recoverable amount [Higher of (i) and (ii)] 522.9 450.0 282.5

© Emile Woolf International 290 The Institute of Chartered Accountants of Pakistan


Answers

21.6. KHYBER LIMITED


(i) Impairment of CGU under IAS 36:

Carrying Fair value Goodwill Impairment Impairment Total


Description less cost to
value sell impairment Round 1 *1 Round 2 impairment
*2

------------------ Rs. in million ------------------


Building 22.00 21.70 *0.30 - 0.30
Machinery 13.00 16.00 *-
Equipment 19.00 3.86 4.38 8.24
License 20.00 18.00 *2.00 2.00
Investment property 22.00 22.00 *- - -
Investment property 8.00 1.62 1.84 3.46
Goodwill 3.00 3.00 - - 3.00
Inventory at NRV 8.00 8.00 *- - -
Carrying value 117.00 3.00 7.78 6.22 17.00
Recoverable amount (100.00)
Impairment required 17.00
Charged to profit or loss 16.70
(17-0.30)
*1Allocationof impairment loss in the ratio of 14(17-3) ÷ 69(22+19+20+8)
*2Allocationof impairment loss in the ratio of 6.22(14-7.78) ÷ 27
*Restricted to fair value less cost to sell

(ii) Impairment of Disposal group under IFRS 5:

Goodwill Impairment of
Description Carrying value
impairment scoped in assets*3
------------------ Rs. in million ------------------
Building 22.00 4.98
Machinery 15.00 3.39
Equipment 19.00 4.30
License 20.00 4.52
Investment property 22.00 **_
Investment property 8.00 1.81
Goodwill 3.00 3.00 -
Inventory at NRV 8.00 **_
Carrying value 117.00 3.00 19.00
Fair value less cost to sell (95.00)
Impairment required 22.00

Charged to profit or loss 22.00


*3 Allocation of impairment loss in the ratio of 19(22-3) ÷ 84(22+15+19+20+8)
** No impairment is allocated due to scope out assets

© Emile Woolf International 291 The Institute of Chartered Accountants of Pakistan


Advanced accounting and financial reporting

CHAPTER 22 - IAS 40: INVESTMENT PROPERTY


22.1. VICTORIA
(a) Treatment in the financial statements for the year ended 31 December Year 8 (IAS16)
Property 1
This is used by Victoria as its head office and therefore cannot be treated as an investment
property. It will be stated at cost minus accumulated depreciation in the statement of
financial position. The depreciation for the year will be charged in the statement of profit or
loss.
Property 2
This is held for its investment potential and should be treated as an investment property. It
will be carried at fair value, Victoria’s policy of choice for investment properties. It will be
revalued to fair value at each year end and any resultant gain or loss taken to the
statement of profit or loss (Rs. 400,000 gain in Year 8).
Property 3
This is held for its investment potential and should be treated as an investment property.
However, since its fair value cannot be arrived at reliably it will be held at cost minus
accumulated depreciation in the statement of financial position. The depreciation for the
year will be an expense in the statement of profit or loss.
This situation provides the exception to the rule whereby all investment properties must be
held under either the fair value model, or the cost model.
(b) Analysis of property, plant and equipment for the year ended 31 December Year 8

Other land Investment Investment


and property property
Total
buildings held held at cost
(W1) at fair value (W2)
Rs. Rs. Rs. Rs.
Cost/valuation
On 1 January Year 8 1,000,000 2,300,000 2,000,000 5,300,000
Revaluation - 400,000 - 400,000
––––––––– ––––––––– ––––––––– –––––––––
On 31 December Year 8 1,000,000 2,700,000 2,000,000 5,700,000
––––––––– ––––––––– ––––––––– –––––––––
Accumulated depreciation
On 1 January Year 8 87,500 - 220,000 307,500
Charge for the year (W1) 12,500 - 40,000 52,500
––––––––– ––––––––– ––––––––– –––––––––
On 31 December Year 8 100,000 - 260,000 360,000
––––––––– ––––––––– ––––––––– –––––––––
Carrying amount
On 31 December Year 7 912,500 2,300,000 1,780,000 4,992,500
––––––––– ––––––––– ––––––––– –––––––––
On 31 December Year 8 900,000 2,700,000 1,740,000 5,340,000
––––––––– ––––––––– ––––––––– –––––––––

© Emile Woolf International 292 The Institute of Chartered Accountants of Pakistan


Answers

Tutorial note
In practice, with a more complex property, plant and equipment table the investment
properties would be included within the land and buildings column with the required
disclosures being given separately in a note to the table.
Workings
(1) Depreciation on Property 1
Rs.
Brought forward (500,000 ÷ 40  7) 87,500
Year 8 (500,000 ÷ 40) 12,500
(2) Depreciation on Property 3
Rs.
Brought forward (2,000,000 ÷ 50  5.5) 220,000
Year 8 (2,000,000 ÷ 50) 40,000

© Emile Woolf International 293 The Institute of Chartered Accountants of Pakistan


Advanced accounting and financial reporting

CHAPTER 24 - FIRST TIME ADOPTION OF IFRS


24.1. IFRS 1
(a) The first IFRS reporting period was the year ended 31 December 2016, and the date of
Transition to IFRS was 1 January 2015.
(b) The procedures which must be followed in order to prepare the Financial
Statements for the year ended 31 December 2016, are as follows:
(i) Choice of accounting policies to be included as part of notes to the Financial
Statements
(ii) Preparation of the opening IFRS Statements of Financial Position by applying the
following rules, except in cases where IFRS grants exemptions and /or prohibits
retrospective application:
Recognise all assets and liabilities required by IFRS
Not recognise assets and liabilities not permitted by IFRS
Reclassify all assets and liabilities and equity in accordance with IFRS
Measure all assets and liabilities in accordance with IFRS
Any gains and losses arising from this exercise should be recognised immediately in
retained earnings as at January 2016
(iii) Since IAS 1 requires that at least one year of comparative prior period financial
information be presented, the opening Statement of Financial Position will be 1
January 2015, if not earlier.
(iv) Preparation of full IFRS Financial Statements for the year ended 31 December
2016, which should include:
 three statements of financial position
 two statements of comprehensive income
 two separate statements of profit or loss (if presented)
 two statements of cash flows
 two statements of changes in equity
 related notes, including comparative information
(c) The reconciliation which the company must include in its financial statements for the year
ended 31 December 2102, to explain how the transition from previous GAAP to IFRS
affect the reported financial position, financial performance and cash flows are as follows:
(i) Reconciliation of equity reported under previous GAAP to equity under IFRS both
(a) at the date of the opening IFRS Statement of Financial Position and (b) the end
of the last annual period reported under the previous GAAP.
(ii) Reconciliation of Total Comprehensive Income under IFRS for the last annual period
reported under the previous GAAP to Total Comprehensive Income under IFRS for
the same period.
(iii) Explanation of material adjustments that were made, in adopting IFRS for the first
time, to the Statement of Financial Position, Statement of profit or loss and
Statement of Cash Flows.
(iv) If errors in previous GAAP financial statements were discovered in the course of
transaction to IFRS, those must be separately disclosed.
(v) If the entity recognised or reversed any impairment losses in preparing its opening
IFRS Statement of Financial Position, these must be disclosed.
(vi) Appropriate explanations if the entity has elected to apply any of the specific
recognition and measurement exemptions permitted under IFRS 1, for example, if
the entity used fair values as deemed cost.

© Emile Woolf International 294 The Institute of Chartered Accountants of Pakistan


Answers

(d) Contents of a typical statement of changes in equity are as follows:


(i) Total comprehensive Income for the period, showing separately amounts
attributable to owners of the parent and to non-controlling interests.
(ii) For each component of equity, the effect of retrospective application or retrospective
restatement recognised in accordance with IAS 8.
(iii) For each component of equity, a reconciliation between the carrying amount at the
beginning and end of the period, separately disclosing
 Profit or loss
 Other comprehensive income
 Transactions with owners in their capacity as owners showing separately,
contributions by and distributions to owners, and changes in ownership
interests in the subsidiaries that do not result in a loss of control.

© Emile Woolf International 295 The Institute of Chartered Accountants of Pakistan


Advanced accounting and financial reporting

CHAPTER 25 - SPECIALISED FINANCIAL STATEMENTS


25.1. LATEEF BANK LIMITED
8 Lendings to financial institutions
Notes 2016 2015

Rs. in million

Call money lending 8.2 850 1,200


Repurchase agreement lending
(reverse repo) 8.3 2,100 2,850

2,950 4,050

8.1 Particulars of lending

In local currency 2,840 3,900

In foreign currencies 110 150

2,950 4,050

8.2
These are unsecured lendings to financial institutions, carrying mark up ranging from
15% to 17% (2015: 10% to 12 % and will mature latest by October 2016.
8.3
These are short term lendings to various financial institutions and are secured against
government securities shown in note 8.4 below. These carry mark up at rates ranging
from 9.5% to 13.2 % (2015:8% to 10.5 %) and will mature on various dates, latest by
October 2016.
8.4
Securities held as collateral against lending to financial institutions

Rs. in million

2016 2015

Held Further Held Further


by given as by given as
bank collateral Total bank collateral Total
Market treasury bills 1,650 - 1,650 1,850 - 1,850

Pakistan investment
bonds 450 - 450 1,000 - 1,000

2,100 - 2,100 2,850 - 2,850

Market value of the above as at September 30, 2016 amounted to Rs. 2,250 million
2015: 2,930 million).

© Emile Woolf International 296 The Institute of Chartered Accountants of Pakistan


Answers

25.2. AL-AMIN BANK LIMITED


Notes to the Financial Statements for the year ended XX/XX/XX
1.1 Particulars on non-performing advances
Advances include Rs. 5,000 million which, as detailed below, have been placed under
non-performing status:
2016
Classified
Category of classification Provisions held
lending
Domestic Rs. in million
Other assets especially mentioned 100 5
Sub-standard 400 70
Doubtful 840 530
Loss 3,400 3,345
4,740 3,950

Overseas
Not past due but impaired 260 50

Total 5,000 4,000

1.2 Particulars of provision against non-performing advances


2016
Specific General Total
Rs. in million
Opening balance 3,320 65 3,385
Charge for the year 802 40 842
Amounts written off (50) - (50)
Reversals (90) - (90)
Exchange adjustments 18 - 18

Closing balance 4,000 105 4,105

25.3. IAS 26
(a) The differences between 1AS 26 - Accounting and Reporting by Retirement Benefit Plan
and IAS 19 - Employee Benefits are:
(i) IAS 26 addresses the financial reporting considerations for the benefit plan itself as
the reporting entity while IAS 19 deals with employers’ accounting for the cost of
such benefits as they are earned by the employees
(ii) These standards are thus somewhat related, but there will not be any direct inter-
relationship between the amounts reported in benefit plan financial statements and
amounts reported under IAS 19 by employers.

© Emile Woolf International 297 The Institute of Chartered Accountants of Pakistan


Advanced accounting and financial reporting

(iii) IAS 26 differs from IAS 19, Employee Benefits, in allowing a choice of measurement
based either on current salary levels or projected salary levels. IAS 19 requires an
actuarial valuation to be based on the latter, whereas IAS 26 requires valuation
based on present value of promised retirement benefits.
(b) Defined Benefit Plan (DBP)
Defined benefit plans are retirement benefit plans under which amounts to be paid as
retirement benefits are determined by reference to a formula usually based on employees’
earnings and/or years of service.
(c) Defined Contribution Plan (DCP)
Defined contribution plans are retirement benefit plans under which amounts to be paid as
retirement benefits are determined by contributions to a fund together with investment
earnings thereon.
(d) Actuarial present value of promised retirement benefits: This is the present value of the
expected payments by a retirement benefit plan to existing and past employees
attributable to the service already rendered.

25.4. SOGO LIMITED


(a) SOGO Limited
Staff Gratuity Fund
Statement of net assets available for benefits as at December 31, 2016
Note 2016
Rupees
ASSETS
Investments 3 159,033,144
Receivable from SOGO Limited 1,147,150
Balances with banks 17,930,120
178,110,414
LIABILITIES
Payable to outgoing members 4,301,017
Accrued expenses 3,822
Withholding tax payable 61,251
4,366,090
NET ASSETS 173,744,324

REPRESENTED BY:
Members' Fund
(Rs. 142,472,122 + Rs. 27,712,441) 170,184,563
Surplus on re-measurement of investments available for sale 3,559,761
173,744,324

© Emile Woolf International 298 The Institute of Chartered Accountants of Pakistan


Answers

(b) SOGO Limited


Staff Gratuity Fund
Statement of changes in net assets available for benefits for the year ended December
31, 2016

General Reserve Fund


Opening Balance 286,754,111
Contribution during the year 10,623,106
Transferred / paid to outgoing members (12,432,973)
142,472,122
Income
Profit from investments 23,389,251
Dividend income 2,696,399
Liabilities no more payable 3,450,000

29,535,650
Expenditure

Bank charges (3,342)


Audit fee (10,000)

(13,342)

Net Income for the year 27,712,441

25.5. JABBAR (PVT) LIMITED


(i) IFRS for SMEs does not allow capitalization of borrowing cost. So capitalizing interest cost
of Rs. 0.3 million should be reversed and charged to profit and loss account.
Consequently, carrying amount of the building i.e. Rs. 3 million does not exceed the
recoverable amount of Rs. 3.1 million and therefore no impairment is required.
(ii) IFRS for SMEs requires that investment properties must be measured subsequently at fair
value, (unless fair value cannot be measured reliably without undue cost or effort) and
PPE must be measured subsequently using the cost model. Based on this, treatment of
both shops should be as follows:

 Shop A should be classified as property, plant and equipment. Revaluation surplus of


Rs. 1.125 million [6 – (5×0.975)] related to this shop should be accounted for in
revaluation surplus on revaluation of assets.

 Shop B should be classified as investment property. Therefore fair value model is


appropriate as being followed by the company. However, depreciation should not be
computed under revaluation model of investment property so depreciation expense of
Rs. 0.1 million (4×5%×50%) and incorrect revaluation of Rs. 1.1 million [5 –
(4×97.5%)] should be reversed and increase in fair value of Rs. 1 million should be
credited to profit or loss account.

© Emile Woolf International 299 The Institute of Chartered Accountants of Pakistan


Advanced accounting and financial reporting

25.6. KARACHI BANK LIMITED


Karachi Bank Limited
Statement of financial position
As on 31 December 2016

Assets: Rs. in million

Cash and balances with treasury banks (9,100+14,500+700+2,300+68) 26,668

Balances with other banks (412+311+1,400) 2,123

Lending to financial institutions (650+6,100) 6,750

Investments – net (24,500+1,200+1,800–222) 27,278

Advances – net (114,200+4,900+679-6,678) 113,101

Operating fixed assets 24,700

Other assets (21,450+3189) 24,639

225,259

25.7. LEOPARD INCOME FUND


Statement of movement in Unit Holders’ Fund
For the year ended 30 June 20X8

Capital value Undistributed


Total
income

----------------- Rs. in million -----------------

Net assets at beginning of the year 9,648 104 9,752

Issuance of 388 million units:

Capital value 7,372 - 7,372

Element of income 70 - 70

Total proceeds on issuance of units 7,442 - 7,442

Redemption of 441 million units:

Capital value (8,382) (8,382)

Element of loss (14) (50) (64)

Total payment of redemption of units (8,396) (50) (8,446)

Total comprehensive income for the year - 214 214

Distribution during the year - (150) (150)

Net assets at end of the year 8,694 118 8,812

© Emile Woolf International 300 The Institute of Chartered Accountants of Pakistan


Answers

Capital value Undistributed


Total
income

----------------- Rs. in million -----------------

Undistributed income brought forward

Realized income 97

Unrealized income 7

104

Accounting income available for distribution

Relating to capital gain 3

Excluding capital gain 161

164

Distribution during the year (150)

Undistributed income carried forward 118

Undistributed income carried forward

Realized income 122

Unrealized loss (4)

118

© Emile Woolf International 301 The Institute of Chartered Accountants of Pakistan


Advanced accounting and financial reporting

CHAPTER 28 - ISLAMIC ACCOUNTING STANDARDS


28.1 SALE AND LEASE BACK TRANSACTIONS
When an asset is sold with an intention to enter into an ljarah arrangement, gain or loss shall be
recorded as follows:
Sold at fair value:
Profit or loss should be recognized immediately.
Sold at below fair value:
If the sale price is below fair value, any profit or loss should be recognized immediately except
that, if the loss is compensated by future lease payments at below market price, it should be
deferred and amortized in proportion to the lease payments over the period for which the asset is
expected to be used.
Sold at above fair value:
If the sale price is above fair value, the excess over fair value should be deferred and amortized
over the period for which the asset is expected to be used.

© Emile Woolf International 302 The Institute of Chartered Accountants of Pakistan

Das könnte Ihnen auch gefallen